Download as pdf or txt
Download as pdf or txt
You are on page 1of 216

OSMOSIS

OTORRINOLARINGOLOGÍA

● PLAYLIST 1
○ An 18-month-old girl is brought to the emergency department because of an
awful sounding cough. She has had a runny nose and a low grade fever for
the past 2 days, with some hoarseness and a rough-sounding cough that
started this afternoon. This evening she began making some high pitched
sounds when taking breaths, and she seemed to be having a bit of trouble
breathing. She is alert and does not appear to be in acute distress. She has a
temperature of 101˚F with a respiratory rate of 50/min and O2 saturation of
97%. There is an audible inspiratory stridor which worsens when she starts to
cry during the examination. She has an occasional barking cough. Her
pharynx is mildly erythematous with normal tonsils and no exudate. Which of
the following is the best step in management? Racemic epinephrine and
intramuscular corticosteroid therapy
○ A 12-year-old boy comes to the hospital because of cold which has lasted for
2 weeks. History from his mother indicate episodes of fevers, nonproductive
cough which is worse at night, purulent nasal discharge, and general facial
pain. He denies any ear pain. He has been using nasal decongestants and
cough syrup to treat his symptoms. Currently, he has a temperature of 102
oF (38.9oC). On physical examination, you observe some mild erythema and
swelling of the nasal turbinates with mucopurulent nasal discharge. Which of
the following is the most likely cause of this patient symptom? Maxillary
sinusitis
○ A 5-year-old boy is brought to the emergency department by his mother
because of neck pain and difficulties swallowing for the past day. The mom
reports that her son "has not been himself." She states that he has been
crying and refusing to move his neck due to pain. He has not eaten all day
due to pain upon swallowing. He also has had a fever for the past 2 days. A
complete blood count is done showing an elevated white blood cell count.
Lumbar puncture is normal. A lateral neck x-ray shows the prevertebral soft
tissue swelling. Which of the following is the most appropriate next step in
management? Start IV antibiotics, IV fluids, and send to operating room
for surgical drainage
○ A 17-year-old girl comes to the pediatric office because of left ear pain that
began 2 days ago and has become progressively worse. The patient
complains that the pain is so severe that disrupts her sleep and persists even
after the use of topical antibiotic medication. Medical history is significant for
type 2 diabetes that is being poorly managed. Examination of the left ear
shows edema and erythema of the auditory canal with a greenish discharge
and manipulation of the left pinna elicits severe pain. The tympanic
membrane can be visualized and appears normal. Examination of the right
ear shows no abnormalities. Which of the following is the most likely
diagnosis? Malignant otitis externa
○ An hour-old newborn boy is brought to the pediatric emergency department
because of respiratory distress for the past 30 minutes. The boy is born to a
primigravid mother at home with a midwife. The mother reports that the baby
turns blue and has difficulty breathing when she tries to nurse him. Physical
examination shows an actively crying baby who is pink all over. When the
baby attempts to suck on a gloved finger, he turns cyanotic with intercostal
and supraclavicular retractions. Which of the following is most appropriate
next best step in management? Attempting to pass a nasal catheter
○ A 4-year-old boy is brought to the emergency department by his mother
because of fever and constant tugging of his left ear. Physical examination
shows a swollen and erythematous left ear canal that is tender to the touch.
An opaque serum-like exudate is seen oozing through the left tympanic
membrane. An audiometry test shows bilateral conductive hearing loss.
Which of the following is the most likely causative microbe? Streptococcus
pneumonia
○ A previously healthy 5-year-old boy comes to the office because of fever for
the past two days. His mother reports onset of vomiting and dysphagia earlier
this morning but denies any diarrhea, rhinorrhea or upper respiratory
symptoms. She notes that he just started kindergarten, and there have been
a few other children out sick. His temperature is 38.8°C (101.9°F). His
posterior oropharynx is erythematous with an inflamed uvula and enlarged
symmetric masses. He also has enlarged, tender anterior cervical
lymphadenopathy. He is quite resistant, constantly moving his head to avoid
the examination. Which of the following is the most appropriate treatment?
Oral penicillin
○ A ten-month-old girl is brought to the emergency department by her parents
with three days of high fever of 104*F°, malaise, anorexia, conjunctivitis,
cough, and a runny nose. The patient and her family recently emigrated from
Eastern Europe to the United States. On oral examination, there is no tonsilar
exudate, but there is a cluster of small granular-appearing white spots on the
patient's buccal mucosa. There are no rashes on the patient's trunk, face, or
extremities. Of the following, which is the most likely diagnosis? Measles
○ A 6-week-old infant comes the office for a routine checkup. His mother says
that he has been breathing loudly over the last week. This is particularly
heard on inspiration and is most noticeable when he is lying on his back,
crying, or during feeding, though the infant does not have difficulty
swallowing. She denies nasal congestion, cough or fever, and says he has
been active as usual. She denies abnormal changes in his skin color. His
temperature is 37.2°C (98.96°F), pulse is 130/min, and respirations are
50/min. Physical examination shows a low-pitched inspiratory stridor
appreciated as he lies on the exam table, but he does not have increased
work of breathing. The lungs are clear to auscultation. Which of the following
is the most likely diagnosis? Laryngomalacia
○ A 10-year-old Asian-American boy comes to your office because of fever,
sore throat, and cough for the past week. His past medical history is
noncontributory. He denies any cold symptoms, rhinnorhea, diarrhea, or
changes in appetite. His mother says he has lost weight since his last visit 6
months ago for immunizations and a well-child visit. Previously, his weight
was at the 35th percentile; now he is at the 20th percentile. Physical
examination shows the patient is afebrile, breathing comfortably, with no
nasal discharge. Posterior oropharynx shows no abnormalities. He has
moderate non-tender cervical lymphadenopathy bilaterally. No murmurs,
rubs, or gallops are heard on auscultation of the heart. Diminished breath
sounds on the right are heard on auscultation of the lungs. A chest x-ray is
ordered. Which of the following is the most appropriate next step in
management? Tuberculin skin test
○ A 3-year-old boy is brought to the pediatrician's office by his mother because
of left ear pain for the past 4 days. Physical examination shows a bulging and
erythematous left tympanic membrane. The use of pneumatic pressure
otoscope shows no movement of the tympanic membrane. Which of the
following organisms is the most likely cause of this patient's symptoms?
Haemophilus influenzae
○ A 3-year-old boy comes to the pediatrician's office because of a cold. His
mother also indicated that he began draining yellow-green fluid from his left
nostril. He has had a fever and other upper respiratory infection symptoms
for the past 10 days. On physical examination, you confirm a mucopurulent
exudate coming from the left nostril. Which of the following will be the likely
complication if left untreated? Orbital cellulitis
○ An 18-year-old woman comes to the emergency department because of
persistent nasal discharge from her right nostril for the past 5 months. She
states that she is healthy but always had an issue with breathing out of the
right nostril. Physical examination shows a right nasal cavity full of thick
yellowish secretions. A nasal patency test shows no air entry on the right
side. A non-enhanced axial computed tomography scan is done showing
failure of communication of the nasal cavity to the nasopharynx. Which of the
following is the most likely diagnosis? Choanal atresia
○ A 14-year-old girl presents to her pediatrician with inflammation of the left
outer ear and severe pain upon manipulation of the left pinnacle. The patient
reports that she is on the high-school swim team and that several of her
teammates have developed ear infections and pruritic, erythematous rashes.
Physical examination reveals an intact tympanic membrane on both sides,
though it is somewhat obstructed by a greenish discharge, and there is
substantial swelling and narrowing of the left ear canal with some associated
hearing loss on the left side. Based on the patient's history and symptoms,
which of the following organisms is most likely responsible for this patient's
infection? Pseudomonas aeruginosa
○ A newborn child is brought to the family medicine clinic for his first well-child
visit. The mother reports an uncomplicated pregnancy, but that the delivery
was complicated by breech presentation. When taking the history from the
mother, she reports a good mood, and that she is surprised at how "not bad"
the crying is. She notes that he tends to cough sometimes while eating, but is
otherwise doing well. Upon examination, the patient has a weak, raspy cry,
but no other abnormalities. What diagnostic test will most likely confirm the
diagnosis in this child? Flexible laryngoscopy
○ A 16-year-old boy comes to the office because of redness in both eyes and
facial swelling for the past four days. He has also been experiencing fever
and myalgia. He states that he has never had his vaccinations as a young
child. Physical examination shows bilateral conjunctival hyperemia, tender
parotid glands and swelling and tenderness of his left testicle. Visual acuity
examination shows no abnormalities. Which of the following is the most likely
diagnosis? Mumps
○ A 2-year-old boy is brought to the office by his mother because of complaints
of fever and ear pain. He began tugging on his ear and complaining of pain
three days ago. The mother reports a temperature of 37.8°C (100.0°F) this
morning, with decreased appetite. Current temperature is 38.6ºC (101.4ºF).
ENT examination shows erythema and decreased mobility of the right
tympanic membrane. Which is the most appropriate pharmacological
management? Amoxicilin
○ An 18-month-old boy is brought to his pediatrician's office by his father
because of fever and ear tugging for the past 3 days. His father reports a
temperature of 37.8°C (100.0°F) this morning and a decreased appetite. The
boy's temperature is 38.6°C (101.4°F), pulse is 110/min, respirations are
22/min, and blood pressure is 90/60 mm Hg. Otoscopic examination shows a
bulging and erythematous right tympanic membrane with decreased mobility
of the right tympanic membrane. Oral amoxicillin is initiated. Four days later,
the child returns to the office with no improvement in symptoms. Repeat
physical examination shows a bulging tympanic membrane bilaterally with a
meniscus of fluid within the middle ear. Which of the following is the most
appropriate next step in management? amoxicillin/clavulanate
○ An 8-year-old boy comes to an urgent care center because of a sore throat
and fever for two days. His mother says this illness seems different from his
others because he has no cough, no rhinorrhea, no ear pain, and no sick
contacts. Examination shows bilateral tender lymph nodes just below the
mandible. Lung fields are clear to auscultation bilaterally, and there is no
rash. His temperature is 38.6℃ (101.5°F). He has no drug allergies and
first-line antibiotic therapy is begun. Which of the following is the most likely
mechanism of action of the prescribed medication? Inhibition of cell wall
synthesis
○ A 4-year-old boy comes to the emergency department with his father
because of a severe nosebleed that started 15 minutes ago. The father says
that his son was watching television with him when suddenly his nose started
bleeding and despite sticking paper up his nose it wouldn't stop. He also says
that there was no preceding trauma, but this has happened five times before
with bleeding stopped after about an hour each time. His temperature is 37ºC
(98.6ºF), pulse is 89/min, respiratory rate is 14/min and blood pressure is
118/74 mmHg. Physical examination shows a well-developed, well-nourished
boy in no distress with blood coming from a single nostril. His father says that
the boy was adopted and the family history is unknown. Which of the
following is the most appropriate next step in management? Pinch on the
anterior soft portion of the nose for 10 minutes
○ A 3-year-old boy is brought to the office because he has been coughing for
the past 3 weeks. His mother states that cough seems to come in bouts
lasting several minutes. The coughing is sometimes severe enough to cause
him to vomit. About 6 weeks ago, she reports that he had a very mild cold
that went away on its own, and that this new cough seemed to happen
spontaneously. Medical history is noncontributory. Examination shows the
child has a fever of 38˚ C (100.4˚ F) but otherwise normal vital signs. His
oropharynx is erythematous but does not have any frank lesions or exudates.
Pulmonary and cardiac auscultation are noncontributory. A chest X-ray is
obtained and is also normal. Which of the following is the most likely
diagnosis? Pertussis
○ A 2-year-old boy is brought to the pediatrician by his mother because of a
low-grade fever for the last 24 hours. The patient's mother states that he has
been complaining of pain when he swallows and that he seems more
fatigued than usual. When asked about recent travel, she mentions that her
family went to visit close relatives out of the state a week before, and he was
around quite a few children his age. The patient currently has a temperature
of 38°C (100.4°F). HEENT examination shows mild pharyngeal and palatal
erythema without exudate, as well as enlarged and erythematous nares.
Which of the following is the best diagnostic test to confirm the cause of the
patient's symptoms? Throat culture test
○ A 16-year old girl comes to your office because of a sore throat and severe
fatigue for the past week. Her mother also notices that her appetite has been
decreased and she has had a mild fever for about the same duration of time.
She is on the field hockey team at school but has missed several practices
due to her illness. Examination shows that her pharynx is erythematous with
sparse petechiae on the palate. Her tonsils are moderately enlarged with no
exudates, and she has tender posterior cervical lymphadenopathy. Her
abdomen is soft with mild tenderness in the left upper quadrant. No
splenomegaly is palpated. Which of the following is the most appropriate next
step in treatment? Counsel her to avoid active physical activity for 3
weeks
○ A 5-year-old girl is brought to the pediatrician's office because of a 3-day
history of fever and throat pain. Her medical history is only relevant to a
penicillin allergy. Upon medical interrogation, the mother says that she
recently started a rash. Physical exam shows a blanching and coarse texture
rash in both palms and soles, circumoral pallor, and erythematous tongue
(see image below). Her temperature is 38.6°C (103.1°F), pulse is 80/min,
respirations are 16/min, blood pressure is 110/70 mmHg. Erythromycin

● PLAYLIST 2
○ A 26-year-old woman comes to the emergency department because of
left-sided redness and swelling of the eyelid with severe pain on eye
movement. On physical examination the left eye has decreased vision and
reduced movement and is proptotic. The patient is diagnosed with orbital
cellulitis and started on aggressive treatment. Which of the following risk
factors is most strongly associated with orbital cellulitis? Sinusitis
○ A 17-year-old boy comes to the office because of allergic rhinitis. He says he
has symptoms of sneezing, nasal congestion, and itching every September
when the new school year beings. He has a family history of childhood
asthma eczema. Which of the following medications is the most appropriate
next step in the management of the patient's symptoms? Intranasal
glucocorticoids
○ A 58-year-old female comes to the emergency department because of nasal
congestion, cough, purulent nasal discharge, and maxillary tooth discomfort
for 2 weeks. In addition, she says that she thinks the pressure in her head is
increased. Past medical history is noncontributory. Examination shows facial
pain and pressure that is worse when the patient bends forward. Endoscopic
examination shows purulent secretions in the middle meatus. Which of the
following is the most likely diagnosis? Acute sinusitis
○ A 25-year-old female comes to the emergency department because of sore
throat, subjective fever, and rhinorrhea of two weeks duration. She also
reports increased tiredness and a dry cough. She also adds that her
symptoms has not improved over 2 weeks. On physical examination, her vital
signs are within normal limits and her lungs are clear to auscultation. There is
tenderness to palpation of her bilateral medial zygomatic and upper maxillary
bones. Which of the following is the next best step in the management of this
patient? Begin amoxicillin-clavulanate

● PLAYLIST 3
○ A 3-year-old boy is brought to the pediatrician's office by his mother because
of left ear pain for the past 4 days. Physical examination shows a bulging and
erythematous left tympanic membrane. The use of pneumatic pressure
otoscope shows no movement of the tympanic membrane. Which of the
following organisms is the most likely cause of this patient's symptoms?
Haemophilus influenzae
○ An 18-month-old boy is brought to his pediatrician's office by his father
because of fever and ear tugging for the past 3 days. His father reports a
temperature of 37.8°C (100.0°F) this morning and a decreased appetite. The
boy's temperature is 38.6°C (101.4°F), pulse is 110/min, respirations are
22/min, and blood pressure is 90/60 mm Hg. Otoscopic examination shows a
bulging and erythematous right tympanic membrane with decreased mobility
of the right tympanic membrane. Oral amoxicillin is initiated. Four days later,
the child returns to the office with no improvement in symptoms. Repeat
physical examination shows a bulging tympanic membrane bilaterally with a
meniscus of fluid within the middle ear. Which of the following is the most
appropriate next step in management? Amoxicillin/clavulanate
○ A 17-year-old girl comes to the pediatric office because of left ear pain that
began 2 days ago and has become progressively worse. The patient
complains that the pain is so severe that disrupts her sleep and persists even
after the use of topical antibiotic medication. Medical history is significant for
type 2 diabetes that is being poorly managed. Examination of the left ear
shows edema and erythema of the auditory canal with a greenish discharge
and manipulation of the left pinna elicits severe pain. The tympanic
membrane can be visualized and appears normal. Examination of the right
ear shows no abnormalities. Which of the following is the most likely
diagnosis? Malignant otitis externa
○ A 2-year-old boy is brought to the office by his mother because of complaints
of fever and ear pain. He began tugging on his ear and complaining of pain
three days ago. The mother reports a temperature of 37.8°C (100.0°F) this
morning, with decreased appetite. Current temperature is 38.6ºC (101.4ºF).
ENT examination shows erythema and decreased mobility of the right
tympanic membrane. Which is the most appropriate pharmacological
management? Amoxicillin
○ A 14-year-old girl presents to her pediatrician with inflammation of the left
outer ear and severe pain upon manipulation of the left pinnacle. The patient
reports that she is on the high-school swim team and that several of her
teammates have developed ear infections and pruritic, erythematous rashes.
Physical examination reveals an intact tympanic membrane on both sides,
though it is somewhat obstructed by a greenish discharge, and there is
substantial swelling and narrowing of the left ear canal with some associated
hearing loss on the left side.
Based on the patient's history and symptoms, which of the following
organisms is most likely responsible for this patient's infection?
Pseudomonas aeruginosa

● PLAYLIST 4
○ A 35-year-old woman comes to the office because her coworkers say that
she should have her hearing checked. She initially thought that it was due to
earwax buildup, but says the symptoms have been worsening for the past 6
months despite improved ear hygiene. Family history reveals that her mother
also began to suffer from progressive hearing loss around the same age.
Otoscopic examination shows a clean auditory canal, no redness, and a cone
of light on the tympanic membrane. Rinne test indicates that bone conduction
is audible longer than air conduction for both ears. Which of the following
additional findings is most likely? Loss of stapedial reflex
○ A 23-year-old woman presents to her primary care physician with the
complaint of episodic dizziness. The dizziness began three days ago and
occurs when she rose up from her bed and when she turns her head fast.
The episodes are associated with nausea and are unlike anything she has
ever experienced. She denies any recent trauma, fevers, headaches, or sore
throat. Her past medical history is insignificant and she takes no medications.
Vital signs reveal a blood pressure of 135/90 mmHg, and pulse of 75/min.
Cardiac and neurological examinations are unremarkable. Which of the
following is the most appropriate treatment in this patient? Epley maneuver
○ A 50-year-old man comes to the office for advice prior to a cross country
flight for work next month. He says that the last few times he has traveled in
an airplane, he felt dizzy and nauseous for the duration of the flight. His
medical history includes asthma and an episode of angle closure glaucoma
for which he was treated with iridotomy last year. He asks if there is anything
that might be able to prevent him from feeling so terrible on his upcoming
flight. He is also concerned about being tired when he arrive because he will
need to present at a conference the next day. Which of the following is the
most appropriate management? Diphenhydramine
○ A 50-year-old man comes to the clinic because of hearing loss and tinnitus in
his right ear which has occurred over the past few years. Physical
examination including Weber and Rinne tests shows sensorineural hearing
loss in the right ear and normal hearing on the left. Magnetic resonance
imaging (MRI) shows a well circumscribed brain tumour. Which of the
following is the most likely tumor found on the MRI? Schwannoma
○ A 45-year-old woman comes to the emergency room with a sudden onset of
dizziness and gait instability that began two days earlier. Since it began, she
has experienced constant dizziness, nausea, and vomiting, despite trying
multiple over the counter medications. The patient also complains of hearing
loss in her right ear. On further questioning, she notes that she had a runny
nose and dry cough approximately 1 week ago that resolved on its own. Vital
signs are within normal limits. Physical examination shows congested nasal
mucosa, oropharyngeal erythema without exudate, and no tenderness to
palpation of the sinuses. Which of the following is the most appropriate
intervention? Meclizine
○ A 54-year-old woman comes to the emergency department because of
vertigo for the past 20 minutes. She has a one-year history of attacks of
vertigo, left ear fullness, left ear hearing loss, and tinnitus. One month ago,
she was treated with oral steroids by another physician and her symptoms
improved. She has subsequently had three more vertigo attacks with similar
symptoms in her left ear. Physical examination shows Rinne and Weber tests
are consistent with sensorineural hearing loss in the left ear. No nystagmus is
present. Which of the following is the most likely diagnosis? Meniere
disease
○ A 35-year-old woman comes to the clinic because of moderate hearing loss
that she first noticed several months ago. She also complains of a ringing
noise in her right ear that is very disturbing. Physical examination shows
right-sided sensorineural hearing loss, an asymmetric smile, and decreased
corneal reflex in her right eye. This patient’s condition suggests an
intracranial mass located in which of the following locations? Between the
cerebellum and the lateral pons
○ A 64-year-old caucasian man comes to the clinic because of dizziness. He
describes the dizziness as a sensation of spinning, even when he is not
physically in motion. He also complains of ringing in his ear and a sensation
of pressure that comes and goes. Which of the following is the most likely
diagnosis? Meniere's disease
○ A 33-year-old man comes to the clinic because he woke up with intense
dizziness upon getting out of bed. He says his feelings of dizziness come and
go in spells, lasting only a few seconds. He cannot identify a trigger. When
asked to described the spells in more detail, he describes them as "feeling
like the room is spinning around him". He denies loss of consciousness,
feeling like his vision blacks out during the spells, or feeling weak or like he is
going to collapse during the spells. His vital signs are normal. Neurological
exam shows no abnormalities. Which of the following is the most appropriate
next step in management? Dix-Hallpike maneuver
○ A 20-year-old man comes to the clinic because of disabling attacks of ringing
in his ears, an illusion of the world spinning around his head, and intermittent
hearing loss. These attacks have lasted hours to days and he is concerned
about the ongoing impact on his ability to work and pay his rent if they
continue. Which of the following is the best long-term treatment for the most
likely condition? Sodium restriction and diuretics
○ A 65-year-old man comes to the clinic with the complaint of gait disturbance.
History reveals he has never experienced this before, and it has become
progressively worse over the past month. He has difficulty initiating
movements and frequently falls. His wife states that he has been more
forgetful recently. He has forgot to take his medication and the names of his
grandchildren. History reveals that he also has trouble controlling his urine,
having multiple episodes of incontinence. An MRI of the head was obtained.
Which of the following is the most likely finding on imaging? Dilated
ventricles without cortical atrophy
○ A 62-year-old man comes to the clinic because of tinnitus in his right ear,
which has occurred over the past few years. He also reports that wife often
complains that his hearing is not very good. Weber and Rinne tests show
sensorineural hearing loss in the right ear and normal hearing on the left.
Magnetic resonance imaging (MRI) shows that the patient has a brain tumor.
Which of the following locations is the most likely site for this patient's tumor?
Cerebellopontine angle
○ A 40-year-old woman comes to the clinic because of a ringing sound in her
ears that has been present for 1 year. She reports that she has trouble
understanding people at times and "cannot hear as well as I could before".
Recently, she has noticed that she keeps losing her balance when walking.
Her MRI shows the following lesion: Which of the following is the most likely
diagnosis? Vestibular schwannoma
○ A 40-year-old female presents to her primary care physician with the
complaint of episodic dizziness. History reveals the dizziness began three
days ago as she was getting out of bed. In addition to rising from bed, she
states that the episodes occur when she turns her head, and resolve
spontaneously after a few minutes of rest. The episodes are associated with
nausea and are unlike anything she has ever experienced. She denies any
recent trauma, fevers, headaches, or sore throat. Her past medical history is
insignificant and she takes no medications. Vital signs reveal a blood
pressure of 135/85 mmHg, and pulse of 75/min. Cardiac and neurological
examinations are unremarkable. The most likely cause of her symptoms is
which of the following? Benign paroxysmal positional vertigo
○ An 85-year-old man comes to the clinic with his wife because she reports the
patient has been having difficulty walking. History reveals trouble
remembering the names of his grandchildren and frequent falling for the past
2 months. The patient has also had several episodes of urinary incontinence.
Past medical history reveals hypertension and hemorrhagic stroke. Physical
examination shows 2+ deep tendon reflexes in the upper and lower
extremities, normal sensation and +5/5 muscle strength throughout. The
patient's stance is unsteady and his gait is shuffling. Which of the following is
the most likely diagnosis? Normal pressure hydrocephalus
○ A 22-year-old woman comes to the emergency department because of
complaints of vertigo, nausea, and tinnitus that has persisted for two days.
Medical history includes a recent infection with Herpes Simplex type 1
(HSV-1) virus that was treated with acyclovir topical cream. Physical
examination reveals mild hearing loss and a positive Dix-Hallpike maneuver.
Based on the patient's history and symptoms, which of the following is the
most likely diagnosis? Labyrinthitis
○ A 35-year-old woman comes to the clinic because of gradual hearing loss
over the past six months. She denies any history of ear infection. Family
history shows that her mother also has a history of hearing loss. Otoscopic
examination is unremarkable. The Rinne test indicates that bone conduction
is audible longer than air conduction in both ears. Which of the following is
most likely to be present? Loss of stapedius reflex on audiometry
○ A 71-year-old man comes to the clinic because of dizziness. The dizziness
started three days ago. Upon further questioning, he describes his dizziness
as a whirling type vertigo, and while it hasn't been getting worse, it has been
constantly disturbing him since its onset. He doesn't complain of nausea or
vomiting, and there is no hearing loss. He has had three episodes of falls to
his right side since yesterday. When you examine him, you notice a vertical
gaze-evoked nystagmus that changes its direction when he is looking to the
right. He also has noticed that his voice has gotten a bit hoarse. Which of the
following is the most likely diagnosis? Vertebrobasilar insufficiency
○ A 65-year-old male presents to the office with the complaint of gait
disturbance. History reveals he has never experienced this before, and it has
become progressively worse over the past month. He has difficulty initiating
movements and frequently falls. His wife states that he has been more
forgetful recently. He has forgotten to take his medication and cannot recall
the names of his grandchildren. History reveals that he also has trouble
controlling his urine, having multiple episodes of incontinence a week. MRI of
the brain shows expansion of the third ventricle. Which of the following is the
most appropriate next step in management? Lumbar puncture
○ A 35-year-old woman comes to the emergency department because of
complaints of vertigo, nausea, and vomiting lasting for the past day. She has
a history of constant ringing in her ears that she attributes to a recent flu-like
illness. Physical examination shows the patient has mild hearing loss. Based
on the patient's symptoms and history, which of the following is the most
appropriate next step in the management of this patient? Prescribe
meclizine
○ A 65-year-old woman comes to the office because of problems with her
hearing and 'ear fullness.' She reports that she has noticed a low pitch noise
that others don't hear that sounds like being by the ocean, and a decrease in
hearing in her left ear. She has also been having intermittent episodes of
dizziness when she feels like the room is spinning. Which of the following is
the most likely diagnosis for this patient? Meniere disease

● PLAYLIST 5
○ A 27-year-old woman comes to the office because of increased sensitivity
and weakness on the left side of her face during the past 2 days. She has
difficulty smiling and reports pain behind her left ear, over the mastoid
process. She has a 15-year history of type 1 diabetes mellitus well controlled
with insulin. Physical examination shows inability to close the left eye. An
MRI shows no signs of tumor or neurological injury. Which of the following is
the most appropriate next step in management? Glucocorticoid therapy
○ A 60-year-old man comes to the office because of increased sensitivity to
loud sounds and change in his sense of taste. He also reports difficulty
blinking and pain in his left ear. Physical examination shows small blisters on
the external ear and facial asymmetry, with drooping of the left lower eyelid
and the corner of the mouth. Which of the following is the most likely
diagnosis? Ramsay-Hunt syndrome
○ A 29-year-old primigravid woman at 34 weeks' gestation comes to the office
because of loss of taste and weakness on the left side of her face during the
past day. She reports difficulty smiling and closing her left eye. MRI shows no
sign of a tumor or neurological injury. Which of the following nerves is most
likely affected in this patient? Cranial nerve VII
○ A 35-year-old woman comes to the office because of frequent biting of her
left cheek while chewing during the past 3 weeks. She also reports
increasing asymmetry of her smile. She has no personal history of serious
medical illness and takes no medications. A tumor in which of the following
glands best explains these findings? Parotid gland
○ A 46-year-old man comes to the emergency department because of the
sudden onset of inability to move the right side of his face. Physical
examination shows drooping of the corner of the mouth. Motor examination
shows difficulty smiling and puffing out the cheeks; no abnormalities are
noted in raising the eyebrows or wrinkling the forehead. Which of the
following is the most likely cause of these findings? Left central facial palsy
○ A 40-year-old man comes to the emergency department because of bilateral
facial weakness during the past 3 days. He also reports a change in taste
sensation. One month ago, he had flu-like symptoms after returning from a
climbing trip in New York. Physical examination shows flattening of the skin
of his forehead. Motor examination shows difficulty with facial movements
bilaterally, including smiling and raising his eyebrows. The remainder of the
examination shows no abnormalities. Which of the following is the most likely
diagnosis? Lyme disease
○ A 45-year-old man comes to the emergency department because of the
sudden onset of left facial paralysis. He has a 10-year history of hypertension
and hyperlipidemia treated with enalapril and atorvastatin. Physical
examination shows drooping of the left side of the face, lack of wrinkling of
the left forehead when asked to raise both eyebrows, and absence of left
nasolabial fold. Which of the following is the most likely diagnosis? Bell's
palsy
○ A 62-year-old man comes to the emergency department because of the rapid
onset of right-sided facial paralysis. He states that his face “doesn’t look
right” but denies any muscle weakness in the arms or legs. He has a 30-year
history of hypertension and hyperlipidemia. His pulse is 80/min, respirations
are 24/min, and blood pressure is 130/85 mm Hg. Physical examination
shows paralysis of the right side of his face. Motor examination shows equal
strength of his arms and legs bilaterally. Which of the following maneuvers is
most appropriate to ask the patient to perform as the next step in diagnosis?
Wrinkling his forehead
○ A 30-year-old man comes to the office because of paralysis of the muscles
on the left side of his face. He is diagnosed with Bell's palsy. On physical
examination, the physician asks him to close his eyes. Which of the following
is the most likely finding in the affected left eye? Left eye looks up and out

● PLAYLIST 6
○ A 6-week-old infant comes the office for a routine checkup. His mother says
that he has been breathing loudly over the last week. This is particularly
heard on inspiration and is most noticeable when he is lying on his back,
crying, or during feeding, though the infant does not have difficulty
swallowing. She denies nasal congestion, cough or fever, and says he has
been active as usual. She denies abnormal changes in his skin color. His
temperature is 37.2°C (98.96°F), pulse is 130/min, and respirations are
50/min. Physical examination shows a low-pitched inspiratory stridor
appreciated as he lies on the exam table, but he does not have increased
work of breathing. The lungs are clear to auscultation. Which of the following
is the most likely diagnosis? Laryngomalacia

● PLAYLIST 7
○ A 64-year-old man presents to his primary care physician's office because of
a thyroid mass. The patient's past medical history is significant for
alcoholism. A fine needle aspiration of the mass is performed. Cytology
reports later showed well-differentiated papillae with orphan Annie eye nuclei
and scattered psammoma bodies. The patient then undergoes a total
thyroidectomy and post-surgical pathology of the gland reveals the following:
papillary carcinoma
○ A 45-year-old woman is brought to her primary care physician's office
because of fevers and weight loss for the past 3 months. Physical
examination shows a hard and nontender thyroid nodule with cervical
lymphadenopathy. A complete metabolic panel and complete blood count is
ordered and the results show no abnormalities. A positron emission
tomography shows pulmonary metastasis. Fine-needle aspiration of the
thyroid mass shows concentric laminar calcified collections. Which of the
following is the most likely diagnosis? Papillary adenocarcinoma of thyroid

● PLAYLIST 8
○ NO HAY PREGUNTAS
__________________________________________________________________________

DERMATOLOGÍA
● PLAYLIST 1
○ A 16-year-old boy presents to the dermatology clinic with acne that is uncontrolled
by over-the-counter facial cleansers and lotions. He has 35 lesions on his chest,
back and face, some of which are cystic and painful. He has been lifting weights
heavily for the past six months and trying “to put on muscle mass.” After his
workouts, he drinks milk with whey protein powder for recovery. Which of the
following may be elevated in this patient’s blood? Insulin-like growth factor 1
○ A 24-year-old woman comes into clinic because of her persistent acne, which she
says is not significantly alleviated by over-the-counter acne treatments. She inquires
about Isotretinoin, which her friend has found helpful. While you would not prescribe
any medications to her directly, what should she be counseled about? Use of
contraception to prevent pregnancy
○ A 17-year-old boy comes to the dermatology clinic with moderate acne that has
been persistent over the past five months. It appears inflammatory, with a moderate
amount of mixed papular and pustular lesions and very few comedonal lesions.
Which of the following would be the most appropriate choice for initial treatment?
Topical retinoid, topical benzoyl peroxide, and a macrolide
○ A 15-year-old boy comes to the office with a facial rash that appeared three months
ago, shortly after the beginning of his football season. The open comedones have
progressively become more numerous. He washes his face multiple times a day and
uses benzoyl peroxide twice a day, but has seen no change. Physical examination
reveals mild comedones in the mandibular region with some extension to the upper
shoulder area. Which of the following is the most likely organism responsible for
these symptoms? Cutibacterium acnes (formerly Propionibacterium acnes)
○ A 15-year-old girl comes to the office for a follow-up appointment about her
treatment for acne vulgaris. She is currently using benzoyl peroxide, topical
retinoids, and erythromycin, which do not seem to be successful. She is started on
oral retinoids. Which of the following is a common side effect of the new treatment?
Teratogenicity
○ Which one of the following structures is considered a skin appendage?
Pilosebaceous unit
○ A 22-year-old man comes to clinic because of a rash on his forearms that he noticed
yesterday. The skin is sore but not itchy. His past medical history is significant for
cystic acne which he has been treating with isotretinoin for the past two months. He
does not take any other medications and has no known drug allergies. He just
returned from a hiking vacation with his family. Physical examination shows both
forearms with well-demarcated, erythematous skin with slight vesiculation, and mild
tenderness to touch. Which of the following was the most likely inciting agent of this
reaction? UVA rays
○ A 62-year-old man comes to the emergency room because he suspects that he has
been poisoned. He was suddenly stricken with severe pancreatitis followed by facial
disfigurement. Levels of 2,3,7,8 tetrachlorodibenzodioxin were reportedly elevated
50,000 times over that of the general population. The lesions on the man’s face
appear as dark pustules. Which of the following best describes the mechanism
behind the development of these lesions? Formation of epidermal cysts

● PLAYLIST 2
○ A 23-year-old Caucasian female comes to the office because of a new, pruritic skin
rash on her back for 2 days. The patient says that the rash appeared without
warning and that the area has not enlarged since she first noticed it. She has had
similar skin lesions in the past on her elbows and knees. She reports no recent
exposure to chemical irritants. The last time she spent extended time outdoors was
over two weeks ago when she went to the pool. Her only medication is an oral
contraceptive pill that she has taken for the past 5 years. Examination shows her
elbows and knees are clear of skin lesions. There is a symmetric, red plaque with
well-circumscribed borders on her back as shown below. Which of the following is
the most likely cause of this patient’s rash? Sunburn
○ A 38-year-old man comes to the office because of ankle pain and skin changes that
have ongoing for the past few months. He reports difficulty walking. He also states
that his left index finger and right wrist felt as if they were sprained. One morning,
the pain in his joints was so intense that he could hardly get out of bed. His primary
care physician ordered ANA, ACPA, and RF titers, all of which came back negative.
Physical examination shows swelling and erythema of several distal interphalangeal
joints on both hands, and also shows scattered silvery plaques on the extensor
surfaces of the skin. Which of the following is the most likely diagnosis? Psoriatic
Arthritis
○ A 46-year-old man comes to the emergency department because of a sore red leg
for the past 6 hours. He says that he woke up from his sleep with pain and he rates
the pain as a 9 on a 10-point scale. The patient says that he scraped the affected
leg 3 days ago while riding his bicycle. Medical history is significant for type II
diabetes. His temperature is 37.5°C (99°F), pulse is 98/min, respirations are 14/min,
and blood pressure is 120/77 mm Hg. Physical examination shows erythema and
swelling affecting the right thigh, right buttock, and lateral aspect of the right leg, and
crepitus on palpation. Which of the following is most likely to confirm the diagnosis?
Surgical exploration and biopsy
○ A 49-year-old woman comes to the office because of new skin lesions on her
elbows for a week. She describes well-marginated, erythematous plaques with
silvery-white scales on the extensor surface of her elbows. She states that she has
had a rash like this before, that resolved on its own. Her father also has a similar
condition. Examination shows further lesions on her knees and heels, as shown in
the exhibit. Which of the following is a nail abnormality most commonly associated
with this patient's condition? Onycholysis
○ A 45-year-old woman comes to the office because of a rash on her buttocks during
the past 3 days. She states that the rash first developed during a cruise, where she
often exercised in the gym and swimming pool. She reports that the lesions are
pustular and pruritic. She has no personal history of serious illness and takes no
medications. Physical examination shows multiple erythematous pustular lesions on
the buttocks. A photograph of a representative area is shown. Which of the following
is the most appropriate next step in management? Reassurance
○ A 19-year-old man comes to the clinic because of a rash that started as a single
lesion a few days ago. It has since spread throughout his chest and back. The
patient had an upper respiratory infection nine days ago. Physical examination
shows salmon pink patches, as seen below. Both VDRL and FTA-AB tests are
negative. Which of the following is the most likely diagnosis? Pityriasis rosea
○ A 30-year-old woman comes to the office because of a painful, pruritic rash on her
legs, elbows, and scalp for 2 weeks. She claims that she had a similar, but mild,
rash eight months prior that resolved on its own. A review of systems also reveals
fatigue, muscle aches and stiffness. She also mentions that she has recovered from
an episode of mononucleosis three weeks prior. Examination shows red plaques
with silvery scales. Which of the following facts about this patient's past medical
history would most likely further support the diagnosis? Family history of skin
disease
○ A 54-year-old postmenopausal woman comes to the gynecology office because of
vaginal burning and copious vaginal discharge. She says it started six weeks ago
and has gradually become more uncomfortable. She is sexually active with her
husband, and experiences pain during intercourse. On speculum examination,
there is a red, ulcerated lesion in her vagina surrounded by an area that appears
white and lacy. Which of the following is the most likely diagnosis? Lichen planus
○ A 30-year-old male comes to the office because of a rash for a month. He states that
it has been gradually worsening and has been very painful with a burning sensation.
He does not recall a recent change in diet or soaps. He has also not had any new
exposures, recent travel or illnesses. While he has never had this rash before, he
does recall that a couple years ago he had a rough, itchy spot on his scalp which
gradually resolved. Examination shows he has several large plaques of scaly skin
on a red base located on his elbows, knees, and trunk. Which of the following is
most likely true about this type of lesion? It bleeds when scales are scraped off
○ A 22-year-old woman comes to a dermatology clinic because of a painful, blistered
right thumb. She injured the thumb when her friend accidentally stepped on it 6
months ago. In the months following the injury, the skin on the tip of her finger
became red, tender and yellow pustules formed. The skin on the tip of the thumb
appears eroded, pustules are present, and the nail has become malformed. This
condition is best classified as which of the following? Autoimmune condition
○ A 50-year-old woman comes to the office because of a spreading, itchy, red, scaly
rash for 5 days. She states that the rash began on her scalp and face and has since
spread to her chest and abdomen. Examination shows her skin is covered almost
entirely in red plaques with dry follicular plugs. Additionally, the skin on her palms
and soles is red and has become tender and she says that she has been losing
some of the hair on her head. Which of the following is most likely true about this
condition? It may be a sign of underlying malignancy
○ A 63-year-old man comes to the dermatology clinic because of rapidly progressing
skin redness which has covered most of his body. He has coronary artery disease
with a 3-vessel CABG, hypertension, and psoriasis. His medications include a
beta-blocker, aspirin, ACE-inhibitor, statin, and topical clobetasol. He has not had
any recent changes to his medications. His temperature is 38.8°C (101.8°F), pulse
is 115/min, respirations are 16/min, and blood pressure is 95/68 mm Hg. His O2
saturation is 97% on room air. He is in no acute distress, but he is shivering. On
examination, his oropharynx is clear without mucous membrane involvement. His
skin exam shows diffuse erythema with an overlying scale covering over 90% of his
body surface area. Which of the following is the most likely diagnosis?
Erythrodermic psoriasis
○ A 72-year-old man comes to the dermatology clinic because of a wart growing on
his left hand. The lesion has been present for about a year and has become more
physically and cosmetically bothersome over time. Physical examination shows a
raised, 1-cm lesion. There are several conical, hyperkeratotic masses atop a pink,
rounded base. Which of the following is the most appropriate morphologic descriptor
for this lesion? Horn
○ A 20-year-old man presents to the office because of joint pain, particularly in his
fingers. Physical examination shows significant swelling of his fingers and pitting in
his fingernails. He is most tender to palpation across his DIP joints. Three months
ago, he visited the office because of a rash, shown below. Besides some itching, he
denied any other symptoms. Topical treatments helped with the itching. Which of the
following haplotypes is most commonly associated with this condition? HLA-B27
○ A 67-year-old woman comes to the clinic because of pustular lesions on her neck.
She has a history of monoclonal gammopathy. She reports no associated systemic
symptoms. Physical examination shows the patient is afebrile. The lesions are
several millimeters in size, contain purulent fluid, and the perilesional skin is
erythematous. Skin bacterial and fungal cultures are negative. Histologic findings
include subcorneal pustules containing neutrophils and sparse eosinophils, with an
absence of significant spongiosis or acantholysis in the dermis. A perivascular
infiltrate of neutrophils is present.Which of the following is the treatment of choice for
this patient’s condition? Dapsone
○ A 60-year-old woman, with a history of gastrointestinal stromal tumor, comes to the
clinic three months after administration of imatinib because of violaceous, polygonal
papules and plaques on the upper chest and arms. The lesions are scaly and
confluent. Which of the following is most likely a characteristic of this rash?
Photodistribution
○ A 17-year-old male come to the office because of red, scaly rashes along both of his
forearms and elbows for a week. He says that the lesions are itchy, and bleed
slightly when scratched. He states that they seem to be getting larger and is worried
that he could have contracted a fungal infection at wrestling camp a few weeks ago.
Medical history is noncontributory. Examination of the skin is shown below. The
scales were prepared with KOH and were found to be negative for mould hyphae.
Which of the following is the most appropriate treatment plan? Topical
corticosteroids, topical emollients, sunlight or UV therapy
○ A 40-year-old man comes to the clinic because of worsening skin on his hands for
the past year. He says recurrent outbreaks of pustular lesions on his palms and
soles make it difficult for him to perform at work. He has previously been diagnosed
with psoriasis. Examination shows dry, fissured skin with lichenification, and a mix of
pustules and macules (shown below). Removal of which of the following exposures
is most likely to benefit his condition? Cigarettes
○ A 55-year-old man comes to the office because of pruritus for a week. He was
diagnosed with psoriasis three years ago, and his symptoms are not improving
despite good adherence to conventional therapy. Examination shows the presence
of erythematous scaly plaques on the extensor surfaces of the knee and elbows.
There is no evidence of flexural involvement. Which of the following is the most
appropriate treatment in this patient? Topical retinoid therapy
○ A 40-year-old female comes to the office because of a pruritic, bright red rash on her
elbow for a week. Examination shows silvery scales on an erythematous plaque
located on the extensor surface of the right elbow. Removal of the scale reveals
pinpoint bleeding. There is also separation of her nails from the nail bed as shown
below. Which of the following is the next best step in management of this patient’s
condition? Topical corticosteroids
○ A 52-year-old perimenopausal woman comes to the gynecology office because of a
rash on her vulva. She says that it is only slightly itchy and also describes that it bled
a little when she scratched it 2 days ago. Examination shows a red, raised, round
patch that is covered with a silvery scale. Which of the following is the most likely
diagnosis? Psoriasis

● PLAYLIST 3
○ A 62-year-old woman comes to the outpatient clinic because of generalized peeling
of her skin. She notes initially she noticed a red patch two days ago which started to
appear in different locations and merged over time. The patient says that her skin
was red and very itchy before it began to slough. She has hypertension, type II
diabetes mellitus, and was diagnosed with osteoarthritis a week ago for which she
has been regularly taking naproxen. Her appetite has been good and she has not
lost weight recently. Her temperature is 37oC (98.6oF), heart rate is 68/min,
respirations are 15/min, and blood pressure is 138/85 mm Hg. Physical examination
shows erythematous raised lesions in over 90% of her body and several linear
crusted erosions on her arms. NSAID use
○ A 16-year-old boy comes to the clinic because of a "pink and patchy" rash on his
face and upper body. The rash began five days prior on his upper arms and trunk,
and has since spread to his forearms and face. The rash is faint and does not itch.
Additional signs and symptoms include exudative pharyngitis and a fever of five
days duration. His only medication is amoxicillin, which he started taking yesterday.
He has no known allergies to any medications, including amoxicillin, which he has
used previously. On physical examination, there is a prominent posterior cervical
lymphadenopathy in addition to the rash. Which of the following is the most likely
etiology of this patient's rash? Viral exanthem
○ A four-year-old girl is brought to the clinic by her mother. A few days ago, the girl
began complaining of a sore throat. She was febrile during this time and was not
eating very well. Today, she has a rash on her hands. Physical examination shows a
restless, well-nourished patients. She seems very uncomfortable, especially upon
examination of the oral cavity. There are small grey blisters on the hands and feet,
as shown below. Which of the following is the most likely causative agent?
Nonenveloped, single-stranded, linear, positive-sense RNA viruses
○ An 8-year-old girl is brought to the urgent care clinic by her father because of a rash
that started 12 hours before and continues to spread. Prior to developing the rash,
the patient had a fever and sore throat for 3 days. Earlier that morning the parents
noticed a rash starting on her neck that progressed downward to her arms and
torso. In addition, the patient has not been eating and states that she is too tired to
play with her friends. Two children at her younger brother's daycare also have
similar symptoms. Physical examination of the patient shows flushed cheeks and a
course diffuse erythematous rash on the patient's neck, chest, and axilla. HEENT
examination shows oropharyngeal injection with tonsillar exudate and bilateral
cervical lymphadenopathy. Based on the patient's presentation, which of the
following is a known complication of the most likely cause of her symptoms? Acute
glomerulonephritis
○ A 9-month-old male is brought in by his mother for evaluation of fever and rash. His
mother noted a fever of 104˚F two days ago. He appeared well and was eating and
playing normally, so his mother was not alarmed. After the fever resolved, he
developed a red rash on his trunk that progressed rapidly over the past 24 hours.
What is the causative agent? Human Herpesvirus 6
○ An 8-year-old boy comes to the emergency department with his mother because of
a 2-day history of rash. His mother says that he was in his usual state of health until
two days ago when he developed a red, raised, extremely itchy rash on his
wrist.More recently, he has also developed similar rashes on his chest, face, and
upper arms. The boy and his family had just returned from camping 2 days earlier.
He has no history of asthma or of allergic rhinitis. One month ago, he was
prescribed a course of amoxicilin for a pharyngeal Streptococcus infection. Which of
the following is the most likely explanation for this patient's physical findings?
Contact Dermatitis
○ A 42-year-old man comes to the emergency department because of a 3-day history
of fatigue, chills, and an itchy rash on his chest and abdomen. He was recently
diagnosed with chronic lymphocytic leukemia and is receiving chemotherapy. His
temperature is 39°C (102.2°F), pulse is 97/min, and blood pressure is 120/80 mm
Hg. A photograph of his abdomen is shown. Which of the following is the most
appropriate next step in management? Intravenous acyclovir
○ A 33-year-old woman comes to the office because of a lesion on her forearm that
appeared yesterday. She had spent the day gardening outside, and later noticed an
itchy red rash with a few small vesicles, as shown here. She has never had such a
lesion before, but says that she recently completed treatment for Lyme disease after
she noticed a circular red rash on her shin. Which of the following is the most likely
cause of this patient's latest rash? Photodermatitis
○ An 8-month-old infant is brought to the family doctor by her mother who has noticed
a red rash developing on her daughter’s face. According to the mother, this rash
appeared a few weeks ago and is worsening. She thinks it may have appeared after
her daughter opened her aunt’s makeup box and smeared mascara over her face.
She has also noticed her daughter scratching at the red areas intermittently. Vital
signs are all within normal range. Examination shows multiple small papules,
pustules, and vesicles around the daughter’s mouth, and scattered lesions near the
nostrils. Which of following is the most appropriate initial treatment? Topical
erythromycin
○ A 3-year-old girl is brought to the office because of a rash. She has been attending
day care until four days ago when she developed fevers and rhinorrhea. Two days
ago, a rash appeared on her face. There are no sick contacts at home but the
patient's mother admits that she has had some aches in her knees and wrists
recently. Physical examination of the daughter shows a macular facial rash which
covers most of the cheeks. Which of the following is a characteristic of the most
likely causative organism? Single-stranded DNA virus
○ A 5-year-old boy comes to the office because of fever, fatigue, and a rash. About a
week ago, the patient started having fevers and "red cheeks." Today, he has a lacy
rash on his arms, legs, and trunk. His past medical history is significant for sickle cell
disease for which he takes daily penicillin and folic acid. What is the most likely
potential complication in this patient? Aplastic crisis
○ A 1-month-old male infant comes to the office because of a thick, scaly plaque on
his scalp. His parents deny any disturbances in feeding or sleep. Physical
examination shows yellow and white, oily scales that rest on an inflamed,
erythematous base. Which of the following is the most appropriate next step in
management? Topical anti-fungal cream
○ A 56-year-old woman is brought to the emergency department 1 hour after the onset
of dizziness and light-headedness. She has noted fatigue, chills, and itching on her
abdomen during the past 6 hours. Her temperature is 39°C (102.2°F), pulse is
97/min, respirations are 16/min, and blood pressure is 120/85 mm Hg. A photograph
of the abdomen is shown. Microscopic examination of the lesions is most likely to
show which of the following? Multinucleated giant cells
○ A 6-month-old infant is brought to the clinic by his mother because he refuses to eat
and has had "bumps" on his palms and soles since yesterday. He is usually a very
happy child that likes to explore his surroundings and place things in his mouth. The
child spent the past week at a daycare center and the mother is unsure whether
other children have developed similar symptoms. Physical examination of the child's
mouth shows erythematous macules on an erythematous base on the mobile oral
mucosa. The pale vesicles on his hands (shown below) and feet are surrounded by
erythema and are tender. Blood tests reveal an elevated C-reactive protein, elevated
leukocyte count, and atypical lymphocytes. Which of the following groups includes
the most likely causative agent(s)? Picornoviridae
○ A 25-year-old female comes to the emergency room because of a bullae on her foot.
Physical examination shows an erythematous, linear, and pruritic rash with vesicles
on her right feet. The only recent change in her routine is a new route for walking
her dog through a local nature preserve. Which of the following is appropriate
concerning her condition? Washing directly after the walk may have prevented
her symptoms
○ A 3-year-old boy comes to the emergency department because of a rash. Five days
ago, he had a fever, runny nose, and diarrhea before developing a rash. His other
symptoms have resolved except the rash. The rash is not pruritic and does not
seem to be bothering the patient. He attends daycare three times per week where
he spends most of his time outside and has a pet cat and turtle. Physical
examination shows a lacy, macular rash located on his trunk. Which of the following
placed the patient at highest risk for this presentation? Day care attendance
○ A 21-year-old woman comes to the dermatologist because of “red bumps” around
her mouth and nose. She says these lesions appear despite washing twice daily
with salicylic acid wash and applying a benzoyl peroxide ointment, as
recommended. Her only prescription medication is an oral contraceptive. The small
lesions are follicular, erythematous, and some are confluent. Physical examination
shows sparing of the vermillion border of the lips. She says the small bumps
occasionally appear around her eyes. She has a history of mild acne vulgaris. Which
of the following drugs is the most appropriate in treating this patient's condition?
Topical metronidazole
○ A 14-year old girl is brought to the office due to several days of pain in her joints,
accompanied by a mild fever. She has a sore throat but no cough. She also
complains that she has had some pain with eye movements for the same amount of
time, but denies watering, itchiness, or headaches and her vision has remained
intact. She is most upset that her facial skin seems to be suddenly breaking out in
bumps. Physical examination shows a mild maculopapular rash on her face which
has started to spread down to her neck and trunk. Pupils are equal and reactive,
and extraocular movements are intact, though there is some pain associated. She
has mild pharyngeal erythema and her tonsils do not appear inflamed. There is
appreciable postauricular and occipital adenopathy. Which of the following is the
most likely diagnosis? Rubella infection
○ A 21-year-old woman comes to the emergency department because of bilateral eye
pain, tearing,and photophobia. These symptoms began about 2 hours ago with
sudden onset. The house the patient lives in recently installed a sun-tanning station.
Physical examination shows normal visual acuity, intact extraocular eye movements,
and pupils are equal, round, and reactive to light. Intra-ocular pressure is within
normal limits. Slit-lamp examination is normal, but fluorescein examination under
cobalt blue light illuminates small dots throughout the cornea. Which of the following
is the most likely diagnosis? Photokeratitis
○ A 5-year-old boy comes to the pediatric clinic with several small bumps on his arm
that have been present for the past few days. His mother says that one of his friends
at school had similar lesions about 2 months ago and that a few of his other
classmates had complained of them recently as well. He does not seem particularly
worried about the bumps and says that they're only a little itchy sometimes, but don't
hurt. Physical examination shows several bowl-shaped lesions on his right wrist and
forearm, each with a central depression. A biopsy of one of the lesions is performed
and histology, shown below, shows characteristic inclusions with keratin in the
central depression. Which of the following correctly characterizes the causative
virus? Double-stranded linear DNA with a complex coat
○ A 7-year-old boy is brought to the emergency department by his mother because he
has a rash on his arms and legs. In addition the patient complains of a headache
and nausea. The mother notes that the rash has progressively gotten worse since
her son returned home from Boy Scout camp in North Carolina about 2 days ago.
Physical examination of the patient shows a tired, febrile young male with a
maculopapular rash over all four extremities involving the palms and soles. The
remainder of the examination is unremarkable. Which of the following is the best
step in the medical management of this patient's condition? Discharge on oral
doxycycline, with close follow-up
○ A 12-month-old girl comes to the clinic one week after her 12-month check-up
because of a rash. The morbilliform rash is spread across the patient's trunk, and is
not itchy or painful. The patient's exam is otherwise unremarkable. Which of the
following is the most likely cause of this presentation? Measles vaccine
○ A 9-year-old Caucasian boy with history of acute lymphoblastic leukemia comes to
the Emergency Department because of a painful rash. His temperature is 36.7°C
(98.1°F), pulse is 105/min, respirations are 22/min, and blood pressure is 108/70
mmHg. Physical examination shows a painful, blistering rash that spans most of his
left arm. Which of the following is the most likely diagnosis? Herpes zoster
○ A 22-year-old woman comes to the clinic because of blisters on her hands. She
recently got a summer job designing costumes for a local theatre production group.
She says that the lesions first appeared on the lateral edges of her fingers and then
spread to her palms. The lesions are very itchy and somewhat tender. Physical
examination shows tense vesicles on her fingers and the palms of her hands without
any associated erythema. Which of the following is true about this condition? It can
be triggered by exposure to nickel
○ A 3-year-old girl comes to the office because of a facial rash which has persisted
over the past 5 days and is associated with a fever. The fever has been measured at
home as 38.6°C (101.5°F). Social history is significant for frequent daycare and
living at home with her father and pregnant mother. Physical examination shows an
uncomfortable girl with an erythematous rash covering her cheeks, and sparing the
rest of the face. This patient's infection places the mother's fetus at highest risk for
which of the following conditions? Hydrops fetalis
○ A 5-year-old boy comes to the office for itching and a rash for the past few days. His
symptoms appear to be worse at night. The patient's mother is currently pregnant
with her second child. Physical examination shows small burrows in the skin on the
patient's face and palms, as well as a small amount of diffusely scattered papules
and vesicles. Which of the following interventions is the most appropriate
management? Treat the child and mother with permethrin cream
○ A 38-year-old woman comes to the gynecology clinic because of vaginal itching
which started 3 weeks ago. She reports that the itching is intense and has gotten
worse. She denies being sexually active. She does state that she started using a
new brand of laundry detergent a month ago. Physical examination shows diffuse
erythema of the labia majora and minora with leathery patches. Which of the
following is the most likely diagnosis? Lichen simplex chronicus
○ A 15-month-old boy comes to clinic with a rash that appeared on his trunk yesterday
morning. Otherwise, the child appears well to the mother, with no changes in
appetite, energy level, mood or temperature. While examining the rash, you note it is
erythematous, macular, blanching and widely distributed across the child's trunk. It
has not spread to his extremities. The patient had his 15 month well check up 10
days ago. Which of the following pieces of advice can you give to this mother? This
reaction is a benign side-effect of the MMR vaccine your son received at his
regular check-up and is not indicative of illness or contagiousness.
○ A 32-year-old man comes to the office because of itching, redness, and scaling in
his scalp and eyebrows. He has tried several over-the-counter dandruff shampoos
with only temporary relief. Physical examination shows greasy scaling on the scalp
and erythema with yellowish scales in the nasolabial folds. Which of the following is
the most likely diagnosis? Seborrheic Dermatitis
○ A 3-year-old boy comes to the office because of a generalized erythematous rash
for the past day. According to his mother, multiple children at his daycare have been
complaining of similar symptoms. The patient's 67-year old grandfather lives with the
family, and the mother wants to know if the grandfather is at risk. Though the
grandfather had chicken pox as a child, she has heard that elderly can develop
chicken pox again. He has no significant health problems, and only takes
Amlodipine daily for hypertension. He has not received any vaccines within the past
ten years. Which of the following is the best course of management for the
grandfather? Administer the zoster vaccine
○ An 8-month-old girl adopted from Ethiopia is comes to the office because of multiple
clusters of papules and vesicles on her palms and soles. The patient is fussy and
appears uncomfortable. A cutaneous viral swab of the lesions shows no
abnormalities. A complete blood count shows eosinophilia. Infection with which of
the following organisms has been linked to the onset of this condition? Scabies
○ A 10-year-old boy presents to the emergency department with a rash. The patient's
mother reports the rash began on the head and spread to the chest. Further history
reveals one week earlier the patient stayed home from school after developing a
cough, runny nose, and high fever. The patient has no known drug allergies, and an
unknown immunization status. Vital signs reveal a temperature of 103.2ºF. Physical
examination reveals a maculopapular rash on the head and trunk of the patient.
Which is the most likely diagnosis in this patient? Measles
○ A 25-year-old female comes to the office because of an itchy skin rash for 3 weeks.
She states that the lesions mostly affect her arms, and have never been on her
scalp. Her past medical history includes atopic dermatitis. Examination shows that
the lesions are discrete, round/oval, erythematous plaques affecting both the legs
and arms. There is noticeable xerosis of the affected skin. A KOH preparation of
lesion scrapings is negative. Which of the following is the most appropriate initial
treatment? Topical steroid ointment
○ A 7-year-old boy is brought to the emergency department with fever, headache and
generalized muscle pains, particularly in his calves, for the past 3 days. He also
complains of mild abdominal pain and nausea. His mother cannot remember
anything unusual in the past couple of weeks, but when probed she recalls removing
an attached tick from his leg last week. On examination he appears toxic and there
is a rosy, blanching maculopapular rash on both his upper and lower extremities
bilaterally, including the palms and soles. His mother states that she first noticed the
rash yesterday over his wrists and ankles, and it has spread. Based on the patient's
symptoms, which of the following is the best approach to managing his condition?
Start regimen of doxycycline

● PLAYLIST 4
○ A 16-year-old girl is hospitalized after a dirt-bike accident. She has a spiral fracture
of the distal tibia and several broken ribs. She is given an IV medication for pain
control. Soon after, she experiences intense pruritus and breaks out in a rash.
Which answer choice best describes the difference in urticaria precipitated by
non-opiate pain medications or penicillin versus urticarial lesions triggered by
opiates?.Opiate-triggered reactions do NOT involve IgE
○ A 40-year-old female comes to the office because of a pruritic, bright red rash on her
elbow for a week. Examination shows silvery scales on an erythematous plaque
located on the extensor surface of the right elbow. Removal of the scale reveals
pinpoint bleeding. There is also separation of her nails from the nail bed as shown
below. Which of the following is the next best step in management of this patient’s
condition?Topical corticosteroids
○ A 75-year-old male comes to his dermatologist with a complaint of itching and
redness of his arms. He states that he has been scratching his inner forearms for 2
weeks and they have become red and inflamed. Physical examination reveals that
the flexor surfaces of his forearms are dry, scaly, and erythematous, with scattered
excoriated patches. Also, dry and scaly patches are present on his cheeks with
prominent bilateral infra-orbital skin folds. Which of the following is the most
appropriate management for this patient's condition? A topical corticosteroid
○ A 73-year-old woman comes to the emergency because of a dangerous cutaneous
drug reaction. Physical examination shows confluent erythema and epidermal
sloughing with application of lateral pressure. 40% of her body surface area is
involved, including her eyes and oral mucosa. She is transferred to the burn unit in
critical condition. Which of the following conditions is the most life-threatening in this
patient? Sepsis
○ A 5-month-old female is brought in by her mother with confluent erythema, papules,
microvesicles, scaling, and crusting on the face. Physical examination shows similar
involvement to a lesser degree on both arms of the infant. The mother denies a
recent history of illness for herself or her infant. Which of the following is associated
with the most likely diagnosis? Family history of asthma
○ A 22-year-old woman comes to the clinic because of sensorineural deafness,
recurrent urticaria, and a periodic fever. Physical examination shows a fever of 38.1°
C, chills, and arthritis. It is discussed that possible treatment options are anakinra
(IL-1 receptor antagonist), rilonacept (dimeric fusion protein,) or canakinumab
(monoclonal antibody against IL-1 beta). The patient has a mutated CIAS1 gene
(cryopyrin protein production). Which of the following is the most probable diagnosis
for this patient? Muckle-Wells syndrome
○ A 6-month-old infant comes to the dermatologist because of red and itchy rash all
over his cheeks and chin. His mother complains that it gets worse when he
scratches his face. Physical exam reveals multiple erythematous papules with
underlying erythema and excoriations with perioral and paranasal sparing. Blood
work shows increased IgE and eosinophilia. His mother also adds that she had
allergies and asthma as a child. Which of the following is his most likely diagnosis?
Atopic dermatitis
○ A 17-year-old boy comes to the clinic because of an itchy rash. The rash started in
the previous day and is not responsive to antihistamines. He also recently
completed a weekend hiking trip in a rural, wooded area. He does not recall any
cuts or insect bites in the affected areas of skin. His mother denies any recent
changes in laundry detergents, soaps, or lotions in the house. No one else has
developed this rash in the household. He is otherwise healthy with no medical
problems. On physical examination, dorsal aspect of his right forearm and the
anterior aspect of his right lower leg is affected. The affected areas are arranged in a
linear pattern and are warm and erythematous with leaking vesicles. There are no
pustules or areas of induration or fluctuance. Which of the following is his likely
diagnosis? T-lymphocyte mediated cytotoxicity
○ A 12-year-old boy is hospitalized after a motor vehicle accident and develops a skin
infection. Culture grows oxacillin-resistant gram-positive cocci and he is treated with
the IV drug of choice for this infection. Soon after, his mother notices that he is
developing a rash on his face and neck. The boy becomes agitated, complains that
he is very itchy and his mouth “feels tingly.”What is the mechanism of this adverse
reaction? Mast cell degranulation
○ A 22-year-old woman comes to the clinic because of blisters on her hands. She
recently got a summer job designing costumes for a local theatre production group.
She says that the lesions first appeared on the lateral edges of her fingers and then
spread to her palms. The lesions are very itchy and somewhat tender. Physical
examination shows tense vesicles on her fingers and the palms of her hands without
any associated erythema. Which of the following is true about this condition? It can
be triggered by exposure to nickel
○ A 7-year-old girl comes to the clinic with her mother because she is very irritable and
cannot stop itching her arms. She says that the itching began after she was playing
outside with her friends over the weekend. Her past medical history non-contributory
and she takes no medications. Her temperature is 37ºC (98.6ºF), pulse is 89/min,
and blood pressure is 90/50 mm Hg. Physical examination shows a well-developed,
well-nourished girl in minimal distress. Cardiac auscultation reveals no murmurs and
a normal S1 and S2. Air entry is good bilaterally with some wheezes. She has a
rash on her right arms with pruritic vesicles in a linear configuration. Which of the
following conditions has the same pathophysiology that is responsible for this
patient's physical findings? Positive tuberculin skin test
○ A 3-year-old boy comes to the emergency department with a sudden onset of itchy
hands while playing outside near some shrubs on a family vacation in South
America. His mom says that she saw no broken glass, nails, or insects on the
ground where the child was playing. Initially, the child’s foot exhibited a white linear
pattern on the sole of the foot. Approximately, three to four hours later, the foot was
erythematous with multiple petechiae. The child also developed a temperature of
39.4°C (103°F). Oral and topical antibiotics were ineffective. Which of the following
is the most likely diagnosis in this patient? Dermatitis
○ A 15-year-old girl comes to the pediatrician’s office because of a gradual onset of
rash in the flexural creases of her elbows. She describes the rash as itchy and her
mother notes that she rubs it constantly. Physical examination shows a lichenified
plaque. Which of the following is the pathogenesis of the most likely diagnosis?
Colonization with Staphylococcus aureus
○ A 17-year-old girl comes to the clinic for a routine health maintenance examination.
She has been well and reports no concerns. She has a history of atopy but requires
no regular medication. Physical examination shows dermatographism. The type of
hypersensitivity involved in dermatographism is the same as which of the following?
Anaphylaxis
○ A 32-year-old man comes to the office because of a large, painful, erythematous
nodule on his right shoulder. Incision and drainage is performed, and the patient is
discharged with empiric antibiotic therapy sufficient to cover methicillin-resistant
Staphylococcus aureus (MRSA). 3 days into his 10-day course of antibiotics, he
comes to the emergency department because of a burning sensation on his back,
chest and face as well as peeling of large areas of skin. Which of the following is
most consistent with the cellular pathophysiology of the peeling skin? Fas-ligand
activation
○ A 35-year-old Caucasian woman comes to the dermatology clinic because of an
itchy rash on her wrists. The rash first appeared three weeks ago. Physical
examination shows violaceous, flat-topped lesions that have a shiny surface. A
histologic examination shows irregular acanthosis and colloid bodies in the
epidermis, as well as lymphocytic and histiocytic infiltrate in the upper dermis. Which
of the following is the most likely prognosis? Regression within 18 months
○ A 73-year-old male comes to the office with an itchy skin rash. The lesions are
round, oval, and erythematous on both legs. There is noticeable xerosis of the
affected skin. A potassium hydroxide preparation of the lesion scrapings is negative.
Which of the following is the most likely diagnosis? Nummular eczema
○ A 20-year-old female comes to the clinic because of a red, scaly plaques over her
antecubital fossae and popliteal fossae. Her dermatologic history is significant for
red, weepy papules, and plaques as a child that were distributed predominantly over
her cheeks and the extensor surface of her arms and legs. Her current rash is
extremely itchy, and the plaques in her anticubital fossae appear noticeably
lichenified. Which of the following modes of management should be initiated?
Topical corticosteroids
○ A 5-year-old boy comes to the emergency department with his mother because of a
skin rash on his face that is very itchy for 20 minutes. His mother says that within
minutes of eating peanut butter, he vomited and developed hives on his face.
Physical examination shows hives that appear red, raised, and are localized to his
face. His medical history is noncontributory and he is up-to-date on his
immunizations. His pulse is 90/min, respirations are 22/min, and blood pressure is
120/85 mm Hg. He does not have stridor on auscultation. Which of the following is
the most likely explanation for this patient’s physical findings? Food Allergy
○ A 16-year-old boy comes to the clinic for a health maintenance examination. He has
no health concerns at this time. He says that he can draw on his skin by firmly
scratching it with his finger nail. On physical examination, pressure is applied to his
forearm and a red wheal appears on the area. Which of the following is the most
likely immune system mechanism responsible for this response? Immunoglobulin
E-mediated
○ A 55-year-old man comes to the office because of pruritus for a week. He was
diagnosed with psoriasis three years ago, and his symptoms are not improving
despite good adherence to conventional therapy. Examination shows the presence
of erythematous scaly plaques on the extensor surfaces of the knee and elbows.
There is no evidence of flexural involvement. Which of the following is the most
appropriate treatment in this patient? Topical retinoid therapy
○ A 68-year-old man comes to the emergency department because of a macular rash
on his palms and soles. He is treated with a high dose of antibiotic administered
intramuscularly. In the days following his treatment, he experiences headache and
malaise followed by the development of red splotches on his skin, which soon form
blisters. He returns to the emergency department because he is unable to eat due to
oral lesions and is admitted for supportive care. Which of the following is the most
likely type of hypersensitivity experienced by the patient? Type IV
○ A 65-year-old woman comes to the clinic because of a pruritic rash on the back of
her lower extremities. The rash has persisted for about six months. She denies use
of any new skincare products, and has not taken any new prescription medication.
The rash partially responded to a topical steroid cream, but never fully resolved. She
has type 2 diabetes mellitus, diabetic retinopathy, chronic foot ulcer, hypertension,
hyperlipidemia, and obesity. Physical examination shows multiple dark plaques that
extend from the left buttock to the back of the left knee. The lesions are shiny with
an erythematous rim. Which of the following is the most likely diagnosis in this
patient? Lichen Planus
○ A 24-year-old woman comes to the emergency department because of urinary
frequency, burning sensation on urination, and pelvic tenderness. A urine culture
shows a lactose-positive, gram-negative rod. Further testing shows a negative
catalase test and positive indole test. The patient is prescribed a paraaminobenzoic
acid (PABA) analog that functions as a competitive inhibitor of dihydropteroate
synthetase and is discharged home. Four days later, she returns with complaints of
a burning rash covering her entire chest, abdomen, and back. Which of the following
conditions is the most concerning for this patient? Toxic epidermal necrolysis
○ A 49-year-old HIV-positive woman comes to the emergency department because of
fever and fatigue for the past 10 days. She has also experienced odynophagia and
"feeling like her skin is burning" for the past day. She denies current or recent history
of headache or cough, but says that she had been treated two weeks ago with
trimethoprim-sulfamethoxazole for a urinary tract infection. Her temperature is
39.5°C (103.1°F), pulse is 80/min, respirations are 15/min, and blood pressure is
125/80 mm Hg. Oral examination is below and shows mucosal abnormalities.
Significant conjunctivitis with a purulent discharge is noted on ocular examination
bilaterally. Skin examination shows confluent, erythematous, nonpruritic target
lesions with associated blisters on her upper arms and a positive Nikolsky sign.
Scalp, palms, and soles appear to be spared with no desquamation noted. The rest
of the examination shows no abnormalities. Which of the following is the most likely
diagnosis? Stevens-Johnson syndrome
○ A 35-year-old female presents to the office with the complaint of right-sided facial
pain. Medical history includes stabbing electric shocks exacerbated by eating,
brushing her teeth and a brisk breeze across her face. The pain lasts a few seconds
and rarely up to a couple of minutes. Physical examination shows normal facial
sensation, normal bulk in the masseters bilaterally, and intact extra-ocular muscles.
Which is the most appropriate treatment in this patient? Carbamazepine
○ A 23-year-old Caucasian female comes to the office because of a new, pruritic skin
rash on her back for 2 days. The patient says that the rash appeared without
warning and that the area has not enlarged since she first noticed it. She has had
similar skin lesions in the past on her elbows and knees. She reports no recent
exposure to chemical irritants. The last time she spent extended time outdoors was
over two weeks ago when she went to the pool. Her only medication is an oral
contraceptive pill that she has taken for the past 5 years. Examination shows her
elbows and knees are clear of skin lesions. There is a symmetric, red plaque with
well-circumscribed borders on her back as shown below. Which of the following is
the most likely cause of this patient’s rash? Sunburn
○ A 35-year-old woman comes to the office because of an itchy rash on her wrist. She
says that it started four days ago and the itching is worsening. Physical examination
shows several small, violet papules located on the anterior wrists. Which of the
following findings is most likely to be seen in this patient? Wickham striae
○ A 35-year-old man comes to the primary care clinic because of an intermittently
itchy rash. He says the rash is very pruritic and often coincides with when he walks
outside in the cold without a warm coat or when he takes a cold shower. The rash is
waxing and waning in nature, and he does not currently have the rash during this
visit. During physical examination, an "X" is drawn on the patient’s back with a
wooden stick. Ten minutes later, the area that was scratched appears exaggeratedly
red and edematous. Which of the following is the most likely diagnosis? Physical
urticaria
○ A patient on IV line complains of intense itching in his arm. On physical examination,
the left forearm is erythematous with scattered small blisters and excoriations. Some
of the blisters have ruptured and are moist with serous fluid. The rash appears in a
linear pattern. Which of the following is his most likely diagnosis? Type IV
hypersensitivity reaction
○ A 16-year-old boy comes to the clinic because of a "red and bumpy" rash on his
arms, back and chest. The rash has persisted for 36 hours and is intensely itchy.
The patient says that he has been recovering from "a cold" over the past two weeks.
Apart from the rash, he complains of a mild sore throat and some fatigue. When
asked about his medications, he says he recently finished a 10-day course of
amoxicillin. A heterophile antibody test is administered and returns positive. Which
of the following is the most likely etiology of this patient's rash? Hypersensitivity
reaction to the antibiotic
○ An 8-year-old boy comes to the clinic because of an itchy rash. He had been eating
lunch at school and developed a small, bumpy rash on the backs of his hands and
on his forearms. The child's medical history is significant for asthma and atopic
dermatitis. Physical examination shows the rash consists of raised, erythematous
plaques with central pallor. There are excoriations diffusely across the affected
areas. The child is otherwise not in any distress and there are no other focal findings
on examination. Which of the following is the most appropriate treatment for this
child? Antihistamines and prescription for epinephrine autoinjector
○ A 49-year-old woman comes to the office because of new skin lesions on her
elbows for a week. She describes well-marginated, erythematous plaques with
silvery-white scales on the extensor surface of her elbows. She states that she has
had a rash like this before, that resolved on its own. Her father also has a similar
condition. Examination shows further lesions on her knees and heels, as shown in
the exhibit. Which of the following is a nail abnormality most commonly associated
with this patient's condition? Onycholysis
○ A 17-year-old man comes to the clinic with a pruritic rash.He says that he noticed a
vesicular rash on his arm after walking in the forest with his friends. His temperature
is 37.1 oC, pulse is 80 /min, respiratory rate is 12/min, and blood pressure is 110/75
mmHg. Physical examination shows clear fluid filled vesicles on his arm in a linear
pattern. He does not have stridor and air entry is good bilaterally. He is up-to-date
on his immunizations and does not have any allergies. He does not smoke and does
not drink alcohol. Which of the following is the most likely explanation for this
patient's physical findings? Rhus dermatitis
○ A 63-year-old man comes to the dermatology clinic because of rapidly progressing
skin redness which has covered most of his body. He has coronary artery disease
with a 3-vessel CABG, hypertension, and psoriasis. His medications include a
beta-blocker, aspirin, ACE-inhibitor, statin, and topical clobetasol. He has not had
any recent changes to his medications. His temperature is 38.8°C (101.8°F), pulse
is 115/min, respirations are 16/min, and blood pressure is 95/68 mm Hg. His O2
saturation is 97% on room air. He is in no acute distress, but he is shivering. On
examination, his oropharynx is clear without mucous membrane involvement. His
skin exam shows diffuse erythema with an overlying scale covering over 90% of his
body surface area. Which of the following is the most likely diagnosis?
Erythrodermic psoriasis
○ A 39-year-old woman is brought to her dermatologist because of a "rough and itchy
rash” for the past week on the flexor surface of her wrists, lower extremities, and
neck. Physical examination shows the following image: Which of the following is
most likely to be associated with this patient's dermatologic condition? Atopic
dermatitis
○ A 30-year-old man comes to the office because of painful and itchy hands. He works
at a hospital that enforces strict hand-washing observance rules and he has been
washing his hands 25-30 times a day. He has no prior medical history and takes no
medications. He smokes a pack of cigarettes daily and has consumed about 2 beers
daily for the past 10 years. His temperature is 36.8ºC (98.2ºF), heart rate is 80/min,
respirations are 15/min, and blood pressure is 110/60 mm Hg. Physical examination
shows his hands are erythematous with areas of fissuring and vesiculation as shown
below. The remainder of his examination is unremarkable. An extensive patch test is
negative. Which of the following is the most likely pathophysiology of this patient's
most likely skin condition? Pro-inflammatory cytokine release from chemical
stimuli
○ A 38-year-old woman comes to the gynecology clinic because of vaginal itching
which started 3 weeks ago. She reports that the itching is intense and has gotten
worse. She denies being sexually active. She does state that she started using a
new brand of laundry detergent a month ago. Physical examination shows diffuse
erythema of the labia majora and minora with leathery patches. Which of the
following is the most likely diagnosis? Lichen simplex chronicus
○ A 30-year-old woman comes to the office because of a painful, pruritic rash on her
legs, elbows, and scalp for 2 weeks. She claims that she had a similar, but mild,
rash eight months prior that resolved on its own. A review of systems also reveals
fatigue, muscle aches and stiffness. She also mentions that she has recovered from
an episode of mononucleosis three weeks prior. Examination shows red plaques
with silvery scales. Which of the following facts about this patient's past medical
history would most likely further support the diagnosis? Family history of skin
disease
○ A 75-year-old woman comes to the emergency department because of a painful
rash on her face. Her husband noted what appeared to be pimples on her right
forehead that gradually grew in size. Physical examination shows vesicular clusters
on her right forehead and most of the right side of her nose. Her temperature is
36.5°C (97.7°F), pulse is 72/min, respirations are 14/min, and blood pressure is
136/80 mm Hg. Which of the following is the next best diagnostic step in this
patient? Ophthalmic fluorescein examination
○ A 30-year-old male comes to the office because of a rash for a month. He states that
it has been gradually worsening and has been very painful with a burning sensation.
He does not recall a recent change in diet or soaps. He has also not had any new
exposures, recent travel or illnesses. While he has never had this rash before, he
does recall that a couple years ago he had a rough, itchy spot on his scalp which
gradually resolved. Examination shows he has several large plaques of scaly skin
on a red base located on his elbows, knees, and trunk. Which of the following is
most likely true about this type of lesion? It bleeds when scales are scraped off

● PLAYLIST 5
○ A 25-year-old woman comes to her primary care physician's office because of hair
loss over the past two weeks. She has type I diabetes mellitus which is controlled
with an insulin pump. She reports that her hairdresser noticed a "bald spot" on her
scalp when she had her hair cut two weeks ago. Physical examination shows a
well-circumscribed oval patch of hair loss in the occipital region with no evidence of
scarring. The surrounding hair around the periphery is broken. Which of the
following is the most likely diagnosis? Alopecia areata
○ A 60-year-old Caucasian male comes to the office because of a growth on his right
upper lip for a year. He states that since he first noticed the lesion, it has slowly but
steadily grown. It is not painful, and it has never bled. Examination shows a
pink/yellow nodule of the right upper lip. A biopsy is obtained. Which of the following
is most likely true regarding this patient's condition? It is not likely to stain positive
for S-100
○ A 26-year-old man comes to the clinic because of pain and purulent drainage from
his left inner buttox. He has had this issue intermittently for the past 10 years, and is
afraid that the lesions will lead to scarring. The patient states that he has had the
same problem in his axillary region. His current medications include atorvastatin and
lisinopril. He is 173 cm (5 ft 8 in) tall, weighs 105-kg (231-lb); BMI is 35.1 kg/m2. His
temperature is 37.2°C (99.0°F), pulse is 86/min, respirations are 12/min, and blood
pressure is 128/80 mm Hg. Physical examination shows erythema, swelling and
multiple pus discharging sinus tract openings on the inner portion of his left thigh.
Examination of his axillae show the presence of linear scarring bilaterally from prior
lesions. Which of the following is the most likely diagnosis? Hidradenitis
suppurativa

● PLAYLIST 6
○ A 72-year-old man is referred to a dermatologist because of an irregular scaly patch
on his right ear. He has a 40-year history of poorly controlled hypertension and type
2 diabetes mellitus. A photograph of the lesion is shown. A biopsy specimen shows
hyperproliferation of epithelial cells confined to the epidermis. Topical therapy with
5-fluorouracil is initiated. Which of the following is the mechanism of action of this
drug? Blocks the action of thymidylate synthase
○ A 70-year-old woman comes to your office because of numerous small "lumps" on
her eyelids. They are neither itchy nor painful, and she has noticed similar growths
on her armpit and groin for the past few years. Physical examination shows multiple
soft, flesh-colored nodules ranging from 2-5mm in diameter, located on her upper
eyelids, axilla, posterior neck, and groin. Which of the following lesions most likely
would require excision? Actinic Keratosis
○ A 60-year old man comes to the dermatology office for a routine skin exam. He has
a history of chronic sun exposure due to his career as a farmer. Dermatologic
examination shows several red, scaly lesions on the scalp (pictured). A topical
treatment that works by inhibiting the action of thymidylate synthase is prescribed.
Which of the following is the patient most likely to experience with application of this
medication? Ulceration, crusting, and hypopigmentation in areas where the
medication was applied
○ A 55-year-old woman comes to the office because of a growth on her left flank for 2
months. She states that she first noticed the lesion a few months ago and since
then, it has grown significantly larger. Examination shows an elevated, firm,
brown-blue lesion. A biopsy with histopathological analysis is obtained. The
dermatopathologist notes atypical melanocytes invading the dermis and epidermis.
Which of the following is most likely true of this patient's condition? It is associated
with higher rates of metastasis.
○ A 65-year-old woman comes to the dermatology clinic because of concern about an
unusual mole on her left arm. She is frightened because several of her family
members have been diagnosed and treated for skin cancer. Her medical history is
otherwise noncontributory. A skin biopsy is performed and shows proliferation of
abnormal melanocytes. Which of the following mutations is the patient most likely to
have? CDKN2A mutation
○ A 40-year-old woman comes to the office because of an itchy, painful bump on her
skin. She remembers having a lesion in that area for years which was always
slightly red. Recently, she has noticed an increase in size as well as pain. Her family
history is positive for skin cancer, but she does not remember which family members
had it or which types. Physical examination shows a red, raised, scaly lesion on the
patient's right forearm that is about 2-cm in diameter. The lesion is not
well-demarcated. The patient has not noted any other similar lesions. Which of the
following is the best treatment option for this patient? Mohs surgery
○ A 65-year-old man comes to the office because of multiple new skin lesions on his
forehead. He has a past medical history of stage 4 renal failure, peritoneal dialysis,
and subsequent renal transplant that took place 3 years ago. Physical examination
shows lesions that are less than 1 cm in diameter, slightly raised, and rough to the
touch. The man says he thought that they were scabs, and they bleed when he
picks at them. He noticed the first one on his forehead 3 months previously and now
has 4 more on his forehead, cheeks, and nose. Which of the following medications
most likely contributes to this patient's skin findings? Azathioprine
○ A 19-year-old man comes to the dermatology clinic because of a 4 mm lesion on his
upper back. Core biopsy shows malignant melanoma, and the patient prepares to
undergo wide local excision of the tumor. Prior to surgery, the correct margins for
resection are determined by which of the following? Breslow thickness
○ A 58-year-old Caucasian man comes to the office because of a non-healing ulcer on
his forearm. He has had an ongoing osteomyelitis infection for approximately two
years now. Physical examination shows an indurated, erythematous,
four-centimeter, round ulceration on the anterior aspect of his right forearm. Upon
further inspection you note a draining sinus tract. Which of the following conditions is
most likely to develop in association with the patient's wound? Squamous cell
carcinoma
○ An 18-year-old woman comes to the dermatology clinic because of multiple dark
papules on her back and shoulders. Physical examination shows that her head is
large and her forehead is broad. Her palms have several pits formed in the skin.
Which of the following is most likely to be found on a biopsy of the lesions on this
patient’s back and shoulders? Basaloid tumor cells
○ A 72-year-old woman comes to the dermatology clinic because of a large, reddish
brown lesion on her nose. She is diagnosed with metastatic nodular melanoma. She
is prescribed a medication that acts by blocking the action of ribonucleotide
reductase. Which of the following medications was most likely prescribed?
Hydroxyurea
○ A 55-year-old man comes to the office because of an ulcerated, scaly lesion on his
left ear. His past medical history is significant for colon cancer, which had been
treated a year prior to this visit with aggressive chemotherapy. He is currently
otherwise healthy. A shave biopsy of the lesion shows atypical cells with nuclear
crowding and disorganization across the full thickness of the epithelium. Which of
the following is true about this patient? The presence of keratin "pearls" on
biopsy is a good prognostic indicator.
○ A 45-year-old woman comes to the office because of the presence of vertical
wrinkles on her upper lip. She complains that none of her friends of the same age
have these fine lines or wrinkles and does not know why she has them. Past
medical history is noncontributory. She reports using a moisturizing lotion with sun
protection factor (SPF) of 15 daily. She has a 20 pack-year history of smoking. She
reports having two glasses of wine socially about once a week. Which of the
following cutaneous features is the patient most likely to also develop? Follicular
plugging
○ A 60-year-old Caucasian male comes to the office because of a growth on his right
upper lip for a year. He states that since he first noticed the lesion, it has slowly but
steadily grown. It is not painful, and it has never bled. Examination shows a
pink/yellow nodule of the right upper lip. A biopsy is obtained. Which of the following
is most likely true regarding this patients condition? It is not likely to stain positive
for S-100
○ A 54-year-old man comes to the office because of a slow-growing reddish spot on
the site of a surgical scar. He broke his arm in his teenage years and required
surgical repair. Over the last several decades, he has noticed a reddish spot
growing on the site of his scar which bleeds when he scratches it. Over the past
year, the spot has become firm and there is often a small ulcer in the center that will
not heal. Which of the following is true regarding the lesion? The lesion is likely an
aggressive tumor
○ An 84-year-old man comes to the office for examination of his skin to detect
pre-malignant or malignant lesions. Past medical history is noncontributory. He
reports smoking about a half-pack of cigarrettes for the past 60 years. Physical
examination shows furrowed, yellowed skin around his eyes and on his forehead. A
punch biopsy is performed. Which of the following features would most likely be
found on histologic examination? Thickened elastic fiber fragments in the upper
dermis
○ A 63-year-old man comes to the clinic because of a large, brown-blue lesion on his
arm. He also says he has headaches and fatigue, and says that he has lost
significant weight over the past few months. He says that his mother died of
melanoma when he was a child. He has periodically taken pictures of the lesion over
the past several months (progression from C to D to B in the image below). Skin
biopsies of the lesion are performed, and a diagnosis of metastatic melanoma is
made. Which of the following would most likely be seen in this patient's skin biopsy?
Abnormal melanocytes with an increased nucleus to cytoplasm ratio
○ A 58-year-old man comes to the office for examination of his skin to detect
pre-malignant or malignant lesions. He has no complaints or concerns. Physical
examination shows indurated, yellowed skin around his eyes and on his forehead.
Which of the following is the most significant risk factor for this skin finding?
Smoking
○ A 1-week-old male newborn delivered at 39 weeks’ gestation is brought to the clinic
by his parents because of a small discolored lump on the side of his scalp. He was
born via an at home water-birth with the assistance of a nurse-midwife. He has not
been scratching the mass and does not appear to be in any pain. He has been
exclusively fed with breast milk since delivery and the appearance of the mass has
not changed. Family history is noncontributory. Physical examination shows an area
of alopecia on the left side of his head. The underlying skin is orange colored and
nodular. Which of the following is the recommended treatment? Removal before
adolescence
○ A 55-year-old woman comes to the clinic for a physical examination required for her
new job. Inspection of her oral cavity reveals the image shown below. The lesion is
tender rather than painful, but she says that she has had recurrent painful canker
sores before. Which of the following is the most appropriate next step in
management? Gingival biopsy
○ A 60-year-old male surfer comes to the office because of a 1-inch mass on the right
side of his forehead. He says that he has had the mass for years and that it has only
gradually grown in size. A punch biopsy is obtained and shows highly atypical cells
spanning the entire thickness of the epidermis with an intact basement membrane.
Nuclear crowding and disorganization is present. Which of the following mutagens
most likely contributed to the formation of the lesion? Ultraviolet B rays
○ A 70-year-old woman comes to the office because of a rapidly growing mass on her
right cheek. A biopsy taken of the lesion shows epidermal hyperplasia, the presence
of large eosinophilic keratinocytes, and a sharp demarcation between the tumor and
surrounding tissue. Which of the following is the most appropriate next step in
management? Schedule surgical excision of the lesion
○ A 60-year-old sailboat captain comes to the clinic because of concern about his
dark-colored skin lesion. He says that the lesion has been growing on the side of his
forehead for several months. A diagnosis of melanoma is confirmed. Which of the
following is the most important preventable risk factor for the development of this
patients condition? Sun Exposure
○ A 75-year-old woman comes to the office because of a growth on her right cheek.
She denies any pain or pruritus associated with the lesion. The lesion has grown
slowly over the past year. Physical examination shows a firm, erythematous, scaly
lesion that is nontender to palpation. Which of the following is the most appropriate
next step in management? Perform a biopsy of the lesion
○ A 37-year-old man comes to the clinic because of a dark spot on his upper back that
has recently changed in shape. He is light-skinned with many freckles and says that
he suffered from frequent severe sunburns as a child. On his right shoulder is an
asymmetric, relatively flat plaque with irregular margins and variegated brown to
black pigmentation measuring 8 mm at its longest length. Skin biopsy of the mole
shows a pagetoid pattern of atypical melanocytes within the epidermis and
superficial papillary body of the dermis. Which of the following may have been the
precursor to this lesion? Dysplastic nevus
○ A 62-year-old man comes to the clinic because he noticed a lesion on his nose three
days ago. He scratched his nose, and thought he had removed a scab because it
was bleeding. He is an avid mountain climber, and loves spending hours climbing in
the sun. Physical examination of his nose reveals a 3mm erythematous area that is
pearly and slightly raised. Biopsy is performed and histology suggests basal cell
carcinoma, superficial type. Which of the following treatments would be most
appropriate for this patient? Electrodesiccation and curettage
○ A 34-year-old comes to the office presents for a routine physical examination.
Physical examination shows a dimpled patch of pink skin, as shown below. The
patient reports that the “pimple” does not hurt, but bleeds when picked at and first
appeared several months ago. Which of the following is the most likely diagnosis?
Basal cell carcinoma
○ A 38-year-old farmer comes to the office for an annual full-body skin examination.
He has had human immunodeficiency virus (HIV) infection for the past five years
and is currently on highly active anti-retroviral therapy. Physical examination shows
a 1-cm, erythematous, translucent plaque with a rolled scaly border on the patient's
back. The patient does not recall any trauma in that area and had not noticed it until
the dermatologist brought it to his attention. Which of the following is the most
appropriate management of the most likely diagnosis? Mohs micrographic
surgery
○ A 57-year-old man comes to the emergency department because of a dark-colored
skin lesion. He has worked as a farmer since he was 17-years-old. He says that the
lesion has been growing on the side of his forehead for several months. Generally
speaking, which of the following is the most effective means of reducing mortality
from this condition? Early clinical detection
○ A 65-year-old man comes to the clinic because of a mole that has been growing on
his right arm. Physical examination shows an 8 mm black-brown asymmetrical
lesion with an irregular surface and border. A skin biopsy of the lesion shows
proliferation of atypical melanocytes that have invaded the epidermis and dermis.
Biopsy of the sentinel lymph nodes is positive for metastasis. Which of the following
proteins would you expect to be elevated in the serum of this patient? S-100
○ A 55-year-old man comes to the dermatology clinic because of an irregular skin
lesion. He is otherwise healthy and without complaints. The lesion is located on the
dorsal aspect of his left forearm. It is asymmetric and heterogeneous in color, with
irregular borders. He says that it has grown larger over past several months. Which
of the following characteristics of this lesion is most appropriate in the determination
of the prognosis? Depth of invasion
○ A 55-year-old man comes to the dermatology clinic for an annual physical
examination. Past medical history is significant for hypertension. Physical
examination shows a dark pigmented lesion on the patient’s back. The lesion has
several different colors including tan, dark brown and nearly black sections. The
lesion measures 7 mm in diameter, and is asymmetric in shape. Which of the
following is the next most appropriate step in management in this patient?
Excisional biopsy
○ A 36-year-old man who recently emigrated from Chile comes to the physician for a
wellness checkup. He states that he had worked as a farmer for many years. His
jobs included spraying plants with pesticide, harvesting crops, and plowing the
fields. The pesticide he worked with contains arsenic. Which of the following skin
conditions is he most at risk for developing? Squamous cell carcinoma
○ A 60-year-old Chinese man comes to the dermatology clinic complaining of
something strange growing on his big toe. He first became aware of its existence
several months ago when he was sitting with his feet propped up, and his wife
noticed a small bump on the bottom of the toe that has since grown. He has worked
in an office for most of his adult life and does not enjoy outdoor activities. Medical
history is negative for trauma to the toe. Physical examination shows a dark brown,
raised lesion with an irregular surface and minimal crusting. The patient says that
the lesion used to be flat, but now it is raised and it hurts when he walks. A biopsy
was taken (results are shown below). Which of the following is true about this
patient's condition? This condition is more common in individuals over the age
of 40

● PLAYLIST 7
○ A 29-year-old man comes to the clinic because of a painful, gluteal rash that has
been present for over one month. He denies intravenous drug use, but admits to
being sexually active with multiple partners of both sexes, including several
unprotected events. Physical examination shows a rash consisting of numerous
small, fluid-filled vesicles alongside multiple ulcerations. Which concomitant infection
would most likely explain why this patient's symptoms are lasting longer than
expected? Human immunodeficiency virus
○ A 20-year-old man comes to the clinic with a rash that began on his hands and feet
that has progressed to his forearms and legs. He also complains of sore, cracked
lips and gums. About 3 days prior, he began having muscle aches, low-grade fever,
and headache. He then began itching and burning, and the rash appeared.
Examination shows scattered bullae and erosions in his oral cavity, in addition to
multiple, annular, symmetric erythematous lesions covering his palms, backs of his
feet, and extensor surfaces of his forearms and legs. The center of each lesion has
a blister. Which of the following is the most common trigger for this condition?
Herpes simplex virus infection
○ A 2-year-old girl is brought to the emergency department with a rash on her trunk
and extremities. History shows the child developed upper respiratory symptoms with
purulent nasal discharge two days prior. She is febrile and irritable. She has a
diffuse erythematous rash on the trunk and upper extremities sparing the face and
mucous membranes. The rash consists of vesicular eruptions that slough off with
sheets of skin when slightly rubbed. Which of the following is the most likely
diagnosis in this patient? Scalded skin syndrome
○ A 64-year-old female has a rash on her right lower extremity several days after
undergoing a right mastectomy. Physical examination shows a red, swollen area that
is sharply demarcated, painful to the touch, and appears similar in consistency to an
orange peel. Which of the following is the most likely infectious agent?
Streptococcus pyogenes
○ A 12-year-old boy is brought to his pediatrician because of a fever and bone pain for
the past 12 hours. The patient was previously healthy except for a slow-healing bite
on his leg from the barn cat that occurred 3 days before. On physical examination
the patient appears fatigued and irritable. Musculoskeletal examination of the
patient's left leg shows focal swelling, pain, and redness at the proximal left tibia and
decreased range of motion in the left knee at the site of laceration. Which of the
following statements is most accurate regarding the microorganism causing the
patient's condition? It is typically treated with Penicillin G
○ A 4-year-old boy is brought to the office by his parents because of blister formation
for the past two days. The mother noticed several small pimples on his cheeks when
he returned from daycare yesterday. This morning the pimples enlarged and filled
with fluid. His temperature is 37.1°C (98.8°F), pulse is 90/min, respirations are
18/min, and blood pressure is 96/64 mm Hg. Physical examination shows a
well-appearing child with multiple bullae filled with clear yellow fluid on his face,
extremities, and in the oral cavity. Administration of which of the following is the most
appropriate treatment for this patient? Cephalexin
○ A 25-year-old woman comes to the emergency department because of pain and
redness in her left breast for the past three days. She gave birth 4 weeks ago and
has been exclusively breastfeeding her son. Vital signs show her temperature is
38.5°C (101.3°F), heart rate is 75/min, respirations are 14/min, and blood pressure
is 140/85 mm Hg. Physical examination shows the left breast is erythematous, warm
and tender around the lateral nipple. There is no fluctuance. Which of the following
is the most appropriate treatment option? Prescribe antibiotics for the mother
while she continues bilateral breastfeeding
○ A 20-year-old female college student comes to the office because of itchy hands for
the past week. She states that she has never experienced this previously. She has
no pets and denies recent travel history. She has multiple sexual partners and states
that she always uses condoms and is currently taking oral contraceptives. Physical
examination shows multiple erythematous linear lesions surrounded by areas of
excoriation on her palms bilaterally as well as the webs of her fingers. Which of the
following modes of transmission is most likely responsible for her disease? Direct
contact
○ An 18-year-old woman comes to the clinic because of a painful rash by the corner of
her mouth. She reports that she felt a “strange tingling” prior to the outbreak a few
days ago. She does not recall having similar lesions previously, but notes that her
boyfriend did have a “weird bump” on his lip a few weeks ago. Which of the following
is true about the most likely cause of her condition? After active infection is over,
the virus will lay dormant in local ganglion nerves and may be reactivated at a
later date
○ A 19-year-old woman comes to the emergency department because of fever,
confusion and a headache. Medical history is significant for herpes simplex virus in
the form of oral herpes. Lumbar puncture reveals increased protein and
lymphocytes, and glucose of 66mg/dl. Magnetic resonance imagining shows
increased signal in the left temporal lobe. Which of the following visual defects might
be present in this patient? Right superior homonymous quadrantanopia
○ A 20-year-old female college student comes to the office because of pruritic papules
on her hands for the past three days. The papules started between her fingers and
have since spread to her wrists. She also has similar lesions in her inguinal skin
folds bilaterally. She has attempted using topical corticosteroids with minimal relief.
She reports recently moving into a dormitory and has noticed that her boyfriend has
similar lesions. Physical examination shows papular lesions with a linear
erythematous track leading away from each one. Which of the following measures is
the most appropriate to prevent disease transmission to others? Washing all
clothing and bedding in hot water
○ A 38-year-old man comes to the emergency department because he is human
immunodeficiency virus-positive and has not been compliant with his antiretroviral
therapy. His friend that brought him to the hospital says that the patient has
appeared progressively more lethargic for several days and his memory appears to
be impaired. In addition, he reports several uncharacteristic behaviors, including
inappropriate sexual behavior and an insatiable appetite. Cerebrospinal fluid
analysis results show glucose levels of 70 mg/dL, protein at 90 mg/dL, and white
blood cells at 10 cells/µL. Which of the following is the most appropriate next step in
the management of his condition? Admit to intensive care unit and start
intravenous acyclovir
○ A 10-year old girl is brought to her pediatrician's office by her parents because of a
new onset of skin lesions for the past 3 days. Her father says that the patient had an
initial lesion of a small pustule which quickly progressed into an extensive rash.
Physical examination shows the following: Which of the following is the most likely
causative organism? Staphylococcus aureus
○ A 5-year-old girl comes to your office because of redness and swelling around the
right eyelid for the past 3 days. Her mother says the patient was scratched by a dog
4 days ago. The patient denies pain in the eye and denies any worsening or
alleviating factors. Physical examination shows extraocular movements are normal.
Additional vision testing in the office is unremarkable. Which of the following is the
most likely diagnosis? Periorbital cellulitis
○ A 22-year-old man comes to the clinic because of a painful and pruritic rash along
the shaft of his penis. The lesions are small, fluid-filled vesicles arranged in clusters.
He has experienced similar symptoms before, and can usually “feel them” before
they appear. He also admits to being sexually active with multiple partners for the
past couple of months. Which of the following has classically been the most
appropriate method of diagnosing the lesion? Tzanck smear
○ A 65-year-old man comes to the clinic with a red, hot, edematous plaque on his left
leg. His lab test results confirmed the diagnosis of a methicillin-sensitive
Staphylococcus aureus infection. He has significant lymphadenopathy in his groin.
The patient has a past medical history of rheumatoid arthritis. Which of the following
is the most likely potential complication of this infection? Sepsis
○ A 45-year-old man comes to the office because of a 3-day history of itching and
erythematous lesions around his mouth. He shaves regularly and has not used any
different soaps or lotions during the past year. He has no history of serious illness
and takes no medications. A photograph of the affected area is shown. Which of the
following is the most likely diagnosis? Folliculitis
○ A 23-year-old woman comes to the clinic because of pain on her genitals. She
states that she is sexually active with multiple men and does not always use
condoms. She denies any other medical history. Temperature is 37.9°C (100.2°F),
pulse is 80/min, respirations are 18/min, and blood pressure is 118/82 mm Hg.
Physical examination shows two small, round areas of ulceration on her right labia
minor. Which of the following is the structure of the genetic code for the most likely
infection? Double-stranded, linear DNA virus
○ A 29-year-old male comes to the office because of a tender, burning rash on his left
leg for the past two days. The patient also reports subjective fever and malaise for
the past week. Physical examination shows a sharply demarcated, raised,
erythematous patch along his left pretibial area. The involved tissue is warm, tender
to palpation, and has no edema or crepitus. Which of the following is the most
appropriate treatment for this condition? Penicillin V potassium
○ A 6-year-old boy is brought to his pediatrician's office by his parents because of
blister formation on his buttocks for the past 3 days. His mother reports seeing
enlarged blisters on his buttocks that appear to be filled with fluid. His temperature is
37.1°C (98.8°F); pulse is 72/min;respirations are 18/min, and blood pressure is
95/60 mm Hg. Physical examination shows the following image: Exfoliative toxin A
○ A 15-year-old boy with a history of asthma comes to the office because of extreme
itching for the past week. His symptoms began shortly after returning from summer
camp, and he notes that they are worse in the evening. He remembers that some of
the other children at the camp experienced similar symptoms. He has never had any
skin issues prior to this event. He denies any allergies and does not take any
medications. Physical examination shows a healthy boy with erythematous and
excoriated papules with some crusting located on the elbows, flexor surface of the
wrist, and the webs and sides of fingers. Which of the following is the most likely
diagnosis? Scabies
○ A 19-year-old male comes to the office because of a rash for the past week. The
rash began with intense itching in his groin, which is worse after a hot shower. He
admits to having several sexual partners over the past year. Physical examination
shows tracks of excoriated papules on his genitals and groin. Which of the following
is the most appropriate treatment? 5% permethrin cream
○ A 15-year-old boy is brought to his pediatrician's office by his mother because of a
new onset of skin lesions for the past 10 days. The patient just returned from
summer camp 10 days ago in central Pennsylvania. Physical examination shows the
following image: The patient is initially treated with mupirocin and his condition
improves. He is brought back to the clinic 3 weeks later because of new onset
brown colored urine, puffy face, and elevated blood pressure. Which of the following
best explains this patient's new symptoms? Group A streptococcal infection
○ A 35-year-old woman comes to the emergency department because of pain and
swelling of her right lower extremity for the past day. She says that she had a fall two
days prior which resulted in a puncture wound to her right lower extremity from
landing on a fallen branch. Physical examination shows redness, swelling,
paresthesias, a large area of gangrenous skin, and crepitusupon palpation in the
right lower leg. Within hours the area of redness extends towards the right knee and
right thigh with extreme tenderness on exam. Which of the following organisms is
most likely to be found in this patient's wound? Streptococcus pyogenes
○ A 65-year-old man comes to the emergency department because of a rash for the
past 2 days. Seven days prior, he had noticed itching and burning pain around his
left shoulder blade which was followed by the development of a painful, vesicular
rash. The rash was treated appropriately. He returns to the office six months later
complaining of a persistent, lancet-like, burning pain in the same area previously
covered by the rash. Physical examination shows no cutaneous abnormalities.
Which of the following is the most appropriate treatment option? Nortriptyline
○ A 25-year-old light-skinned man presents to your office because of several white
patches on his upper back and chest for the past year. He works as a roofer. He first
noticed the patches at the end of last summer, but they disappeared as the weather
got colder. This summer the patches appeared as soon as he started working. His
chest lesions are as shown below. Which of the following is the most appropriate
treatment for this condition? Ketoconazole 2% cream
○ A 37-year-old woman comes to the emergency department because of a laceration
on the arm. She reports that she was bitten by her dog about six hours ago. Her
temperature is 36.8°C (98.3°F), pulse is 74/min, respirations are 18/min, and blood
pressure is 110/78 mm Hg. Physical examination shows a superficial laceration
3.2-cm in length without hemorrhage. Which of the following is the most appropriate
management? Wound irrigation with normal saline
○ A 45-year-old woman comes to the emergency department because of right eye
pain and redness for 1 day. She has photophobia and watery discharge from the
eye. She does not wear glasses or contact lenses and has no prior eye problems.
On examination, the patient’s visual acuity is 20/20 in the left eye and 20/70 in the
right eye. She has conjunctival injection around the cornea and clear watery
discharge. When fluorescein is applied, a branching, white-colored epithelial defect
is seen. The remainder of the head examination is normal and the patient has no
cutaneous lesions. Which of the following is the most appropriate treatment for this
patient? Topical antiviral medication
○ A 19-year-old man comes to the clinic because he has several stubborn periungual
warts. The warts have becoming more numerous and progressed in size over the
past several months. Unsuccessful attempts to permanently eradicate the lesions
included salicylic acid, cryotherapy and electrosurgery. Intralesional injection of
which of the following compounds may be effective in treating recalcitrant warts?
Bleomicine
○ A 64-year-old woman has a rash on her right lower extremity several days after
undergoing a right mastectomy. Physical examination shows a red, swollen area that
is tender to touch and appears similar in consistency to an orange peel. A similar
rash is shown below. Which of the following is the most likely diagnosis? Erysipelas
○ A 20-year-old man comes to the clinic because of painful ulcers on his penis. He is
sexually active and reports consistently not using condoms. He denies significant
past medical history. He reports current headache and fatigue for 3 days. His
temperature is 38.6 °C (101.5°F), pulse is 68/min, respirations are 16/min, and blood
pressure is 128/78 mm Hg. Physical examination shows three 2x1cm painful ulcers
on the left lateral shaft of the penis and tender inguinal lymphadenopathy. Cultures
of the ulcers are obtained. Which of the following medications is the next best step
in management? Valacyclovir
○ An 11-year-old girl is brought to the clinic by her worried mother because of a
worsening rash. She returned home from summer camp two days prior with bumps
on her face that her mother had never seen before. When the patient woke up this
morning, the lesions were present on her arms as well. Physical examination shows
multiple flesh-colored, dome-shaped papules around her eyes and inner forearms.
The papules are not pruritic or painful. A biopsy of one of the lesions is performed
and the histological appearance is shown below. Which of the following is the most
appropriate treatment for children with this condition? Reassurance
○ A 10-year-old girl is brought to the outpatient clinic because of a rash that seems to
be getting worse. Her mother says that the patient returned home from gymnastics
camp two days prior with small bumps on her arms, but now they appear larger and
are on her face as well. Physical examination reveals multiple flesh-colored,
dome-shaped papules around her eyes and inner forearms, as shown in the image
below. The patient says the papules are only a little bit itchy, but are not painful. A
biopsy of a small lesion using a curette is performed. Which of the following is most
likely to be seen on histological analysis of this papule? Henderson-Paterson
(inclusion) bodies
○ A 66-year-old man comes to the office because of fever and thigh pain for the past
day. The patient noticed intense pain followed by redness and swelling in a small
area of the right lateral thigh and it has progressively increased to the current state.
Physical examination shows erythema and swelling affecting the right thigh, right
buttock, and lateral aspect of the right leg, and crepitus on palpation. Leukocyte
count is 14,000/mm3. Which of the following organisms is the most likely etiologic
agent of this patient's condition? Group A Streptococcus
○ A 18-year-old man presents to the emergency department with periorbital edema
and a black necrotic lesion on his right cheek. The patient recently came back from
Iran where he was living on a farm. He has been well until 14 days ago when he
noticed a small, painless, pruritic papule on his face that quickly enlarged and
developed a central vesicle. The vesicle burst, leaving a painless necrotic ulcer with
a black, depressed eschar. He has extensive bilateral edema of the eyelids which
began 5 days ago. He is afebrile with no lymphadenopathy. A Gram's staining of the
ulcer revealed gram-positive spore-forming bacilli. Which of the following is the best
management for this patient? 7-day course of Ciprofloxacin
○ A 10-year-old boy comes to the emergency department with progressive right
periorbital swelling for the past two days. He also complains of fever, nasal
congestion, and blurry vision. On physical examination, his right eye is swollen
closed. He also has proptosis, conjunctival injection, and is unable to abduct his
right eye. The left eye is normal. Which of the following is the most likely diagnosis
in this patient? Orbital cellulitis
○ A 50-year-old man who recently returned from vacation in a tropical area comes to
the office because of multiple white spots on his face and trunk. Physical
examination shows several hypopigmented macules on his face and upper trunk
that are about 4 mm in diameter. A potassium hydroxide preparation shows short
mycelial and round yeast forms and is shown below. Which of the following is the
most appropriate treatment? Selenium sulfide
○ A 65-year-old man comes to the emergency department because of a rash for the
past 2 days. Seven days prior, he had noticed itching and burning pain around his
left shoulder blade which was followed by the development of a painful, vesicular
rash. The rash was treated appropriately. He returns to the office six months later
complaining of a persistent, lancet-like, burning pain in the same area previously
covered by the rash. Physical examination shows no cutaneous abnormalities.
Which of the following is the most appropriate treatment option? Nortriptyline

● PLAYLIST 8
○ A 25-year-old African American woman comes to the clinic because of lesions on
her legs. History reveals the lesions are painful, red nodules that have been present
for the past five days. She also reports increased fatigue, unintentional weight loss,
painful joints and blurry vision. Physical examination reveals a well-developed
female with multiple erythematous macules and nodules on her shins. Which is the
best initial test to detect the most likely diagnosis? Biopsy of the rash
○ A 56-year-old man of Nepalese origin attends the clinic complaining of skin rashes
which have been troubling him for years. On examination, there are numerous
poorly demarcated skin lesions present on all parts of the body. There is also
evidence of significant facial thickening, eyebrow loss, and symmetrical sensory
neuropathy in a 'glove and stocking' distribution. An examination of the hands
reveals bilateral weakness. A skin biopsy is taken from one of the lesions and the
culture is positive for acid-fast bacilli. Which of the following pharmacological
therapies is involved in the treatment of this condition? Dapsone
○ A 24-year-old woman comes into clinic because of her persistent acne, which she
says is not significantly alleviated by over-the-counter acne treatments. She inquires
about Isotretinoin, which her friend has found helpful. While you would not prescribe
any medications to her directly, what should she be counseled about? Use of
contraception to prevent pregnancy
○ A 22-year-old man comes to his primary care provider's office because of facial skin
discoloration for the past 2 years. His paternal uncle has a similar skin abnormality
on his hands. Physical examination shows sharply demarcated patches of
depigmentation around his lips as shown. Which of the following is the next best
step in management of this patient's condition? Topical calcineurin inhibitors
○ A 25-year-old woman comes to the family medicine clinic because of progressive
depigmentation of her hands for the past 3 years. Her past medical history is
significant for type 1 diabetes mellitus. She works as an elementary school teacher.
She reports that initially there were small spots of depigmentation on both of her
hands which progressively involved her whole hand. Physical examination shows
nonpruritic demarcated patches of depigmentation on both hands bilaterally. Which
of the following best describes the pathophysiology of this patients condition?
Autoimmune
○ A 16-year-old boy presents to the dermatology clinic with acne that is uncontrolled
by over-the-counter facial cleansers and lotions. He has 35 lesions on his chest,
back and face, some of which are cystic and painful. He has been lifting weights
heavily for the past six months and trying “to put on muscle mass.” After his
workouts, he drinks milk with whey protein powder for recovery. Which of the
following may be elevated in this patient’s blood? Insulin-like growth factor 1
○ An eight-year-old girl is brought to the office for a check-up. The patient is from Peru
and she has a significant number of hyperpigmented, hyperkeratotic lesions on her
face and on other sun-exposed areas of the body. Ophthalmic exam shows clouded
eyes and corneal ulcers. Her parents say that she has had these lesions since
infancy. She is otherwise healthy and playful. Which of the following is the most
likely diagnosis? Xeroderma pigmentosum
○ A 17-year-old boy comes to the dermatology clinic with moderate acne that has
been persistent over the past five months. It appears inflammatory, with a moderate
amount of mixed papular and pustular lesions and very few comedonal lesions.
Which of the following would be the most appropriate choice for initial treatment?
Topical retinoid, topical benzoyl peroxide, and a macrolide
○ A 32-year-old Caucasian woman comes to her primary care provider's office
because of non-pruritic skin changes for the past year. Her past medical history is
significant for type 1 diabetes mellitus. She states that there have been small
patches of depigmentation of the back of her hands and arms for as long as she
could remember. She also reports of recent constipation and fatigue. She states that
she has felt colder than usual. A lipid panel done 2 years ago showed normal
laboratory values. Physical examination shows a well-appearing female with
widespread macules and patches on her hands, arms, and legs. These lesions are
amelanotic, symmetrical, and measure up to 6 cm in diameter. Which of the
following would be the next bests step in management for this patient? Thyroid
stimulating hormone
○ An 8-year-old boy is brought to the office because of a chief complaint of numerous
coffee-colored spots on his body. Mother of the patient says that he has had them
for years, but they just would not go away. Physical examination shows scoliosis and
ten separate spots, which are larger than 5 mm, in the back. Which of the following
signs is most likely part of the confirmation criteria for this patient's condition? Lisch
nodules
○ An 8-year-old boy is brought to his pediatrician's office by his parents for a well child
check. His past medical history is significant for a congenital melanocytic nevus.
Physical examination shows a nevus which has not grown since the last visit. There
is, however, a new patch of depigmentation surrounding the nevus. The remainder
of the physical examination is unremarkable and the child has met all developmental
milestones. The patient is most at risk for which of the following conditions? Vitiligo
○ A 57-year-old man comes to the emergency department for evaluation of a painful
skin rash. The patient had a mole removed from the tip of his nose last week. He
broke out in a painful facial rash over the weekend and has subsequently been
experiencing fevers and chills. Past medical history is notable for poorly controlled
type II diabetes mellitus, hypertension, lymphedema, and a left kidney transplant for
which he is on tacrolimus. The patient develops hives when he takes penicillin. At
arrival, temperature is 38.0°C (100.4°F), pulse is 104/min, respirations are 16/min,
and blood pressure is 161/82 mmHg. Physical examination is notable for the
following finding: Which of the following is the most appropriate treatment for this
patient’s clinical condition? Intravenous cefazolin
○ A 25-year-old woman with a history of Addison disease comes to the office because
of patches of light skin around her lips and on her hands. The lesions have been
gradually spreading for the past three months. She denies any recent travel or
changes in cosmetics. Diagnostic studies show an elevated anti-melanocyte
antibody level. Which of the following is the most likely diagnosis in this patient?
Vitiligo
○ A six-year-old boy comes to the pediatrician clinic for a scheduled check-up. He and
his parents recently immigrated to the U.S. from Africa. His parents report that he
frequently gets sunburned when playing outside. The parents also say that their
child is otherwise healthy with no other medical issues. Physical examination shows
a well-appearing boy with extremely pale skin, white hair, and blue eyes. Both his
parents are dark skinned. Genetic testing indicates that he has a disorder
associated with an increased risk of developing squamous cell carcinoma. Which of
the following best describes the disease pathophysiology in this patient? Abnormal
tyrosinase enzyme
○ A 5-hour-old male infant is evaluated in the nursery for a skin lesion present at birth.
He was born at term to a 32-year-old woman who had consistent prenatal care. The
lesion is located next to the umbilicus and is shown below. The parents are worried
that it might affect their child cosmetically. They would like to know what is the most
appropriate treatment and prognosis for their child’s condition. Which of the
following is the most appropriate response by the doctor at this time? Observation,
as this lesion is likely to regress without treatment
○ A 26-year-old man comes to the office because of numerous lesions in his trunk and
extremities. Patients medical history is contributory for an autosomal dominant
condition associated with the deletion of a tumor suppression gene on chromosome
17. Which of the following lesions is most likely characteristic in this patient's
condition? Multiple neurofibromas
● PLAYLIST 9
○ A 57-year-old man comes to the office because of a painful rash on his face.
Three days ago he developed a fever and itching on his forehead. The itchy
area became painfull, and earlier today he noticed the appearance of the
rash over the painful area. He currently takes medications for hypertension
and diabetes. Physical examination shows vesicular lesions on his forehead
and face. Which of the following features is the most concerning regarding
the patient's rash? Lesions on the nose
○ A 50-year-old man comes to the urgent care facility with a two-day history of
a progressive, red, painful rash on the left leg. He describes a rapid onset of
pain and rash that started on the anterior shin and spread outward. He also
has had fevers and chills. Physical examination shows a large area below the
left knee of erythema and edema that is warm to the touch and severely
tender. There are patchy areas of unaffected skin. There are no
abnormalities on the right leg. There is maceration in between the toes on
both feet. Which of the following processes is most likely involved in the
development of the patient's symptoms? Spreading infection of Dermi and
subcutaneous tissue by Staphylococcus aureus
○ A 16-year-old girl comes to the office because of skin lesions on her face,
neck and back for 2 weeks. She states that the lesions are itchy, and that she
has also had some ulcers in her mouth over the last 2 months. Her past
medical history includes a seizure disorder that began when she was 12 for
which she takes carbamezapine. Examination shows bullae which are
flaccid. There is crusting of some the lesions and erythema is present at their
base. Which of the following findings is most likely to be found in this
patient? Suprabasal acantholysis
○ A 55-year-old man comes to the office because of burning back pain for 3
months. He states that pain is severe, and that he has never had anything
similar before. He does not recall any back trauma, but did have a rash on
his back when the pain began. He says that the rash has now disappeared,
but produces a photo of what it looked like. Which of the following types of
pain does this patient most likely now have? Neuropathic pain
○ A 54-year-old woman comes to the office for routine follow-up after starting a
new medication. Her past medical history includes type II diabetes mellitus.
At her most recent appointment one month ago, she was found to have
microalbuminuria and was started on captopril. Her only other medication is
metformin. Examination at this visit shows flaccid blisters are present on the
epidermis and oral mucosa. Light stroking of the skin causes the epidermis to
peel away. There are also multiple areas where the blisters have ruptured,
leaving denuded skin. Which of the following antibodies are most likely
involved in the pathogenesis of this disease? IgG that binds to the
desmosone
○ A 53-year-old woman, with a history of type II diabetes mellitus, comes to the
emergency department with a two-day history of febrile illness and a
progressive, red, painful rash on the left lower extremity. The affected area is
warm and tender to the touch. There is no crepitus. The right leg appears to
be unaffected. Which of the following is the most likely causative agent of this
skin disease? Streptococcus pyogenes
○ A 70-year-old man comes to the dermatology clinic because of a blister that
was treated on his right upper arm two weeks ago. He has completed his
course of clindamycin, as prescribed, with little to no improvement. During
the last two weeks, a similar rash has appeared on his right forearm. He
denies any travel, trauma, new hygiene products, or insect bites. He has had
no fever or chills, gastrointestinal symptoms, joint pain, or numbness.
Physical examination of his extremity reveals a large erythematous patch
with clusters of bullae. There is no inguinal lymphadenopthy or mucosal
involvement. Nikolsky sign is negative. Which of the following is the most
appropriate treatment option for this patient at this time? TOPICAL
CORTICOSTEROID
○ A 67-year-old woman comes to the emergency room because of a
widespread, blistering rash. She denies use of any new products or recent
burns. Physical examination shows the affected areas to be negative for
Nikolsky sign, with blisters being hard, tense and difficult to burst. Which of
the following would be the most appropriate first-line treatment of this
condition? Systemic steroids and immunosuppressive agents, such as
prednisone and azathioprine, respectively
○ A 6-year-old girl is brought to the office because of a 5-day history of fever
and malaise. During the past 3 days, she has also developed small blisters
over her face, chest, and extremities that are intensely itchy. She frequently
plays in the woods but has no history of recent travel. She has not had
regular well-child visits but has no personal history of serious medical illness.
Which of the following is the most appropriate next step in management?
Antihistamine
○ A 24-year-old man comes to his primary care provider's office because of a
rash on the back of his head and neck for the past 4 days. He denies
significant sun exposure or any new medications. Examination shows
multiple golden crusts with surrounding erythema on his posterior neck and
scalp as shown below. Impetigo
○ A 32-year-old male comes to the emergency department because of a
blistering rash on his trunk, limbs and mucus membranes for 10 hours. He
states that the rash is itchy, and painful. He also says that when he touches
the blisters they easily break. He is currently taking no medications, but was
treated two months ago treated with an antibiotic. Examination shows bullae
that are are flaccid and erythematous. Which of the following findings is most
suggestive of drug-induced pemphigus? Positive Nikolskys’s sign
○ A 54-year-old man comes to the office because of large, painful blisters and
bright-red lesions on his skin for 4 days. Prior to the presence of the lesions,
he also reports having had multiple, painful oral lesions for the past couple of
weeks. Physical examination shows separation and blistering of unaffected
skin following light rubbing of the area. A punch biopsy and
immunofluorescence are obtained. Which of the following is the most
appropriate first-line treatment for this condition? Oral corticosteroids
○ An 80-year-old woman comes to the emergency department because of
widespread blisters along her abdomen and arms. The patient’s caretaker
denies any recent burns, and her reported past medical history is
unremarkable aside from an upper respiratory infection a few weeks ago.
The blisters are tense, and filled with either blood-stained or clear fluid. They
appear to be pruritic, as evidenced by the patient’s frantic scratching and the
presence of multiple scratch marks along the blisters and adjacent skin.
Rubbing of the blister and neighboring skin did not result in breakage or new
blister formation. Her wounds are dressed, and the patient is prescribed both
oral prednisone and topical corticosteroids. Which of the following is the most
likely pathophysiology of this disease? Autoantibodies attack Type XVII
collagen, causing a split in between the dermis and epidermis
○ A 73-year-old man comes to the clinic because of itchy and sensitive skin,
which started acting up a few weeks ago. He believes that he has been
having allergic reactions, along his abdomen to a new lotion his wife had
recently purchased. He is worried because over-the-counter allergy
medications have not been working, and blisters began forming along the
lesions. Physical examination reveals multiple, stable blisters and hive-like
lesions along the abdomen. Adjacent skin was negative for Nikolsky sign.
Which of the following would most likely be seen on a skin biopsy?
Histological analysis shows subepidermal bullae with dense, eosinophilic
infiltrate
○ A 55-year-old woman with a history of type 2 diabetes mellitus, comes to the
emergency department because of a 2-day history of a progressive, red,
painful rash on the left leg. Physical examination shows a large area below
the right knee of erythema and edema that is warm to the touch and severely
tender. There are patchy areas of unaffected skin. There is no drainage or
discharge associated with the rash. The left leg has no abnormalities. There
is maceration in the web spaces of the toes on both feet. Which of the
following empirical antimicrobial therapies would be best for this patient?
Cephalexin
○ A 22-year-old man comes to the clinic because of a lesion on his lip. The
lesion started with tingling and redness 4 days ago followed by increasing
pain, emergence of a cluster of vesicles which then burst to create a weeping
ulcer that has started to crust. Which of the following is true of his current
lesion? Oral antiviral therapy is unlikely to provide benefit
○ A 16-year-old boy presents to the emergency department for evaluation of
right middle finger pain. Several days ago, the patient noticed small fluid-filled
sacs in the right middle finger, which progressed into painful lesions
yesterday. His mother, who is accompanying him, states, “He has got a finger
infection from chewing on his nails all day.” The patient is otherwise healthy
and takes only a multivitamin daily. Temperature is 37.0°C (98.6°F), blood
pressure is 115/73 mmHg, and pulse is 78/min. Physical examination is
notable for the following finding: Which of the following is the most
appropriate treatment for this patient’s clinical condition? Acyclovir
○ A 60-year-old woman comes to the emergency department because of pain
in her right eye and a burning sensation over half of her forehead and scalp.
Physical examination shows a linear patch of grouped vesicles on an
erythematous base on her scalp and forehead. There are also a few vesicles
located at the tip of the patient’s nose. The rest of her examination shows no
abnormalities. Which of the following is the most concerning complication of
this patient’s clinical presentation? Ophthalmic involvement leading to
anterior uveitis or corneal scarring

● PLAYLIST 10
○ A 3-year-old child is brought to the emergency department by his mother because of
a 6-day high fever. The fever has not responded to treatment with paracetamol. His
temperature is 39°C (102.2°F). Examination shows an irritable appearing child with
cracked lips and peeling skin on his fingertips. There is also injected conjunctiva
without discharge, a swollen tongue, swollen cervical lymph nodes, and a
maculopapular rash on his trunk. Which of the following complications is this child
most likely at risk of if prompt treatment is not initiated? Coronary artery
aneurysm
○ A 35-year-old woman comes to the clinic for a health maintenance exam. She states
she is currently feeling well. Current medications include a lipid-lowering agent and
a corticosteroid. Her temperature is 37.1°C (98.8°F), pulse is 76/min, respirations
are 18/min, and blood pressure is 132/84 mm Hg. Physical examination is
unremarkable except for a malar rash, which she reports is exacerbated by sun
exposure. For which of the following sequelae is this patient at increased risk?
Osteonecrosis of the femoral head
○ A 70-year-old woman comes to the emergency department because of painless
vision loss in the right eye. She reports a 2-month history of difficulty dressing due to
shoulder stiffness. She has also experienced right-sided headaches and jaw
tightness when eating. Past medical history is significant for carotid atherosclerosis,
diabetes mellitus, and hypertension. Vitals are within normal limits. Physical
examination shows tenderness to palpation on the right side of the head. Visual
acuity in her right eye is 20/200, and a right afferent pupillary defect is noted.
Laboratory studies show an erythrocyte sedimentation rate of 65 mm/h. Which of the
following is the most appropriate treatment for this patient’s current condition?
Methylprednisolone
○ A 56-year-old woman comes to the hospital because of a unilateral headache, jaw
claudication, and transient vision loss. Laboratory studies show an erythrocyte
sedimentation rate of 89 mm/h. Empiric treatment with glucocorticoids is initiated,
and an MRI with contrast is obtained. Which of the following is the most appropriate
next step in diagnosis? Histology
○ An 85-year-old woman comes to the emergency department because of a throbbing
right-sided headache for the past 3 hours. She describes exquisite tenderness over
her right temple without associated nausea or photophobia. Physical examination
shows a pulsating right temporal artery. Which of the following conditions is the
patient at an increased risk for developing? Vision loss
○ A 16-year-old girl comes to the clinic with a 9-month history of persistent fevers,
malaise, and headache. Recently, she has noticed that her urine is occasionally
dark or brown in color. She also notes new onset shortness of breath when
exercising. She denies any illicit or prescription drug use. Her temperature is 38.3°C
(101.0°F), pulse is 70/min, respirations are 16/min, and blood pressure is 110/76
mm Hg. Physical examination shows a faint erythematous rash over the upper
cheeks and the bridge of the nose. Microscopy of a urine sample collected today is
notable for the presence of several RBC casts. Which of the following is the most
appropriate next diagnostic test? Anti-dsDNA and anti-Smith serum antinuclear
antibodies
○ A 65-year-old woman comes to the office because of a 3-month history of neck and
shoulder pain. She initially noticed symptoms only on the left side but has
experienced bilateral pain and stiffness for the past two months. The stiffness is
worst in the morning and lasts more than an hour. She reports a 4.5-kg (10-lb)
weight loss during the past three months. Physical examination shows swelling of
shoulders with limited range of motion. Muscle strength is normal in all extremities.
Laboratory studies show an erythrocyte sedimentation rate of 70 mm/h. Which of the
following is the most appropriate next step in management? Administration of
prednisone
○ An 80-year-old woman comes to the emergency department because of pain and
tenderness over the left temple for the past 3 hours. She reports a 2-month history
of episodic jaw pain during eating and occasional episodes of vision loss. She has a
40-year history of hypertension well controlled with lisinopril. Which of the following
is the most appropriate next step in management?. Glucocorticoid therapy
○ A 4-year-old boy is brought to the emergency department by his mother because of
a 1 day history of rash. She says that he had Streptococcal pharyngitis several
weeks ago, but was in his normal state of health until 5 days ago when he began
complaining of right hip pain. This subsided and was replaced by right knee and
ankle pain one day ago. At the same time, he developed a rash of dark red spots
scattered on his legs, buttocks, and forearms. He also began complaining of nausea
and stomach pains 2 hours ago. Physical examination shows a purpuric rash
located in dependent areas, and a swollen right knee without effusion, erythema, or
warmth. Urinalysis shows microscopic hematuria with proteinuria. Serum
electrolytes, including creatinine, are within normal limits. Which of the following is
the most likely diagnosis? Henoch-Schönlein Purpura
○ A 32-year-old man comes to the clinic because of a 1-year history of pain in his feet
and hands. The pain now occurs at rest, and worsens with activity and exposure to
cold temperatures. He also describes symptoms suggestive of Raynaud
phenomenon. He currently smokes 20 cigarettes per day. His wristand foot pulses
are weak, and he has a positive Allen test. Diabetes mellitus and autoimmune
disease are excluded. An angiogram is obtained and shows corkscrew collateral
vessels around segmental occlusions in the small and medium arteries of the feet
and hands, no atherosclerosis, and no aneurysmal disease. Which of the following
is the most appropriate next step in management? Advising and supporting
smoking cessation
○ A 7-year-old boy comes to the clinic because his mother has noticed "bumps on his
bottom" since last week. He has no major past medical history. He is unvaccinated
due to his parents' personal preferences. He says that both of his knees hurt him.
His mother reports that he has had a stomach ache that has come and gone a few
times since yesterday. Physical examinations shows diffuse tenderness of the
abdomen to palpation, but no distention or masses. A dermatologic survey reveals
palpable purpura over the buttocks and upper thighs. Which of the following is the
most important sequela of the pathology affecting this patient? Intussusception
○ A 38-year-old woman comes to the emergency department because of shortness of
breath and cough, productive of blood-tinged sputum for 3 days. She has been
unresponsive to antibiotic therapy from her family doctor. Her temperature is 38.0°C
(100.4°F), pulse is 92/min, respirations are 18/min, and blood pressure is 110/68
mm Hg. Laboratory studies show an elevated erythrocyte sedimentation rate (ESR)
and circulating cytoplasmic anti-neutrophil cytoplasmic antibodies (c-ANCA).
Urinalysis shows dysmorphic red cells and red blood cell casts. Which of the
following antibodies is most likely to be found in the patient's serum?
Anti-proteinase-3 antibodies
○ A 70-year-old woman comes to the emergency department because of painless
vision loss in the right eye. She reports a 2-month history of difficulty dressing due to
shoulder stiffness. She has also experienced episodic right-sided headaches and
jaw tightness when eating. Physical examination shows tenderness to palpation on
the right side of her head. Visual acuity in her right eye is 20/200, and a right afferent
pupillary defect is noted. Laboratory studies show an erythrocyte sedimentation rate
of 65 mm/h. Which of the following is the most appropriate next step in
management? Methylprednisolone therapy
○ A 60-year-old woman comes to the office because of progressively worse
headaches and blurred vision over the past week. She also reports a history
of low-grade fevers during the past month. She has a 20-year history of
systemic lupus erythematosus. Physical examination shows tenderness to
palpation in the temporal area. Laboratory studies show serum alkaline
phosphatase activity of 140 U/L and erythrocyte sedimentation rate of 55
mm/h. Temporal artery biopsy shows no abnormalities. Ultrasonography
shows a halo sign. Which of the following is the most appropriate next step
in management? Administration of glucocorticoids
○ A 65-year-old woman comes to the office because of a 1-month history of fever,
malaise, and weight loss. During the past 3 months, she has experienced pain and
stiffness in her hips and shoulders, especially when she wakes up. She reports
occasional unilateral headaches and worsening vision. She denies any history of
head trauma. Neurological examination shows no abnormalities. Administration of
which of the following is the most appropriate next step in management?
Prednisone
○ A 33-year-old man comes to the clinic because of new skin lesions for the past 9
months. The patient says that the skin lesions have been growing in number and
that they tend to blister and then form a black center. Medical history is contributory
for asthma for which he requires an albuterol inhaler as needed. Physical
examination shows bleeding areas on the arms and legs with a central black dimple.
Laboratory test shows 11% of eosinophils in the blood. Which of the following
serologies is most likely to be positive or elevated in this patient's condition?
P-ANCA
○ A 64-year-old woman comes to the emergency department because of sinusitis, a
runny nose, and hemoptysis for the past 3 months. The nasal drainage is often
purulent and occasionally contains blood. The patient has also experienced new
joint pains and fatigue during the same time-period. Examination shows tenderness
to percussion over the maxillary sinuses; however, is otherwise non-contributory.
Urinalysis shows 3+ microscopic haematuria and 2+ proteinuria. Chest X-ray is
shown below. What is the most likely diagnosis? Granulomatosis with polyangiitis

● PLAYLIST 11
○ A 68-year-old man comes to the emergency department because of a macular rash
on his palms and soles. He is treated with a high dose of antibiotic administered
intramuscularly. In the days following his treatment, he experiences headache and
malaise followed by the development of red splotches on his skin, which soon form
blisters. He returns to the emergency department because he is unable to eat due to
oral lesions and is admitted for supportive care. Which of the following is the most
likely type of hypersensitivity experienced by the patient? Type IV
○ A 20-year-old man comes to the clinic with a rash that began on his hands and feet
that has progressed to his forearms and legs. He also complains of sore, cracked
lips and gums. About 3 days prior, he began having muscle aches, low-grade fever,
and headache. He then began itching and burning, and the rash appeared.
Examination shows scattered bullae and erosions in his oral cavity, in addition to
multiple, annular, symmetric erythematous lesions covering his palms, backs of his
feet, and extensor surfaces of his forearms and legs. The center of each lesion has
a blister. Which of the following is the most common trigger for this condition?
Herpes simplex virus infection
○ A 35-year-old man comes to the emergency department because of a burning rash
on his neck, face and upper torso. He also experiences sensitivity to light. He admits
to feeling flu-like symptoms including fever, arthralgias and headache one week ago.
His past medical history is significant for bipolar disorder, post-traumatic stress
disorder and anxiety. He was started on a new psychiatric medication three weeks
ago, of which he cannot recall the name. Physical examination shows conjunctival
injection and inflamed oral mucosa. Which of the following drugs is the most likely
cause of his current symptoms? Lamotrigine
○ A 73-year-old woman comes to the emergency because of a dangerous cutaneous
drug reaction. Physical examination shows confluent erythema and epidermal
sloughing with application of lateral pressure. 40% of her body surface area is
involved, including her eyes and oral mucosa. She is transferred to the burn unit in
critical condition. Which of the following conditions is the most life-threatening in this
patient? Sepsis
○ A 26-year-old man comes to the clinic because of a burning rash, fever, productive
cough and headache. The rash began on his face three days ago and spread to his
upper torso. On examination, you note lesions of two colors that have a target-like
appearance. The oral mucosa is involved. Which of the following findings can most
likely distinguish the condition seen in this patient from the self-limited condition
known as erythema multiforme? Concentration on the skin of the face and trunk
○ A 25-year-old woman comes to the clinic because of a skin condition of sudden
onset in her extremities. Examination shows target-like lesions on her arms and
legs, including the palms and soles of her feet. The lesions are a hypersensitivity
reaction that is self-limited. Which of the following might be expected on a more
detailed history? History of herpes simplex virus infection
○ A 24-year-old woman comes to the emergency department because of urinary
frequency, burning sensation on urination, and pelvic tenderness. A urine culture
shows a lactose-positive, gram-negative rod. Further testing shows a negative
catalase test and positive indole test. The patient is prescribed a paraaminobenzoic
acid (PABA) analog that functions as a competitive inhibitor of dihydropteroate
synthetase and is discharged home. Four days later, she returns with complaints of
a burning rash covering her entire chest, abdomen, and back. Which of the following
conditions is the most concerning for this patient? Toxic epidermal necrolysis
○A 49-year-old HIV-positive woman comes to the emergency department because of
fever and fatigue for the past 10 days. She has also experienced odynophagia and
"feeling like her skin is burning" for the past day. She denies current or recent history
of headache or cough, but says that she had been treated two weeks ago with
trimethoprim-sulfamethoxazole for a urinary tract infection. Her temperature is
39.5°C (103.1°F), pulse is 80/min, respirations are 15/min, and blood pressure is
125/80 mm Hg. Oral examination is below and shows mucosal abnormalities.
Significant conjunctivitis with a purulent discharge is noted on ocular examination
bilaterally. Skin examination shows confluent, erythematous, nonpruritic target
lesions with associated blisters on her upper arms and a positive Nikolsky sign.
Scalp, palms, and soles appear to be spared with no desquamation noted. The rest
of the examination shows no abnormalities. Which of the following is the most likely
diagnosis? Stevens-Johnson syndrome
○ A 32-year-old man comes to the office because of a large, painful, erythematous
nodule on his right shoulder. Incision and drainage is performed, and the patient is
discharged with empiric antibiotic therapy sufficient to cover methicillin-resistant
Staphylococcus aureus (MRSA). 3 days into his 10-day course of antibiotics, he
comes to the emergency department because of a burning sensation on his back,
chest and face as well as peeling of large areas of skin. Which of the following is
most consistent with the cellular pathophysiology of the peeling skin? Fas-ligand
activation
____________________________________________________________________________

URGENCIAS

● PLAYLIST 1
○ A 71-year-old man arrives at the emergency department after his friend found him
unconscious in his apartment. You begin by evaluating his conscious state using the
Glasgow Coma Scale (GCS). You call the patient by his name and get no response.
After rubbing your knuckles against his chest, he opens his eyes and closes them.
You then ask him a question, the patient doesn't utter a word or sound. Finally, you
place pressure over the patient's left index finger nail bed. He moves his hands
away due to the stimulus. What's this patient's Glasgow Coma Scale (GCS) score?
GCS 7
○ A 35-year-old man comes to the emergency department for a worsening headache.
He was drinking heavily this evening when he fell over and hit his head. He got up
immediately and felt fine, however over the ensuing hours his headache worsened.
Physical exam shows a dilated left pupil. A head CT is obtained, the results of which
are shown below. Which of the following sets of vitals would most likely be seen in
this patient? Temperature: 37.5ºC, pulse: 55/min, blood pressure: 154/90 mm
Hg, respiratory : 20/miA 40-year-old woman presents to the emergency
department complaining of fever. Her past medical history reveals diabetes and
morbid obesity. Vitals reveal a blood pressure of 80/60 mmHg, a heart rate of
140/min, a respiratory rate of 35/min, an oxygen saturation of 94% on room air, and
a temperature of 40°C (104°F). Urine analysis indicate positive for UTI. Sepsis is
suspected. A right internal jugular central venous catheter is placed without
complication under ultrasound guidance for vascular access in preparation for
administration of vasopressors. The patient’s blood pressure has improved to
100/70 mmHg with saline running through a peripheral line. A chest radiograph is
obtained subsequent to placement of the central line placement. Which of the
following is the best next step in management in this patient subsequent to seeing
this X-ray image? Remove the central line after confirming the correct
placement of a new central line
○ A 27-year-old man comes to the emergency department because of a blunt head
trauma. The paramedic stated that the patient was hit on the temple with a baseball
bat by a woman claiming to be acting in self-defense. The man lost consciousness
for about 2 minutes after he was struck but then regained consciousness and was
asymptomatic for a while before becoming stuporous and then progressively
unresponsive. Which of the following is the most likely appearance on this patient's
head CT scan? Biconvex lens shaped hematoma
○ A 20-year-old male presents to the emergency department after being involved in a
motor vehicle accident as an unrestrained passenger. Upon arrival, the patient
becomes obtunded and goes into respiratory distress requiring intubation and
mechanical ventilation. A head CT is obtained as shown in the exhibit. Neurory
evacuates the epidural hematoma and places an intracranial pressure (ICP)
monitoring probe which reveals that his ICP remains elevated. Post-op vital signs
include a temperature of 36.9ºC (98.4ºF), a blood pressure of 120/80 mmHg, a heart
rate of 89/min, a respiratory rate of 14/min, and an oxygen saturation of 100% on
30% FiO2. Which of the following would be an appropriate treatment to lower this
patient's intracranial pressure? Increase the patient's respiratory rate so that his
PaCO2decreases to 26-30 mmHg
○ A 55 year-old man comes to the emergency department because of a head injury
sustained at work. The patient is a construction worker and he fell from a ladder
from a 3 m height. He lost consciousness for about 30 seconds and then was
asymptomatic for a few hours before becoming confused and increasingly
unreponsive. Head CT scan shows a 40 mm x 30 mm x 10 mm hematoma with a
convex shape and a 2 mm midline shift. Which of the following would confer the best
prognosis for this patient? Glasgow Coma Scale score > 8
○ A 31-year-old man comes to the emergency department because of blunt trauma to
his perineum and penis from falling off a ladder. He fell approximately 5 feet and
landed on concrete, resulting in immediate pain lasting several minutes. Since the
injury, the patient experiences burning with urination and notes large amount of
blood mixed with urine. He denies pain with ambulation and does not recall any
trauma to the pelvis during the fall. Genitourinary examination shows blood at the
urethral meatus. Which of the following is the most appropriate study to order?
Retrograde urethrogram
○ A 30-year-old man is brought into the emergency room after an automobile accident.
His blood pressure is 110/80, pulse 66, respiratory rate 16, temperature 36.9. He
has a scalp laceration over his left temple as well as other cuts and bruises.
On physical exam, he opens his eyes to nearby, loud speech, responds to speech
with incoherent, isolated words, and tries to avoid a painful pinch to his nailbed by
pulling away from it. You order a CT of his head and ask to be paged when the CT is
complete.
Fifteen minutes later, before the CT has happened, the nurse calls you to report that
the patient seems worse. He no longer opens his eyes in response to nearby, loud
speech but does open his eyes when a painful pinch is applied to his nailbed.
Further, when his nailbed is pinched, he flexes his forearm but no longer pulls away.
He is no longer speaking but rather moaning.
What is the next step in management? Endotracheal intubation
○ A 35-year-old man comes into the emergency room after a motor vehicle accident.
His chart shows a pulse of 56, blood pressure of 160/90, respiratory rate of 16, and
pulse oximetry of 98% on room air. You find him lying flat on a bed in the ER. He is
oriented to person but not to place or time. You note that his speech seems odd
because he is breathing very irregularly. What is the next step in management?
Order stat head CT and prepare to intubate
○ A 73-year-old man comes to the emergency department because of severe eye pain
and loss of vision in his left eye for an hour. He states that he has never had
symptoms like this before, and has vomited twice. He takes hydrochlorothiazide and
tamsulosin for hypertension and benign prostatic hyperplasia. Ophthalmic
examination shows conjunctival injection and a hazy cornea. Intraocular pressure is
elevated. Which of the following structures is most likely obstructed? Trabecular
meshwork
○ A 60-year-old man comes to the emergency department because of a severe
headache for 30 minutes. He describes a left-sided headache and eye pain and
says he has vomited twice. The patient has a long history of migraines, but says his
migraines do not usually include eye pain. His temperature is 36.5°C (97.7°F); pulse
is 84/min; respirations are 12/min, and blood pressure is 134/86 mm Hg. His
neurologic examination is normal. Ophthalmic examination shows his left eye is
mid-dilated and nonreactive. His left cornea is also cloudy. His best corrected visual
acuity is 20/50 in the left eye and 20/20 in the right eye. Which of the following is the
most appropriate next step? Check intraocular pressure
○ A 15-year-old male sustains a head to head collision during a football game. Vital
signs reveal a respiratory rate of 24/min. Physical examination in the field reveals
the patient is unconscious for a time and responds minimally to verbal stimuli.
Respirations are labored. Which is the correct initial management of this patient?
Remove the face mask, leaving the helmet in place, followed by proper airway
management
○ A 66-year-old woman comes to the Emergency Department because of a
decelerating motor vehicle accident that resulted in injury to her neck. Her
temperature is 37.1°C (98.8°F), pulse is 90/min, respirations are 20/min, and blood
pressure is 130/85 mm Hg. Physical examination shows bilateral loss of pain and
temperature sensation in her upper extremities with triceps and biceps reflexes are
weak bilaterally and strength 2/5. Lower extremity reflexes are present, with 4/5
muscle strength. Which of the following is a risk factor for the most likely diagnosis?
Cervical spondylosis
○ A 21-year-old woman comes to the emergency department because of bilateral eye
pain, tearing,and photophobia. These symptoms began about 2 hours ago with
sudden onset. The house the patient lives in recently installed a sun-tanning station.
Physical examination shows normal visual acuity, intact extraocular eye movements,
and pupils are equal, round, and reactive to light. Intra-ocular pressure is within
normal limits. Slit-lamp examination is normal, but fluorescein examination under
cobalt blue light illuminates small dots throughout the cornea. Which of the following
is the most likely diagnosis? Photokeratitis
○ A 55-year-old male presents to the emergency department after falling from a
ladder. The patient was standing on a six-foot ladder when he suddenly lost
balance, fell, and landed on his right side. He currently reports pain in the shoulder
and over the right rib cage. He does not take any medications and does not have
other medical conditions. Temperature is 37.0°C (98.6°F), pulse is 101/min, and
blood pressure is 132/63 mmHg. Respiratory rate is 20/min and oxygen saturation is
92% on room air. Physical examination reveals decreased breath sounds at the right
lung base, as well as a 5 cm x 5 cm area of ecchymosis over the lateral rib cage. A
chest CT is obtained, and the results are shown below: Fourth or fifth intercostal
space midaxillary line
○ A 16 year-old boy comes to the emergency department after falling from a wall. The
patient and several friends were sitting on a stone wall when the patient lost his
balance and fell forward. The patient admits to a short loss of consciousness, but
then "felt fine." Examination shows slight drowsiness and confusion. Which of the
following diagnostic tools is the least useful in ruling out an emergency in this case?
Lumbar puncture
○ A 30 year-old woman comes to the emergency department because of a persistent
severe headache. While skiing today, she fell and hit the side of her head against a
rock. She lost consciousness briefly and then got up. Her friends, worried about her
progressive headache, dizziness, and nausea, and told her that she should come
and get checked out. Examination shows ipsilateral pupil dilation and crepitus over
the left temporal region. The patient becomes increasingly lethargic throughout the
examination. Which of the following investigations is the most appropriate in
confirming the most likely diagnosis? Head CT without contrast
○ A 25-year-old man is brought to the emergency department after a motor vehicle
collision. Upon arrival to the hospital, his vital signs are within normal limits, but he
experiences some post-traumatic amnesia and complains of a headache and
nausea. He believes that he may have lost consciousness for 1-2 minutes. A
computed tomography (CT) scan is obtained and is shown below: concussion
○ A 30-year-old man is brought to the hospital after being involved in a motor vehicle
collision. He was an unrestrained passenger in a vehicle that was hit when another
vehicle ran a red light. He is unconscious and intubated on the scene. He presents
to the hospital with a pulse of 120/min and blood pressure of 85/40 mm Hg. Focused
assessment with sonography for trauma (FAST) is positive for blood within the
peritoneum. He is rushed to the operating room for an exploratory laparotomy. Prior
the procedure the anesthesiologist places a central venous catheter to assist with
rapid fluid and blood product administration. Which of the following is the best
technique to place a subclavian central venous catheter? Insert the needle inferior
to the midpoint of the clavicle, pointing the needle medially
○ A 67-year-old man comes to the emergency department after a fall at home walking
to the kitchen from the living room. His daughter reports that he fell on his head, and
later reported a very bad headache. The initially drowsy patient becomes comatose,
and his breathing becomes apneic. His systolic blood pressure rises steeply. Which
of the following findings would also be associated with Cushing triad? Bradycardia
○ A 50-year-old man comes to the emergency department because of injuries after
falling off a ladder. He describes falling about 5 feet off the ground and suffering
blunt trauma to his perineum and penis, resulting in immediate pain lasting 5
minutes. Following the injury, the patient experienced burning with urination and
noted large amounts of hematuria. He denies pain with ambulation and does not
recall any trauma to the pelvis during the fall. Genitourinary examination reveals
blood at the urethral meatus. Which of the following is the most appropriate initial
step in management? Placement of a suprapubic catheter
○ An 85-year-old woman comes to clinic with an unresponsive, dilated pupil. She
reports falling two days ago while trying to walk up the stairs to her house. She felt
"fine" afterward and did not feel that she required any treatment. Earlier today, a
visiting nurse noticed that the patient's pupils were different sizes and sent her to the
clinic. Which of the following additional findings, if present, most strongly indicates
that the patient should undergo surgery? The patient begins vomiting
○ A 24-year-old woman comes to the emergency department because of diplopia,
headache, vertigo and hearing loss in her right ear after falling down several flights
of stairs. Physical examination shows clear rhinorrhea, which makes a "halo sign"
when it contacts the fabric of her pillow. Which of the following additional conditions
is most commonly associated with this type of injury? Epidural hematoma
○ A 42-year-old woman who was hit by a car while riding her bike presents to the
emergency department with a pulse of 100 and a BP of 90/75. She is complaining of
pain in the left shoulder and wrist, with the latter showing an obvious deformity. A
FAST scan is positive for free fluid in the abdomen. Which of the following organs is
most likely injured? Spleen
○ A 75-year-old female was brought to the emergency department by EMS. The son
arrived shortly and reports that he found the patient unconscious at her home.
History reveals the patient was initially unconscious when paramedics arrived.
Physical examination reveals the patient to be confused and disoriented. Patient
opens her eyes spontaneously and obeys commands. Which is the most
appropriate Glasgow coma scale score in this patient? 14
○ A 45-year-old man is wheeled into the operating room for a surgical evacuation of
an epidural hematoma. Which of the following hemodynamic changes would be
indicators concerning for an increase in intracranial pressure (ICP)? Hypertension
and bradycardia
○ A 7-year-old boy comes to the emergency department with his mother. His mother
says he dove head-first into a shallow pool and was unconscious for a period of
thirty seconds. Upon awakening, the patient had one episode of emesis. The boy
does not recall the event. Physical examination reveals a confused male with
several small superficial lacerations on his scalp. A computed tomography of the
patient's head reveals no intracranial pathology.
Based on the patient's history, symptoms, and imaging, which of the following is the
most likely diagnosis? Concussion
○ A 30-year-old man is brought into the emergency room after an automobile accident.
His blood pressure is 110/80, pulse 66, respiratory rate 16, temperature 36.9 C. He
has a scalp laceration over his left temple as well as other cuts and bruises.
On physical exam, he opens his eyes to pain, responds to speech with incoherent,
isolated words, and tries to avoid a painful pinch to his nail-bed by pulling away from
it.
Which of the following developments would indicate a decrease in his Glasgow
coma scale score? The patient begins to extend his limbs in response to a
pinched nailbed instead of pulling away from it.
○ A 45 year-old woman comes to the emergency department because of a severe
headache following a skiing accident. Immediately after the fall, she refused medical
evaluation because she had no difficulty with speech or movement and "felt fine."
Hours later she developed a severe headache and was taken to the hospital, but
despite medical intervention she passed away. Which of the following is the most
likely cause of this patient's symptoms? Epidural hematoma caused by a middle
meningeal artery tear
○ A 15-year-old male comes to the office because of a sore throat and fever for two
days. He says his throat started hurting several days ago and he has been
experiencing increased fatigue. His medical history is noncontributory, and he has
no allergies. His temperature is 38.5°C (101.3°F), pulse is 87/min, respirations are
18/min, and blood pressure is 117/78 mm Hg. Physical examination shows a
erythematous posterior pharynx, tonsillar edema with exudates, palatal petechiae,
and several enlarged posterior cervical lymph nodes. Rapid group A beta-hemolytic
streptococci screen and Monospot tests are performed in the office and both are
negative. The patient begins course of amoxicillin, but calls the next day saying that
he has developed a diffuse rash. Of the following potential complications associated
with this patients condition, which most commonly results in death? Splenic rupture
○ An 18 year-old man comes to the emergency department after being mugged and
hit on the head with a blunt object. He lost consciousness for approximately 10
minutes, then woke up and walked home. He was fully conscious for 20 minutes but
then gradually became lethargic. Which of the following is the most likely site of his
associated skull fracture? Temporal region
○ An 88-year-old woman falls in her bathroom and is taken to the emergency room.
Before the fall, neighbors and friends attested to the fact that she was very active
and a very healthy woman. After the fall, she complains of fatigue and drowsiness,
and often stares at the wall with a blank stare during the interview. Neurological
exam discloses no lateralizing or focal findings. Plain x-ray of the skull was found to
be negative. Complete blood count, comprehensive metabolic panel, and TSH
levels were all within normal limits. Which of the following is the most likely
diagnosis? Subdural hematoma
○ A 53-year-old man is brought to the clinic because of a severe headache and
decreased mental status. 20 minutes later patient is obtunded. Medical history is
contributory for hypertension for the past 14 years. Ophthalmic examination shows
anisocoria and a blown pupil. CT findings contraindicate the use of thrombolytics.
Which of the following is the most likely diagnosis? Uncal herniation
○ A 47-year-old man comes to the emergency department because of a
sudden-onset, exploding headache. He states that the pain developed in seconds
and that he has never had anything like this before. Medical history includes
uncontrolled hypertension. Examination shows no focal neurological deficits at the
time of arrival. A non-contrast CT scan of the head is obtained. Which of the
following is the most common complications of this condition? Vasospasm
○ A 50-year-old man comes to the emergency department because of a leg injury
following a motor vehicle accident. His past medical history is significant for
hypertension and hyperlipidemia. Physical examination shows minor contusions and
a tibia-fibula compound fracture, requiring surgery. Post-operatively, the patient
experiences severe abdominal pain and the passing of an abundance of bright red
blood and mucus from his rectum. An emergent abdominal X-ray shows
thumb-printing in the colon and dilation of the small intestine. Which of the following
is the most likely structure affected? Splenic flexure
○ A 30-year-old male presents to the emergency department after being found by
paramedics on a sidewalk below a three-story balcony. History reveals the patient
was drinking on the balcony and fell over the side. Physical examination reveals the
patient opens his eyes when his name is called, but has no verbal or motor
response to painful stimuli. Which is the most appropriate Glasgow Coma Scale
score in this patient? 5
○ A 43-year-old man comes to the emergency room because of sharp chest pain that
is worse on inspiration and shortness of breath for 10 minutes. His symptoms
developed after he crashed his motorcycle. Witnesses say that he hit his chest on
the front of the bike before coming off the bike and landing on his back. Examination
shows that he is fully alert, oriented, and can move all four extremities. Chest X-ray
is obtained and is shown below. Which of the following is the most appropriate first
step in treatment? Insertion of a chest tube
○ A 25-year-old man comes to the emergency department because of a penetrating
stab wound to the lower abdomen. He is diaphoretic and tachypneic. His blood
pressure is 70/40 mm Hg. Physical examination shows a distended and diffusely
tender abdomen. Intravenous fluids are administered. Which of the following is the
most appropriate next step in management? Laparotomy
○ An 82-year-old man is brought to the emergency department because he suddenly
started "acting strange during dinner". His son says that 45 minutes ago he was
"completely normal". Medical history is noncontributory. His temperature is 37.8°C
(100°F), pulse is 75/min, respirations are 12/min, and blood pressure is 160/90 mm
Hg. Oxygen saturation is 98% on room air. Physical examination shows a right facial
droop and right arm weakness, as well as an expressive aphasia. Laboratory
studies show an INR of 1.0 IU and undetectable troponin levels. Electrocardiogram
is interpreted as within normal limits. A non-contrast head CT scan is obtained and
is read as normal. Which of the following is the most appropriate next step in
management? Give IV tissue plasminogen activator
○ A 15-year-old boy comes to the emergency department because of dazed feelings.
These feelings began after he was hit in the head during football practice. The
impact to his head gave him a "ringing" sensation, after which he felt a bit dazed,
found it difficult to think clearly, and felt his vision had become blurred. He also felt
out of balance. His Glasgow coma score is 15. He did not lose consciousness or
vomit. Which of the following is the most appropriate next-step in management?
Refrain from football practice

● PLAYLIST 2
○ … the best additional pharmacological intervention for this patient's condition?
Dantrolene
○ A 53-year-old male comes to the emergency department because of extremely high
fever. He is diagnosed with severe sepsis and his state continues to depreciate.
After 6 days, he becomes unresponsive to painful stimulus and brain death is
suspected. Which of the following is not associated with the evaluation of brain-stem
reflexes? Withdrawing O2 supply until PaCO2 is above 50 torr
○ … Her temperature is 37.0°C (98.6°F), pulse is 68/min, respirations are 18/min, and
blood pressure is 118/78 mm Hg. Her medications include levothyroxine,
prednisone, acetaminophen, and sertraline. After beginning intravenous
maintenance fluid administration, the first-year resident on duty notices her severe
hyponatremia (100mmol/L) and gives her a rapid infusion of hypertonic saline. The
patient is discharged but returns three days later in an acutely altered condition.
Physical examination shows that the patient is unable to speak, is
○ disoriented, and cannot move her legs. Which of the following most likely explains
this series of events? Osmotic demyelination syndrome

● PLAYLIST 3
○ A 25-year-old man comes to the emergency department because of
dizziness and decreased urination during the past 2 days. He joined the
army and just started the first week of summer boot camp. He appears
lethargic. His pulse is 120/min, respirations are 18/min, and blood pressure
is 100/60 mm Hg. Physical examination shows sunken eyes and chapped
lips. Serum studies show a urea nitrogen concentration of 45 mg/dL and a
creatinine concentration of 2 mg/dL. Urine studies show a fractional
excretion of sodium of 0.8%. Which of the following abnormalities is the
most likely cause of this patient's condition? Hypovolemia
○ A 45-year-old woman in acute distress is brought into the emergency
department by her son because of altered mental status beginning an hour
ago. The patient is rapidly deteriorating, and her son says that she has been
having respiratory problems for the past 2 days. Physical examination
shows tachycardia, tachypnea, and diffuse purpura. The patient was given
two boluses of normal saline, and her temperature is 39°C (102.2°F), pulse
is 110/min, respirations are 35/min, and blood pressure is 88/45 mm Hg.
Pulse oximetry on room air shows an oxygen saturation of 86%. Laboratory
studies show an anion gap acidosis, with a venous lactate of 6.5 mmol/L.
The patient is started on intravenous antibiotics, non-invasive positive
pressure ventilation, and fluid resuscitation with crystalloids. Which of the
following parameters when re-evaluated serially over time is the best
option for guiding further resuscitation efforts? Venous lactate
○ A 26-year-old man is rushed to the emergency department following a
gunshot wound shot to the chest. The wound is 2 cm below the right nipple.
His initial blood pressure was 80 / 10 mm Hg, but he has since responded to
3L of crystalloid solution; his current blood pressure is now 100 / 30 mm
Hg. Abdominal examination is unremarkable and chest X-ray reveals air
underneath the diaphragm. Which of the following is the next best step in
management? Exploratory laparotomy
○ A 15-year-old girl who started menstruating a week ago is brought into the
emergency department because of a fever, abdominal pain, and confusion for the
past 2 days. The patient's mother states that she was overall healthy prior to this,
though she has been dealing with a stressful semester at school. Patient history
shows that the patient has been using tampons and unknowingly left one in for 3
days. While in the emergency department, she begins vomiting and is admitted to
the hospital for further workup. Which of the following types of rash is associated
with her condition? Diffuse erythema that desquamates as the patient recovers
○ A 30-year-old primigravid woman at 28 weeks' gestation comes to the emergency
department because of severe abdominal pain for 2 hours. Her last visit to the
physician was 2 weeks ago. Her pregnancy had been uncomplicated. Her
temperature is 37.1°C (98.8°F), pulse is 112/min, respirations are 22/min, and blood
pressure is 94/58 mm Hg. Abdominal examination shows diffuse tenderness with
board-like rigidity. Ultrasound shows a large subcapsular mass within the liver.
Which of the following is the most likely diagnosis? Hepatic adenoma
○ A 75-year-old male is currently maintained on a ventilator in the intensive care unit
due to a case of urosepsis. In making a choice of medications to provide sedation
and/or analgesia to the patient, which of the following should be avoided?
Etomidate
○ A 71-year-old male is scheduled to undergo coronary artery bypass
surgery following a myocardial infarction. You are assigned as the
anesthesiologist to his case. Midway through the procedure, you notice a
dangerous drop in his mean arterial pressure, indicating acute
hypotension. Which choice of parameters will give you the largest
increase in mean arterial pressure and help resolve this patient's
hypotension? Increasinc contractility and increasing heart rate
○ A 15-year-old girl comes to the emergency department because of
enlarged, edematous, lips for the past 10 minutes. Her symptoms
developed after she was stung by a bee at her softball game. Her blood
pressure is 90/60 mm Hg. Diffuse wheezes are heard bilaterally. A
diagnosis of anaphylactic shock is made. Which of the following best
describes the effect of the most appropriate treatment for the patient's
condition? α1 activation increases peripheral vascularresistance, β1
activation increases heart rate and contractility, and β2 activation causes
bronchodilation

○ A 45-year-old man comes to the emergency department with a gunshot wound to
the abdomen. His wife says that he was getting dressed in his closet when his
loaded handgun fell from the top shelf and unloaded one round when it struck the
floor. His temperature is 37.2°C (99.1°F), pulse is 132/min, respirations are 22/min,
and blood pressure is 65/45 mm Hg. The patient is non-responsive to verbal and
painful stimuli. Physical examination shows that skin is clammy and cool to touch
with dry mucous membranes. A bullet entrance wound is visible in the epigastric
region. Diffuse ecchymosis and distention of the abdomen are also present. A Foley
catheter is placed and dark brown urine accumulates in the collection bag. Which
type of renal cast is most likely to be seen on urinalysis? Granular casts
○ A day after undergoing elective abdominal surgery, a 65-year-old man has
decreased urine output. His urine output has been 10 mL/h during the past 2 hours.
The 3-hour operation had no complications. Currently, his temperature is 37.5°C
(99.5°F), pulse is 112/min, respirations are 23/min, and blood pressure is 90/62 mm
Hg. Pulse oximetry shows an oxygen saturation of 95%. Laboratory studies show a
hemoglobin concentration of 12 g/dL. Which of the following is the most appropriate
next step in management? Intravenous fluid resuscitation
○ A 55-year-old man comes to the emergency department because of chest pain and
difficulty breathing over the past 2 hours. Over the past 7 days, he has felt increased
shortness of breath on exertion, with right-sided chest pain which occurs mainly with
deep breathing. He also has a cough productive of yellow-green sputum, and
intermittent chills. He has received his influenza vaccine. The patient is lethargic but
arousable, and his temperature is 39°C (102.2°F), pulse is 110/min, respirations are
30/min, and blood pressure is 105/65 mm Hg. Pulse oximetry shows an oxygen
saturation of 91% on 4 L of oxygen. His serum lactate is measured to be 3.5 mg/dL
by venous blood gas. A plain film chest radiograph shows multifocal consolidative
opacities. Which of the following interventions is most important in the prevention of
morbidity and mortality in this patient? Vancomycin, azithromycin and ceftriaxone
○ A 34-year-old man comes to the emergency department because of a rash and
trouble breathing for the past 20 minutes after he was stung by a bee at a picnic. He
has a history of anaphylaxis secondary to bee stings. He is anxious and is
vigorously scratching his skin. His temperature is 37.2°C (99.1°F), pulse is 140/min,
respirations are 22/min, and blood pressure is 86/50 mm Hg. He has generalized
hives, periorbital edema, and stridor in the sound of his breath. His tongue and uvula
are edematous. Which of the following is the most appropriate initial treatment?
Epinephrine
○ A 42-year-old man and his 11-year-old daughter are brought to the
Emergency Department by EMS following a motor vehicle accident. The
father is bleeding from a large laceration in his head. Bedside ultrasound
confirms the diagnosis of splenic rupture. Upon examination of the
patient's personal effects, you learn that he is a Jehovah's Witness and
carries a document stating that he does not wish to receive blood
products due to his religious beliefs. He is able to communicate these
beliefs briefly before losing consciousness. The document also states that
his daughter should not receive blood products. The father's vitals are P
130, BP 70/palpable, RR 14. The daughter is unconscious and trauma
radiographs reveal a pelvic fracture. Her vitals are P 160, BP 90/70 RR
30. Labs reveal a hemoglobin of 6.8 gm/dL. Which of the following is the
most appropriate management for both patients in regard to blood
transfusion? Do not transfuse the father but transfuse the daughter

○ A 78 year old male is undergoing an emergent repair of a dissecting aortic
aneurysm. His cardiac index is determined to be 1.8 L/min/m2. Which of the
following inhaled anesthetics would the patient be more likely to overdose on if given
the standard general anesthetic dose? Halothane
■ Blood/Gas Coefficients
■ Nitrous oxide = 0.47
■ Halothane = 2.4
■ Isoflurane = 1.4
■ Desflurane = 0.42
■ Sevoflurane =0.65
○ A 6-year-old girl comes to the emergency department because of a motor vehicle
accident. She is covered in blood and is unconscious secondary to hemorrhagic
shock. Her parents urge the physician to do everything possible but implore that no
blood products be used to treat their daughter. Which of the following is the most
appropriate next step for the physician? Transfuse blood as needed and explain
the situation to the parents

● PLAYLIST 4
○ A 64-year-old man comes to the emergency department because of 2 hours of
persistent right facial droop and slurred speech. He denies any previous episodes of
this nature. Past medical history shows essential hypertension that is well controlled
with hydrochlorothiazide. Physical examination shows an asymmetrical smile and
tongue deviation towards the right. Which of the following is the next most
appropriate step in management? Head CT without contrast
○ A 76-year-old man comes to the office for a pre-surgical assessment. He has no
new medical symptoms. He has a history of diabetes and hypertension, for which he
takes metformin, and hydrochlorothiazide. Physical examination shows no
abnormalities. An ECG is obtained, the results of which are shown below. Follow-on
ambulatory monitoring for six weeks confirms that this is a persistent rhythm. Which
of the following is the most appropriate next step? Initiate anticoagulation therapy
with warfarin
○ A 60-year-old woman comes to the emergency department with the complaint of a
severe headache, nausea, and sensitivity to light. Past medical history reveals a
seizure disorder secondary to a left temporal A-V malformation. Soon after arriving
in the emergency department, the patient develops loss of consciousness requiring
endotracheal intubation. Vital signs reveal a blood pressure of 185/90 mm Hg, a
pulse of 95/min, and a temperature of 37.5ºC (99.5ºF). Which of the following is the
most appropriate initial study to support the diagnosis for this patient? computed
tomography of head without contrast
○ A 56-year-old man is brought to the office for a checkup. Three months ago, patient
suffered from an ischemic stroke and after this his family noticed that he no longer
has outbursts of anger, and surprisingly, has completely quit smoking. His fear of
heights has also disappeared. Which of the following part of his brain is most likely
affected by this patient's stroke? Insular cortex
○ A 61-year-old man presents to the emergency department following a 30-minute
episode of left hand weakness. During the episode, he was unable to pick up any
object at home. He denies any difficulty in ambulation or speaking during the
episode. He has a past medical history significant for hypertension and a
25-pack-year smoking history. He drinks 1-2 beers on the weekend. His temperature
is 37°C (98.6°F), pulse is 92/min, and blood pressure is 138/88 mmHg. BMI is 21
kg/m2. He is alert and oriented, and his speech is fluent and clear. Motor strength is
normal in all extremities, and there are no focal deficits. Noncontrast head CT shows
no abnormalities. Electrocardiogram demonstrates normal sinus rhythm. Which of
the following interventions is most appropriate for long-term risk mitigation given this
patient's clinical presentation? Initiation of aspirin and atorvastatin
○ An 83-year-old woman presents to the emergency department with the complaint of
temporary loss of vision in her left eye. The patient had 5 minutes of visual loss and
describes it as if a “shade” was pulled over her left eye. The patient’s medical history
include hypertension and type II diabetes mellitus. Neurologic and ophthalmologic
examinations are normal. Which of the following is the most informative study in
assessing this patient? Carotid duplex ultrasonography
○ A 65-year-old man comes to the emergency department because of right arm
weakness. He reports that the weakness lasted approximately 25 minutes and then
resolved. He generally feels well, but does have occasional palpitations. His
temperature is 37.5°C (99.5°F), heart rate 82/minutes, respiratory rate 16/minute,
and blood pressure 140/62 mm Hg. Physical examination shows normal muscle
strength bilaterally in the upper extremities and normal sensation. Which of the
following clinical findings is most consistent with his medical history? Atrial
fibrillation
○ A 70-year-old man is brought to the emergency department because of left-sided
weakness for one hour. The patient reports that he suddenly could not move his left
leg at home. The patient denies any history of a bleeding disorder, active bleeding,
or recent surgery. His past medical history is significant for hypercholesterolemia,
hypertension, atrial fibrillation, and tobacco abuse. Physical examination shows 5/5
strength in the left upper extremity and 1/5 strength in the left lower extremity. There
is decreased sensation in the left leg and foot. A computed tomography scan of the
brain shows no hemorrhages. Which of the following is the next best step in
management? Alteplase
○ A 72-year-old woman is brought to the emergency department because she fell
suddenly while mowing the lawn. The patient says that she was in her garden when
all of the sudden she felt dizzy, nauseous, and fell without losing consciousnesses.
She has past medical history of hypertension, diabetes mellitus, atrial fibrillation, and
atherosclerosis. Physical examination shows a right-sided ptosis, loss of lacrimation,
and a pupil of 2 mm and minimally responsive to light. Her left eye shows no
abnormalities. Neurological examination shows decreased sensation to pain and
temperature on the right side of the face and left side of her upper and lower
extremities, dysphagia, and ataxia. Which of the following arteries is most likely
involved in her condition? Posterior inferior cerebellar artery
○ A 25-year-old woman, gravida 2, para 1, with congestive heart failure at 40 weeks
gestation is brought into the labor and delivery unit after her membranes ruptured
that morning. Physical examination reveals the patient’s cervix is dilated to 5 cm (2
in), and her contractions are 3 minutes apart. The patient is prepared for delivery,
and she requests an epidural analgesic due to the intensity of her pain. Within
minutes of being administered the medication, she complains that she can not feel
or move her legs, and appears to have lost bowel control. Which of the following is
the most appropriate intervention? Surgical decompression
○ A 45-year-old female comes to the emergency department because of the “worst
headache of her life” for 4 hours. The headache came on suddenly while she was
writing on the computer. She says that the emergency department is far too bright,
and asks for the lights to be switched off. Medical history is non-contributory. Her
temperature is 36.8°C (98°F), pulse is 87/min, respirations are 18/min, and blood
pressure is 117/78 mm Hg. Examination shows Brudzinski's sign is positive. A
CT-scan of the head is reported as normal. Which of the following is the most
appropriate next step in this patient's care? Lumbar puncture
○ A 78-year-old man comes to the emergency department because he crashed his car
an hour ago. He says he is feeling confused and is unsure what caused him to
swerve off the road. He denies a loss of consciousness. He says he had a lot of
difficulties filling out his admission paperwork. Particularly, he found writing
extremely difficult, and could not sign the document. His vital signs show no
abnormalities. Physical examination shows a man with some minor bruises and
scrapes. Neurological examination shows an inability to differentiate his left and
right, to write, and to distinguish his fingers from one another. There is no further
motor or sensory loss. In which of the following is the lesion most likely to be
located? Left parietal lobe
○ A 60-year-old comes to the emergency department because of right-sided
weakness and slurred speech. The patient's symptoms began two hours ago, last
30 minutes, and have since fully subsided. Physical examination shows 5/5 muscle
strength in all extremities, ambulation without loss of balance, and no slurred
speech. A CT of the head is negative. A carotid duplex reports 80% stenosis in the
left carotid artery and 50% stenosis in the right carotid artery. Which of the following
is the most appropriate next step in management? Surgical carotid
endarterectomy
○ A 55-year-old male presents to the emergency department after losing his footing on
the rung of his ladder and landing on his feet with his back hyper-extended. The
patient reports developing symptoms in his legs immediately after the fall, and a
computerized tomograph scan shows vertebral bony fragments compressing the
anterior spinal artery. The most likely physical finding associated with this patient's
condition includes which of the following? Paraplegia of the legs, loss of
temperature and pain sensation bilaterally, with intact pinpoint and vibratory
discrimination
○ A 57-year-old male is brought to the emergency department after suddenly
collapsing while helping his wife making their dinner. His wife says he lost
consciousness for approximately 10 minutes and then the ambulance arrived. His
medical history is significant for hypertension, hypercholesterolemia, and Type II
Diabetes. He has been smoking 1 pack per day for the past 12 years. He had
appendectomy when he was 21 years old.
His medications include Atenolol, Atorvastatin, Metformin and Aspirin.
On examination there is paralysis of left upper and lower extremities, absent
vibration and position on the left side, positive Babinski’s sign on the left foot, pain
and temperature is intact bilaterally, tongue deviated to the right upon protrusion and
his speech is difficult to understand.
Lesion of which of the following arteries is most likely responsible for these findings?
Anterior Spinal Artery (ASA)
○ A 45-year-old man comes to the emergency department because of the “worst
headache of his life” for 4 hours. The headache came on suddenly while he was
writing on the computer. He says that the emergency department is far too bright,
and asks for the lights to be switched off. Medical history is non-contributory. His
temperature is 36.8°C (98°F), pulse is 87/min, respirations are 18/min, and blood
pressure is 117/78 mm Hg. Examination shows Brudzinski's sign is positive. A
CT-scan of the head is reported as normal. Which of the following findings would
most likely be seen in this patient? The patient will often report another sudden
and severe headache about one to three weeks prior.
○ A 70-year-old man is brought to the emergency department because of left-sided
weakness. The weakness began an hour ago, while at home, when he noticed that
he was suddenly unable to move his left leg. Medical history is contributory for
hypercholesterolemia, hypertension, and tobacco-dependence. Physical
examination shows 0/5 strength in the left lower extremity and 5/5 strength in the left
upper extremity. The left leg and foot also have decreased sensation. A CT scan of
the brain shows no hemorrhage. Which of the following arteries is most likely
affected by this patient's condition? Right anterior cerebral artery
○ A 50-year-old man comes to the clinic with his wife because of right face and arm
weakness. He has a lot of difficulty trying to speak so his wife says that he has
hypertension and diabetes. His temperature is 37.1°C (98.8°F),pulse is 81/min,
respirations are 12/min, and blood pressure is 123/75 mm Hg. Physical examination
shows a decreased right nasolabial fold at rest and decreased movements of the
right face with sparing of the forehead. He also has no right arm movement with only
slight flexion. His is speech appears to be garbled and he cannot phonate a
complete sentence. He appears to understand what is said to him, but he gets
frustrated that he cannot appropriately respond. Which of the following is the most
likely artery and lobe involved in his stroke? Left middle cerebral artery, frontal
lobe
○ A 45-year-old female comes to the emergency department because of a severe
headache for 20 minutes. She states that she often gets headaches, but that this
one feels different. Medical history includes hypertension. CT-scan is obtained.
Lumbar puncture shows xanthochromia. Which of the following is the most likely
vasculature associated with her condition? Anterior communicating artery and
anterior cerebral artery junction
○ A 75-year-old male comes to the community health clinic with his son, who
complains that his father has been getting increasingly forgetful, distracted, and
irritable after he had fallen on ice three weeks ago. History reveals a constant
headache. The most appropriate imaging modality will reveal which of the following?
Crescent shaped area of bleeding on head CT
○ A 65-year-old man comes to the emergency department because of right arm
weakness. He reports that the weakness lasted approximately 25 minutes and then
resolved. His temperature is 37.5°C (99.5°F), heart rate 82/minute, respiratory rate
16/minute, and blood pressure 140/62 mm Hg. Physical examination shows normal
muscle strength bilaterally in the upper extremities and normal sensation. Bruits in
the neck are heard upon auscultation. Which of the following is the most appropriate
next step in management? Ultrasound
○ A 75-year-old woman is brought to the emergency department because of sudden
loss of the upper half of vision in her left eye that lasted 2 minutes. The episode
occurred 5 hours ago and is not associated with pain or loss of consciousness.
Medical history is significant for hypertension and hyperlipidemia. Fundoscopic
examination of the left eye shows Hollenhorst plaques, lodged in a retinal artery with
restored distal blood flow. Visual acuity testing shows 20/20 vision in both eyes and
there are no visual field deficits by confrontation. Which of the following is the most
appropriate next step in management? Doppler ultrasound of the carotid
○ A 46-year-old man was brought to the emergency department for evaluation of a
severe headache for the past hour. The patient has never had a similar episode in
the past. His past medical history includes end-stage renal disease due to
autosomal dominant polycystic kidney disease. Temperature is 37°C (98.6°F), pulse
is 102/min, and blood pressure is 172/104 mmHg. Non-contrast CT-brain
demonstrates hyperlucent density within the subarachnoid space. The patient is
subsequently admitted to the neuro-critical care unit for further monitoring. Four
days following the incident, the patient’s nurse notes new onset left-sided
hemiplegia and worsening somnolence. Administration of which of the following
medication classes could have prevented this patient’s current clinical condition?
Calcium channel blocker
○ A 47-year-old man comes to the emergency department because of a
sudden-onset, exploding headache. He states that the pain developed in seconds
and that he has never had anything like this before. Medical history includes
uncontrolled hypertension. Examination shows no focal neurological deficits at the
time of arrival. A non-contrast CT scan of the head is obtained. Which of the
following is the most common complications of this condition? Vasospasm
○ A 65-year-old man comes to the emergency department because of a sudden onset
of aphasia and dysarthria for the past four hours. He has since developed left-sided
hemiplegia. Patient's medical history is significant for hypercholesterolemia,
hypertension, and diabetes mellitus type 2. Physical examination shows spastic
hemiplegia with hyperreflexia on deep tendon reflexes in the left upper and lower
extremities. CT scan without contrast shows no abnormalities. Which of the
following imaging modalities would most likely detect a diffusion restriction within the
brain at the earliest stage? Diffusion weighted MRI
○ A 75-year-old man is brought to the emergency department because of bilateral
visual loss and muscle weakness for the past 4 hours. The man has an extensive
history of poorly controlled hypertension and does not recall any precipitating events
to this episode. A physical examination shows visual loss in both eyes. He has a
muscle strength of 1/5 in his right shoulder and thigh. Which of the following is the
next appropriate step in management for this patient? Non-contrast computed
tomography scan
○ A 55-year-old right hand dominant man comes to the office for a yearly physical. His
past medical history is significant for hypertension and hyperlipidemia. He is
supposed to take lisinopril, rovastatin, and low-dose aspirin, but has not been
compliant. He says that he has noticed increased difficulty when writing or lifting
things with his right hand over the last three months. Physical examination shows a
slight tremor when the right arm is extended but not when the left arm is.
Examination shows no other abnormalities. Which of the following is the most likely
location of the lesion causing this patient's tremor? Right cerebellar hemisphere
○ A 65-year-old man is evaluated in the hospital following an ischemic stroke. He has
oculomotor nerve palsy with mydriasis of the left side of his face. On the right side of
his face, he cannot feel when someone touches him. He also cannot feel vibration or
the position of someone's finger on his face. He lacks two-point discrimination in this
region too. The right side of his body has signs of tremor and athetosis. Benedikt
syndrome
○ An 80-year-old woman comes to the emergency department because of left sided
paralysis for the past two hours. Her husband says her symptoms began suddenly
and that she is also unable to speak. Her temperature is 36.8°C (98°F), pulse is
90/min, respirations are 18/min, and blood pressure is 150/98 mm Hg. An ECG is
obtained and is shown below. Which of the following is the most likely cause of the
patient's paralysis? Cardioembolic stroke
○ A 60-year-old man is brought to the emergency department because of right upper
and lower extremity weakness and the inability to speak for the past day. Medical
history is significant for hypertension and coronary artery disease. Physical
examination shows that he is able to follow verbal commands, understands what
you are saying to him, but patient is unable to form words. Further physical
examination shows 3/5 strength in his right lower extremity, 0/5 strength in the right
upper extremity, and 5/5 on the left upper and lower extremities. Which of the
following arteries is most likely affected in this patient's condition? Left middle
cerebral artery
○ A 30-year woman comes to the office because of a 1-week history of a worsening,
nonpulsatile, and diffuse headache. She also reports bilateral lower extremity
weakness. She has no history of trauma, neurological disease, or migraine. Her only
medication is the oral contraceptive pill. A T2 MRI shows a hypointense signal in the
cerebral venous system with surrounding edema. MR venography confirms
thrombosis of a cerebral vein. Which of the following most likely characterizes this
patient's condition? Papilledema on fundoscopic exam
○ A 59-year-old man is brought to the emergency department for evaluation of sudden
onset of weakness of the right upper and right lower extremity for three hours. He
does not have chest pain, shortness of breath, or leg pain. His past medical history
includes recurrent infections when he lived in Mexico as a child, all of which were
resolved with “home remedies.” He immigrated to the United States in his 30’s and
was told by several physicians he has a cardiac murmur. He does not smoke
cigarettes, drink alcohol, or use illicit drugs. His temperature is 37°C (98.6°F), pulse
is 94/min and irregularly irregular, and blood pressure is 141/78 mmHg. The
patient’s pulse oximetry is 99% on room air. He has marked weakness and sensory
loss of the right arm and right leg. There is no warmth, swelling or increased
diameter of either leg. Which of the following is the next best step in the
management of this patient? CT of the brain without contrast
○ A 75-year-old man comes to the emergency department because of inability to
move his right upper and lower extremity after waking from a nap one hour ago.
Physical examination shows mild aphasia and left-sided facial droop. A non-contrast
head CT scan is obtained and reveals hypo-attenuation within the middle cerebral
artery distribution. During physical examination the patient develops a sudden
headache, right pin-point pupil and poor gag reflex with increased respiratory effort.
Which of the followings is the most appropriate next step in management?
Immediate intubation
○ A 65-year-old man presents to his primary care physician for a routine checkup. He
has a 40-pack-year history of cigarette smoking and known chronic obstructive
pulmonary disease. Physical examination reveals a distended abdomen with visible
pulsations along the midline. An abdominal ultrasound is obtained and reveals an
abdominal aortic aneurysm of 6.5 cm. The patient is referred for surgery; however,
during the repair the surgeon accidentally injures an artery which supplies the
anterior spinal cord near thoracic vertebra 8. The neurological deficit most likely
associated with this complication includes which of the following? Decreased motor
strength and pain sensation in the lower extremities
○ A 74-year-old man is brought to the emergency department because of an acute
onset of left arm and left facial paralysis. His medical history is significant for
hypertension, coronary artery disease, hypercholesterolemia, and atrial fibrillation.
Neurological examination shows left facial sensory loss, left arm strength 2/5,
hyperreflexia on the bicipital reflex, and loss of sensation. His head computed
tomography (CT) is shown below. Which of the following is the most likely
diagnosis? Cerebrovascular accident
○ A 52-year-old man comes to the clinic because of difficulty initiating speech. He is
accompanied by his sister, who is the patient's primary caretaker, and she denies
head trauma. His speech is labored and disjointed. His sentence construction is
poor and he is omitting function words and inflections. The patient comprehends
words and understands what is said to him. His temperature is 37.8°C (100°F),
pulse is 72/min, respirations are 21/min, and blood pressure is 145/90 mm Hg.
Physical examination shows no signs of head trauma and auscultation with pulse
palpation shows an irregular rhythm. Which of the following is the most likely cause
of this patient's condition? Cerebrovascular accident
○ A 70-year-old man is brought to the emergency department because of slurred
speech, confusion, and weakness in his right side for the past hour. He has
complete hemiparesis of his right upper and lower extremities. Patient's medical
history is significant for hypertension, atrial fibrillation, and diabetes for the past 26
years. His temperature is 37.8°C (100°F), pulse is 100/min, respirations are 14/min,
and blood pressure is 150/92 mm Hg. Oxygen saturation is 94% in room air.
Physical examination shows a right-sided facial droop to the lower portion of his
face, symmetric forehead wrinkles, and symmetric eyebrow elevation. Which of the
following is the most likely diagnosis? Acute ischemic stroke
○ A 61-year-old man is sitting down the cardiology clinic waiting room when he falls to
the left side of the chair onto the floor. He did not hit his head with the fall, but
continues to have leg weakness. He has a DM type 2, and had a myocardial
infarction two years ago. His medications include metformin, atorvastatin, lisinopril
and aspirin. On physical examination his vitals are within normal limits, he is alert
and oriented, and there is profound weakness and absent sensation of his entire left
leg. He is taken to the emergency department for further management in
consultation with neurology. Glucose level is 103 mg/dL, and a non-contrast head
CT is negative for intracranial bleeding. Administer tissue plasminogen activator
(tPA)
○ A 64-year-old man comes to the emergency department because of 2 hours of
persistent right facial droop and slurred speech. He denies any previous episodes of
this nature. Past medical history shows essential hypertension that is well controlled
with hydrochlorothiazide. Physical examination shows an asymmetrical smile and
tongue deviation towards the right. Which of the following is the next most
appropriate step in management? Head CT without contrast
○ A 40-year-old man is brought to the clinic because of progressive clumsiness and
the inability to appropriately use his right hand for the past five days. He is currently
under medication for hypertension and diabetes, and patient's compliance to
treatment is unsure. His temperature is 37.8°C (100°F), pulse is 75/min, respirations
are 15/min, and blood pressure is 148/85 mm Hg. Physical examination shows
hyperreflexia, decreased sensation, and 3/5 muscle strength in the upper and lower
right extremities. Further physical examination shows 10-15 beats of right ankle
clonus, right-sided positive Babinski sign, and a positive Romberg test. An MRI
shows multiple lacunar infarcts. Which of the following is the most likely location of
the lacunae and the corresponding vessel involved in this patient's condition? Basal
ganglia and the deep penetrating lenticulostriate arteries
○ A 73-year-old woman is brought to the emergency department by her daughter for
evaluation of sudden-onset weakness in the left side of her body that began two
hours ago. The patient does not have chest pain, shortness of breath, or leg pain.
Her past medical history includes hypertension and a 35-pack-year smoking history.
She does not take anticoagulants, have any recent surgeries, or have a history of
gastrointestinal bleeding. Vital signs are temperature is 37°C (98.6°F), pulse is
81/min and regular, and blood pressure is 166/104 mmHg. She has marked
weakness and sensory loss in the left arm and left leg. Non-contrast CT-brain is
without evidence of hemorrhage or mass effect. Which of the following is the next
best step in managing this patient’s clinical condition? Recombinant tissue
plasminogen activator
○ A 62-year-old woman comes to the office because of left sided weakness. She was
in her usual state of health when, suddenly, she was unable to lift her arm while
eating breakfast. These symptoms resolved within 15 minutes. She has a history of
diabetes mellitus and hypertension. Her temperature is 37.5°C (99.5°F), heart rate
80/minutes, respiratory rate 18/minute, and blood pressure 145/92 mm Hg. Which of
the following is the most appropriate next step in management? Admit for MRI
neuroimaging (stroke protocol), initiate anti-platelet therapy
○ A 61-year-old woman comes to the clinic because of difficulty repeating words and
phrases for the past 2 weeks. She is able to comprehend words and sentences with
a simple syntactic structure, but she is unable to form fluent speech. The patient
also has severe difficulty when she is asked to name objects. Her past medical
history is significant for hypertension, diabetes mellitus type 2, and dyslipidemia. Her
temperature is 37.8°C (100°F), pulse is 72/min, respirations are 21/min, and blood
pressure is 145/90 mm Hg. Physical examination shows no signs of head trauma
and an irregular rhythm is heard on auscultation. Which of the following is the most
likely diagnosis? Expressive aphasia
○ A 30-year-old male presents to the emergency room after being stabbed in the back
during a bar fight. Inspection reveals a wound on the right lower back at vertebral
level L1. Physical examination reveals spastic paralysis of the right lower extremity,
loss of proprioception on the right side below the level of the lesion, and a positive
Babinski sign on the right foot. Magnetic resonance imaging (MRI) confirms the
presence of a spinal cord lesion exclusively on the right side.
Based on the patient's history and physical examination, which of the following
additional deficits would most likely be present below the lesion? Loss of pain
sensation on the left side
○ A 56-year-old man is brought to the emergency department because of sudden
onset acute vision loss, and confusion. Patient says he feels dizzy and with nausea.
His son says that he has been bumping into objects since these symptoms started.
Medical history is significant for atrial fibrillation and hypertension. Physical
examination shows a fixed, bilateral, homonymous visual-field cut. Which of the
following is the most likely diagnosis? Posterior cerebral artery stroke
○ A 25-year-old male presents to the emergency room with a gunshot wound to the
back. Physical examination reveals a loss of proprioception and spastic paralysis on
the left side as well as a loss of pain and temperature on the right side, all below the
level of the lesion. Magnetic resonance imaging (MRI) shows a left-sided lesion to
the spinal cord at vertebral level T10.
Based on the patient's symptoms and imaging, which of the following is the most
likely diagnosis? Brown-Séquard syndrome
○ An 85-year-old woman comes to the emergency department due to temporary loss
of vision in her left eye. History reveals the patient had a five minute episode of
visual loss and describes it as if a “shade” was pulled over her left eye.
Past medical history reveals hypertension and diabetes. Neurologic and
ophthalmologic examinations are normal. Which of the following is the most likely
cause of this condition? Atherosclerosis
○ A 53-year-old man with a past medical history of hypertension and hyperlipidemia is
hospitalized following an ischemic stroke. He is alert and responsive. On initial
neurological exam, he is asked to stick out his tongue. He looks puzzled, as if he is
trying very hard to follow the instruction but cannot. Two minutes later he sticks his
tongue out spontaneously. Which of the following is the most likely diagnosis?
Apraxia
○ A 65-year-old man comes to the emergency department with right upper and lower
extremity weakness. He also has had an inability to speak over the past day. Her
daughter says that he has hypertension and coronary artery disease. Physical
examination shows that he is able to follow verbal commands and seems to
understand what is said to him, but he is unable to form words. He is unable to
repeat the phrase, “no ifs, ands, or buts." He has 3/5 strength in his right lower
extremity, 0/5 strength in the right upper extremity, and 5/5 on the left upper and
lower extremities. Which of the following areas of the brain has most likely been
affected? Left inferior frontal gyrus
○ A 35-year-old male comes to the emergency department because of a stiff neck,
severe headache, and vomiting. He states that these symptoms developed
suddenly, while he was out for his daily run. Medical history is noncontributory. His
temperature is 37.3°C (99°F), pulse is 87/min, respirations are 18/min, and blood
pressure is 117/78 mm Hg. Examination shows a fit appearing man. A non-contrast
CT-scan is obtained. Which of the following is the most likely diagnosis?
Subarachnoid hemorrhage

● PLAYLIST 5
○ A 57-year-old man comes to the emergency department with his wife
because of syncope 30 minutes ago. His wife reports that he had just
returned home from work when he began experiencing severe shortness of
breath. He coughed up small amounts of bright red blood just prior to
passing out. He is awake but in severe respiratory distress. His left leg is
swollen, erythematous, and tender. His temperature is 37.2°C (99°F), pulse
is 120/min, respirations are 23/min, and blood pressure is 86/45 mm Hg. A
basic metabolic panel shows normal kidney function and troponin
concentration is 0.0 ng/mL. An electrocardiogram shows a new onset right
bundle branch block with right axis deviation. Which of the following the
most appropriate next step in management for this patient? Intravenous
alteplase
○ A 23-year-old male presents to the emergency department after being
stabbed in the chest. The patient's primary survey is notable for decreased
breath sounds over the right chest. Secondary survey reveals a single stab
wound to the right lateral thorax, tracheal deviation, and jugular venous
distension. Temperature is 37.0°C (98.6°F), pulse is 126/min, and blood
pressure is 77/42 mmHg. Respiratory rate is 26/min, and oxygen saturation
is 90% on room air. Which of the following best describes the correct
anatomic space for emergent intervention to manage this patient's
condition? Second intercostal space midclavicular line
○ A 35-year-old man comes to the operating room for elbow surgery. Aside
from his musculoskeletal injury, he is well-appearing and his medical history
is noncontributory. For post-operative pain management, a local anaesthetic
is injected around the nerve plexus in his shoulder. The injection went
through the skin just above the collar bone. Which of the following is the
most likely complication from this anesthetic procedure? Pneumothorax
○ A 27-year-old man comes to the emergency department because of chest
pain and profuse bleeding from a 4-cm scalp laceration. He sustained these
injuries during a bar fight 30 minutes ago. The chest pain is worse on
inspiration. The right hemithorax is tender to palpation with decreased
breath sounds on auscultation. While awaiting assessment in the
department, he becomes acutely short of breath and hypotensive. At this
time, he has distended neck veins and his right hemithorax is
hyper-resonant to percussion. Which of the following is the most
appropriate next step in management? Needle decompression to the
anteromedial chest wall
○ A 29-year-old man comes to the emergency department because of chest
pain and shortness of breath for two days. The difficulty in breathing started
suddenly while he was watching television. His temperature is 36.8°C
(98°F), pulse is 87/min, respirations are 18/min, and blood pressure is
117/78 mm Hg. Cardiac, pulmonary, and abdominal examinations are
noncontributory. ECG shows sinus rhythm. Chest X-ray is obtained. A repeat
chest X-ray 6 hours later shows that the pathology has decreased slightly in
size. Which of the following is the most appropriate next step in
management? Discharge the patient with follow-up in 24 hours
○ A 24-year-old man comes to the emergency department because of acute
shortness of breath and left-sided chest pain following a severe motor
vehicle crash an hour ago. He was a restrained driver and the airbags were
deployed, but there was severe damage to the driver's side of the vehicle.
His temperature is 36.8°C (98°F), pulse is 87/min, respirations are 32/min,
and blood pressure is 117/78 mm Hg. Glasgow Coma Score is 14.
Pulmonary auscultation shows decreased breath sounds on the left side,
with normal lung sounds on the right. There is no jugular vein distention or
tracheal deviation. CT scan shows several broken ribs and free air in the
pleural space. It is decided that a thoracostomy tube will be placed. Which
of the following is most likely the proper location for tube insertion? Left
mid-axillary line, 5th intercostal space
○ A 62-year-old man comes to the emergency department because of acute
onset shortness of breath and lightheadedness for the past hour. He
underwent a total knee replacement surgery a month ago. He reports that
he experiences chest pain upon deep inspiration. His temperature is 36.4°C
(97.5°F), pulse is 120/min, respirations are 22/min, and blood pressure is
140/85 mm Hg. An emergent spiral computed tomography scan is obtained
and is shown below. Which of the following is the most likely diagnosis?
Saddle pulmonary embolism
○ A 19-year-old man comes to the emergency department because of severe
shortness of breath for the past hour. He was lying in bed trying to fall
asleep when he felt a sudden, sharp pain on his right side, followed by
shortness of breath and difficulty breathing. Using his roommate’s “asthma
inhaler” did not help resolve his symptoms. He says he also has right-sided
pleuritic chest pain which worsens with inspiration. He is tall and thin
appearing. Oxygen saturation is 88% on room air. Vital signs otherwise
show no abnormalities. Pulmonary auscultation shows no wheezes, rales, or
rhonchi. The right hemithorax is hyperresonant to percussion compared to
the left. Which of the following is the most appropriate next step? Chest
radiograph
○ A 45-year-old man comes to the emergency department because of fever
and scrotal pain for 2 days. Medical history includes diabetes mellitus and
morbid obesity. His temperature is 40.0°C (104°F), pulse is 130/min,
respirations are 35/min, and blood pressure is 80/60 mm Hg. Physical
examination shows a large area of ecchymosis, edema, and crepitus to his
perineal area. Fournier gangrene is suspected. A right internal jugular
central venous catheter is placed without complication under ultrasound
guidance for vascular access in preparation for administration of
vasopressors. Which of the following is the most appropriate next step?
Obtain an immediate portable chest radiograph to evaluate line placement
○ A 19-year-old male presents to the emergency department with shortness
of breath and chest pain. The patient reports that he was sitting on the
couch watching television when the symptoms started. He does not take
any medications and is otherwise healthy. Temperature is 37.0°C (98.6°F),
pulse is 101/min, and blood pressure is 122/64 mmHg. Respiratory rate is
18/min, and the patient has an oxygen saturation of 94% on room air.
Physical examination reveals a tall thin male in no acute distress. A portable
chest x-ray is ordered and reveals pleural markings approximately 1-2
centimeters away from the left chest wall. The original image is shown
below: Which of the following is the most appropriate management for this
patient’s condition? 100% Fi02 and observation
○ A 33-year-old woman, gravida 2, para 2, in the postpartum unit is now
having acute shortness of breath for the past 30 minutes. The patient gave
birth to a healthy baby boy by elective repeat cesarean section 2 days ago.
She says that she is still in extreme pain from the operation and has only left
her bed a few times to use the restroom. Her temperature is 36.4°C (97.5°F),
pulse is 107/min, respirations are 25/min, and blood pressure is 121/84 mm
Hg. Physical examination shows clear lung fields bilaterally and a swollen,
tender, and erythematous left calf. Which of the following would most likely
have prevented this patient's complication? Postpartum early ambulation
○ A 65-year-old woman is admitted to the hospital because of severe
shortness of breath that suddenly began 1 hour ago. She also has left-sided
chest pain that is worse on inspiration. Medical history includes dilated
cardiomyopathy and congestive heart failure. She was also recently
mechanically ventilated in the intensive care unit for 2 weeks after
developing a severe infection following resection of an ischemic portion of
small bowel. Her temperature is 36.8°C (98°F), pulse is 119/min,
respirations are 27/min, and blood pressure is 91/78 mm Hg. Examination
shows that the left side of her chest is hyper-resonant to percussion. Chest
X-ray is obtained. Which of the following is most likely a risk factor for her
current condition? Chronic mechanical ventilation
○ A 35-year-old man comes to the emergency department because of
dyspnea, chest pain, and bruising over his torso for an hour. His symptoms
developed after he was involved in a motor vehicle accident. His
temperature is 36.8°C (98°F), pulse is 87/min, respirations are 18/min, and
blood pressure is 117/78 mm Hg. Examination shows an uncomfortable
appearing man. Pulmonary auscultation shows absent breath sounds on his
right side. The trachea is at the midline. Which of the following tests is the
most accurate in the diagnosis of this patient? CT chest
○ A 64-year-old man comes to the emergency department because of severe
shortness of breath for 2 days. Medical history includes severe chronic
obstructive pulmonary disease. He deteriorates despite short-acting
bronchodilators, supplemental oxygen, and non-invasive positive pressure
ventilation support. The decision is made to intubate. The patient is
pre-oxygenated and rapid sequence intubation is successfully performed on
the first try. Five minutes later the patient goes into cardiac arrest.
Examination shows tracheal deviation to the left and absence of breath
sounds on the right. The patient rapidly becomes hypoxemic and
hypotensive. Which of the following most likely went wrong during the
intubation? Rapid pre-oxygenation ventilation rate
○ A 22-year-old man comes to the emergency department because of
left-sided chest pain for three days. He felt a small “pull” in his upper left
chest while walking to work. He denies any trauma. The pain has persisted,
and is worse when he takes a deep breath or coughs. He is more short of
breath than usual when climbing stairs. His past medical history is
noncontributory, he has smoked a pack of cigarettes a day for the last two
years. His vital signs show no abnormalities. He is tall and thin in
appearance. Pulmonary auscultation shows breath sounds are reduced on
the left side in the upper zone. Cardiovascular examination is
noncontributory. Chest X-ray shows a small left sided pneumothorax. At the
level of the hilum, 1.5 cm (0.6 in) of space lies between the chest wall and
the pleura. Which of the following is the most appropriate next step in
management? High flow oxygen (10 L/min)
○ A 17-year-old boy comes to the emergency department because of acute
shortness of breath for 20 minutes. Medical history includes four idiopathic
spontaneous pneumothoraces in the past 3 years. Examination shows a tall,
thin appearing male. Pulmonary examination shows hyperresonance to
percussion on the right compared to the left. The patient says that he does
not want this to occur again and asks if there is any way to prevent future
episodes. Which of the following is the most appropriate treatment?
Video-assisted thoracoscopic surgery pleurodesis
○ A 43-year-old man comes to the emergency department because of a stab
wound to the right side of his chest 20 minutes ago. He says that in addition
to right-sided chest pain, he feels shortness of breath. His temperature is
36.8°C (98°F), pulse is 114/min, respirations are 26/min, and blood pressure
is 88/62 mm Hg. He is having trouble finishing his sentences due to his
shortness of breath. Pulmonary examination shows right-sided
hyperresonance and pulmonary auscultation shows decreased breath sound
also on the right-hand side. His neck veins are also distended. Which of the
following is the most likely diagnosis? Tension pneumothorax

● PLAYLIST 6
STEP 1
1. A 68-year-old man comes to the emergency department because of 4 hours of
acute onset retrosternal chest pain, radiating to the left arm. It began while he
was playing soccer with his grandson. His past history is significant for type 2
diabetes mellitus controlled with insulin, and hypertension controlled with
enalapril. Further history revealed that he had ingested a sildenafil tablet
yesterday. Physical examination shows a temperature of 37°C (98.6°F), pulse of
69/min, respirations of 18/min, blood pressure of 130/70 mm Hg, and O2
saturation of 95%. An electrocardiogram shows sinus rhythm and ST-segment
elevation in leads V4-V6. Which of the following medications is contraindicated
for use in this patient for treating his symptoms?
a. Nitroglycerin
2. A 74-year-old woman comes to the emergency department because of severe
"heartburn" for 40 minutes. In
addition to her "heartburn," she says that she feels very fatigued and feels
numbness in her jaw. She has vomited twice in the last 30 minutes. She has a
history of insulin dependent diabetes mellitus and hypercholesterolemia. An
ECG is obtained and is shown below. Which of the following is the most likely
diagnosis?
a. Inferior ST-elevation myocardial infarction
3. A 60-year-old man is reviewed on the cardiology ward because of hypotension
and prominent jugular vein distention for 10 minutes. He is on the ward because
of a large myocardial infarction five days ago. Medical history includes type 2
diabetes mellitus, uncontrolled hypertension, and hypercholesterolemia. His
temperature is 36.8°C (98°F), pulse is 112/min, respirations are 21/min, and
blood pressure is 94/78 mm Hg. His pulse is weaker to palpation during
inspiration, and cardiac auscultation shows muffled heart sounds. Which of the
following is the most likely diagnosis?
a. Myocardial wall rupture
4. A 66-year-old man comes to the emergency department because of left-sided
chest pain that began at rest and lasted for 15 minutes before resolving. He
says a similar episode occurred at rest yesterday. Medical history includes
hypertension and type II diabetes mellitus. Current medications are amlodipine,
glyburide, and aspirin. His temperature is 36.8°C (98°F), pulse is 87/min,
respirations are 18/min, and blood pressure is 117/78 mm Hg. Cardiac
examination shows carotid upstroke is normal, there are no cardiac murmurs,
and the lung fields are clear. ECG shows ST-depression in leads V2, V3, and V4.
Laboratory investigations show elevated CK-MB and serum troponin I level.
Chest X-ray is normal. Which of the following is the most likely diagnosis?
a. Non–ST-elevation myocardial infarction
5. A 71-year-old woman comes to the emergency department because of severe
central chest pain for 30 minutes this morning. She says the pain was cramping
in nature and radiated down her left
arm. She has a history of atrial
fibrillation and type 2 diabetes mellitus.
Her temperature is 36.8°C (98°F), pulse
is 97/min, respirations are 18/min, and
blood pressure is 163/91 mm Hg.
Cardiovascular examination shows no abnormalities. ECG is obtained and is
shown below. Which of the following biochemical measures would most likely
be elevated and remain elevated for a week after this acute event?
a. Troponin I
6. A 60-year-old man is brought to the emergency department because of
crushing substernal chest pain for the past 45 minutes. He received 325 mg of
aspirin en route. Nitroglycerin does not relieve his pain. He has a history of
diabetes and hypertension. Medications include carvedilol and sildenafil. His
temperature is 36.8°C (98°F), pulse is 99/min, respirations are 18/min, and blood
pressure is 192/88 mm Hg. He appears diaphoretic. ECG shows ST-segment
elevation in leads V1, V2, and V3. Which of the following is the most
appropriate next step in management?
a. Percutaneous coronary intervention

STEP 2
1. A 64-year-old man comes to the emergency department because of severe
central chest pain for two hours. He says the pain is heavy in nature and radiates
to his jaw. He has a history of uncontrolled hypertension and diabetes mellitus
type 2. His temperature is 36.8°C (98°F), pulse is 97/min, respirations are
18/min, and blood pressure is 163/91 mm Hg. Cardiovascular examination
shows no abnormalities. ECG shows no ST-elevation. Which of the following
factors is a specified criterion to predict the outcome in patients with this
diagnosis?
a. Age ≥65
2. A 48-year-old man comes to the emergency department because of severe
substernal chest pain radiating to his left arm that began 1.5 hours prior to
arrival. The pain is accompanied by diaphoresis and shortness of breath. His
blood pressure is 165/94 mm Hg, pulse is 82/min, and respiratory rate is 18/min.
An acute myocardial infarction is suspected. Which of the following tests is the
most important tool in the initial evaluation of this patient?
a. Electrocardiogram
3. A 73-year-old woman comes to the emergency department because of severe
"heartburn" for 45 minutes. She says that she feels very fatigued and has
numbness in her jaw. She has vomited twice in the last 30 minutes. She has a
history of insulin dependent diabetes mellitus and hypercholesterolemia. An
ECG is obtained and is shown
below. The hospital she is
currently in does not have an
interventional catheterization
laboratory and the nearest
hospital with such capabilities is
an hour and a half away. Which
of the following is a component of the most appropriate intervention for this
woman?
a. Aspirin
4. A 60-year-old man is brought to the emergency department because of
crushing substernal chest pain for the past 45 minutes. The patient received 325
mg of aspirin in the ambulance. Medical history includes diabetes mellitus type
2 and asthma, for which the patient
takes albuterol as needed. Other
medications include carvedilol and
sildenafil. The patient’s temperature is
36.8°C (98°F), pulse is 99/min,
respirations are 18/min, and blood
pressure is 160/90 mm Hg. The patient
appears diaphoretic. An ECG is
obtained and shown below:
Which of the following is the most appropriate next step in management?
a. Percutaneous coronary intervention
5. .A 46-year-old man comes to the office because of a four-month history of
intermittent episodes of chest discomfort and tightness. His symptoms typically
occur only on exertion, such as when gardening or playing with his children, and
are relieved by rest. He says the episodes last less than 20 minutes. He has
otherwise been healthy and his personal medical history is noncontributory. His
father has coronary artery disease, but is alive and well. Physical exam,
laboratory studies and ECG are normal. The medication most appropriate for
treating his acute episodes acts by which of the following mechanisms?
a. Venodilation
6. An 85-year-old man with hypertension and hyperlipidemia comes to the
emergency department because of chest discomfort and shortness of breath
that started two hours ago while shoveling snow. During the past 10 years, he
has had similar episodes of chest pain, which
resolved with rest. His pulse is 94/min,
respirations are 22/min, and blood pressure is
134/86 mm Hg. An ECG is obtained. Treatment
with oxygen, morphine, nitroglycerin, aspirin,
and metoprolol is initiated. The patient reports
feeling faint, and his blood pressure falls to
92/54 mm Hg. Which of the following is the most likely cause of his
hypotension?
a. Nitroglycerin
7. A 67-year-old woman comes to the emergency department because of chest
heaviness and shortness of breath for 4 hours. She states the discomfort began
rapidly. Medical history is significant for hypertension and type II diabetes
mellitus. Medications include amlodipine, lisinopril, and metformin. Her
temperature is 36.7ºC (98.0ºF), pulse is 110/min, respirations are 20/min, and
blood pressure is 82/45 mm Hg. Electrocardiogram shows normal sinus rhythm,
with ST-segment elevation in leads II, III, aVF, as well as V1 and V2. Troponins
are pending, and the cardiac catheterization service has been alerted. Which of
the following is the most appropriate next step in management?
a. Bolus 0.9% saline
8. A 35-year-old male presents to the emergency department with crushing
substernal chest pain for 2 hours. He is taken to the cardiac catheterization lab
and immediately rushed to surgery for coronary artery bypass surgery. Prior to
being placed on cardiac bypass he is anticoagulated with unfractionated
heparin. At the end of the procedure the patient is weaned off of the
cardiopulmonary bypass by warming, lung ventilation, and heart defibrillation.
The venous return of the bypass machine will be slowed down, allowing the
heart to fill up. While this process is occurring, global cardiac function and
hemodynamics must be monitored. Which of the following is the most
appropriate technique for performing this function?
a. Transesophageal echocardiography
9. A 75-year-old man comes to the emergency department because of severe
central chest pain for 2 hours. He says the pain began during breakfast, is heavy
in nature, and radiates to his jaw. He has a history of uncontrolled hypertension
and type 2 diabetes mellitus. Social history includes long-term cigarette
smoking. His temperature is 36.8°C (98°F), pulse is 97/min, respirations are
18/min, and blood pressure is 163/91 mm Hg. Cardiovascular examination
shows no abnormalities. ECG shows ST-elevation in leads V2, V3, and V4. Which
of the following treatments is most likely associated with improved survival in
this condition?
a. Oral metoprolol
10.A 67-year-old man comes to the office because of chest pain when shoveling
snow that morning. The pain does not radiate and lasted 5 minutes, improving
on rest. He rates the pain as a 4 on a 10-point scale. He has a 20-year history of
hypertension and hyperlipidemia. He smoked a pack of cigarettes daily for 20
years but quit 30 years ago. ECG shows Q-waves and T-wave inversion in leads
II, III, and aVF. No prior ECGs are available for comparison. Which of the
following is the most appropriate next step in management?
a. Send to emergency department for evaluation of serum biomarkers
11.A 45-year-old male presents to the hospital, complaining of chest pain upon
exertion. Chest pain has been noted to be more severe recently. Coronary
angiograph was performed and it indicated moderate to severe occlusion of the
left anterior descending artery causing a myocardial infarction. Coronary artery
bypass graft (CABG) is advised for this patient. Which of the following blood
vessels are not commonly used as the grafting vessel?
a. Internal jugular vein
12.A 55-year-old man with hypertension and hyperlipidemia comes to the
emergency department because of chest pain that started an hour ago while
shoveling snow. He describes it as a crushing pain that radiates down his left
arm and rates it as a 7 on a 10-point scale. His pulse is 115/minute, respirations
are 24/min, and blood pressure is 112/68 mm Hg. Cardiac examination shows
no murmurs. A 12-lead ECG with right-sided leads is obtained. Which of the
following medications, if administered, is most likely to cause serious
complications in this patient?
a. Nitroglycerin

○ An 85-year-old man comes to the emergency department because of chest


discomfort and shortness of breath that started two hours ago while
watching television. Past medical history includes hypertension and
hyperlipidemia. During the past 10 years, the patient has had similar
episodes of chest pain, which resolved with rest. The patient’s pulse is
94/min, respirations are 22/min, and blood pressure is 134/86 mmHg. An
ECG is obtained (see image below). Treatment is administered, and the
patient suddenly reports feeling faint and loses consciousness. Which of the
following medications is most likely the cause of the patient’s worsened
condition?
■ Nitrates
○ A 60-year-old man is brought to the emergency department due to chest
pain that started suddenly 15 minutes ago while playing with his
grandchildren. The patient describes the pain as “pressure and tightness”
located in the center of his chest. The patient reports that he occasionally
experiences mild chest discomfort when going up the stairs. Medical history
is significant for a 30-pack-year smoking history, hypercholesterolemia, and
hypertension. The patient is given a sublingual medication while in the
emergency department, and the symptoms resolve within a few minutes.
Which of the following sets of hemodynamic changes is most likely to be
seen in this patient following the administration of this medication?

○ A 70-year-old woman comes to the emergency department due to an acute
onset of epigastric pain. The pain started 30 minutes ago while she was
gardening. She has been feeling nauseous and vomited once since the pain
started. She denies chest pain or shortness of breath. Her medical history is
significant for a 20-pack-year smoking history, hypertension, peptic ulcer
disease, type 2 diabetes mellitus, and cholelithiasis. Her temperature is
37.0°C (98.6°F), pulse is 80/min, respirations are 18/min, and blood pressure
is 145/85 mmHg. The patient appears distressed and diaphoretic. Physical
examination shows normal heart sounds with no murmurs or gallops on
auscultation. Lung auscultation reveals normal breath. Abdominal
examination shows a soft, nontender abdomen with normal bowel sounds.
Which of the following investigations should be immediately performed in
this patient?
■ Electrocardiogram
○ A 60-year-old man comes to the emergency department due to 20 minutes
of crushing chest pain and diaphoresis that began suddenly while watching
television. According to the patient’s partner, lately he has had occasional
chest pain occurring during rest that resolved after a few minutes. Medical
history is significant for hyperlipidemia, hypertension, and diabetes mellitus.
The patient has not been compliant with the medications. The patient’s
temperature is 37.0°C (98.6° F), pulse is 80/min, respirations are 20/min, and
blood pressure is 135/85 mm Hg. The patient appears to be in acute
distress and in pain. Cardiac examination shows an additional sound just
prior to S1 on auscultation. ECG demonstrates ST segment elevation in
leads V1-V5. While being prepared for percutaneous coronary intervention,
the patient suddenly dies. Following this patient’s death, an autopsy is
performed to better understand the underlying pathology. At the time of this
patient's death, which of the following sets of findings is most likely to be
seen in the anterior wall of the myocardium?

○ A 32-year-old man is brought to the emergency department due to chest
discomfort that started suddenly 20 minutes ago while he was trying to fall
asleep. The patient describes the pain as a “squeezing” in the center of the
chest that moves to the neck and jaw; he denies any changes in pain with
respiration or position. The patient denies any recent travel, sick contacts, or
illnesses. Medical history is significant for a 5-pack-year smoking history,
and the patient denies the use of alcohol or illicit drugs. The patient’s
temperature is 37.0°C (98.6°F), pulse is 100/min, respirations are 20/min,
and blood pressure is 145/85 mmHg. The patient appears to be in distress
and is diaphoretic. Electrocardiogram shows ST-segment elevation in the
anterior leads. The patient is given nitroglycerin and the pain resolves within
a few minutes. Which of the following findings is most likely underlying
pathophysiology responsible for this patient’s presentation?
■ Minimal atherosclerotic plaque and coronary artery spasm
○ A 20-year-old male college student suddenly collapses and dies while
playing in a football game. His teammate reports that the patient mentioned
“feeling off” just before the game started but attributed it to being nervous
for the game. Medical history is noncontributory, and the parents report that
he was completely healthy and took no medications. According to friends,
the patient drank approximately 2-3 beers on weekends and did not smoke
cigarettes or use illicit drugs. Which of the following is the most likely cause
of death in this patient?
■ Ventricular tachyarrhythmia
○ A 65-year-old man is brought to the emergency department due to chest
pain that started 12 hours ago while watching television. The patient
describes the pain as “heavy pressure” located in the center of the chest that
radiates up to the neck and jaw. He states that the pain has waxed and
waned, although he has felt some relief from taking sublingual nitroglycerin.
Medical history is significant for coronary artery disease, stable angina,
hyperlipidemia, and type 2 diabetes mellitus. The patient’s temperature is
36.7°C (98.0°F), pulse is 110/min, respirations are 24/min, and blood
pressure is 135/93 mmHg. Pulse oximetry on room air shows an oxygen
saturation of 97%. Chest x-ray shows no abnormalities, cardiac enzymes are
negative, and ECG shows ST-segment depression in the anterior leads.
Which of the following is the most likely diagnosis?
■ Unstable angina
○ A 52-year-old man comes to the clinic due to an episode of chest pain and
shortness of breath. The patient describes substernal chest pain that started
suddenly when he was walking up the stairs and resolved after a few
minutes of rest. Medical history includes type 2 diabetes mellitus and
hypercholesterolemia. The patient does not use tobacco, alcohol, or illicit
drugs. The patient’s temperature is 36.5°C (97.7°F), pulse is 80/min,
respirations are 20/min, and blood pressure is 130/85 mmHg. He does not
appear to be in distress. Physical examination shows normal heart sounds
with no murmurs or gallops heard on auscultation. Which of the following
sets of findings would most likely be seen in this patient if diagnostic
investigations had been obtained during the episode of pain?


○ A 60-year-old man comes to the emergency department due to 20 minutes
of crushing chest pain and diaphoresis that began suddenly while watching
television. According to the patient’s partner, lately he has had occasional
chest pain occurring during rest that resolved after a few minutes. Medical
history is significant for hyperlipidemia, hypertension, and diabetes mellitus.
The patient has not been compliant with the medications. The patient’s
temperature is 37.0°C (98.6° F), pulse is 80/min, respirations are 20/min, and
blood pressure is 135/85 mm Hg. The patient appears to be in acute
distress and in pain. Cardiac examination shows an additional sound just
prior to S1 on auscultation. ECG demonstrates ST segment elevation in
leads V1-V5. While being prepared for percutaneous coronary intervention,
the patient suddenly dies. Which of the following is the most likely cause of
death in this patient?
■ Ventricular arrhythmia

Myocardial infarction
STEP 1

1. A 74-year-old woman comes to the emergency department because of severe


"heartburn" for 40 minutes. In addition to her "heartburn," she says that she
feels very fatigued and feels numbness in her jaw. She has vomited twice in the
last 30 minutes. She has a history of insulin dependent diabetes mellitus and
hypercholesterolemia. An ECG is obtained and is shown below. Which of the
following is the most likely diagnosis?
a. Inferior ST-elevation myocardial infarction
2. A 47-year-old man comes to the emergency department because of a
40-minute episode of central chest pain that occurred 2 hours ago and has since
subsided. This is the first time that the patient has experienced this type of pain.
His troponin and CK-MB levels are normal and an electrocardiogram (ECG) is
shown below. Which of the following types of acute myocardial infarction is the
patient at most risk for?
a. Anterior wall myocardial infarction
3. A 64-year-old man comes to the emergency department because of severe
central chest pain for 2 hours. He says the pain is heavy in nature and radiates to
his jaw. He has a history of uncontrolled hypertension and type 2 diabetes
mellitus. His temperature is 36.8°C (98°F), pulse is 97/min, respirations are
18/min, and blood pressure is 163/91 mm Hg. Cardiovascular auscultation
shows a holosystolic murmur that seems loudest at the apex. ECG shows
ST-elevation in leads V2, V3, and V4. Which of the following will most likely
occur as a result of this pathology?
a. Regurgitant valve disease
4. A 60-year-old man is brought to the emergency department because of
crushing substernal chest pain for the past 45 minutes. He received 325 mg of
aspirin en route. Nitroglycerin does not relieve his pain. He has a history of
diabetes and hypertension. Medications include carvedilol and sildenafil. His
temperature is 36.8°C (98°F), pulse is 99/min, respirations are 18/min, and blood
pressure is 192/88 mm Hg. He appears diaphoretic. ECG shows ST-segment
elevation in leads V1, V2, and V3. Which of the following is the most
appropriate next step in management?
a. Percutaneous coronary intervention
5. A 55-year-old man comes to the emergency department because of chest pain
that began about 18 hours ago. He describes it as substernal pressure with
electrical shooting pains down his left arm. He has a past medical history of
type II diabetes mellitus, hypertension, and hyperlipidemia. His medications
include metformin, pravastatin, lisinopril and hydrochlorothiazide. Laboratory
studies show a troponin concentration of 1.5 ng/mL. Despite comprehensive
resuscitation efforts, the patient dies. An autopsy is performed and the
following illustration represents the macroscopic appearance of the patient’s
heart. Which of the following best characterizes the histological findings of the
patient’s myocardium?
a. Karyopyknosis, hypereosinophilia of myocytes, contraction band necrosis
6. A 55-year-old man comes to the emergency department because of severe
central chest pain for 2 hours. He says the pain is heavy in nature and radiates to
his jaw. Medical history includes uncontrolled hypertension and type II diabetes
mellitus. His temperature is 36.8°C (98°F), pulse is 97/min, respirations are
18/min, and blood pressure is 163/91 mm Hg. Cardiovascular examination is
noncontributory. An ECG is obtained and it shows ST-elevations in leads V1-V6,
as well as reciprocal changes in leads III and aVF. Which of the following
complications is most likely to occur, and result in death during this admission?
a. Ventricular tachycardia
7. A 65-year-old man comes to the emergency department because of severe
central chest pain for two hours. He says the pain began while he was sitting at
the breakfast table, is heavy in nature, and radiates to his jaw. He has a history
of uncontrolled hypertension and diabetes mellitus type 2. He also has a history
of cigarette smoking. His temperature is 36.8°C (98°F), pulse is 97/min,
respirations are 18/min, and blood pressure is 163/91 mm Hg. Cardiovascular
examination shows no abnormalities. ECG shows sinus rhythm at 97/min
without ST-segment or other changes. Cardiac biomarkers taken six hours after
admission are not elevated. Which of the following is the most likely cause of
his symptoms?
a. Unstable angina
8. A 71-year-old woman comes to the emergency department because of severe
central chest pain for 30 minutes this morning. She says the pain was cramping
in nature and radiated down her left arm. She has a history of atrial fibrillation
and type 2 diabetes mellitus. Her temperature is 36.8°C (98°F), pulse is 97/min,
respirations are 18/min, and blood pressure is 163/91 mm Hg. Cardiovascular
examination shows no abnormalities. ECG is obtained and is shown below.
Which of the following biochemical measures would most likely be elevated
and remain elevated for a week after this acute event?
a. Troponin I
9. A 72-year-old woman is rushed to intensive care unit because of sudden
hemodynamic instability. The patient is currently on the 3rd day of her hospital
stay. Her medical history is relevant for a recent primary percutaneous coronary
intervention due to a myocardial infarction. She currently takes dual antiplatelet
therapy and statins. Physical exam shows pale and cold extremities and
peripheral cyanosis. Her temperature is 35.1°C (95.1°F), pulse is absent,
respirations are irregular, and blood pressure is 50/30 mmHg and pulse
oximetry on room air shows an oxygen saturation of 75%. After a cardiac arrest
and several cycles of cardiopulmonary resuscitation, the patient dies 30 minutes
later. The autopsy revealed hemopericardium along with necrotic myocardium (a
yellowish lesion with hyperemic borders). Which of the following is the most
likely cause of death in this patient?
a. Myocardial rupture
10.A 72-year-old man comes to the emergency department because of severe
upper abdominal pain for the past two hours. He says the pain is heavy in
nature, and that he has vomited twice. Medical history includes uncontrolled
hypertension and type II diabetes mellitus. His temperature is 36.8°C (98°F),
pulse is 97/min, respirations are 18/min, and blood pressure is 163/91 mm Hg.
Cardiovascular examination shows no abnormalities. ECG is obtained and is
shown below. Which of the following is the most likely diagnosis?
a. Acute inferior myocardial infarction
11.A 73-year-old male comes to the office because of chest pain for 2 hours. He
returned to America yesterday from a holiday in New Zealand. He states that
the pain feels heavy and is located behind his sternum. He also feels nauseated,
and as a result hasn't eaten lunch. His medical history includes type II diabetes
mellitus, dyslipidemia, and hypertension. His temperature is 36.5°C (97.7°F),
pulse is 78/min, respirations are 17/min, and blood pressure is 120/78 mm Hg.
Examination shows a man who appears pale and diaphoretic. Cardiovascular
and respiratory examinations are noncontributory. An ECG is obtained. Which of
the following is the most likely diagnosis?
a. Coronary artery disease
12.A 66-year-old man comes to the emergency department because of left-sided
chest pain that began at rest and lasted for 15 minutes before resolving. He
says a similar episode occurred at rest yesterday. Medical history includes
hypertension and type II diabetes mellitus. Current medications are amlodipine,
glyburide, and aspirin. His temperature is 36.8°C (98°F), pulse is 87/min,
respirations are 18/min, and blood pressure is 117/78 mm Hg. Cardiac
examination shows carotid upstroke is normal, there are no cardiac murmurs,
and the lung fields are clear. ECG shows ST-depression in leads V2, V3, and V4.
Laboratory investigations show elevated CK-MB and serum troponin I level.
Chest X-ray is normal. Which of the following is the most likely diagnosis?
a. Non–ST-elevation myocardial infarction
13.A 62-year-old Caucasian woman comes to the office because of shortness of
breath for the past six hours. She had a myocardial infarction five days ago,
which was treated medically because she failed to come to the hospital in time
for primary percutaneous coronary intervention. On cardiac auscultation, a
high-pitched holosystolic murmur is heard at the apex, when she is lying on her
left side. Coarse crackles are heard at the lung bases bilaterally on pulmonary
auscultation. Which of the following muscles is most likely damaged?
a. Papillary muscles
14.A 60-year-old man is reviewed on the cardiology ward because of hypotension
and prominent jugular vein distention for 10 minutes. He is on the ward because
of a large myocardial infarction five days ago. Medical history includes type 2
diabetes mellitus, uncontrolled hypertension, and hypercholesterolemia. His
temperature is 36.8°C (98°F), pulse is 112/min, respirations are 21/min, and
blood pressure is 94/78 mm Hg. His pulse is weaker to palpation during
inspiration, and cardiac auscultation shows muffled heart sounds. Which of the
following is the most likely diagnosis?
a. Myocardial wall rupture
15.A 64-year-old man comes to the emergency department because of chest
discomfort for the past 5 hours. When the patient is asked where the pain is
located, he places a clenched fist to indicate a squeezing over his substernal
region. The pain radiates to the neck, left shoulder, and left arm. His
temperature is 36.8°C (98°F), pulse is 55/min, respirations are 17/min, and blood
pressure is 117/78 mm Hg. Examination shows a diaphoretic male. ECG shows
inverted U-waves in leads V5 and V6. The cardiac biomarker test shows an
elevated concentration of troponin. Which of the following is most likely
consistent with this patient's diagnosis?
a. Elevated creatine kinase-MB concentration
STEP 2
1. A 45-year-old male presents to the hospital, complaining of chest pain upon
exertion. Chest pain has been noted to be more severe recently. Coronary
angiograph was performed and it indicated moderate to severe occlusion of the
left anterior descending artery causing a myocardial infarction. Coronary artery
bypass graft (CABG) is advised for this patient. Which of the following blood
vessels are not commonly used as the grafting vessel?
a. Internal jugular vein
2. A 73-year-old woman comes to the emergency department because of severe
"heartburn" for 45 minutes. She says that she feels very fatigued and has
numbness in her jaw. She has vomited twice in the last 30 minutes. She has a
history of insulin dependent diabetes mellitus and hypercholesterolemia. An
ECG is obtained and is shown below. The hospital she is currently in does not
have an interventional catheterization laboratory and the nearest hospital with
such capabilities is an hour and a half away. Which of the following is a
component of the most appropriate intervention for this woman?
a. Aspirin
3. A 55-year-old man with hypertension and hyperlipidemia comes to the
emergency department because of chest pain that started an hour ago while
shoveling snow. He describes it as a crushing pain that radiates down his left
arm and rates it as a 7 on a 10-point scale. His pulse is 115/minute, respirations
are 24/min, and blood pressure is 112/68 mm Hg. Cardiac examination shows
no murmurs. A 12-lead ECG with right-sided leads is obtained. Which of the
following medications, if administered, is most likely to cause serious
complications in this patient?
a. Nitroglycerin
16.A 56-year-old man comes to the emergency department because of chest pain
and pressure he experienced an hour ago while shoveling snow. He has had
recurrent bouts of this pain over the past year, and he says that it usually
radiates to his jaw and left arm, and goes away when he sits down to rest.
Today's episode lasted five minutes, he has come to the ED at the urging of his
family. He has a 10-year history of hyperlipidemia and type 2 diabetes mellitus.
Current medications include atorvastatin and metformin. His temperature is
37.1°C (98.8°F), pulse is 87/min, respirations are 19/min, and blood pressure is
130/85 mm Hg. Initial laboratory studies show a troponin I level of less than
0.01 ng/mL. Chest X-ray and ECG show no abnormalities. Which of the
following is the most likely diagnosis?
a. Stable angina
STEP 1
1. A 65-year-old man comes into the emergency room with chest pain and
pressure that started 16 hours ago. The pain radiates up to his jaw and down
his left arm. He states that the pain has waxed and waned, although he has felt
some relief from taking sublingual nitroglycerin. His medical history is significant
for coronary artery disease, stable angina, hyperlipidemia and diabetes mellitus
type 2. His temperature is 36.7°C (98.0°F) , pulse is 110/min, respirations are
24/min, and blood pressure is 135/93 mm Hg. Pulse oximetry on room air shows
an oxygen saturation of 97%. Chest X-ray shows no abnormalities, cardiac
enzymes are negative, and ECG shows ST-segment depression in the lateral
leads. Which of the following is the most likely diagnosis?
a. Unstable angina

STEP 2
1. A 65-year-old male comes into the emergency department because of chest
pain and pressure that started about three hours ago. The pain radiates up to his
jaw and down his left arm. He says that it has waxed and waned over the past
three hours, but has never gone away despite him taking three doses of
sublingual nitroglycerin. Chest X-ray shows no abnormalities and
electrocardiogram (ECG) shows ST-segment depression in the lateral leads.
Troponin measurements are positive, above the upper reference limit. Which of
the following medications is least appropriate for the patient at this present
time?
b. Thrombolytics
2. A 56-year-old man with long-standing hypertension and a 20-year history of
type 2 diabetes mellitus undergoes a coronary angiogram after an episode of
unstable angina. Angiogram reveals stenotic lesions greater than 75% in the
proximal left anterior descending artery and left circumflex artery. His coronary
vasculature is right-dominant. The surgeon determines that the patient has a
high cardiovascular risk and is eligible for a coronary artery bypass graft. During
the procedure, the surgeon detaches a vessel from the chest and creates which
of the following connections?
c. Between the distal end of the left anterior descending artery and the
subclavian artery
3. A 62-year-old man comes to the emergency department after experiencing an
hour of crushing, left-sided chest pain that improved with aspirin and sublingual
nitroglycerine tablets. The pain radiated to his left arm and in addition he had
shortness of breath and diaphoresis. Cardiac enzyme tests show no
abnormalities. An electrocardiogram (ECG) of the patient is shown below.
Which of the following is the most appropriate next step in the management?

d. Anti-coagulation with urgent cardiac catheterization


STEP 1
1. A 28-year-old man comes to the emergency department because of intermittent
chest discomfort for six hours. He describes chest pain and pressure that
radiates to the jaw. Episodes of pain last 5-15 minutes and are associated with
nausea, diaphoresis, dizziness, and dyspnea. He has not noticed any
precipitating factors. He has a 5-year history of migraines, for which he takes an
abortive medication. He has smoked a pack of cigarettes daily for 10 years and
uses cocaine occasionally. His temperature is 37°C (98.6°F), pulse is 99/min,
respirations are 22/min, and blood pressure is 135/93 mm Hg. Initial ECG shows
no abnormalities, but a repeat ECG 30 minutes later during an episode of chest
discomfort shows ST-segment elevation in leads V2 through V6. Laboratory
studies at that time show a troponin I level of 0.01 ng/mL. Which of the
following is the most likely diagnosis?
a. vasospastic angina
2. A 28-year-old man comes to the emergency department because of persistent
chest discomfort for the past 6 hours. He describes pain and pressure in the
center of his chest that radiates to his jaw. Painful episodes last 5-15 minutes
and are accompanied by nausea, diaphoresis, dyspnea, and dizziness. His
medical history is significant for migraine headaches controlled with an abortive
medication that he cannot currently name. His history is also significant for
occasional cocaine use in addition to a 10 pack year tobacco smoking history. His
temperature is 36.6°C (98.0°F), pulse is 99/min, respirations are 22/min, blood
pressure is 135/93 mm Hg, and oxygen saturation is 97% on room air.
Electrocardiogram on arrival shows no abnormalities, but 30 minutes later,
during a pain episode, a repeat electrocardiogram shows ST elevation in leads
V2-V6. Current troponin-I is 0.01. Which of the following is the most
appropriate intervention for this patient?
a. Calcium channel blockers and nitrates
3. A 50-year-old woman comes to the emergency department because of acute
severe chest pain for 30 minutes. She states that the pain woke her from sleep
and that she is very concerned she is having a ‘heart attack.’ She states she has
had similar chest pains in the past, usually also at night, but never this severe.
Cardiopulmonary examination is noncontributory. An ECG is obtained which
shows ST-elevations in leads V1, V2, V3, and V4. Laboratory investigations
show her cardiac enzyme marker levels are within normal limits. At angiography,
no pathological findings are found. Which of the following is the most likely
diagnosis?
a. Variant angina

Atherosclerosis and arteriosclerosis:


Pathology review
STEP 1
1. A 60-year-old man is brought to the emergency department due to acute onset
of chest pain. Medical history is significant for smoking 1 pack per day for the
past 30 years, diabetes mellitus type 2, and hypertension. An ECG shows
ST-segment elevation on V1-V4 leads. Cardiac catheterization is performed and
shows 80% occlusion of the left anterior descending artery. The first step in the
pathogenesis of this patient’s condition most likely involves which of the
following cell types?
a. Endothelial cells
2. A 62-year-old man is brought to the emergency department by his partner after
experiencing crushing substernal chest pain radiating to the left arm for the past
hour. The patient reports experiencing nausea and diaphoresis during this time.
An ECG is performed, which shows sinus rhythm with ST-segment elevation in
leads II, III and aVF. Before cardiac catheterization could be performed, the
patient develops ventricular tachycardia with subsequent hemodynamic
instability. Despite appropriate life-saving measures, the patient dies. Autopsy
shows yellow lipid plaques on the intimal surface of the right coronary artery
with an overlying, completely occlusive thrombus. Examination of several other
coronary arteries demonstrate similar lipid plaques. Which of the following
locations is most likely to show similar lesions?
a. Abdominal aorta
3. A 75-year-old man is brought to the emergency department due to
sudden-onset confusion and headaches accompanied by nausea and vomiting.
The patient has a history of poorly controlled hypertension and hyperlipidemia
due to noncompliance with medications. The patient has smoked a pack of
cigarettes daily for 40 years. The patient’s temperature is 37.0°C (98.6°F), pulse
is 100/min, and blood pressure is 205/140 mmHg. On physical examination, the
patient is dyspneic. Fundoscopic examination reveals bilateral papilledema.
Laboratory results are obtained and shown below: Which of the following
histologic findings would likely be seen if this patient’s renal artery was
biopsied?

a.
4. A 60-year-old woman comes to the clinic due to hip pain that began after she
slipped and fell on her side while going down the stairs 2 hours ago. Past
medical history is significant for osteoarthritis. The patient has had recent
difficulty walking due to hip pain but is otherwise healthy. Vitals are within
normal limits. On physical exam, the right hip is tender to touch and peripheral
pulses are intact. An x-ray of the lower extremities is obtained (shown below)
and no fractures or displacements are seen. Which of the following is
responsible for the appearance of this patient’s femoral arteries?
a. Deposition of calcium without luminal narrowing
e. CREO QUE FALTAN ALGUNAS PREGUNTAS DE ESTE
Prinzmetal angina
17.

● PLAYLIST 7

Abdominal pain: Clinical Practice


STEP 1

1. A 20-year-old woman comes to the emergency department with lower


abdominal pain for the past three days. She says the pain is present on both
sides of her lower abdomen, and is sharp in nature. She also reports irregular
vaginal bleeding and lower back pain for the past seven days, before which she
had an intrauterine contraceptive device placed. Examination shows adnexal
tenderness, uterine tenderness and a mucopurulent cervical discharge. Urine
dipstick shows no abnormalities. Gram stain from a cervical swab shows
Gram-positive filamentous rods. The organism responsible for causing this
infection can be characterized by the presence of which of the following?
a. Sinus tracts containing sulfur granules
2. A 3-year-old boy is brought to the clinic because of irritability, vomiting, fever,
and abdominal pain for the past month. His temperature is 39.2°C (102.56°F),
pulse is 110/min, respirations are 25/min, and blood pressure is 110/70 mm Hg.
Physical examination shows an upper right quadrant abdominal mass on
palpation. A CT scan image is obtained and is shown below. Which of the
following is the most likely diagnosis?
a. Wilms tumor
3. A 10-year-old girl comes to the outpatient clinic because of bloating and
excessive flatulence. Her symptoms worsen when she eats cheese or yogurt.
She has been growing at average pace and does not show signs of malnutrition.
She is afebrile. Physical examination shows abdominal distention, but no signs
of tenderness or guarding. Which of the following is the most appropriate next
step in diagnosis?
a. Hydrogen breath test
4. A 20-year-old woman comes to the emergency department because of bilateral
lower abdominal pain for the past two days. She states that she had an
intrauterine device placed about 2 weeks before, and she has been experiencing
irregular vaginal bleeding and lower back pain ever since. Pelvic examination
shows adnexal tenderness and a mucopurulent cervical discharge. Gram stain
shows gram-positive filamentous rods, and a pregnancy test was negative.
Which of the following is the most likely microorganism causing the patient's
condition?
a. Actinomyces israelii
5. A 35-year-old woman comes to the clinic for a fertility consultation. The patient
has been married to her husband for the past 10 years and they have been
trying to have a child for the past 2 years without success. She has been
receiving hCG injections for the past 4 months and she says that she has been
experiencing nausea, abdominal bloating, excessive weight gain, darker urine,
dyspnea, and lower abdominal pain for the past 2 weeks. The patient says that
symptoms have been worsening. Which of the following is the most likely
diagnosis?
a. Ovarian hyperstimulation syndrome
6. A 23-year-old previously healthy woman comes to the emergency department
because of right sided lower abdominal pain and vaginal bleeding for the past
24 hours. She denies nausea, vomiting, fever, anorexia, and diarrhea. Her last
menstrual period (LMP) was 7 weeks ago. She is sexually active and has
multiple sexual partners, using condoms inconsistently. She is afebrile, pulse is
80/min, respirations are 14/min, and blood pressure is 115/80 mm Hg. Physical
examination shows mild tenderness in the right lower quadrant and right
adnexa, with no rebound tenderness nor guarding. The cervix is closed with
scant blood visible at the os; there is no active bleeding. A quantitative serum
ßhCG is 2000 IU/L. Which of the following is the most likely diagnosis?
a. Ectopic pregnancy
7. A 27-year-old woman, gravida 2, para 1, abortus 1, comes to the emergency
room because of right lower quadrant pain and nausea. She has a history of
ectopic pregnancy and is concerned that this could be the problem. Her last
menstrual period (LMP) was 6 weeks ago. A quantitative b-hCG concentration is
1,851 IU. Which of the following imaging modalities would you use for further
investigation?
a. Transvaginal ultrasound
8. A 26-year-old man is brought to the emergency department an hour after being
stabbed in the abdomen during a street fight. The patient is hemodynamically
stable. Abdominal examination shows a knife wound in the right lower
quadrant, guarding, and rebound tenderness. A full body computed tomography
scan is done and shows the following image:
a. Pneumoperitoneum
9. A 56-year-old man comes to the emergency department because of onset of
severe epigastric pain. He has intractable retching without vomiting. Physical
examination shows a distended abdomen that is dull to percussion. An X-ray
with barium contrast is ordered and the results are shown below. Which of the
following is the most likely diagnosis?
a. Organo-axial volvulus
10.A 29-year-old pregnant woman comes to the emergency department because
of acute, severe right upper quadrant abdominal pain. She has been receiving
regular prenatal care and has had no complications in her pregnancy thus far.
She denies any tobacco use and has not consumed any alcohol for the past 11
months. Her hepatojugular reflex is negative. Abdominal examination shows
hepatomegaly with an abdominal fluid wave. Urinalysis shows no abnormalities.
Serum laboratory studies show a creatinine concentration of 1.6 mg/dL and an
albumin concentration of 2.5 g/dL. Liver enzyme studies show alkaline
phosphatase concentration of 40 U/L, AST concentration of 159 U/L, and ALT
concentration of 184 U/L. Complete blood count shows a leukocyte count of
8,500/mm3. Which of the following is the most likely diagnosis?
a. Hepatic vein thrombosis
11.A 4-year-old boy is brought to the emergency department by his parents
because of severe abdominal pain and constipation for 2 days with no
improvement. Prior to this episode his mother states that he has become
increasingly irritable and lethargic over the past 2 weeks to the point where he
has stopped speaking. Review of the patient's medical history shows that he
failed to meet developmental milestones over the past year. When asked about
this, the parents mention that he has been having a hard time adjusting since
they moved into an older home with many of the original fixtures 15 months
before. Physical examination of the patient shows marked lethargy and is
otherwise normal. Based on the patient's symptoms, which of the following is
the most appropriate treatment?
a. Dimercaprol
12.A 24-year-old woman comes to the office because of intermittent diarrhea and
crampy abdominal pain over the past year. During these episodes, she notices
an increased frequency of bowel movements and softer stool consistency. Her
symptoms get worse with stress and improve when she takes loperamide. The
abdominal pain is decreased after defecation. She denies bleeding, weight loss,
early satiety, family history of colon cancer, and smoking.
Esophagogastroduodenoscopy and colonoscopy with biopsy are unremarkable.
Which of the following is the most likely diagnosis?
a. Irritable bowel syndrome
13.A 56-year-old man comes to the emergency department because of significant
abdominal pain, which started eight hours ago. On general examination, he is in
severe distress, diaphoretic, and nauseated. Physical examination reveals
absent bowel sounds and severe rebound tenderness. An abdominal computed
tomography is performed. Which of the following is the most likely diagnosis?
a. Small bowel infarction with subsequent lumenal air entering into
portal circulation
14.A 25-year-old woman comes to the emergency department because of
abdominal pain. She describes it as intermittent dull, throbbing pain in the left
lower quadrant. She complains of nausea and vomiting in the morning that
usually goes away in the middle of the day. She denies the possibility of being
pregnant as she uses oral contraceptive pills. Her past medical history includes
treatment of a sexually transmitted infection with doxycycline one year ago, and
a history of depression, currently being treated with paroxetine. Serum
beta-hCG levels are 2225 mIU/mL. A transvaginal ultrasound scan (TVUS) is
obtained and is negative for an intrauterine gestation. Which of the following
features on history put this patient at an increased risk for the most likely
diagnosis?
a. Prior infection with Chlamydia trachomatis
15.A 77-year-old woman comes to the emergency department because of acute
abdominal pain. She rates the pain a 10 on a 10-point scale and localizes it to
her umbilicus. She also says she experiences nausea and vomiting. She denies
diarrhea or constipation. She has a past medical history of congestive heart
failure. She has never had any surgeries. She has a 10-pack-year history of
smoking cigarettes. Her temperature is 37.0ºC (98.6ºF), pulse is 80/min,
respirations are 16/min, and blood pressure is 120/90 mm Hg. Physical
examination shows a soft, nontender, nondistended abdomen with no rebound
tenderness or guarding. The lower extremities have 1+ pitting edema. Which of
the following is the most likely pathological process underlying this patient’s
condition?
a. Occlusion of vasculature supplying the bowel
16.A 35-year-old woman comes to the emergency department because of right
lower quadrant abdominal pain that started suddenly and has been present for
the past 3 hours. She also reports a small amount of vaginal bleeding. Her last
period was 6 weeks ago. She denies fever, nausea, or vomiting. She reports that
she is sexually active and had a tubal ligation 2 years ago. Her temperature is
37°C (98.6°F), pulse is 77/min, respirations are 12/min, and blood pressure is
110/70 mm Hg. A urine pregnancy test is obtained and is positive. Serum hCG is
1950 mIU/mL. Which of the following is the most appropriate next step for
diagnosis?
a. Transvaginal ultrasound
17.A 38-year-old woman, with a history of gastric bypass two months ago, comes
to the emergency department because of colicky abdominal pain, severe nausea,
and diarrhea. She has a blood pressure of 86/50 mmHg, her heart rate is 120
bpm, and her skin is sweaty. Her pupils are somewhat dilated but are equal,
round, and reactive to light. She reports a rapid onset of symptoms following a
meal out with friends at a local fast food restaurant, where she had a milkshake
and several cookies. Which of the following complication of bypass surgery is
she experiencing?
a. Dumping syndrome
18.A 60-year-old man with a history of coronary artery disease and tobacco use
comes to the emergency department because of acute abdominal pain. He has
had diffuse mild abdominal pain after meals for the past three months.
However, recently he has experienced more frequent episodes of abdominal
pain and has seen blood in his stool intermittently for the past two weeks. He
rates the pain a 7 on a 10-point scale. Abdominal examination shows no
abnormalities. A computed tomography scan of the abdomen scan shows bowel
wall edema around the splenic flexure extending to the descending colon.
Decreased blood flow in which of the following arteries would most likely
account for the patient's clinical features?
a. Inferior mesenteric artery
19.A 64-year-old man comes to the emergency department because of a 2-day
history of uncontrollable nausea, vomiting, and generalized abdominal pain. His
temperature is 36.8°C (98.2°F), pulse is 112/min, respirations are 20/min, and
blood pressure is 104/64 mm Hg. Physical examination shows a distended
abdomen with decreased bowel sounds that is tympanitic to percussion. An
abdominal radiograph is obtained and is shown below. Which of the following is
the most common underlying cause of the patient's symptoms?
a. Adhesions
20.A 26-year-old woman comes to the emergency department because of lower
abdominal pain and vaginal bleeding for the past 24 hours. Her last menstrual
period was 7 weeks ago. She is sexually active with her boyfriend and they use
condoms intermittently. Physical examination reveals mild tenderness in the left
lower quadrant and left adnexa. The cervix is closed with dried blood visible at
the os; there is no active bleeding. A quantitative serum B-HCG is 1,500 IU/L.
Transvaginal sonography of the patient's left adnexa reveals an extra-uterine
gestational sac with a diameter of 2 cm. Which of the following is the most
appropriate management option for this patient?
a. Intramuscular methotrexate
21.A 51-year-old woman comes to the clinic because of abdominal pain for the
past 12 months. The pain has been vague, located in the epigastric region and
left upper quadrant of the abdomen. She says she has weekly nausea and
vomiting, and intermittent pain in her left shoulder. An arterial angiogram is
obtained and is shown below. Which of the following is the most likely
diagnosis?
a. Splenic artery aneurysm
22.A 35-year-old woman, grava 1, para 0, at 35 weeks' gestation comes to the
emergency department because of painful vaginal bleeding for the past 2 hours.
She says the pain began suddenly and is located in her lower abdomen and
lower back. She is also experiencing contractions every 10 minutes, which have
not increased in frequency or intensity. Her past medical history is significant for
hypertension that has been well managed throughout her pregnancy with
labetalol. She has smoked 10 cigarettes a day for the past 12 years, and has
continued to smoke throughout her pregnancy. She does not drink alcohol, and
has not used illicit drugs. Physical examination shows blood in the vaginal vault
and a closed cervix. Which of the following is the most likely diagnosis?
a. Placental abruption

STEP 2.

1. A 22-year-old woman comes to the emergency department with lower


abdominal pain for the past five days. She says the pain is present on both sides
of her lower abdomen, and is sharp in nature. She also reports irregular vaginal
bleeding and lower back pain for the past seven days, before which she had an
intrauterine contraceptive device placed. Examination shows adnexal
tenderness, uterine tenderness and a mucopurulent cervical discharge. Urine
dipstick shows no abnormalities. Gram stain from a cervical swab shows
Gram-positive filamentous rods. Which of the following is the most appropriate
pharmacological treatment for this patient?
a. Penicillin G
2. A 66-year-old man is brought to the emergency department because of
worsening left lower quadrant pain and diarrhea over the past 2 days. His
temperature is 38.7°C (101.6°F), pulse is 115/min, respirations are 20/min, and
blood pressure is 145/85 mm Hg. Physical examination shows left lower
quadrant tenderness. Laboratory studies show leukocytosis. Which of the
following is the most appropriate next step in diagnosis?
a. Computed tomography scan
3. A 13-year-old boy is brought to the emergency department for evaluation of
testicular pain and swelling. The patient was running when he felt sudden onset
groin pain. Since then, he experienced severe scrotal pain with associated
nausea and vomiting. He has had similar episodes of pain over the past two
months that have all self-resolved without intervention. He has no significant
past medical history and has received all age-appropriate vaccinations.
Temperature is 37.0°C (98.6°F), pulse is 94/min, respirations are 21/min, and
blood pressure is 141/80 mmHg. Physical examination demonstrates an
uncomfortable adolescent male in significant pain. Examination demonstrates an
edematous and erythematous right scrotum. Stroking of the right inner thigh
does not result in elevation of the right testis. The pain is worsened with
elevation of the scrotum, and the scrotum does not transilluminate. Which of the
following is the best next step in management?
a. Surgical fixation of the bilateral testicles
4. A 42-year-old man comes to the emergency department because of 3-day
abdominal pain. The pain occurs intermittently throughout the day, but is not
associated with meals or sleep. He has a history of rheumatoid arthritis, for
which he takes multiple non-opioid pain relief medications per day. He
otherwise appears well in spite of his acute pain. His vitals show no
abnormalities and he has epigastric tenderness without distention. Which of the
following diagnostic tests is the most appropriate next step in the management
of this patient?
a. Complete blood count
5. A 58-year-old man comes to the emergency department because of severe
abdominal pain and vomiting. He has felt this pain for months, but it has
become intolerable over the last 8 hours. He has a history of gastroesophageal
reflux disease (GERD), hypertension, and asthma. He has not had a bowel
movement or passed flatus in 3 days. Since the pain worsened, he is now having
trouble moving. His temperature is 39.1°C (102.5°F), pulse is 115/min, and
blood pressure is 135/75 mm Hg. Physical examination shows a distended
abdomen with high pitched bowel sounds. He appears to be in severe distress,
secondary to pain, and refuses to straighten his legs because it hurts too much.
Which of the following is observed on the barium follow-through due to this
patient disease?
a. Dilated small intestines
6. A 3-year-old boy is brought to the emergency department by his mother for
abdominal pain and irritability. She states that for the past two weeks he has
been increasingly lethargic and has had frequent spells of colicky abdominal
pain with nausea and vomiting. She also relates that he has recently been
fixated on eating ice and soil, and has been behaving strangely. The patient has
not had any recent dietary or lifestyle changes, and recently went with his
parents on fishing trip to Mexico. The patient's vital signs are within normal
limits. Physical examination of the patient shows a well developed male with
marked pallor and lethargy. Laboratory studies show a white blood cell count of
4,000/mm3, hemoglobin of 8.2 g/dL, and a platelet count of 400,000/mm3.
Which of the following is most likely to be associated with the patient's clinical
presentation?
a. Lead poisoning
7. A 12-year-old previously healthy boy is brought to the emergency department
by his mother because of nausea, vomiting, fever, non-bloody diarrhea and
cramping abdominal pain for the past 24 hours. His mother states that he woke
up the day before with a fever of 38.6°C (101.6°F) and has been having 5-6
loose stools per day ever since. She also mentions that he was helping her cook
chicken cutlets two days ago. Physical examination shows a dehydrated male in
moderate distress, and abdominal examination shows diffuse tenderness to
palpation without rebound. Stool culture grows gram-negative,
lactose-negative rod that form translucent colonies on MacConkey agar. Which
of the following is the best course of treatment?
a. Replace fluids and electrolytes
8. A 12-year-old boy comes to the emergency department with severe right lower
quadrant abdominal pain. Parents note the patient has felt hot for 3 days. His
temperature is 38.9°C (102.0°F). Laboratory studies show leukocytosis. The
patient is brought to the operating room for an appendectomy, and the surgeon
finds visible inflammation around the appendix, terminal ileum, and mesenteric
lymph nodes, but the appendix itself is normal. A week later the patient begins
complaining of joint pain, dysuria, headache, and malaise. He has had bloody
diarrhea for the past week. What is the most likely source of the pathogen
causing his illness?
a. Contaminated pork or milk
9. A 62-year-old man with a history of atrial fibrillation and previous myocardial
infarction comes to the emergency department because of abdominal pain,
nausea, vomiting, and loss of appetite for the past five hours. The patient rates
the pain an 8 on a 10-point scale. He says that the pain is constant and “all
over” his abdomen, unable to point to a specific location. He took 500 mg of
acetaminophen but did not notice any improvement in his symptoms. He also
says he had an episode of hematochezia prior to coming to the emergency
department. Which of the following is the most appropriate diagnostic step in
the management of this patient?
a. Contrast CT
10.A 50-year-old woman comes to the office because of nausea, vomiting, and
decreased appetite for the past four days. Her medical history shows systemic
lupus erythematosus (SLE) and fibromyalgia, for which she requires chronic
opiate therapy. Current medications include prednisone, hydroxychloroquine,
and a daily multivitamin. She recently was treated for community-acquired
pneumonia with azithromycin. She works as a nurse at a local hospital and
denies use of drugs, alcohol, or tobacco products. Abdominal examination
shows a soft, distended abdomen that is diffusely tender to palpation. Which of
the following is the most likely underlying cause of her complaints?
a. Hypomotility of the gastrointestinal tract
11.A 23-year-old nulliparous woman comes to the office because of chronic
episodic pelvic pain. The pain begins 2-3 days before menses and continues
throughout the menses after which it subsides. Menarche was at age 14. She
has a regular menstrual cycle of 25 days with 5-6 days of moderate bleeding.
Medical history is significant for recurrent migraine headaches with aura. She is
not sexually active. Physical examination shows a fixed anteverted uterus and
nodularity in the cul-de-sac. The physician offers surgical therapy, which the
patient declines at this time. posteriorWhich of the following is the most
appropriate option to treat this condition medically?
a. Progestin-only pills
12.A 1-week-old infant is referred to a neonatal consultant because of abdominal
distension and rectal bleeding. The infant was observed in the neonatal
intensive care unit after premature birth at 35 weeks' gestation. Abdominal
radiography series shows pneumatosis intestinalis. Which of the following is the
most likely diagnosis?
a. Necrotizing enterocolitis
13. A 60-year-old man comes to the emergency department because of fever and
severe right-sided upper and lower abdominal pain for the past 12 hours. He
has had no bowel action for the past 5 days. He was recently diagnosed with
lymphoma of the liver and was discharged several days ago after starting his
first cycle of Hyper-CVAD chemotherapy. His abdomen is severely distended
and there rebound and percussion tenderness exists. A nasogastric tube is
inserted, he is started on IV fluids and antibiotics, and a CT scan is ordered.
Which of the following is the most likely diagnosis?
a. Neutropenic enterocolitis
14.A 26-year-old woman, gravida 2, para 2, comes to the office because of episodic
right abdominal pain for the past week. She says the pain is dull and
intermittent, located in the right upper quadrant. Her medical history is
significant for well controlled hypertension, an appendectomy at the age of 14,
and two prior cesarean deliveries. Her medications include cilazapril and the
combined oral contraceptive pill. Her vital signs show no abnormalities.
Examination shows moderate tenderness in the right upper quadrant without
guarding or rebound tenderness. A abdominal computed tomography scan
shows a 5cm heterogenous, well-circumscribed lesion in the right lobe of her
liver (shown below). Which of the following is the most likely diagnosis?
a. Hepatic adenoma
15. A 62-year-old woman comes to her outpatient provider seeking evaluation for
postprandial abdominal pain. The symptoms started 6 months ago and have
progressively worsened. She reports that the pain begins 30-40 minutes after
meals. She has been eating less due to the pain, and she has experienced a 5 kg
(11 lb) weight loss since symptom onset. The patient denies nausea, vomiting,
and diarrhea. Past medical history is notable for hypertension and
hyperlipidemia. The patient has a 40-pack-year smoking history. Her younger
sister was diagnosed with colorectal cancer at age 54. Laboratory testing
reveals the following findings
a. Perform CT angiography
16.A 74-year-old woman arrives via ambulance to the emergency department
complaining of increasing abdominal pain, lethargy, and anorexia beginning
three days ago. She reports some nausea, but denies vomiting. Her last bowel
movement was two days ago. Upon arrival in the emergency department, her
vital signs are as follows: T 100.0 F P 75 bpm R 14 bpm BP 100/70 SaO2: 99%
(room air). Physical exam shows an ill-appearing elderly woman with a
distended, diffusely tender abdomen that is tympanic to palpation. An
abdominal X-ray is ordered, which shows the following:
a. Reduction via flexible sigmoidoscopy
17.

Apendicitis
STEP 1

1. A 25-year-old man comes to the emergency department because of


excruciating abdominal pain. The abdominal pain is localized to the right lower
quadrant. A day ago, the abdominal pain was located in the periumbilical region
and was less severe. His temperature is 37°C (98.6°F), pulse is 95/min,
respirations are 18/min, and blood pressure is 130/80 mm Hg. Abdominal
examination shows tenderness with deep palpation. Laboratory studies show a
leukocyte count of 15,000/mm3. Which of the following is the most sensitive
finding for the patient's likely diagnosis?
a. Pain 3-5 cm from the anterior superior iliac spine on a straight line
from the anterior superior iliac spine to the umbilicus
2. A 19 year-old woman comes to the emergency department because of intense
right lower quadrant abdominal pain. The abdominal pain is sharp in character
and has been present for two hours. Her temperature is 37°C (98.6°F), heart
rate is 90/min, respirations are 16/min, and blood pressure is 120/70 mm Hg.
Abdominal examination shows tenderness with deep palpation at the McBurney
point. There are no peritoneal signs. Laboratory test shows a leukocyte count of
13,000/mm3. Which of the following symptoms is most likely characterized in
this patient?
a. Nausea and vomiting
3. A 29-year-old woman, gravida 1, para 0, at 19 weeks gestation, comes to the
emergency department because of nausea, vomiting, fever, and abdominal pain
for the past two days. She says she has lost her appetite. Her temperature is
38.8°C (101.8°F), pulse is 92/min, respirations are 17/min, and blood pressure is
135/80 mm Hg. Physical examination shows moderate tenderness to palpation
in the right lower quadrant and right flank tenderness. Which of the following is
the most appropriate next step in management?
a. Graded compression ultrasound
4. A 21-year-old man comes to the emergency department because of abdominal
pain. The pain started 12 hours ago, started around the umbilical region and has
since moved to his right lower quadrant (RLQ). He also has nausea and anorexia
which also started 12 hours ago. On examination, his temperature is 36.7°C
(98.1°F), heart rate is 90/min, respirations are 16/min, and blood pressure is
120/70 mm Hg. Abdominal examination shows tenderness with deep palpation
at the McBurney point. There are no peritoneal signs. Which of the following is
the most likely diagnosis?
a. Appendicitis
5. A 47-year-old man comes to the emergency department because of intense
abdominal pain. The abdominal pain is sharp in character and associated with
nausea and vomiting. He has type 1 diabetes, which he has managed with
insulin since childhood. The patient further says that he had been experiencing
abdominal pain all day but thought it was secondary to the flu. His temperature
is 36.7°C (98.1°F), heart rate is 70/min, respirations are 18/min, and blood
pressure is 125/70 mm Hg. Abdominal examination shows right lower quadrant
tenderness with palpation. Which of the following is the most appropriate
choice to confirm this patient's diagnosis?
a. Abdominal CT scan
6. A 66-year-old man comes to the emergency department because of severe pain
in his right lower back. For several days before this back pain began, he had
been feeling abdominal discomfort in his right lower quadrant. He denies any
history of heavy lifting, weight loss, and dysuria. His temperature is 38.8°C
(101.8°F), heart rate is 90/min, respirations are 16/min, and blood pressure is
120/70 mm Hg. Physical examination shows tenderness to palpation in the right
lower quadrant, rebound tenderness, and no costovertebral angle tenderness.
An ultrasound of the right lower quadrant shows a fluid collection surrounding
an organ. Which of the following is the most likely diagnosis?
a. Appendiceal abscess
7. A 23-year-old man comes to the emergency department because of nausea,
fever, and abdominal pain. The abdominal pain started around his umbilicus and
has since moved to the right lower quadrant. On examination, his temperature is
38.8°C (101.8°F), pulse is 90/min, respirations are 16/min, and blood pressure is
120/70 mm Hg. Physical examination shows positive Rovsing and McBurney
signs. After surgery, the patient recovers uneventfully. Which of the following
bacterium is least likely to cause symptoms similar to that of this patient's
presentation?
a. E. coli O157:H7
8. A 49-year-old man comes to the emergency department because of intense
abdominal pain. The pain is sharp in character and associated with nausea and
vomiting. He has type 1 diabetes, which he has managed with insulin since
childhood. He has also been experiencing abdominal pain all day but thought it
was secondary to the flu. His temperature is 38.8°C (101.8°F), pulse is 70/min,
respirations are 18/min, and blood pressure is 125/70 mm Hg. Abdominal
examination shows right lower quadrant tenderness with palpation and a
positive psoas sign. Which of the following is the most appropriate next step in
management?
a. Laparoscopic removal of the infected organ

Appendicitis: Clinical practice


STEP 1

1. A 35 year-old investment banker with a history of liver injury is undergoing


surgery for an appendectomy. You are his anesthesiologist; which of the agents
below is best suited for the patient’s anesthesia?
a. Sevoflurane

STEP 2

1. A 48-year-old woman is reviewed on the surgical ward because of fever and


wound discharge for 30 minutes. She had an open appendectomy for
gangrenous appendicitis 5 hours ago and her post-operative recovery has been
uneventful so far. Her medications include metformin and intermediate-acting
insulin. Her temperature is 38.0°C (100.4°F), pulse is 87/min, respirations are
18/min, and blood pressure is 117/78 mm Hg. Physical examination shows that
the area around the bandage has become extremely edematous and has taken
on a dusky grayish-purple color. There is also some thin, watery brown
discharge that is draining from the wound. Which of the following is the most
appropriate next step in management?
a. Surgical debridement
2. A 24-year-old woman presents to the emergency department with nausea,
vomiting, and worsening abdominal pain for the last 12 hours. She states the
pain started all of sudden with no inciting factors. Her symptoms were not
associated with any particular meal, and she denies any recent travel. The
patient has a history of mild asthma, for which she takes an albuterol inhaler as
needed. Her temperature is 39°C (102.2°F), pulse is 84/min, respirations are
18/min, and blood pressure is 128/74 mmHg. Physical examination shows right
lower abdominal pain that is elicited with deep palpation. There is no
hepatosplenomegaly. Cardiac and pulmonary exams are noncontributory. A
urine β-hCG test is ordered and returns negative. Leukocyte count is
12,000/mm3. Which of the following is the next best step in management?
a. Computed tomography of the abdomen
3. A 66-year-old man comes to the emergency department because of severe
abdominal pain in his right lower quadrant. The abdominal pain is sharp in
character and has been present for three hours. His temperature is 37°C
(98.6°F), pulse is 95/min, respirations are 18/min, and blood pressure is 130/80
mm Hg. Abdominal examination shows tenderness with deep palpation at
McBurney point. There are no peritoneal signs. Laboratory studies show a
leukocyte count of 16,000/mm3. Which of the following is the definitive
treatment?
a. Appendectomy
4. A 24-year-old man presents to the emergency department with worsening
abdominal pain that began all of a sudden after eating pizza two nights ago. The
patient reports the pain has progressively worsened and is now 8 out of 10 in
severity. He reports no change in bowel movements during this time, but he has
had a decreased appetite and moderate nausea. Medical history includes lactose
intolerance. His temperature is 37.9°C (100.2°F), pulse is 72/min, respirations
are 14/min, and blood pressure is 108/66 mmHg. Abdominal examination
shows right lower quadrant pain with passive right hip extension. Which of the
following will most likely provide definitive treatment for this patient’s
condition?
a. Surgical intervention
5.
Bowel obstruction
STEP 1

1. A 50-year-old woman comes to the emergency


department with severe colicky abdominal pain
that has worsened over the past 48 hours. She
has a past history of endometriosis and a surgical
history of appendectomy and a cesarean section.
Physical examinations shows a distended
abdomen that is tympanitic to percussion. An
abdominal radiograph is obtained and is shown
below. Which of the following is the most likely
diagnosis?
a. Small bowel obstruction
2. A 30-year-old woman gravida 1, para 1, comes to the office because of colicky
abdominal pain for 12 hours. She states that along with the pain she has felt
nauseated and vomited twice. She has never experienced these symptoms
before, but her medical history includes appendicectomy 2 years ago. Her
temperature is 36.5ºC (97.7ºF), pulse is 87/min, respirations are 18/min, and
blood pressure is 116/82 mm Hg. Examination shows abdominal distension.
The abdomen is tender to palpation, especially in the right lower quadrant.
Bowel sounds are present and high-pitched. There are no signs of peritonism.
Which of the following is the most likely diagnosis?
a. Small bowel obstruction
3. A 62-year-old man is brought to the emergency department complaining of
severe cramping abdominal pain which he has been experiencing since last
night. He has had 3 episodes of vomiting
since this morning and no bowel movements.
On examination, he is vitally stable and his
abdomen is mildly distended with
hyperactive bowel sounds on auscultation.
His past medical history includes celiac
disease and his surgical history is significant
for a cholecystectomy 3 weeks ago. An abdominal X-ray is ordered and shown
below. Which of the following is the most likely underlying etiology causing this
patient's presentation?
a. Post-surgical adhesions

STEP 2

1. A 50-year-old woman comes to the emergency department because of colicky


abdominal pain for the past hour. She has a medical history of endometriosis
and she has had an appendectomy and multiple cesarean sections. Physical
examination shows a distended abdomen that is tympanitic to percussion. An
abdominal radiograph shows dilated loops of bowel with air-fluid levels. Which
of the following is the most likely underlying condition in this patient
a. Surgical adhesions

Bowel obstruction: Clinical practice


STEP 1

1. You are an anesthesia resident discussing your upcoming patient with your
attending. Your patient is a 70-year-old male with bowel obstruction. What
would be the adverse effects of nitrous oxide use in this patient?
a. Nitrous oxide is highly diffusable into air spaces and could exacerbate
this patient's bowel obstruction
2. A 35-year-old man comes to the emergency department because of severe
periumbilical, colicky pain and abdominal distention for the past two days. He is
nauseous and reports vomiting light green emesis eight times since yesterday
morning. He has not had a bowel movement or passed any gas for the past
three days. His past medical history is remarkable for ventral hernia repair five
years ago. His temperature is 38°C (100.4°F), pulse is 110/min, respirations are
24/min, and blood pressure is 120/90 mm Hg. The abdomen is distended and
mildly tender to deep palpation. Bowel sounds are high pitched and tinkling.
Which of the following is the most likely diagnosis?
a. Small bowel obstruction
3. A 68-year-old man comes to the emergency department because of complaints
of gas, abdominal pain, nausea, and vomiting. He says he has not "pooped in 3
days" and nothing seemed to make it better. Yesterday he started to feel more
uncomfortable and his abdomen became more distended. Physical examination
shows guarding, diffuse tenderness, and positive rebound tenderness. His
abdomen is diffusely tympanic. Abdominal CT was inconclusive so a barium
enema was ordered. A bird's beak sign is noted. Which of the following is the
most likely diagnosis?
a. Sigmoid volvulus
4. A 45-year-old man comes to the emergency department because of sudden
onset of severe epigastric pain. He has intractable retching without vomiting.
The nasogastric intubation tube is unable to pass and put into place. He also
complains of severe pain at the top of his left shoulder. An upper
gastrointestinal contrast radiographic study is ordered. Which of the following
was most likely seen on the radiograph?
a. Mesenteroaxial volvulus
5. A 45-year-old man comes to the emergency department because of sudden
onset of severe epigastric pain. He has intractable retching without vomiting.
The nasogastric intubation tube is unable to pass and appropriately placed. He
also complains of severe pain at the top of his left shoulder. Which of the
following is the most likely diagnosis?
a. Gastric volvulus

STEP 2

1. A 35-year-old woman comes to the emergency department because of nausea,


vomiting, and abdominal pain over the past 24 hours. Her last bowel movement
was three days ago. She describes her emesis as non-bloody and non-bilious.
Her temperature is 36.5oC (97.7oF), pulse is 85/min, respirations are 20/min,
blood pressure 116/80 mmHg (sitting), 95/65 mmHg (standing). Examination
shows a diffusely tender, distended abdomen with hyperactive bowel sounds.
There is no rebound tenderness or guarding. Initial laboratory results are as
follows: Hematocrit: 45% Leukocyte count: 8,000/mm Sodium: 145 mEq/L
Potassium: 3.1 mEq/L Creatinine: 0.9 mg/dL AST: 20 U/L ALT: 12 U/L An
abdominal radiograph is obtained and is shown below. Which of the following
historical findings would most likely explain this patient’s current presentation?
a. Appendectomy as a child
● PLAYLIST 8
STEP 1

1. A 27-year-old female comes to the emergency department because she was


involved in a house fire 50 minutes ago. She is conscious, but in significant pain.
Examination shows that she has significant burns to her entire head and neck,
anterior chest and abdomen, and entire right leg. Her temperature is 36.8°C
(98°F), pulse is 135/min, respirations are 26/min, and blood pressure is 98/60
mm Hg. Which of the following most likely represents the total percentage of
her total body surface involved in the burn?
a. 45%
2. A 9-year-old boy is brought to the emergency department because he sustained
burns to his torso 40 minutes ago. His mother states that she was cooking and
drinking a cup of coffee while the boy was on the kitchen table playing with
toys. Sometime later, she went to the bathroom and when she came back the
child was screaming and crying with the coffee spilled over the front of his
pajamas. His vital signs are within the normal ranges. Examination shows that
the patient is crying because of pain. There is a non-blanching erythematous,
wet looking wound with a few thick-walled blisters that are painful to the touch
on his chest. There are no other burn sites on the patient. Which of the
following is the most likely diagnosis?
a. Deep partial thickness burn
3. A 19-year-old woman comes to the clinic because of a rash on her thighs. She
has just returned home from her first semester of college for the holidays. She
states that she first began to notice the rash when she first went to college and
began living in her dorm room. The rash is most prominent after she has been
studying on her laptop. The affected skin is mildly itchy and has a "lace-like"
pattern.
a. Ultraviolet radiation burn
4. A 56-year-old man comes to the emergency department because he burned his
left hand two days ago. He has now noticed that the wound is producing a blue
green pus. His temperature is 39℃ (102.2℉), pulse is 90/min, respiratory rate is
16/min, and blood pressure is 126/82 mm Hg. Culture of the bacteria shows a
gram negative, oxidase positive bacteria that does not ferment lactose on
MacConkey agar. The man gives a history of consuming mostly raw, uncooked
vegetables in the last few days. Which of the following best describes the
associated virulence factor?
a. Exotoxin A

STEP 2

1. A 35-year-old man comes to the emergency department because of severe


burns sustained in a house fire an hour ago. The patient is conscious and states
that he is in significant pain. His temperature is 36.8°C (98°F), pulse is 123/min,
respirations are 23/min, and blood pressure is 117/78 mm Hg. Examination
shows that he has sustained partial and full-thickness burns to both arms, his
right leg, and his anterior chest. During the first 24 hours following injury, the
appropriate strategy for fluid resuscitation begins with which of the following?
a. Lactated Ringer's solution
2. A 27 year old female arrives at the trauma bay after being involved in a house
fire. She has significant burns to her entire head/neck, anterior chest/abdomen
and right leg. Her vitals are P 135, BP 98/60, RR 26, O2 Sat 95% and GCS 10.
She weighs approximately 60 kg. Which of the following choices best
represents the resuscitation needs of this patient?
a. Give her 5.4 L of fluid in the first 8 hours, then another 5.4 L over the
next 16
3. A 14-year-old boy comes to the emergency department because he was caught
in a house fire 30 minutes ago. He is currently spontaneously breathing at a
rate of 35/min and his oxygen saturations are 88% on room air. Examination
shows he has some superficial and partial thickness burns on all four
extremities. He also has deep burns on his forehead and occipital region. His
estimated total burn area is about 75%. His nares appear inflamed, with singed
hair. Which of the following is most likely the most important step in the
management of this patient?
a. Endotracheal Intubation
4. A 25-year-old man comes to the emergency department after escaping from a
burning building 40 minutes ago. He states that he is in severe pain. Physical
examination shows burn injuries on the anterior and posterior surfaces of the
right arm, the anterior and posterior right leg, the chest, and the abdomen. His
temperature is 37.8°C (100°F), pulse is 110/min, respirations are 23/min, and
blood pressure is 117/78 mm Hg. Which of the following is the most important
to monitor for adjusting fluid administration?
a. Hourly urine output
5. A 19-year-old man comes to the emergency department after being rescued
from a burning building 20 minutes ago. His right lower leg was trapped under
a burning beam, resulting in a severe full-thickness and circumferential burn. He
sustained no other injuries and he is currently stable on room air. In the ED, he
begins feeling severe pain in his right foot and toes. Examination shows there is
increased edema of the lower leg, pain with passive movement of the toes, and
subjective "tingling" in the right foot. Dorsalis pedis and posterior tibial pulses
are palpable bilaterally. There is no cyanosis. Which of the following is the most
appropriate next step in management?
a. Emergency escarotomy
6. A 45-year-old man comes to the emergency department because of partial
thickness burns to his anterior upper thighs and medial forearms after his grill
exploded 20 minutes ago. There is no singed hair on his face and his airway is
intact. Examination shows that the wounds are patchy on the thighs, face, and
forearms and that <10% of the patient's body is covered in burn wounds. Which
of the following is most likely correct regarding further treatment of the patient?
a. Intact blisters should be left alone
7. A 17-year-old boy comes to the emergency department because of a burn on
his left foot and left hand from a skillet 35 minutes ago. The boy states that he
tried to grab the skillet, but the handle burned his hand and he dropped it on his
foot. Examination shows that the wounds on his hand and foot go through the
epidermis and into the dermis, are red in appearance, and are dry. He states that
the wounds don't hurt as much as he expected, just around the edges. Which of
the following is the most appropriate treatment?
a. Skin graft, with careful attention to potential graft contraction given
the site of the wound.
8. A 25-year-old male is brought to the emergency department after escaping
from a burning building. Physical examination reveals burn injuries on the
anterior and posterior surfaces of the right arm, anterior and posterior right leg,
chest, and abdomen. His blood pressure is 120/75 mm Hg, and his only
complaint is severe pain. The patient's current weight is 50 kg (110 lbs). Which
the most appropriate fluid management in this patient?
a. 200 X 45 mL lactate ringer plus maintenance in 24-hours
9. A 43-year-old man calls emergency medical services minutes after splashing
bleach in his face and eyes. He works with a partner, who is able to drive him to
the hospital. Which of the following is the most appropriate recommendation to
give the patient for immediate management, prior to coming to the emergency
department?
a. Irrigate the eye

Burns: Clinical Practice


STEP 1

1.

STEP 2

Toxidromes: Clinical practice


STEP 1

1. A 24-year-old male is brought to the emergency department by his roommate.


He was found in his room in an agitated state and appears to be experiencing
visual hallucinations. His roommate discloses that the patient spent the
weekend in the woods hunting for mushrooms and wonders whether he might
have tried the ones he found. He is unaware of any other past history,
medications or allergies. The patient himself is confused and doesn't focus on or
respond to your questions but does keep mentioning wanting to use the
bathroom. On examination the patient's skin is quite pink and dry. His vitals
show he is febrile, with a heart rate of 104 and a blood pressure of 90/58.
Neurological examination is difficult without the patient's cooperation but you
observe that his movements are uncoordinated and he has bilateral pupillary
dilation that does not respond to accommodation. You organize intravenous
access, supplemental oxygen, cardiac monitoring and continuous pulse
oximetry, What medication is most appropriate in this case?
a. Physostigmine
2. A 44-year-old woman with bipolar disorder develops proteinuria after a recent
change in medication. Assuming that the proteinuria is a result of a drug
reaction, which of the following medications is most likely to be responsible for
this symptom?
a. Carbamazepine
3. A 45-year-old woman comes to the emergency department because of
dysfunctional uterine bleeding. A hysterectomy is performed without
complication. The morning after her hysterectomy, she complains of nausea,
headache, and tremulousness. During examination, she suddenly loses
consciousness and experiences a brief tonic-clonic seizure that stops without
intervention. The patient has never experienced symptoms like these before.
She has no history of seizures. Which of the following examination findings is
most likely to be observed in this patient?
a. Cirrhosis
4. A 50-year-old woman comes to the office with the complaint of loss of visual
acuity. History reveals the patient also has increased sweating and leg pain.
Social history reveals the patient eats a diet with a lot of fish. Vital signs reveals
a blood pressure of 142/82 mmHg, a heart rate of 82/min, and a respiratory rate
of 18/min. Physical examination shows decreased visual fields and decreased
sensation to light touch and pinprick in the lower extremities. Laboratory data
reveals a creatinine of 2.2 mg/dL and a BUN of 25 mg/dL. Which of the
following is the most appropriate treatment for this patient?
a. Succimer
5. A 33-year old male comes to the emergency department screaming about
"spaceships" and clutching his chest and abdomen saying that aliens "probed"
him. He was found by concerned relatives in a field near his home. On physical
exam his pupils do not respond to light, he has dry mucous membranes, and
suprapubic fullness. He is rapidly given sedation with 10 mg of IV midazolam.
Which of the following should also be administered?
a. Physostigmine
6. A 42-year-old man comes to the emergency department for evaluation after a
motor vehicle accident. Paramedics say he jumped in front of a car that was
leaving a parking garage at low speed. Past medical history is significant for
hepatitis C, cirrhosis, and alcoholism. Physical examination shows an intoxicated
male with slurred speech and lethargy, unable to fully cooperate with the exam.
Cervical spine is non-tender, and there are several superficial abrasions on his
limbs. Cardiopulmonary exam shows no abnormalities. 8 hours after initial
physical examination, the patient becomes febrile, diaphoretic, tachycardic,
agitated, and tremulous while remaining fully alert and oriented. Which of the
following is the most likely etiology of his current symptoms?
a. Alcohol withdrawal
7. A 25-year-old woman with a history of a panic disorder is brought to the
emergency department by her partner. She is unable to walk, responding
minimally to questions, and appears to be falling asleep. Her partner reports
that the patient is disorientated and has noticed slurring of her speech over the
past day. Her medications include albuterol, alprazolam, and gabapentin.
Assuming that her symptoms represent a medication overdose, which of the
following can be used as an antidote for her toxicity?
a. Flumazenil
8. A 25-year-old man comes to the emergency department because of worsening
visual hallucinations for the past week. He says that he has schizophrenia and
has been noncompliant with his medications. In the emergency department he is
administered a 5-mg injection of haloperidol. Six hours later he has diffuse
muscular rigidity and diaphoresis. His temperature is 39.4ºC (103.0ºF), pulse is
89/min, respiratory rate is 18/min, and blood pressure is 130/91 mm Hg. Over
the next 24 hours, the patient becomes increasingly obtunded. Which of the
following is the most likely diagnosis?
a. Neuroleptic malignant syndrome
9. An 85-year-old man with a history of congestive heart failure is brought to the
emergency department by his daughter due to abdominal pain, nausea,
dizziness, and confusion that started a few hours before. The patient denies
recent falls, chest pain or shortness of breath. His daughter produced his
medication list which includes digoxin, spironolactone, a daily multivitamin and
omega-3 fish oil. The daughter usually cues the patient to take his medication,
but states the patient took them on his own this morning. Physical examination
shows bradycardia and hypotension. Electrocardiography shows high-grade 2nd
degree atrioventricular block (A:V ratio of 4:1). Which of the following is the
most likely diagnosis?
a. Digoxin toxicity
10.A 19-year-old man with a history of opioid dependence comes to the office to
express his interest in getting clean and commencing rehabilitation. His medical
history is relevant to heroin abuse. Upon medical interrogation, the patient
claims that he has previously attempted to rehabilitate on his own, using only
his will to stop using heroin, to which he reports that it has been a waste of
time. Which of the following pharmacological agents should the physician
recommend?
a. Methadone
11.A 70-year-old man comes to the internal medicine outpatient clinic for regular
follow-up. 10-years ago, he underwent a distal gastrectomy for gastric
carcinoma. He reports that he has been feeling fatigued for the past 3 months.
He also notes a tingling sensation in his toes and fingers. Physical examination
shows bilateral brisk knee reflexes and loss of proprioception bilaterally. Which
of the following is the most likely diagnosis?
a. Vitamin B12 deficiency
12.A 55-year-old woman comes to the office complaining of sudden visual
changes. History reveals the patient also has increased sweating and leg pain.
When asked about her diet, she reports changing to a strict pescatarian diet last
year. Her vitals include a blood pressure of 140/80 mmHg, a heart rate of
80/min, and a respiratory rate of 20/min. Physical examination shows decreased
visual fields and decreased sensation to light touch and pinprick in the lower
extremities. Laboratory data shows a creatinine of 2.5 mg/dL and a BUN (blood
urea nitrogen) of 25 mg/dL. Which of the following is the most likely diagnosis?
a. Mercury poisoning
13.A 35-year old woman with chronic back pain presents with several days of
constipation. She is on a regimin of hydrocodone for her back pain. She has tried
various laxatives, but none have been adequate. She now presents with
minimally active bowel sounds, and a physical exam remarkable only for mild
fullness in the area of the sigmoid colon. An abdominal x-ray does not show any
evidence of obstruction other than mild fullness and normodistributed gas in
areas of the large bowel. What pharmacological option could be explored to
treat her constipation and increase motility?
a. Naloxegol
14.A 65-year-old man is found to have a
deep venous thrombosis in the right
common femoral vein post-operatively
following a right total hip replacement.
He is initially placed on a weight-based
heparin protocol and remains in the
hospital for three days without further
complications. He is discharged home
with an oral medication to continue anticoagulation therapy. Two days following
discharge, the patient presents to the emergency department with a necrotic
skin lesion of the left lower extremity, as shown in the exhibit. His skin findings
are most likely due to which of the following?
a. Improper dosing of warfarin
15.A 79-year-old man with a history of acne vulgaris, hypertension,
gastroesophageal reflux disease and osteoarthritis comes to the hospital for an
elective knee replacement surgery. While being prepped, physical examination
shows a darkening of his arms and neck (see image below). The patient states
that this has been happening over the last few years and that it hasn't been a
problem. Medication reconciliation shows that he is taking atenolol, felodipine,
celecoxib, oxybutinin, rabeprazole, and minocycline regularly. While in surgery,
the orthopedist notices greenish pigmentation of his bones. Which medication is
most likely to be responsible for these skin changes?
a. Minocycline
16.A 44-year-old woman is observed with a physiologic tremor in the psychiatric
ICU. Her current medications are Vitamin B12 supplements, zolpidem,
diphenhydramine, and fluoxetine. Caffeinated food products are not permitted in
the ICU. Assuming the tremor is medication induced, substitution of which drug
is most likely to eliminate or reduce the observed tremor?
a. Fluoxetine
17.A 52-year-old man is brought to the emergency department by police after he
was found behind a local restaurant vomiting. Upon presentation the patient is
combative, uncooperative, and smells of alcohol. Physical examination reveals
signs of decreased sensation of the lower extremities. When gazing to the left
the right eye only minimally adducts while the left eye abducts with nystagmus.
The patient is most likely suffering from a deficiency in which of the following?
a. Vitamin B1
18.Your patient is brought in via EMS, as she was found unresponsive next to an
empty bottle of her prescription for amitryptiline. Medics estimate that this
apparent overdose occurred about 3 hours ago. On arrival, the patient is
hypotensive and remains unresponsive. You place an IV line, administer 1L of
normal saline, and order an EKG, which shows a widened QRS. Which of the
following is the next best step in the management of this patient?
a. Administer sodium bicarbonate
19. A 35-year-old man comes to the emergency department because of a seizure.
The patient has been incarcerated for a day and hit his head on the floor during
a single episode of seizing, lasting about 3 minutes, witnessed by prison staff.
Past medical history is significant for gastroesophageal reflux disease. Physical
examination shows a tachycardic, hypertensive, anxious, diaphoretic, tremulous
man who is fully alert and oriented. There is evidence of urinary incontinence.
Neurological exam shows tongue fasciculations and a resting tremor.
Laboratory studies show
a. Alcohol withdrawal
20.A 45-year-old man comes to the emergency department via ambulance after
being pulled from a burning building. He is very lethargic and complains of
vertigo and abdominal pain. His temperature is 37 ºC (98.6 ºF), pulse is 110/min,
respirations are 24/min, and blood pressure is 100/70 mm Hg. Physical
examination shows a confused male with soot in the nares and mouth. His
breath also has an odor of bitter almonds. There is decreased air entry into the
lungs bilaterally with diffuse wheezes. Cardiac examination shows no murmurs
and a normal S1 and S2. Laboratory data shows a serum lactate of 15 mmol/L.
Which of the following is the most appropriate next step in management?
a. Hydroxocobalamin
21.A 34-year-old male is prescribed an antidepressant and is told to absolutely
avoid wine and cheese. Which of the following drugs was most likely prescribed
to this patient?
a. phenelzine
22.A 14-year old male is brought to his primary care provider by his mother with a
complaint of red eye and the worry that the male may have an infection. The
patient denies any blurry vision, foreign-body sensation, or discharge. He says
his eyes feel "dry" but denies any itching or burning. Physical exam reveals
injected conjunctivae but no discharge. Which of the following is the most likely
diagnosis?
a. Marijuana use
23.You are called to see a 15-year-old male in the emergency department. Upon
your arrival, he is tachycardic and hypertensive. The patient is very combative
towards the staff and complains of severe itching and formications. An EKG is
performed showing sinus tachycardia, but is otherwise normal. Based on the
above information, which toxication is most likely the patient is suffering from?
a. Cocaine
24.A long term psychiatric patient presents to his psychiatrist with complaints of a
sore throat. The psychiatrist decides to immediately discontinue the patient's
prescription of clozapine and order blood draws. What is likely to be seen on
CBC and peripheral blood smear?
a. Agranulocytosis
25.A 5-year-old boy comes to the emergency department because of altered
mental status. He was being watched at a neighbor's house and was found by
his babysitter laying on the floor of the bathroom, unresponsive, with several
prescription medication bottles open next to him: acetaminophen, clonazepam,
glyburide, and metropolol. Pulse is 120/min, respirations are 6/min, blood
pressure is 110/60 mmHg, and oxygen saturation is 99% on non-rebreather
mask. Physical examination shows an unresponsive male with no signs of
external trauma. Pupils are 6 mm, equal, and reactive to light bilaterally.
Laboratory studies show blood glucose of 110 mg/dL. Which of the following is
the most likely etiology?
a. Benzodiazepine overdose
26.A 35-year-old man is brought to the emergency department because he was
found obtunded and hypoactive on the side of the road 30 minutes ago. He says
he is homeless, but can not remember when he last ate, or his medical history.
Blood sugar level is normal. Examination shows an ulcer on the lateral aspect of
the right heel (shown below). There is decreased sensation to light touch and
pain around the site of the ulcer. Which of the following conditions is a risk
factor for the development of this type of ulcer?
a. Alcoholism
27.A 25-year-old woman with schizophrenia presents with restlessness, agitation,
and a fixed upward stare. She states that her eyes hurt and cannot make them
look forward. Which is the most appropriate treatment?
a. Benztropine
28.A 50-year-old male, with a 25 year history of alcohol use disorder, comes to the
emergency department complaining of a loss of appetite, abdominal pain and
fever for the past 24 hours. He states that he had consumed twelve beers and a
bottle of vodka two days ago. Laboratory studies are obtained, and results show
the following
a. Vitamin B1 deficiency
29.A 2-year-old boy is brought to the pediatrician for a check up. The boy's mother
states that he has been rather lethargic and weak for the past few months.
Recently, his eyes have been tinged with yellow, and he has a mild fever. The
patient has not traveled recently or had any sick contacts. His parents are
vegetarian and so he eats mostly homegrown fruits and vegetables. On physical
examination the patient appears underdeveloped for his age, pale, and
confused. A peripheral blood smear shows schistocytes and basophilic stippling.
Which of the following biochemical processes is the most likely cause of the
patient's condition?
a. Inhibited ferrochelatase activity
30.A 45-year-old man comes to the emergency department after being pulled from
a burning upholstery factory in the city. He is very anxious when he is being
interviewed and complains of headache, nausea, and a metallic taste in his
mouth. His temperature is 37.1 oC pulse is 110/min, a respiratory rate of
24/min, and blood pressure of 148/70mmHg. He is put on pulse oximetry and it
shows an oxygen saturation of 96% on room air. Physical examination shows a
confused man with soot visible in the nares and mouth. He also has a breath
odor of bitter almonds. Respiratory examination shows decreased air entry
bilaterally with some diffuse wheezes. His arterial blood gas sample shows a
serum lactate of 11mmol/L. Which of the following is the most likely
explanation for the patient's physical findings?
a. Cyanide poisoning
31.A 39-year-old woman is brought to the emergency department by her husband
because of severe muscle pain and weakness for the past week. She also
complains that she has been having difficulty concentrating at work for the past
2 months and attributed it to being distracted with her daughter moving away
for college. For the past 10 years she has worked at an incinerator plant for the
marijuana interdiction program. In addition, her husband states that he has
noticed she has been very irritable for the past few months and her libido
decreased dramatically. Physical examination of the patient shows decreased
sensation to light touch, pain and temperature bilaterally in the upper and lower
extremities. Examination of her gums shows blue pigmentation. Which of the
following tests should be ordered to confirm the patient's diagnosis?
a. Blood lead level
32.A 40-year-old woman comes to the emergency department with severe right
arm pain after a bad fall earlier in the day. Her temperature is 37.5°C (99.5°F),
pulse is 70/min, respirations are 14/min, and blood pressure is 132/75 mmHg .
Physical examination shows marked pain on palpation of the wrist and she does
not want to move her hand because of significant pain. A radiograph of the right
hand and arms shows a fracture of the scaphoid and radial head. A dual-energy
x-ray absorptiometry (DEXA) scan shows a T-score of -1.0. Laboratory results
show a parathyroid hormone level of 830 pg/mL, serum phosphate of 2.1 mg/dL,
and a serum calcium of 6.5 mg/dL. Which of the following is the most likely
cause of her fractures?
a. Defective mineralization of osteoid
33.AA 5-year-old boy comes to the emergency department because of altered
mental status. He was being watched at a neighbor's house and was found by
his babysitter laying on the floor of the bathroom, unresponsive, with several
prescription medication bottles open next to him: acetaminophen, clonazepam,
glyburide, and metropolol.23–year-old woman is brought to the emergency
department because of nausea and vomiting after swallowing several packets of
acetaminophen tablets. The patient says that she swallowed the pills about 45
minutes ago. Past medical history is significant for major depressive disorder
and is treated with fluoxetine. Physical examination shows minimal epigastric
tenderness. Which of the following is the most appropriate next step in
management?
a. Administration of oral activated charcoal

STEP 2

1. A 34-year-old male is admitted to the emergency department after being


brought to the hospital by his friends. The patient had been drinking alcohol
"excessively" at a party and was found unresponsive on a couch. His pulse
150/min, blood glucose concentration is 60 mg/dL, and serum anion gap is
normal. Laboratory studies show
a. Positive serum ketone test
2. A 42 year-old woman comes to her primary care
provider's office because of numbness and tingling in
both her hands for the past month. She states that
she also feels constantly fatigued and lightheaded.
She also reports that her mood has been unstable
and always fluctuates throughout the day. She is
vegan and her diet consists of beans, green
vegetables, and starches. An magnetic resonance
imaging study is done and shows the following:
a. Vitamin B12 deficiency
3. A 22-year-old man presents to the emergency department for evaluation of
nausea and vomiting. The patient consumed marijuana and later had several
episodes of non-bloody, non-bilious emesis. He has a history of cyclic vomiting
syndrome and feels that his current symptoms are similar to those of prior
episodes. Past medical history is also notable for cocaine use and bipolar
disorder. The patient's vital signs are within normal limits, and abdominal
examination is otherwise benign. He is subsequently given intravenous
metoclopramide and fluids for symptomatic relief. Shortly after administration of
metoclopramide, the patient’s neck involuntarily contracts to the right. He also
begins experiencing associated neck pain and discomfort. Which of the
following is the most appropriate first-line treatment for this patient’s new
symptoms?
a. Diphenhydramine
4. A 79 year old woman is brought to the emergency department via EMS after her
husband found her unresponsive this morning. The patient has non-insulin
dependent diabetes and hypertension and had a total hip arthroplasty 4 days
ago. She takes metformin, lisinopril, and recently started taking
oxycodone/acetaminophen for post-operative pain. Her husband states that he
gave her an extra oxycodone/acetaminophen tablet last night to help her sleep.
He states that she may have woken up to take more during the night, but he is
not sure. He awoke in the morning to find her unresponsive and slightly blue, so
he called EMS. Her vitals are: P 54, BP 90/60, RR 6, and T 98.2°F. Finger stick
glucose is 156. Penlight examination of her eyes is shown below. She does not
respond to questioning. She retracts in pain in response to sternal rubbing.
What the most appropriate next step in treatment?
a. Naloxone
5. An 84-year old male develops severe dry mouth while in the hospital for
influenza A and viral gasteroenteritis with vomiting. His current medications
include IV insulin, heparin, famotidine, and scopolamine. Spot glucose is 96
mg/dL. If his symptoms are due to a medication effect and not organic medical
condition, what medication class should be discontinued?
a. Muscarinic Antagonists
6. A 27-year-old woman is reviewed on the psychiatric inpatient ward because of
involuntary muscle contractions for 6 hours. She was admitted for overt
psychosis as a result of her schizophrenia and has been gradually improving.
Over the last 2 days she has had less hallucinations and paranoid delusions due
to adjustments to her medications. Her minder tells you she suddenly developed
involuntary contraction of her neck muscles, and involuntary eye blinking 6
hours ago. Examination shows diplopia, and an upward deviation of her eyes.
Which of the following is the most appropriate pharmacological management?
a. Give her diphenhydramine immediately.
7. A 17-year-old girl is brought to the emergency department for evaluation after
a suicide attempt. Per report, the patient's partner broke up with her, and she
decided to end her life by overdosing on an unknown quantity of “some pills.”
The patient is currently complaining of blurry vision and confusion. Her
temperature is 38.9 C° (102 F° ), respirations are 20/minute, pulse is
125/minute, and blood pressure is 125/85 mmHg. On physical examination, the
patient is not oriented to place or time. The pupils are dilated and nonreactive to
light. Her skin appears flushed and dry, and bowel sounds are absent. An ECG
demonstrates sinus tachycardia. She does not have evidence of clonus.
Administration of which of the following medications is the most appropriate
intervention given this patient’s clinical presentation?
a. Physostigmine
8. A 65-year-old male comes to the emergency department because of chest pain
and difficulty breathing. History reveals the patient was hit by a car and
sustained a diffuse trauma to his chest. Past medical history reveals multiple
hospitalizations for alcohol-related injuries and seizures. The patient is
intubated in the emergency department for a flail chest segment and spends
one week sedated in the intensive care unit. After a successful extubation, he is
transferred to the floor where it is noted that he appears to be confused, has
unsteady gait, and is easily agitated. Physical examination reveals asymmetric
eye movements, most notably a lateral rectus palsy. The patient is oriented to
himself and is unable to walk without assistance. Which of the following is the
most appropriate treatment in this patient?
a. IV thiamine
9. A 15-year-old male is brought to the emergency department by his parents
because he was confused all morning. The patient goes into a coma 15 minutes
later. His parents say he was drinking a homemade alcoholic beverage known as
“moonshine” last night at this classmate’s house. The patient’s airway,
breathing, and circulation are controlled. Laboratory results reveal anion gap
metabolic acidosis. Which of the following is the next best step in management
in this patient?
a. Fomepizole administration
10.A 65-year old man with a history of severe
depression is brought to the emergency
department by his partner after she found him
unconscious on the bathroom floor. The patient’s
partner suspects the patient may have attempted suicide, as she found empty
pill bottles next to him. Temperature is 38.1 C° (100.5 F°), pulse is 110/minute,
and blood pressure is 80/60 mmHg. On physical examination, the pupils are
dilated, his skin appears flushed, and bowel sounds are decreased. ECG is
shown below. During the evaluation, the patient has a generalized tonic-clonic
seizure, which terminates with administration of lorazepam. Which of the
following interventions is most appropriate given this patient’s clinical
presentation?
a. Sodium bicarbonate
11.A 52-year-old woman comes to the office because of increasing shortness of
breath and nonproductive cough for five months. Her respiratory rate is 23/min
at rest. Chest X-ray shows an increase in intercostal spaces bilaterally and an
extensive increase in interstitial markings. Spirometry shows marked restrictive
ventilatory pattern. Chest CT scan shows areas of honeycombing mostly in
peripheral lung bases and cysts bilayers. She was diagnosed with psoriasis
vulgaris ten years ago, for which she is taking methotrexate. Which of the
following is the most appropriate next step in diagnosis?
a. Right heart catheterization
12.A 16-year-old boy is hospitalized after a dirt-bike accident. While recovering
from his injuries, he develops a cellulitis. Culture grows catalase-positive
gram-positive cocci. A susceptibility test reveals resistance to
penicillinase-resistant penicillins. He is treated with the drug of choice for this
infection. Soon after, his mother notices that his face and neck are turning red.
His blood pressure drops rapidly. What is the mechanism of action of the
antibiotic that caused this adverse reaction?
a. Interruption of cell wall synthesis
13.A 17-year old boy is brought to the emergency department for evaluation of
altered mental status. Earlier this morning, the patient was complaining of
ringing in his ears, abdominal pain, and vomiting. His parents report he has
seemed more depressed lately, and they are worried he may have taken
something to hurt himself. Per EMS, the patient was found with an empty pill
bottle belonging to his grandmother. Temperature is 38.5 C° (101.3 F°), pulse is
106/minute, respirations are 10/minute, and blood pressure is 110/70 mmHg.
On physical examination the patient is minimally responsive to pain, pupils are
reactive and equal bilaterally, and no clonus is appreciated. During the
evaluation, the patient has a generalized tonic-clonic seizure, which terminates
with administration of lorazepam. Administration of which of the following
medications is the most appropriate treatment for this patient’s suspected
medication toxicity?
a. Sodium bicarbonate
14.A 50-year-old man with altered mental status is brought to the emergency
department by his estranged wife after being found stumbling and mumbling
incoherently in his home. He lives alone and has not been seen by
acquaintances for some time. His speech is unintelligible, and he is unable to
walk steadily. He is lucid for a few moments but cannot remember who he is, or
where he lives. He has a long history of alcoholism but urine toxicology is
negative for alcohol ingestion, benzodiazepines, barbituates, opiates, or other
substances. His symptoms are most likely to be resolved with the administration
of what substance?
a. Thiamine
15.A 40-year-old homeless man presents to the emergency department with
altered mental status. He is afebrile, is hyperventilating, and endorses a visual
disturbance. He is unable to provide additional history. Physical examination
reveals an afferent pupillary defect but is otherwise noncontributory. Which of
the following is the most appropriate management in this patient?
a. Fomepizole
16.A 51-year-old man with a 20-year history of alcoholism comes to the
emergency department because of abdominal pain, fever, and loss of appetite
for the past 24 hours. He also complains of anxiety and insomnia. Additionally,
he had a seizure 10 minutes prior to arrival. He says he has decided to stop
drinking; he last drank 2 days ago, consuming 12 beers and a bottle of vodka.
Laboratory studies show:
a. Chlordiazepoxide
17.A 55-year-old man presents to his primary care physician for evaluation of
tremor and ataxia. He is unable to walk without his partner’s assistance in the
clinic. In addition, his partner reports “confusion” for the past two days. The
patient has a history of severe bipolar disorder, for which he takes lithium. One
month ago, he started using an over-the-counter medication for arthritic knee
pain. He does not use tobacco or alcohol. Pale mucous membranes are noted on
physical examination. Laboratory analysis demonstrates elevated serum
creatinine and blood urea nitrogen. In addition to intravenous fluids, which of the
following is the most appropriate next step in treatment of this patient's clinical
condition?
a. Hemodialysis
18.A 65-year-old man is being evaluated for hematuria following urinary
catheterization. The patient was hospitalized four days ago with severe chest
pain and left lower limb swelling, and he was subsequently diagnosed with
venous thromboembolism. The patient was admitted to the floor and initiated
on heparin anticoagulation therapy. Today, the nursing staff noted bright red
blood following urinary catheterization. The patient’s past medical history
includes chronic kidney disease, for which he receives dialysis on Monday,
Wednesday, and Friday. The patient’s blood pressure is 120/75 mmHg, heart
rate is 100/minute, and respirations are 18/minute. The patient’s serum analysis
is demonstrated below. Which of the following is the most appropriate next
step in management?
a. Administration of protamine sulfate
19.A 24-year-old woman comes to the emergency department because of slurred
speech, ataxia, and hypoventilation. She claims to have overdosed on
lorazepam, her anti-anxiety medication. She takes no other medications and
denies suicidal ideation, homicidal ideation, or thoughts of harming herself or
others. Which of the following is the most appropriate next step in
management?
a. Flumazenil
20.A 40-year-old intravenous drug user presents with severe hypoventilation,
miosis, and vomiting presents at the emergency department. Which of the
following is the appropriate management in this patient?
a. naloxone
21.A 25-year-old female presents to the emergency room with involuntary
contractions in her neck, involuntary eye blinking, diplopia, and upward
deviation of her eyes. The patient had been discharged from the hospital five
days earlier, when she was diagnosed with schizophrenia and put on a
low-dose antipsychotic medication, chlorpromazine, which she has continued to
use as directed. The patient indicates that the medication has significantly
improved her psychiatric symptoms, which included hallucinations and paranoid
delusions. Physical examination reveals posterior and lateral cervical flexion of
the neck with extraocular muscle paralysis and ocular pain. A complete blood
count (CBC) and toxicology screen come back normal. Based on the patient's
history and symptoms, what is the most likely diagnosis?
a. Acute dystonia
22.A 45-year-old man comes to the emergency department because of auditory
and visual hallucinations that have progressively worsened over the past week.
He has a history of schizophrenia and has not been compliant with his
medications. He is started on the appropriate pharmacologic therapy after his is
admitted. Shortly after the nurse administers his medication he becomes
restless and agitated. His temperature is 37.1 oC, pulse is 80 /min, respiratory
rate is 12/min, and blood pressure is 110/75 mmHg. Physical examination
shows that his eyes are stuck in an upward deviation. Which of the following is
most appropriate next step in management?
a. Diphenhydramine
23.A 25-year-old man comes to the office for a pre-travel medical examination. His
medical history is noncontributory. Upon medical interrogation, the patient
mentions that he is going to be traveling to several countries in South America.
Physical exam is noncontributory. His temperature is 36.5°C (97.7°F), pulse is
101/min, respirations are 15/min, blood pressure is 110/60 mmHg. After
finishing the medical exam, the patient requests to receive prophylaxis for
malaria. Based on this information, which of the following should be indicated
before prescribing prophylactic treatment in this patient?
a. Test for G6PD deficiency
24.A 60-year-old man presents to the hospital with bradycardia, hypotension, and
dyspnea after overdosing on his blood pressure pills. An on-call psychiatry
resident is consulted to manage this patient. Which of the following is the most
proper management in this patient?
a. glucagon
25.A 50-year-old man is brought to the emergency department by his partner for
evaluation of altered mental status. His partner states the patient woke up
today with knee pain, for which he took an “over-the-counter pain reliever” and
his prescription hydrocodone-acetaminophen. Several hours later, the partner
noticed he was less responsive and unaware of where he was. The patient has a
history of chronic lower back pain and chronic alcohol use. On physical
examination, the patient is not oriented and responds only to pain. The patient
grimaces with palpation of the right upper quadrant. AST and ALT are 2136 and
2467 IU/L, respectively. Which of the following is the most appropriate
therapeutic treatment for this patient's clinical condition?
a. N-Acetylcysteine

Bites and stings: Clinical practice


STEP 1

1. A 15-year-old boy comes to the emergency department because of a bee sting.


The patient has been stung once before with only a local reaction. Physical
examination shows audible wheeze bilaterally, edema of the posterior pharynx,
and diffuse cutaneous hives. Which of the following is the most appropriate
initial step in the management of this patient?
a. Intramuscular epinephrine 1:1,000
2. A 35-year-old woman comes to her primary care office complaining of a growth
that appeared on her arm one month ago. She reports that prior to the
appearance of the growth, she had been stung by a bee and had had a mild
allergic reaction. When the dermatologist palpates the lesion, a small dimple
appears in the center of the lesion. A biopsy shows thickening of the epidermis,
with an increased number of histiocytes and fibroblasts in the dermis. Given this
information, which of the following is the most likely diagnosis?
a. Dermatofibroma
3. A 17 year-old boy comes to the emergency department because of dehydration.
For the past day, he has experienced severe spasms in his throat when trying to
drink. He reports being bitten on the left upper arm by a dog in the local park
one month ago. During the previous week he experienced several episodes of
fever, weakness, fatigue, insomnia and headache. He has also experienced some
dysphagia in the past two days, but onset of his spasms was very sudden.
Physical examination shows a facial grimace and hyperextension of the neck.
Which of the following is the most likely diagnosis?
a. Rabies encephalitis
4. A 67-year-old man is brought to the emergency department because of a skin
infection. He says that he was bitten by a cat four days ago on his left forearm
and that he feels pain, fatigue, and generalized body aches for the past day.
Physical examination shows a 1 cm laceration on the left forearm with
surrounding erythema, swelling, and increased warmth. Which of the following
pathogens is the most likely cause of this patient's condition?
a. Pasteurella multocida

STEP 2

1. A 48-year-old woman comes to the emergency department because of an


insect bite. She states she was mowing her lawn in a red T-shirt and shorts
when an insect attacked her and stung her several times on the back of the leg.
She began to develop itching and some difficulty breathing so decided to seek
medical attention. Which of the following is most accurate?
a. IgE mediates anaphylaxis after a bee or wasp sting
2. A 17-year-old boy from Oklahoma is brought to the emergency department
with two days of fever to 39.3°C (102.7°F), myalgia, abdominal pain, and
vomiting. A maculopapular rash is noted on the trunk, back, extremities, palms,
and soles. He had previously been in good health, with no unusual dietary or
travel exposures and no sick contacts. He has been sexually active with multiple
partners; he drinks socially but denies use of recreational drugs. He is not taking
any medications and has no known drug allergies. Which of the following is the
most appropriate next step in management?
a. Order empiric treatment with doxycycline
3. A 5-year-old girl is brought to her pediatrician because of a 3 day history of a
tender swollen lump underneath her right armpit. The patient's father states
that she was previously healthy with no problems. She does not have a fever
and states that she is otherwise feeling "good". Physical examination shows
tender right axillary lymph nodes ranging from 2-3cm in diameter, but no
overlying erythema or fluctuance is noted. She also has several healed scratch
marks on the dorsum of her right hand, with 3-5mm reddish raised papules in a
linear distribution surrounding the scratches. When asked about the scratches,
she says that she found a ‘little kitty’ two weeks ago in the backyard and tried to
feed her a saucer of milk, but the cat ‘was naughty’ and scratched her. Based on
the patient's presentation, which of the following is the most appropriate
management for her condition?
a. Observance and reassurance with supportive therapy

● PLAYLIST 9

Mycoplasma Pneumoniae
STEP 1

1. A 19-year-old man presents to the clinic with low-grade fevers, fatigue,


paleness, shortness of breath, and a nonproductive cough for the past 2 weeks.
His temperature is 38.4°C (101.1°F), pulse is 86/min, respirations are 20/min,
and blood pressure is 122/84 mmHg. Chest auscultation reveals mild wheezing
and crackles bilaterally. Chest X-ray shows bilateral patchy infiltrates. Direct
Coombs test is positive. Which of the following classes of antibodies is most
likely implicated in this disease process?
a. Immunoglobulin M (IgM)
2. A 19-year-old navy recruit comes to the office because of non-productive cough,
headache, fever, rhinorrhea, and myalgias for a week. He says that many of the
other recruits in his unit are also sick with the same symptoms. His temperature
is 37.8ºC (99.3ºF), pulse is 110/min, respirations are 22/min, and blood pressure
is 116/82 mm Hg. Widespread rales are heard on chest auscultation. There is
also a red rash present on his limbs and torso. Blood cultures are ordered.
Which of the following growth medium would be most effective in culturing the
common organism responsible for the patient’s condition?
a. Eaton Agar
3. A 24-year-old man comes to the clinic because of a 2-week history of
nonproductive cough and intermittent headaches. The patient has been feeling
severely fatigued for the past few weeks, even though his daily regimen has
remained the same. He is currently training to become an artillery officer and
lives in army barracks. Past medical history is noncontributory, and he takes a
multivitamin daily. He does not use tobacco, alcohol, or illicit substances.
Temperature is 38.2°C (100.8°F), pulse is 96/min, and blood pressure is 128/85
mmHg. Physical examination reveals conjunctival pallor and pale mucous
membranes. Bilateral crackles are heard on chest auscultation. Chest radiograph
reveals diffuse bilateral infiltrates. Laboratory results are as follows:

Laboratory value Result

Complete blood count

Hemoglobin 9.6 g/dL

Hematocrit 30%

Leukocyte count 9,100/mm3

Platelet count 250,000/mm3

MCV 88 µm3

Haptoglobin 23 mg/dL (N= 41-165 mg/dL)

Further testing in this patient is likely to reveal which of the following findings?

a. Positive direct Coombs test


4. A 22-year-old woman comes to the office due to a week of low-grade fever,
fatigue, headaches, and persistent non-productive cough. She was feeling
healthy prior to symptom onset. Past medical history is noncontributory. She
currently lives in a college dormitory. She does not use tobacco products,
consume alcohol, or use illicit substances. Temperature is 38.0°C (100.4°F),
pulse is 96/min, and blood pressure is 121/75 mmHg. Physical examination is
notable for rhinorrhea and bilateral cervical lymphadenopathy. Chest radiograph
reveals reticulonodular opacities and patchy consolidations in the lung fields.
Laboratory findings reveal a mild anemia, and subsequent direct Coombs test
and cold-agglutinin titers are found to be positive. Which of the following is the
most likely cause of this patient’s condition?
a. Close quarters living arrangement
5. A 28-year-old previously healthy woman comes to the office because of a
1-week history of recurrent headaches and generalized muscle pain. Her
medical history is noncontributory. Upon further interrogation, the patient also
reports having a persistent, dry, and irritating cough lasting more than a week.
The patient currently takes acetaminophen, dextromethorphan, and guaifenesin,
to which she claims no relief of her symptoms. On physical examination, the
patient looks well-appearing. Auscultatory findings reveal right basal crackles
on inspiration. Her temperature is 39.7°C (102.2°F), pulse is 106/min,
respirations are 17/min, blood pressure is 120/80 mmHg, oximetry on room air
shows an oxygen saturation of 98%. Laboratory findings are normal. A
conventional chest radiograph is obtained and shown below. Which of the
following is the most likely causal organism for this patient's condition?
a. Mycoplasma pneumoniae

Pneumonia: Pathology Review


STEP 1

1. A 56-year-old woman goes to the emergency department for evaluation of


productive cough and shortness of breath, which have been going on for the
past 3 days. She has also experienced chest pain that worsens with deep
inspiration. Past medical history is significant for persistent atrial fibrillation that
has been well-controlled with amiodarone. She has smoked a pack of cigarettes
daily for the past 20 years. She does not drink alcohol or use illicit substances.
Temperature is 38.0°C (100.4°F), pulse is 98/min, blood pressure is 138/85
mmHg, and oxygen saturation is 92% on room air. A chest x-ray is obtained and
demonstrates the following finding: Image reproduced from Wikimedia
Commons. Which of the following is the most likely diagnosis?
a. Lobar pneumonia
2. A 54-year-old woman comes to the clinic for evaluation of a nonproductive
cough and fever for the past 3 weeks. She also has been experiencing
arthralgias in her wrists, knees, and elbows. The patient is a farmer and
regularly comes in contact with various animals, including cows and pigs. Past
medical history is notable for hypertension that is well-controlled with losartan.
She does not smoke cigarettes, drink alcohol, or use illicit substances.
Temperature is 38.0°C (100.4°F), pulse is 98/min, and blood pressure is 138/85
mmHg. The patient's pulse oximetry shows 94% on room air. Physical
examination reveals expiratory wheezes bilaterally. Chest radiograph reveals
right lower and middle lobe consolidation. Sputum and blood cultures are
obtained and show no growth after incubation for several days. Which of the
following pathogens is most likely responsible for this patient’s condition?
a. Coxiella burnetii
3. A 58-year-old man goes to the emergency department for evaluation of fever
and dry cough for the past week. He has also become progressively more
dyspneic, and walking a single block makes him short of breath. He underwent
a liver transplantation 4-months ago for treatment of autoimmune hepatitis and
is currently taking several immunosuppressant drugs. Temperature is 38.5°C
(101.3°F), pulse is 104/min, respirations are 20/min, blood pressure is 122/88
mmHg, and oxygen saturation is 89% on room air. Lung auscultation reveals
bilateral crackles and rhonchi. Chest radiograph reveals bilateral, diffuse
interstitial infiltrates extending from the perihilar region. Leukocyte count is
13,000/mm3 and serum lactate dehydrogenase level is 460 U/L. Further testing
is most likely to reveal which of the following findings?
a. Presence of organisms in sputum that stain positive for methenamine
silver
4. A 42-year-old man comes to the clinic for evaluation of a productive cough with
purulent secretions. His symptoms have been going on for the past three days.
He also has been experiencing subjective fevers during this same time period.
Past medical history is notable for hypertension, which is well controlled with
lisinopril. He has a 15-pack-year smoking history and drinks 2-3 beers on the
weekends. Temperature is 38.5°C (101.3°F), pulse is 104/min, respirations are
18/min, and blood pressure is 122/88 mmHg. Physical examination
demonstrates dullness to percussion and bronchial breath sounds over the left
lung. Chest radiograph reveals a left lower lobe consolidation. Which of the
following describes the epidemiologically most likely pathogen responsible for
this patient’s illness?
a. Gram-positive diplococci
5. A 73-year-old man with a past medical history of COPD and hypertension is
brought to the emergency department because of dyspnea over the past 2 days.
He has a 25-pack year smoking history. On arrival, the patient has altered
mental status and impending respiratory failure. Endotracheal intubation is
performed in the emergency department. A chest radiograph demonstrates lung
hyperinflation but no consolidation or infiltrates. The patient is started on
bronchodilators, antibiotics, and corticosteroid therapy. On the fifth day of
hospitalization, while still intubated, the patient develops a fever as well as
yellowish, purulent secretions. Temperature is 38.9°C (102°F), pulse is 104/min,
and blood pressure is 151/85 mmHg. Repeat chest radiograph reveals new right
middle lobe infiltrates. Which of the following organisms is the most likely
cause of this patient’s current symptoms?
a. Pseudomonas aeruginosa
6. A 22-year-old woman comes to the office due to a week of low-grade fever,
fatigue, headaches, and persistent non-productive cough. She was feeling
healthy prior to symptom onset. Past medical history is noncontributory. She
currently lives in a college dormitory. She does not use tobacco products,
consume alcohol, or use illicit substances. Temperature is 38.0°C (100.4°F),
pulse is 96/min, and blood pressure is 121/75 mmHg. Physical examination is
notable for rhinorrhea and bilateral cervical lymphadenopathy. Chest radiograph
reveals reticulonodular opacities and patchy consolidations in the lung fields.
Laboratory findings reveal a mild anemia, and subsequent direct Coombs test
and cold-agglutinin titers are found to be positive. Which of the following is the
most likely cause of this patient’s condition?
a. Close quarters living arrangement
7. A 54-year-old woman comes to the clinic for evaluation of fevers and a
productive cough that have been ongoing for the past three days. She has not
experienced hemoptysis, recent weight loss, or night sweats. Past medical
history is notable for type II diabetes mellitus. She has a 20-pack-year smoking
history. She worked as a plumber for 30 years. Temperature is 38.5°C
(101.3°F), pulse is 106/min, respirations are 18/min, and blood pressure is
132/88 mmHg. Physical examination reveals dullness to percussion and
bronchial breath sounds over the right lung. Chest radiograph reveals a right
lower lobe consolidation. Sputum microscopy shows Gram-positive diplococci.
Which of the following histopathological changes is most likely present in the
affected lung lobe?
a. Alveolar spaces filled with neutrophils and fibrin
8. A 38-year-old man comes to the emergency department for evaluation of
fever and cough that have been going on for the past four days. The cough is
productive and yields purulent, foul-smelling sputum. Temperature is 38.9°C
(102°F), pulse is 102/min, and blood pressure is 121/65 mmHg. The patient’s
pulse oximetry shows 91% on room air. Crackles are heard bilaterally. Chest
radiograph reveals infiltrations in the basilar region of both lungs. Sputum
Gram-stain is notable for neutrophils and Gram-negative organisms.
Subsequent sputum culture yields growth of Klebsiella and Bacteroides
species. Which of the following is most likely to be found upon further
interviewing this patient?
a. History of alcohol intoxication
9. A team of researchers is attempting to develop a new pharmacotherapy for the
treatment of respiratory infections. Pathogen X, as identified by the
researchers, is introduced to an in vitro model consisting of human
nasopharyngeal epithelial cells grown within a culture container. Pathogen X
is found to produce an enzyme that binds sialic acid residues on the surface of
nasopharyngeal epithelial cells. This enzyme allows pathogen X to
subsequently enter the epithelial cells. Clinically, pathogen X causes
symptoms including shortness of breath, fevers, myalgias, and joint pain.
Furthermore, infection by pathogen X can predispose patients to subsequent
infection by Staphylococcus aureus. Which of the following is the most likely
identity of pathogen X?
a. Influenza virus
10. A 62-year-old man is brought to the emergency department for dyspnea and
wheezing. The patient is diagnosed with a COPD exacerbation. Despite
appropriate medical management, endotracheal intubation is required for 2
days. On the 6th day of hospitalization, as he is recovering, he develops a fever
with increasing productive cough. Temperature is 38.5°C (101.3°F), pulse is
104/min, and blood pressure is 121/75 mmHg. A chest radiograph reveals
new, dense infiltrates in the left lower lobe with no blunting of the
costophrenic angle. Sputum microscopy shows Gram-negative bacilli. Which
of the following physical findings are likely to be present on physical
examination of this patient?
Percussion Fremitus

Dull Increased
11. A 24-year-old man comes to the clinic because of a 2-week history of
nonproductive cough and intermittent headaches. The patient has been feeling
severely fatigued for the past few weeks, even though his daily regimen has
remained the same. He is currently training to become an artillery officer and
lives in army barracks. Past medical history is noncontributory, and he takes a
multivitamin daily. He does not use tobacco, alcohol, or illicit substances.
Temperature is 38.2°C (100.8°F), pulse is 96/min, and blood pressure is
128/85 mmHg. Physical examination reveals conjunctival pallor and pale
mucous membranes. Bilateral crackles are heard on chest auscultation. Chest
radiograph reveals diffuse bilateral infiltrates. Laboratory results are as
follows:

Laboratory value Result

Complete blood
count

Hemoglobin 9.6 g/dL

Hematocrit 30%

Leukocyte count 9,100/mm3

Platelet count 250,000/mm3

MCV 88 µm3

Haptoglobin 23 mg/dL (N= 41-165 mg/dL)

Further testing in this patient is likely to reveal which of the following findings?

a. Positive direct Coombs test


12.A 71-year-old woman is brought to the emergency department because of
dyspnea for the past 2 days and is determined to have a COPD exacerbation.
She has a 25-pack-year smoking history. Despite appropriate medical
management, endotracheal intubation is required in the emergency
department. On the fifth day of hospitalization, the patient develops a fever
and increasingly purulent oral secretions. Temperature is 38.9°C (102°F), pulse
is 106/min, and blood pressure is 111/65 mmHg. A chest radiograph reveals
new right middle lobe infiltrates. Sputum microscopy is notable for
Gram-negative bacilli. Which of the following organisms is most likely
responsible for this patient’s current condition?
a. Pseudomonas aeruginosa
13.A 59-year-old man comes to the clinic for evaluation of worsening productive
cough and fevers over the last few days. The patient has a past medical history
of hypertension that is well-controlled with metoprolol. He recently completed
a course of antibiotics for cellulitis of the left shin. He actively smokes, has a
15-pack-year smoking history, and drinks 2-3 beers on the weekends. He
rarely exercises. Temperature is 38.5°C (101.3°F), pulse is 101/min,
respirations are 16/min, and blood pressure is 125/85 mmHg. Physical
examination reveals dullness to percussion and bronchial breath sounds over
the left lung. Chest radiograph demonstrates a left lower lobe consolidation.
Sputum microscopy shows Gram-positive diplococci. Which of the following is
the most important risk factor for this patient’s current condition?
a. Cigarette smoking
14.A 49-year-old woman comes to the emergency room for evaluation of fever
and a cough productive of thick, purulent sputum. The patient’s symptoms
have been ongoing for the past two days. She has not experienced
hemoptysis, unintentional weight loss, or night sweats. Past medical history is
notable for type II diabetes. She has a 15-pack-year smoking history and does
not consume alcohol. Temperature is 38.5°C (101.3°F), pulse is 104/min,
respirations are 19/min, and blood pressure is 122/88 mmHg. Physical
examination demonstrates dullness to percussion and bronchial breath sounds
over the right lung. Chest radiograph reveals a right lower lobe consolidation.
Sputum Gram stain reveals gram positive diplococci. Which of the following is
a feature of the organism most likely responsible for this patient’s symptoms?
a. Bile-soluble and optochin-sensitive
15.A 52-year-old man goes to the emergency department for evaluation of fever,
cough, and shortness of breath for the past several days. The patient has also
experienced several episodes of diarrhea daily. He recently went on a business
trip and stayed in a large hotel. The patient’s past medical history is significant
for hypertension and type II diabetes mellitus that are well-managed with
lisinopril and metformin, respectively. Temperature is 38.5°C (101.3°F), pulse is
104/min, respirations are 18/min, and blood pressure is 132/88 mmHg.
Physical examination demonstrates dullness to percussion and bronchial
breath sounds over the left lung. Chest radiograph reveals multifocal opacities
in the right lung. Laboratory results are as follows:

Laboratory value Result

Serum chemistry

Sodium 128 mEq/L

Potassium 4.2 mEq/L

Chloride 95 mEq/L

Creatinine 0.9 mg/dL

Aspartate aminotransferase 74 U/L

Alanine aminotransferase 88 U/L

Which of the following media is required for culturing the pathogen most likely
causing this patient’s current condition?

a. Charcoal yeast extract agar

Pneumonia: Clinical practice


STEP 1

1. A 14-year-old girl with hereditary spherocytosis comes to the emergency


department following a fall from a horse. Her injuries consisted of three
fractured ribs, a punctured right lung, and a ruptured spleen that required
surgical removal. Three months later the patient comes to the office because of
fatigue, cough, and shortness of breath. Physical examination shows dullness to
percussion and decreased breath sounds in the right lower lobe. A chest
radiograph is obtained and shows a right lower lobe infiltrate with patchy
opacity. The patient is started on antibiotic therapy. Which of the following
organisms is the most likely causative organism in this patient?
a. Streptococcus pneumoniae
2. A 66-year-old man comes to the emergency department because he has had
shortness of breath and a cough. He says that he had a deceased donor kidney
transplantation done 2 months ago because of stage 5 chronic kidney disease.
He has a history of diabetes and hypertension. He does not drink alcohol or
smoke. During the interview he goes into respiratory arrest and an emergency
tracheostomy is done. This patient is at risk of developing which of the
following?
a. Pneumonia
3. A 52-year-old man comes to the emergency department with his wife because
he was found unresponsive at home in a pool of vomit and bloody sputum. His
wife says that he drinks approximately a case of beer everyday. His temperature
is 39.1 oC (102.4°F), pulse is 105 /min, respiratory rate is 25/min, and blood
pressure is 111/81 mmHg. Physical examination shows significant difficulty
breathing with a very foul smelling breath. A cardiac examination shows no
murmurs with a normal S1 and S2. A chest X-ray is provided below. Which of
the following is the most likely pathogen responsible for his condition?
a. Klebsiella pneumoniae
4. A 57-year-old man with a history of alcoholism comes to the urgent care clinic
because of stabbing chest pain on inspiration for the last 2 days. The patient
also complains of cough, fever, fatigue and joint pain for the last week. He
received a liver transplant 5 weeks ago with no surgical complications. Physical
examination of the patient shows a fever of 38.1°C (100.6°F) and is otherwise
unremarkable. Results from a chest radiograph are shown below. Given the time
frame, which of the following best explains this patient's symptoms?
a. Cytomegalovirus infection
5. A 68-year-old man comes to the emergency department with difficulty
breathing, high fever, and intermittent diarrhea. Several other elderly individuals
have been reported being admitted to other hospitals in the same area. All
patients recently shopped at the local grocery store, where they purchased fresh
vegetables stored under the water mist machines. There is some difficulty in
obtaining sputum samples. Which of the following samples should be obtained
in order to test for the suspected causative organism?
a. Urine sample
6. A 67-year-old man comes to the emergency department because of fever and
shortness of breath for the past 6 days. He has associated chills and a cough
productive of multiple tablespoons of thick green sputum each morning. He has
a history of diabetes and end-stage renal disease requiring regular dialysis
which he gets at a center near his home. He lives at home with his wife, has had
no recent travel or sick contacts, and no recent hospitalizations. His temperature
is 38.5°C (101.3°F), pulse is 100/min, respirations are 22/min, and blood
pressure is 100/72 mm Hg. Pulse oximetry on room air shows an oxygen
saturation of 92%. He can speak in 3-4 word sentences. Physical examination
shows bibasilar crackles on auscultation, and diminished breath sounds at the
right middle and right lower lung fields. Which of the following is the most
likely diagnosis?
a. Community-acquired pneumonia
7. A 75-year-old man is brought to the emergency department because of a
productive cough and a fever of 39.4°C (103°F) for the past 3 days. The man has
dementia and is brought to the hospital from a nursing home. A sputum gram
stain shows lancet shaped gram positive diplococci. A chest radiograph shows
the following:
a. Lobar Pneumonia
8. A 34-year-old woman presents come to the office for a check-up after falling off
of her bike an hour ago. She complains of mild lateral chest pain worsened by
inspiration. She has a history of tuberculosis treated 15 years ago, but is
otherwise healthy and does not smoke. A Chest X-ray is obtained. Which of the
following is the most likely cause of these x-ray findings?
a. Aspergillus fumigatus

STEP 2

1. A 39-year-old female with a history of HIV presents to the infectious disease


specialist for follow-up care after multiple rounds of treatment for active
tuberculosis. She has been intermittently compliant with the medication
regimen and continues to have persistent symptoms despite multiple rounds of
therapy with different antimicrobials. The infectious disease specialist informs
her that she likely has extensively drug-resistant tuberculosis. Which of the
following treatment options is most appropriate for this patient?
a. Pretomanid, linezolid, and bedaquiline
2. A 73-year-old man comes to the emergency department because of a 2-day
history of fever, cough, and breathlessness. He has a history of congestive heart
failure. He works as a singer and is currently on tour. He is unable to tell the
doctor what city he is in, though he attributes that to being on the road so much.
Vital signs show his temperature is 38.5°C (101.3°F), pulse is 91/min,
respirations are 32/min, and blood pressure is 96/58 mm Hg. Which of the
following is the most appropriate next step in management?
a. Admit the patient for further work-up
3. A 55-year-old man comes to the clinic because of cough, shortness of breath,
and nausea since the past two days. He denies chills, abdominal pain, vomiting,
diarrhea, or rhinorrhea. He recently returned from an extended business trip and
has stayed in several hotels. He has not been ill or exposed to any sick person
recently but has been feeling ‘hot’. He did not take his temperature. Medical
history is significant for chronic obstructive pulmonary disorder treated with an
ipratropium inhaler. Vital signs show BP 134/95 mm Hg, 89 beats/min, 19
breaths/min, 39 C. General examination shows a thin man quietly breathing
through his mouth without using accessory muscles. Physical examination is
unremarkable, except for rales on auscultation. Pulse oximetry shows 98% on
room air. Chest X Ray reveals diffuse bilateral interstitial opacities. Which of the
following is the most appropriate diagnostic step?
a. Urine antigen
4. A 44-year-old man comes to his physician for evaluation of a nonproductive
cough, headaches, and myalgias for the past three weeks. He also reports chest
pain that is aggravated by deep inspiration. The patient works in construction
and is regularly involved in excavating caves. He recently traveled to Ohio for
work one month ago. Past medical history is noncontributory. Temperature is
37.8°C (100.1°F), pulse is 96/min, respirations are 16/min, and blood pressure is
125/85 mmHg. Lung auscultation reveals diffuse rales. Chest radiograph
demonstrates bilateral diffuse reticular opacities, as well as mediastinal and
hilar lymphadenopathy. The patient is started on empiric amoxicillin for the
treatment of community-acquired pneumonia. However, he returns after one
week of treatment due to persistent symptoms. Urinary antigen testing is
performed and subsequently confirms the diagnosis. Which of the following
therapies is most appropriate to treat this patient’s condition?
a. Itraconazole
5. A 26-year-old man comes to your clinic because of persistent cough for the past
6 years. The cough is usually non-productive, but he rarely notices blood (3-4
times a year). His mother also had a long-standing cough and passed away
when he was fifteen. He works as a dentist in the Southeastern US, but it
considering moving out West because dry air improves his symptoms. His
temperature is 37.8°C (100.2°F), pulse is 90/min, respirations are 15/min, and
blood pressure is120/80 mm Hg. His BMI is 23 kg/m2. Lungs are clear to
auscultation bilaterally. Which of the following is the most appropriate course of
treatment for this patient?
a. Isoniazid, rifampin, ethambutol, and pyrazinamide
6. A 67-year-old man comes to the emergency department because of fever,
cough, and shortness of breath for the past 4 days. He says he has been
coughing up "multiple tablespoons of thick green sputum each day." He has a
history of diabetes and undergoes hemodialysis at a dialysis center three times
per week for end-stage renal disease. He lives at home with his wife, has had no
recent travel or sick contacts, and no recent hospitalizations. His temperature is
38.5°C (101.3°F), pulse is 100/min, respirations are 30/min, blood pressure is
100/82mmHg, and oxygen saturation is 88% on air. He can speak in 3-4 word
sentences and his lung examination shows diminished breath sounds on the
right middle and right lower lung fields. Chest X-ray is obtained and is shown
below. Which of the following is the most appropriate initial therapy?
a. IV ceftriaxone and azithromycin
7. A 35-year-old man comes to the clinic because of fever and productive cough
for the past 2 days. He has no chest pain, dyspnea, and shortness of breath. He
has a history of HIV infection and on highly active antiretroviral therapy (HAAR).
Three weeks ago, his CD4 count was 400/mm3. His temperature is 38.5°C
(101.3°F), pulse is 100/min, respirations are 15/min, and blood pressure is
110/70 mm Hg. Crackles are heard on the left lower lung on auscultation.

Laboratory studies show:

Chest X-ray shows consolidation of the lower lobe of the left lung. Which of the
following is the mechanism of action of the most appropriate treatment?

a. Inhibits DNA topoisomerase II


8. A 51-year-old female presents for a follow-up appointment with an infectious
disease specialist. The patient has a history of HIV and has been intermittently
compliant with antiretroviral therapy. She has been staying in a shelter with
several individuals who have active tuberculosis infection. Her most recent CD4
count was 167/mm3. The patient undergoes a screening Mantoux skin test,
which is negative. However, there is strong suspicion for underlying anergy.
Which of the following is the most appropriate treatment regimen for this
patient?
a. Isoniazid monotherapy
9. A 25-year-old man comes to his primary care provider's office because of cough
and increasing shortness of breath while exercising for the past 4 months. He
says that he is a healthy individual who enjoys running marathons. However,
during his training for the past 4 months, he noticed that he has been
increasingly fatigued and out of breath during his training regimen. He also has
been coughing and lost 10 pounds unintentionally. He is an owner of a parrot
rescue sanctuary and does mention that his job can be stressful at times.
However, he denies anything out of the ordinary. Physical examination shows a
well-built man in no acute distress. Pulmonary auscultation shows inspiratory
crackles. Chest radiograph findings are consistent with chronic interstitial
inflammation with some patches of fibrosis. Which of the following is the next
best step in management for this patient?
a. An extended leave of absence from work
10.A 47-year-old male presents to the primary care physician with several weeks
of worsening fevers, cough, and night sweats. He recalls that several months
ago, he was volunteering at a medical facility in Cambodia. The patient is
otherwise healthy and does not take illicit drugs or drink alcohol. Temperature is
38.0°C (100.4°F), pulse is 90/min, respirations are 16/min, blood pressure is
120/64 mmHg, and O2 saturation is 96% on room air. Physical exam is
otherwise unremarkable. A chest x-ray demonstrates a reticulonodular right
upper lobe opacity. Which of the following tests will help definitively establish
this patient’s underlying disease process?
a. Lung biopsy and culture
11.A 56-year-old woman who immigrated from China 3 years ago comes to the
emergency department because of substantial hemoptysis. Initial work up
includes a chest x-ray which shows several cavitary lesions in the upper lung
fields bilaterally. Further testing confirms a diagnosis of tuberculosis. Proper
airborne precautions are initiated and the patient is placed in isolation. Which of
the following is the most appropriate initial treatment, assuming the TB strain is
not multi-drug resistant?
a. Combination therapy with rifampicin, isoniazid, ethambutol and
pyrazinamide
12. A 41-year-old male presents to the primary care physician after having a
positive interferon-gamma assay. The patient works as a nurse and received the
test during a routine onboarding evaluation screening. Several months ago, he
had contact with an individual with active tuberculosis, but he never developed
any symptoms. He is otherwise healthy and does not take any medications. The
patient subsequently undergoes sputum analysis and chest radiography, both of
which are within normal limits. Which of the following is the most appropriate
treatment regimen for this patient’s condition?
a. Rifampin monotherapy
13. A 42-year-old woman goes to the clinic for evaluation of fever and productive
cough with purulent secretions. These symptoms have been going on for the
past three days. Her past medical history is notable for hypertension and
hyperlipidemia, which are well-controlled with amlodipine and rosuvastatin.
She has no known drug allergies. Temperature is 38.5°C (101.3°F), pulse is
104/min, respirations are 18/min, and blood pressure is 132/88 mmHg. Physical
examination reveals dullness to percussion and bronchial breath sounds over
the right lung. A chest radiograph is obtained and reveals right middle lobe
consolidation. Which of the following medications is recommended in first-line
treatment for this patient’s condition?
a. Amoxicillin
14.A 48-year-old woman comes to the emergency department for evaluation of
fever and productive cough with purulent, foul-smelling sputum. Her symptoms
have been going on for the past 4 days. She has a history of alcohol abuse and
drinks a pint of vodka daily. She was brought to the emergency department
several weeks ago for alcohol intoxication after being found unconscious on the
sidewalk. Temperature is 38.9°C (102°F), pulse is 106/min, blood pressure is
121/65 mmHg, and pulse oximetry is 90% on room air. Oral examination reveals
poor dentition. Crackles are heard over the right lung base. Chest radiograph
reveals a right lower lobe infiltrate. Sputum Gram-stain shows neutrophils,
Gram-positive rods, and Gram-negative rods. Which of the following is the most
appropriate therapy for this patient’s condition?
a. Amoxicillin-Clavulanate

Chlamydia Pneumoniae
STEP 1

1. A 79-year-old male is brought to the clinic with dyspnea, pleuritic chest pain,
and a dry cough. He currently resides in a nursing home. He says he has had a
runny nose, sinus congestion, and hoarseness for the past 3 weeks, but his
symptoms have been getting worse. Chest x-ray shows diffuse patchy areas of
infiltration in the lungs. He is diagnosed with pneumonia caused by a
gram-negative, obligate intracellular bacteria. Which of the following is also a
characteristic of this organism?
a. It has been implicated in the pathogenesis of atherosclerosis
Klebsiella Pneumoniae
STEP 1

1. A 36-year-old man comes to the emergency department because of fever, chills,


and breathing issues. He recently suffered from strep throat, for which he was
prescribed 10 days of antibiotics. He is coughing with significant sputum that
looks like red currant jelly. Which of the following is the most likely diagnosis?
a. Klebsiella pneumoniae

STEP 2

1. A 67-year-old man comes to the emergency department because of confusion


and a persistent cough. He says that he has had a very foul smelling sputum
that has some tinge of blood when he coughs. He says that he drinks
approximately 5 drinks a day and does not smoke. His temperature is 38.2ºC
(102.0ºF), pulse is 112/min, respiratory rate is 22/min, and blood pressure is
110/85 mm Hg. Physical examinations shows decreased breath sounds at the
right upper lung field. Cardiac examination shows no murmurs and a normal S1
and S2. A sputum sample shows stains positive for short, plump, gram-negative
bacilli and initial cultures grow Klebsiella pneumonia. Sensitivities are still
pending. Which of the following is the best next step in management?
a. Ceftriaxone

Streptococcus Pneumoniae
STEP 1

1. A 35-year-old man comes to the office because of a 5-day history of a


productive cough, shaking chills, and fever. His medical history is relevant for
occasional smoking, bipolar disorder, and a splenectomy when he was 10. Upon
further interrogation, the patient reports having a "rusty" sputum whenever he
coughs. On physical examination, auscultatory findings reveal inspiratory
crackles on inspiration and increased tactile and vocal fremitus. His temperature
is 39.7°C (102.2°F), pulse is 106/min, respirations are 24/min, blood pressure is
120/80 mmHg, oximetry on room air shows an oxygen saturation of 97%. A
conventional chest radiograph is obtained and shown below. This patient is at
increased risk for developing which of the following?
a. Sepsis
2. A 13-year old boy is brought to the emergency department because of a fever
and a productive cough for the last 24 hours. The patient states that he is
having difficulty breathing and is short of breath even without exertion. His
mother noticed that he has a poor appetite, has been low in energy, and has
missed several soccer practices this week. His mother also mentions that
several students in his classroom are also experiencing the same symptoms.
Physical examination of the patient shows dullness to percussion in the lower
lobes and crackles on chest auscultation. Laboratory analysis of a sputum
sample shows gram-positive catalase-negative diplococci. Which of the
following is the most likely microorganism causing the patient's symptoms?
a. Streptococcus pneumoniae
3. A 24-year-old woman comes to the emergency department because of
headache and neck pain for the past two
days. She has two cats at home. She denies
any recent head trauma or travel. Her
temperature is 38.9°C (102°F), respirations
are 18/min, and blood pressure is 125/80 mm
Hg. Physical examination shows a positive
Kernig sign. A lumbar puncture shows:
Which of the following describes the most
likely pathogen?
a. Gram positive coccus, optochin sensitive
4. A 6-year-old boy is brought to the pediatrician for evaluation of left ear pain.
The patient's mother states, “He has been tugging on his left ear for the past
three days; he just cannot seem to get any relief.” The patient has been taking
tylenol at home for symptomatic relief. Temperature is 38.0°C (100.4°F), pulse is
99/min, respirations are 16/min, and blood pressure is 100/65 mmHg. Physical
examination is notable for the following finding: Image reproduced from
Wikimedia Commons Which of the following pathogens is the most likely cause
of this patient’s clinical presentation?
a. Streptococcus pneumoniae
5. A 34-year-old man presents to his primary care physician for evaluation of sinus
pressure. The patient has had a stuffy nose with purulent nasal discharge for
the past two weeks. In addition, he has been experiencing headaches and ear
fullness. His symptoms began improving around 11-12 days after onset but
then took a turn for the worse. Past medical history is notable for type II
diabetes mellitus, for which he takes metformin daily. Temperature is 38.9°C
(102°F), pulse is 101/min, respirations are 19/min, and blood pressure is 132/71
mmHg. Physical exam demonstrates tenderness to palpation over the bilateral
maxillary sinuses. Which of the following organisms is the most likely cause of
this patient’s clinical presentation?
a. Streptococcus pneumoniae
6. An 18-year-old girl comes to the emergency department with her roommate
because she has a severe headache and altered mental status. She started
having the headache 2 hours ago. She is living in the dormitory with her
roommate and shares a lot of items with other tenants. She does not smoke or
drink alcohol. Her medical history is non-contributory. Her temperature is 38.1°C
(100.7°F), pulse is 118/min, respirations are 18/min, and blood pressure is
139/88 mm Hg. Physical examination shows nuchal rigidity with a positive
Kernig and Brudzinski sign. A bacterial pathogen is suspected. Which of the
following cerebrospinal fluid (CSF) analyses would confirm a bacterial
pathogen?
a. Decreased glucose, increased protein and white blood cells –
predominantly neutrophils
7. A 50-year-old man is brought to the emergency department accompanied by his
partner due to sudden onset of confusion and altered mental status. The
patient’s partner mentions that the patient was in his usual state until 2 days
ago when he started complaining of headache and fever. Since then, he has
vomited twice, and yesterday he became confused and disoriented. Temperature
is 39.0°C (102.2°F), pulse is 100/min, and blood pressure is 135/85 mmHg. On
physical examination, the patient is disoriented, and nuchal rigidity is noted. No
focal signs are present, and Babinski sign is negative bilaterally. The results of
the CSF analysis are shown below:
Cerebrospinal fluid

Opening pressure 350 mm H2O

Protein 250 mg/dL

Glucose 30 mg/dL

RBCs 6 cells//mm3

WBC count 1200/µL

Neutrophils 90%

Lymphocytes 10%

Which of the following is most likely to be found on gram stain of the cerebrospinal
fluid of this patient?

a. Gram-positive; lancet-shaped diplococci

STEP 2

1. A 19-year-old woman comes to the emergency department because she has a


fever and headache. She says that 12 hours ago she began having a headache
with nausea and vomiting. Her temperature is 38.2°C (100.7°F), pulse is
118/min, respirations are 15/min, and blood pressure is 140/82 mm Hg. Physical
examination shows significant neck stiffness but passive flexion of the neck
does not elicit spontaneous hip flexion. Neurological exam shows no focal
deficits. She does not have a rash. She has no known drug allergies. She does
not smoke or drink alcohol. A lumbar puncture is performed with the CSF
analysis below: WBC - 1245/mm3, 85% PMN, RBC - 25/mm3 in tube 1, 5/mm3
in tube 4 protein - 300 mg/dL, glucose - 25 mg/dL (concurrent serum glucose -
80 mg/dL). Which of the following is the most appropriate next step in
management?
a. Dexamethasone followed by vancomycin and ceftriaxon

● PLAYLIST 10

Metabolic Acidosis
STEP 1

1. A 54-year-old man comes to the emergency department because he is currently


confused, sluggish, unable to answer questions, and has rapid and shallow:
breathing. A arterial blood gas test is performed, and the results are as follows:
pH 7.04 (7.35 - 7.45)PaCO2 21 (35 - 45 mmHg)PaO2 85 (80-100 mmHg)
HCO3 13 (22 - 26 mEq/L. Which of the following is the most likely diagnosis?

a. Metabolic acidosis with respiratory alkalosis

2. A mother rushes her 4 year old son into the emergency department after finding
him passed out in the garage. She says that she found him unresponsive, lying
face down on the floor of their garage, and when she tried to revive him by
giving mouth to mouth, she noted a sweet taste on his lips. On physical exam,
the patient is unarousable with temperature of 37.5 degrees, BP 110/60, HR 95
and RR of 20. The rest of the exam is unremarkable. A stat metabolic panel
reveals: Na: 135 K: 3.4 Cl: 99 HCO3: 16 BUN: 18 Cr: 0.9 Glucose: 99. A urine
microscopy reveals envelope-shaped crystals. What was the most likely cause
of this child's coma?

a. Ethylene glycol

3. A 70-year-old woman is admitted to the emergency


room with a one-week history of diarrhea and general
weakness. She appears confused and disoriented upon
admission. Her lying blood pressure is 110/70 mm Hg
and sitting blood pressure is 75/45 mm Hg. Physical
examination reveals that she is clinically dehydrated.
Test results reveal the following: Based on the patient’s
symptoms and serum anion gap, which of the following
is the most likely diagnosis?

a. Hyperchloremic metabolic acidosis


4. A 20-year-old woman is rushed to the emergency department after being found
unconscious in her bedroom. Her brother, who found her unresponsive next to
her bed reports that the patient had been feeling depressed since a recent
breakup. He additionally mentions that there were a few white pills next to her
nightstand. A physical exam shows dry skin, scratch marks on both forearms,
and irregular respirations. Her temperature is 36.7°C (98°F), pulse is 130/min,
respirations are 32/min, blood pressure is 105/70 mmHg. Arterial blood gas
analysis on room air shows: Which of the following is most likely seen in this
patient?

a. Mixed high anion gap metabolic acidosis and respiratory alkalosis

5. A 3-year-old boy comes to the emergency department because he has acute


vomiting, diarrhea and melena since earlier that day. His parents describe a
period of time after these symptoms where he appeared to be well. His
condition then deteriorated several hours afterwards, and he had convulsions,
drowsiness and is now comatose. It is later revealed that the patient had
swallowed some of his mothers tablets. Which option would most likely have
caused the neurological signs in this patient?

a. Iron tablets

6. A 34-year-old man comes to the clinic immediately after running a marathon


because of rapid breathing for the past 15 minutes. He used to run marathons
when he was younger, but decided to participate in this one after preparing for
only one week. His temperature is 37.0°C (98.6°F), pulse is 92/min, respirations
are 26/min, and blood pressure is 124/80 mm Hg. Physical examination shows
a pale appearing man who is breathing rapidly. Laboratory studies show a
serum pH of 7.30 and a pCO2 of 28 mm Hg. Which of the following is the most
likely cause of this condition?

a. Lactic acid increase

7. A 30-year-old woman comes to the emergency department because she


attempted to commit suicide by swallowing a handful of aspirins. She has rapid
breathing, vomiting, dehydration, fever, double vision, and is feeling faint. She is
also complaining of ringing in her ears and loss of hearing. Which of the
following acid-base disturbances will be most likely observed in this patient?

a. Mixed respiratory alkalosis with metabolic acidosis

8. A 70-year-old man comes to the emergency department because of worsening


lethargy for the past 2 days. He also complains of nausea, shortness of breath,
and dizziness. Physical examination shows no abnormalities. Serum chemistry
shows: pH: 7.2, Sodium: 145 mEq/L, Chloride: 105 mEq/L, Potassium: 4.8
mEq/L, Bicarbonate: 15 mEq/L, Which of the following is the most likely
diagnosis?

a. Renal failure

9. A 45-year-old man comes to the emergency department because he is


unresponsive. Vital signs reveal a blood pressure of 95/50 mmHg, a heart rate
of 130, and a respiratory rate of 20/min. The patient’s pupils are noted to be
fixed and dilated, and a fruity breath odor is present. Physical examination
shows the liver edge is palpable 4cm below the rib border. Laboratory data
reveals a metabolic acidosis with a high anion gap and normal glucose levels.
Which of the following is the most likely diagnosis in this patient?

a. Alcoholic ketoacidosis

10.A 40-year-old man comes to the emergency department unresponsive. Vital


signs show a blood pressure of 90/60 mmHg, a heart rate of 125/min, and a
respiratory rate of 24/min. The patient’s pupils are fixed and dilated and a fruity
breath odor is present. Physical examination reveals the liver edge is palpable 5
cm below the rib border. Laboratory data is consistent with metabolic acidosis
with a high anion gap and normal glucose levels. Which of the following is the
most likely diagnosis?

a. Alcoholic ketoacidosis

STEP 2

1. An arterial blood gas is drawn on a 15-year-old boy who presented to the


hospital with symptoms of diabetic ketoacidosis. He has no past history of lung
disease, and no wheeze or rhonchi is auscultated on examination. However, he
is tachypneic. In a hurry, the physician has the second year medical student
urgently obtain an arterial blood sample for analysis. The partial pressure of
oxygen (PO2) is found to be 30 mm Hg (normal >80 mm Hg) with an oxygen
saturation (SaO2) of 70% (normal >94%). At the time of the arterial blood
sampling, the patient was in no acute distress with an oxygen saturation of 99%
while on room air. Which of the following best explains the patient's current
presentation?
a. The sample was drawn medial to the femoral artery

Metabolic Alkalosis
STEP 1

1. A 15-year-old girl is brought to the clinic by her mother because of lethargy and
low mood. She has recently dropped out of all extracurricular activities at school
and feels as though she is unattractive and overweight. Physical examination
shows ragged-appearing enamel over the front teeth and callus formation over
her knuckles. She has a height and weight in the 94th and 86th percentile,
respectively. Which of the following acid-base disorders would be the most
likely to be found in this patient?
a. Metabolic alkalosis
2. A 5-week male infant is brought to the
emergency department because of severe
vomiting. The baby was prematurely born to a
gravida 1, para 0, mother via cesarean section
following a pregnancy complicated by
polyhydramnios. The mother said that she’s
been changing the baby’s diaper 20-30 times a
day. A physical exam shows dry skin, sunken
fontanel, and facial dysmorphism (large ears
and drooping mouth). His temperature is
36.7°C (98°F), pulse is 130/min, respirations
are 30/min, blood pressure is 100/50 mm Hg, and pulse oximetry on room air
shows an oxygen saturation of 90%. Arterial blood gas analysis on room air
shows: Which of the following is the most likely mechanism for these laboratory
findings?
a. Loss of function mutation in the Na-K-Cl
cotransporter
3. A 12-year-old boy comes to the office for a
follow-up visit after he was noted to have a
blood pressure of 157/93 mm Hg on his last
office visit. The patient's medical and social
history are unremarkable. The patient's family
history includes a father with hypertension.
The patient continues to feel well and currently
has no complaints. The patient's vital signs
show a temperature of 37°C (98.6°F), pulse
86/min, respirations 14/min, blood pressure
163/97 mm Hg. Physical examination shows
no abnormalities. Laboratory studies show:
Which of the following is the most likely
diagnosis?
a. Liddle syndrome

Metabolic and respiratory alkalosis:


Clinical Practice
STEP 1

1. A 43-year-old woman comes to the emergency department because of


palpitations for the past hour. An arterial blood gas is drawn and shows the
following results: PaO2: 98 mmHg (13.1 kPa), pH: 7.64, PaCO2: 20 mmHg (2.71
kPa), HCO3-: 23 mmol/L. Which of the following is most consistent with her
results?
a. Respiratory alkalosis
2. A 27-year-old woman comes to the emergency department because of severe
vomiting for the past two days. She is 9 weeks pregnant and began
experiencing regular morning sickness in the form of mild nausea, with
occasional episodes of vomiting, 7 weeks into her pregnancy. Her episodes of
vomiting have since become more frequent. In the past 2 days, she says she has
vomited over 25 times. Her other complaints include fatigue, dizziness, and
epigastric pain associated with vomiting. Physical examination shows she is
afebrile, pulse is 107/min, and blood pressure is 120/74 mmHg. Her arterial
blood gas shows: pH 7.51, pCO2 46 mmHg, pO2 80, Bicarbonate 32. Which of
the following acid-base disturbances is present in this patient?
a. Metabolic alkalosis
3. A 15-year-old boy comes to the emergency department with his mother
because of rapid, uncontrollable breathing. His mother states that two hours
earlier the boy's girlfriend ended their relationship. The boy had been sobbing
profusely when he started breathing rapidly. The patient states that his mouth
feels tingly. His temperature is 37.1°C (98.8°F), pulse is 96/min, respirations are
32/min, and blood pressure is 122/86 mm Hg. Physical examination shows
perioral pallor and carpopedal spasm. Which of the following is the most likely
etiology of his symptoms?
a. Respiratory alkalosis
4. A 3-week-old infant is brought to the emergency department because of
projectile vomiting after feeding. She has vomited like this for three days, but her
mother says the infant has a good appetite despite losing weight. The mother
remembers that there may have been a similar problem with the baby's paternal
uncle, who needed surgery when he was an infant. Her temperature is 37.0°C
(98.6°F), pulse is 140/min, respirations are 45/min, and blood pressure is 70/45
mm Hg. The abdomen is non-tender and non-distended, but a small mass can
be palpated below the costal margin. Which of the following findings is most
likely a characteristic sign of this condition?
a. Hypochloremic hypokalemic alkalosis

STEP 2

1. A 33-year-old man has suffered severe head trauma in a motor vehicle accident.
Which of the following precautionary treatments would be most appropriate?
a. Hyperventilation
Metabolic and respiratory acidosis:
Clinical Practice
STEP 1

1. A 45-year-old man comes to the emergency department with left lower


quadrant pain, fever. This is the 4th episode this year. A diagnosis of recurrent
diverticulitis is made and a left colectomy is scheduled for tomorrow. During
anesthesia, an arterial blood gas is drawn and shows: pH: 7.3, PaCO2: 70 mm
Hg, HCO3-: 30 mEq/L. Which of the following best describes the patient's
acid-base disorder?
a. Respiratory acidosis with compensation

STEP 2

1. A 14-year-old boy comes to the emergency department because of worsening


respiratory distress. This patient was seen 3 days ago with a 2-day history of
shortness of breath and fever; he was diagnosed with pneumonia and
discharged with amoxicillin-clavulanate, azithromycin, and instruction to follow
up with his primary care physician. In the past day he has developed additional
symptoms of abdominal pain, nausea, non-bilious emesis, and polyuria. His
temperature is 37.9°C (100.3°F), pulse is 106/min, respirations are 26/min, and
blood pressure is 110/74 mm Hg. Physical examination shows a
well-developed, ill-appearing patient in respiratory distress with Kussmaul
respirations and fruity breath odor. Chest x-ray shows consolidation of the
lower left lobe. Laboratory studies show: Na+: 130 mEq/L, K+: 3.4 mEq/L, Cl-:
92 mEq/L, HCO3-: 13 mEq/L, Blood Urea Nitrogen: 30, Creatinine: 1.5, Amylase:
162 U/L. Arterial blood gas shows: pH: 7.30, PaO2: 95 mm Hg, PCO2: 28 mm
Hg. Which of the following is the most appropriate next step in management?
a. Serum acetoacetate

Ketone body metabolism


STEP 1
1. A prisoner goes on a hunger strike to protest the conditions of his detainment.
After ten days without food his glycogen stores are depleted, and
glycogenolysis has slowed to a halt. Gluconeogenesis has peaked to fuel his
energy requirements. His blood glucose level is now 50 mg/dL. In addition to the
glucose, which of the following molecules is his liver producing and releasing
into the bloodstream to meet the energy demands of his brain?
a. ß-hydroxybutyrate

● PLAYLIST 11
○ A 35-year old male is to undergo heart surgery and will be placed on cardiac
bypass. He is anticoagulated with unfractionated heparin prior to and during bypass.
At the end of the procedure, the patient is weaned off the cardiopulmonary bypass
by warming, lung ventilation, and heart defibrillation. The venous return of the
bypass machine will be slowed down, allowing the heart to fill and the external pump
to be decelerated. While this is occurring, global cardiac function and
hemodynamics must be monitored. Finally, anticoagulation must be reversed and
hemostasis achieved. What should he used to achieve this? Protamine sulfate
○ A 23-year-old woman is brought to the emergency department with fever and loss of
consciousness. Her temperature is 39.0°C (102.2°F), pulse is 103/min, respirations
are 22/min, and blood pressure is 100/60 mm Hg. Physical exam shows an
ill-appearing woman with pallor with a port-o-cath in her left chest. She had begun
chemotherapy for leukemia 10 days ago. Blood cultures are still pending, but
preliminary blood analysis shows: Hemoglobin 13.0 g/dL, Leukocyte count
1400/mm3, Segmented neutrophils 30%, Bands 5%, Eosinophils 5%, Basophils 2%,
Lymphocytes 50%, Monocytes 8%, Platelet count 200,000/A 25-year-old
Japanese male is hospitalized because of neutropenic fever. Physical
examination shows a nodular skin lesion on his leg. Biopsy of the lesion
shows a dermal lymphocytic infiltrate with migration into the
epidermis.mm3. G-CSF
○ A 45-year-old woman presents at a the doctor's office visit with fatigue and
weakness. These symptoms have persisted for the past three weeks. She
complains of excessive sweating at night. Physical examination reveals
splenomegaly. Fluorescence in situ hybridization cytogenetic testing detects a 9;22
translocation and the BCR-ABL fusion gene within somatic cells, and she receives
Imatinib as treatment. Which of the following describes the mechanism by which this
drug works? Inhibition of tyrosine kinase
○ A 5-year-old boy comes to the office because of red urine for the past six hours. He
is currently being treated for acute lymphoblastic leukemia with chemotherapy that
includes ifosfamide. His last dose of this round of chemotherapy was 48 hours ago.
His temperature is 36.8°C (98°F). pulse is 87/min, respirations are 18/min, and
blood pressure is 117/78 mm Hg. Urine sample shows macroscopic hematuria.
Which of the following medications could have most likely prevented this adverse
effect of treatment? Mesna
○ A 40-year-old man comes to the emergency department because of fatigue, fever
and a sore throat for the last 3 days. He states that he has also noticed that he has
been bruising more easily. Physical examination of the patient shows a middle-aged
male in moderate distress with marked pallor and shortness of breath. Initial
laboratory studies shows thrombocytopenia and normocytic normochromic anemia.
A peripheral blood smear shows many immature leukocytes. A few hours later the
patient begins to bleed spontaneously from several orifices, and several large
petechiae develop on his arms and legs. Laboratory studies show low platelets,
increased bleeding time, prothrombin time and partial thromboplastin time. Which of
the following translocations is associated with the etiology of the patient's condition?
t(15:17)
○ A 14-year-old boy is brought to the clinic for evaluation of his persistent fevers and
bruising. His mother reports his fevers have come and gone for the past month. The
boy also notes his thighs and upper arms are painful, and the pain feels "really
deep" within the limbs. On exam, you observe a thin boy with mild limb tenderness
and several non-tender cervical lymph nodes measuring 21 mm each. There is
extensive bruising bilaterally on the lower extremities. When spoken with privately,
the boy reports that he feels safe and supported at home. Laboratory studies show
the following. Penoneral smear shows several immature leukocytes. Which of the
following tests is needed to confirm the diagnosis? Bone marrow biopsy
○ …. White blood cell count 21.5 x 10^3/mm3, Hemoglobin 7.4 g/dL, Platelet count
101.000/mm, Sodium135 mEa/L, Chloride100 mEg/L, Urea nitrogen, serum 18
mg/dL, Glucose 80 mg/dl, Potassium 6.4 mEa/L, HC03 18 mEq/L, Creatinine1.3
mg/dL, Calcium9.0 mg/dL, Phosphate4.5 mg/dL, Uric acid 9.0 mg/dL. Which of the
following is the most likely cause of this patient's elevated creatinine? Uric acid
crystal deposition
○ … Which of the elements of the dermis are primarily responsible for the distribution
of this man's condition? Eccrine glands
○ A 28-year-old G1P0 woman at 14 weeks gestation comes to the prenatal clinic for a
routine visit. Since the last time she saw you, this patient has had blood drawn for
her routine prenatal laboratory workup. Her blood type is O negative and she says
that the father of the baby has the same blood type. Assuming that she is correct
about the father of the baby's blood type, how would you advise this patient
regarding Rh-D alloimmunization? This patient should not receive any doses of
anti-D immune globulin
○ A 70-year-old man comes to his primary care physician with complaints of a
dragging sensation in his abdomen when he walks. Physical examination reveals
massive splenomegaly. A peripheral blood smear shows several unusual cells that
stain positively for tartrate-resistant acid phosphatase. Which of the following
additional features is most likely present in these cells? Hair-like cytoplasmic
projections
○ … Physical examination shows mild splenomegaly. There are no signs of
lymphadenopathy. A complete blood count shows a leukocyte count of 30.000/mm3
and a hemoglobin concentration of 9.1 g/dL. A blood smear and stain show early
myeloid cells. A fluorescence in situ hybridization test shows the presence of the
Bcr-Abl tyrosine kinase. A bone marrow biopsy is shown. A diagnosis of chronic
myelogenous leukemia is made and the patient is started on imatinib. Which of the
following best describes the mechanism of action of this medication? Tyrosine
kinase inhibitor
○ A 27-year-old woman is brought by ambulance to the resuscitation room of the
emergency department after a high-speed road traffic crash. She has major injuries
including a lacerated left arm and has suffered massive intra-abdominal
hemorrhage. She has hemorrhagic shock and blood transfusion is required
immediately. What blood group of red cells should she receive? O Rh(D) negative
○ A 27-year-old woman is reviewed in the postoperative care unit because of acute
onset of fever and chills for 10 minutes. She underwent total colectomy for Crohn's
disease an hour ago. She was given a dose of perioperative antibiotics and received
one unit of packed red blood cells during the surgery for excessive blood loss. Her
temperature is 39°C (102.2°F), blood pressure is 120/80 mmHg, pulse is 95/min,
respirations are 14/min, and SaO2 is 99% on room air. She has an indwelling
urinary catheter, arterial line, and peripheral IV; all of which were placed at the time
of surgery. Which of the following is the most likely cause of her fever? Transfusion
Reaction
○ A 25-year-old G1P0 woman presents to the office for her first prenatal visit. History
reveals the patient is blood type O negative and she has never received blood
products in the past. The patient refuses Rho(D) immune globulin because it is
derived from human plasma and she does not want the risk of contracting HIV.
Which of the following is the most appropriate advice to this patient? Rho(D)
immunoglobulin is unnecessary if the father isRh-negative
○ … a lesion is obtained. Which of the following is most likely to be found
histologically? Neutrophilic infiltrate
○ A 25-year-old Japanese male is hospitalized because of neutropenic fever. Physical
examination shows a nodular skin lesion on his leg. Biopsy of the lesion shows a
dermal lymphocytic infiltrate with migration into the epidermis. The nuclei of the
lymphocytes display a characteristic lobulated, appearance. With which of the
following viruses is this patient most likely infected? Human T-lymphotropic virus
ntr
● PLAYLIST 12
○ A 77-year-old man comes to the emergency department because he
fainted 20 minutes ago. He states that he has had 4 months of increasing
dyspnea on exertion and lower extremity swelling. His temperature is
36.5°C (97.7°F), pulse is 88/min, respirations are 18/min, and blood
pressure is 138/88 mm Hg. Examination shows distended neck veins,
crackles in both lung bases on auscultation. Cardiac auscultation shows a
crescendo-decrescendo murmur heard loudest at the right sternal edge,
in the second intercostal space. Which of the following medications
should most likely be avoided in the management of this patient’s
condition? Isosorbide mononitrate
○ A 68-year-old man comes to the emergency department because of feelings like his
"heart is beating out of his chest" for the past 2 hours. He has a history of
hypertension, congestive heart failure, and two prior myocardial infarctions but he
says that this particular incident feels different. His temperature is 36.8°C (98°F).
pulse is 145/min, respirations are 18/min, and blood pressure is 92/64 mm Hg.
Cardiac examination shows an irregularly irregular pulse but no murmurs, rubs or
gallops. Below is an ECG with findings similar to those of the patient. Which of the
following is the most appropriate next step?cardioversion
○ An 80-year-old man comes to the emergency department because of worsening
shortness of breath and edema He has a history of diabetes melitus, chronic kidney
disease. and congestive heart failure. Upon arrival, he becomes unresponsive and
CPR is started. ECG shows ventricular fibrillation, initial cardioversion, is
unsuccessful, but subsequent shock results in the rhythm shown on the ECG below.
Which of the following is most appropriate next step in management? Check the
patient's pulse
○ A 55-year-old man comes to the surgical ward because of tachycardia and a feeling
of heart palpitations for the past 2 hours. He has a history of coronary artery
disease, for which he underwent a coronary artery bypass surgery three days ago.
He is alert and says his main issues are wound pain and shortness of breath. His
temperature is 36.8°C (98°F). pulse is 130/min, respirations are 23/min, and blood
pressure is 105/78 mm Hg. Which of the following is the most likely rhythm that will
be seen on ECG? Atrial fibrillation
○ … Physical examination shows tachycardia and mild hypotension. The patient's
electrocardiogram is obtained. Which of the following drugs is the best choice for
first line treatment of the patient's condition? Magnesium sulfate
○ A 33-year-old woman is brought to the emergency department by police because of
multiple syncopal episodes witnessed by the police. Her speech Is rapid and
nonsensical, and gaining any information from the patient is futile. She has a
medical record that shows frequent psychiatric hospitalizations and a prescription for
lithium and antipsychotics. She is hemodynamically stable upon presentation, and
remains so until she has another witnessed syncopal episode. She is breathing and
has a palpable pulse. An ECG is obtained at this time: Administer magnesium
sulfate via intravenous bolus
○ A 68-year-old man comes to the emergency department because of feelings of his
"heart beating out of his chest" for the past 2 hours. His medical history includes two
prior myocardial infarctions but he reports that this feels different. His temperature is
36.8°C (98°F). pulse is 113/min, respirations are 18/min, and blood pressure is
117/78 mm Hg. Cardiac examination shows an irregularly irregular pulse but no
murmurs, rubs or gallops. An ECG is obtained and is shown below. Assuming this
arrhythmia is new in onset, which of the following laboratory tests should most likely
be performed? Thyroid-stimulating hormone
○ … The dizziness is worse with standing. He has never had an episode like this
before. He has a history of diabetes mellitus and hypertension for which he takes
metformin and chlorthalidone. Physical examination shows abnormalities. Troponins
are negative. TSH concentration is normal. An ECG is obtained, the results of which
are shown below. Previous ECG results have shown no abnormalities. Which of the
following Is the most appropriate next step? IV diltiazem
○ A 23-years-old man comes to the clinic for his routine check-up. He has no
complaints regarding his health and his medical history is noncontributory. Physical
examination and laboratory assessment show no abnormalities with the exception of
a notched ('M'-shaped) R-wave in lead V6 of his electrocardiogram. Which of the
following is the most appropriate next-step in management? Explaining to the
patient that he has nothing to worry about
○ A 66-year-old man comes to the office because of feelings that his heart has been
"fluttering" and "skipping beats" for the past 8 hours. He drinks 8-9 alcoholic drinks
per day and has hypertension for which he is prescribed lisinopril. He says two
nights ago, he went to a party and drank "a lot more than usual." He is not aware of
having heart problems and he asks you whether he requires treatment. His
temperature is 36.8°C (98°F), pulse s 135/min, respirations are 18/min. And blood
pressure is 117/78 mm Hg. Examination shows an irregular heart rate. An ECG is
obtained and is shown below. Which of the following is the most appropriate
response to the patient? Your symptoms will likely resolve in 24 hours
○ A 57-year-old woman comes to the emergency department because of recurrent
episodes of palpitations with associated dizziness, nausea, chest pain, and cold
○ sweats over the past 2 days. Polymorphic ventricular tachycardia, characterized by
shifting sinusoidal waveforms is seen on ECG. What medication could you
administer to treat your patient's condition? Magnesium sulfate
○ A 66-year-old man comes to the office because of feelings that his heart has been
"fluttering" and "skipping beats" for the past 8 hours. He drinks 8-9 alcoholic drinks
per day and has hypertension for which he is prescribed lisinopril. He says two
nights ago, he went to a party and drank "a lot more than usual”. He is not aware of
having heart problems and he asks you whether he requires treatment. His
temperature is 36.8°C (98°F), pulse is 135/min, respirations are 18/min, and blood
pressure is 117/78 mm Hg. Examination shows an irregular heart rate. An ECG is
obtained and is shown below. Which of the following is the most appropriate
response to the patient?Your symptoms will likely resolve in 24 hours
○ A 26-year-old man presents to the emergency department because of chest
palpitations and lightheadedness. These symptoms began to appear one hour ago.
The patient notes he does not take medications, has occasional beer on weekends,
and does not smoke tobacco. He also denies any recreational drug use.
Examination shows that the blood pressure is 112/66 mm Hg, pulse is 175/min,
respirations are 16/min, and oxygen saturation is 98% on room air. Cardiac
examination reveals S1 and S2 heart sounds without murmurs, and pulmonary
exam is within normal limits. An ECG reading reveals a heart rate of 175/min with a
QRS-complex of 95 milliseconds in regular rhythm. There are no discernable
P-waves. A carotid sinus massage is performed, but has no effect on his symptoms
or ECG. Which of the following is the most appropriate next step in management?
Adenosine
○ A 64-year-old man comes to the office for a pre-operative evaluation
because he is scheduled for an elective cholecystectomy in 2 weeks. He
has a history of diabetes mellitus and hypertension. Medications include
metoprolol, atorvastatin, and metformin. His temperature is 36.8°C
(98°F), pulse is 76/min, respirations are 18/min, and blood pressure is
136/78 mm Hg. An ECG is obtained and is shown below. Which of the
following is the most appropriate adjustment to this patient's
medications? No medication adjustment needed
○ A 72-year-old man with a history of hypertension, atherosclerosis, a
myocardial infarction status post triple coronary artery bypass graft
surgery, and an ejection fraction of 42% is brought in to the emergency
department due to palpitations and anxiety. On physical examination the
patient is disoriented with a temperature of 37°C (98.6°F), undetectable
pulse, respirations of 4/min, blood pressure of 80/50 mmHg, and SpO2 of
88%. Cardiac enzymes have been ordered, and an electrocardiogram
(ECG) strip is shown below. Which of the following treatments is the
best choice for managing this patient's condition? Inmediatamente
defibrillation
○ A 61-year-old man comes to the emergency department with two weeks of fevers
and chills. He also reports a poor appetite and has noticed a weight loss of 2.3 kg (5
lb) over the past month. He also complains of malaise with intermittent headaches,
muscle aches, and night sweats over this same time period. Medical history is
significant for chronic kidney disease (CKD) stage II, due to hypertension.
Medications include lisinopril and hydrochlorothiazide. He was born in India, and he
does not drink alcohol or use recreational drugs. His temperature is 39.4°C
(102.9°F), pulse is 45/min, respirations are 23/min, blood pressure is 100/50 mmHg,
and oxygen saturation is 95% on room air. He appears ill. Physical exam shows a
regularly irregular heartbeat with a high-pitched, early diastolic decrescendo
murmur. Lung auscultation reveals crackles at the bilateral lung bases. Laboratory
studies show renal function at baseline and normal cardiac enzymes. An ECG is
obtained and shows the following: Transcutaneous pacing
○ A 65-year-old woman presents to the emergency department with palpitations,
lightheadedness, and shortness of breath. She explains she has experienced these
symptoms for the past three days and that her symptoms are not associated with
physical exertion. The patient has a history of hypertension, hyperlipidemia, and
type 2 diabetes mellitus. The patient denies any loss of consciousness, muscle
weakness, sensory deficits, chest pain, vomiting, fever, and chills. Examination
shows that her temperature is 37°C (98.6°F), blood pressure is 150/85 mm Hg,
pulse is 160/min, respirations are 17/min, and oxygen saturation is 98% on room air.
Cardiac examination reveals an irregularly irregular rhythm and tachycardia, which
are both confirmed with an ECG. The patient promptly received IV metoprolol and is
closely monitored. Examination later reveals resolution of her dyspnea and a heart
rate of 95 beats per minute. The consulting cardiologist reports that the patient is at
moderate-high risk of thromboembolic events and recommends prophylaxis. Which
of the following is the most appropriate next step in management? Warfarin
○ Ejection fraction is 33%, SpO2 is 99%, cardiac enzymes are negative for myocardial
infarction, and an ECG was obtained and is shown below. Which of the following is
the most appropriate treatment option for managing this patient's condition?
Intravenous lidocaine
○ A 20-year-old male college student comes to the clinic with an ECG
obtained at a health fair, where he was told the reading was abnormal. He is
a competitive track and field athlete and excels in long-distance running. He
denies chest pain, shortness of breath, or loss of consciousness. He has no
prior medical history and takes no medications other than a daily
multivitamin. He denies family history of sudden unexpected deaths. His
temperature is 37.0°C (98.6°F), pulse is 65/min, respirations are 20/min, and
blood pressure is 115/75 mmHg. He appears thin and comfortable. Physical
exam shows a normal heart rate without murmurs and 2+ regular carotid
and radial pulses bilaterally. Lungs are clear. ECG shows the following:
Which of the following is the most appropriate management of this patient? No
treatment is required
__________________________________________________________________________

GASTROENTEROLOGÍA

● PLAYLIST 1
○ A 28-year-old man comes to the emergency department because of vomiting,
sudden onset severe retrosternal chest pain, and strange popping sensations under
his fingers when he pushes on the skin in his upper chest. He was well until
yesterday evening when he developed nausea and vomiting several hours after a
fast-food meal. Physical examination shows subcutaneous Emphysema. Which of
the following is the most likely diagnosis? Boerhaave syndrome
● A 62-year-old man who immigrated 10 years ago from Colombia is referred to a
gastroenterologist because of difficulty swallowing for the past 5 months. The
patient denies smokin, drinking, or pertinent family medical history. He does
mention that lately he has been experiencing shortness of breath and swelling of
both of his legs. A barium swallow study is completed and the results are shown
below. Esophageal manometry confirmed the diagnosis. What is the most likely
underlying cause of this patient's condition? Chagas disease
● A 32-year-old man comes to the clinic because of a progressive history of painful
swallowing of solids and liquids. He has putrid breath and esophageal
manometry shows an elevated lower esophageal sphincter pressure. A barium
swallow study is obtained and the results of which are shown below. Which of the
following is the most likely diagnosis? Achalasia
● A 55-year-old man comes to the clinic due to the recent onset of throat pain,
fever, and substernal burning pain. Medical history is notable for hypertension,
hyperlipidemia and end-stage renal disease, for which he received an allograft
renal transplantation from a deceased donor 6 months ago. Medications include
azathioprine and TMP-SMX. Temperature is 38.4°C (101.1°F), pulse is 80/min,
and blood pressure is 145/85 mmHg. Cardiopulmonary examination is within
normal limits. Examination of the oropharynx is normal without exudate or
erythema. The abdomen is soft with mild epigastric tenderness. Leukocyte count
is 5000/mm3. An endoscopy is obtained and reveals multiple shallow linear
ulcers in the distal esophagus. Which of the following best characterizes the
organism responsible for this patient's findings? Enveloped double stranded DNA
virus
● A 60-year-old man comes to the clinic for evaluation of difficulty swallowing
accompanied by chest pain. The patient had a few of these episodes in the past
but did not seek treatment. The episodes occur while he is having a meal, but
only occasionally. He describes difficulty in swallowing both liquids and solids. He
has not noticed any pain during exertion. Medical history is noncontributory. The
patient does not smoke or use illicit drugs, and he drinks alcohol socially.
Temperature is 37.0°C (98.6°F), pulse is 75/min, and blood pressure is 125/85
mmHg. ECG shows no abnormalities or Ischemic changes. Two subsequent
troponin measurements are within normal limits. Which of the following is most
likely seen in this patient on further evaluation? Simultaneous contractions of the
lower esophagus on manometry
● An 80-year-old man comes to the clinic due for a follow-up after hospitalization.
He was hospitalized two weeks ago for pneumonia and was treated with
ampicillin-sulbactam. Medical history is notable for type 2 diabetes mellitus and
aortic stenosis. Further medical history reveals 2 prior hospitalizations in the past
year for pneumonia. The patient reports difficulty swallowing and regurgitation of
undigested food particles. A barium esophagram is performed and shown below:
D
● A 43-year-old man comes to the clinic because of a 7-month history of
progressive pain upon swallowing. He has trouble tolerating solids and liquids
and has developed increasingly foul breath. He often regurgitates undigested
food and has had a chronic cough. A barium swallow is obtained and is shown
below. Which of the following types of peristalsis pattern is characterized by this
patient's condition? Tertiary peristalsis
● A 55-year-old man comes to the emergency department because of several
episodes of vomiting. The patient complains of retrosternal chest pain, back pain,
and shortness of breath. Auscultation shows diminished breath sounds bilaterally
at the bases, and a crackling sound heard best directly over the heart. Palpation
of the anterior chest is best described as feeling like there are "rice crispies"
underneath the skin. Which of the following tests would confirm the diagnosis?
Gastrografin swallow study
● A 62-year-old man who immigrated 10 years ago from Colombia is referred to a
gastroenterologist because of difficulty swallowing for the past 5 months. The
patient denies smoking, drinking, or pertinent family medical history. He does
mention that lately he has been experiencing shortness of breath and swelling of
both of his legs. A barium swallow study is completed and the results are shown
below. Esophageal manometry confirmed the diagnosis. What is the pathological
agent behind a common cause for the secondary form of this disease?
Trypanosoma cruzi
● A 33 year-old man comes to the gastroenterologist because of progressive
dysphagia over the last 3 months. Barium swallow shows a dilated esophagus
with an area of distal stenosis. Upon surgical treatment of the patient's condition,
which of the following is the patient at the greatest risk of developing?
Gastro-esophageal reflux disease
● A 35-year-old man comes to the office due to difficulty swallowing, food
regurgitation, and chest pain for the past several months. The patient describes
difficulty swallowing both liquids and solids. He often regurgitates large pieces of
undigested food mixed with saliva. History is notable for drinking 2 beers every
evening and smoking 1 pack of cigarettes per day for the past 5 years. He has
had no recent illness, weight loss, or fatigue. He has not traveled out of the
country recently. Vital signs are within normal limits. Physical examination,
including cardiopulmonary examination, is normal. Which of the following is the
most likely diagnosis? Achalasia
● A 52-year-old woman comes to the urgent care clinic because of progressive
dysphagia for solids and liquids. She has putrid breath and esophageal
manometry shows elevated lower esophageal sphincterpressure. A barium
swallow study is obtained and the results of which are shown below. Which of the
following organisms is most likely to be implicated? Trypanosoma Cuzi
● A 60-year-old man comes to the clinic due to intermittent difficulty swallowing for
the past few months. The patient describes intermittent difficulty swallowing
solids, which is often relieved by drinking fluids and after self-induced
regurgitation. There is no associated weight loss or fatigue. Medical history is
notable for a sliding hiatal hernia, hypertension, and long term gastroesophageal
reflux disease. Medications include proton pump inhibitors and thiazides. The
patient does not smoke or use alcohol or illicit drugs. Vitals are within normal
limits. The rest of the physical examination is normal. Endoscopy is performed,
and the results are equivocal. Barium esophagogram is obtained and reveals the
following: Esophageal ring
● A 47-year-old man comes to the clinic because of food sticking to his throat, a
sensation that has grown gradually worse over the last 5-7 years. He has
dysphagia for solid food and liquids, and often spits food back up undigested,
along with a large amount of saliva. He reports severe discomfort after a large
meal and has lost 6.8-kg (15-lb) over the past three years unintentionally. He
says he has substernal chest pain and cannot remember the last time he
vomited, although he does frequently hiccup. He does not want to lose additional
weight and wants to understand the situation and what treatment options are
available. Which of the following tests is most appropriate to confirm the probable
diagnosis? Manometry
● A 40-year-old woman comes to the office due to difficulty swallowing, food
regurgitation, and reflux for the past several months. The patient describes
difficulty swallowing both liquids and solids. She occasionally regurgitates chunks
of undigested food. Medical history is unremarkable except for a C-section 8
years ago. She has had no recent illness, weight loss or fatigue, and she does
not smoke cigarettes or use illicit drugs. She drinks alcohol socially. She has not
traveled out of the country recently. A barium swallow test is performed and
shown below: B
● A 35-year-old man comes to the office due to dysphagia accompanied by food
impaction and upper abdominal pain for the past few months. He had tried
multiple over the counter antacids with minimal relief. Past medical history is
notable for gastroesophageal reflux disease and seasonal allergies. He has
smoked 1 pack of cigarettes per day for 10 years. An endoscopy is performed,
and the results are shown below: Eosinophilic infiltration of the esophagus
● A 49 year-old woman comes to the clinic because of tightening of the skin on her
hands and face. She has a history of hypertension and asthma. She is currently
taking amlodipine, losartan, and salbutamol. She does not smoke or drink
alcohol. Her temperature is 37.5°C (99.5°F), pulse is 70/min, respirations are
14/min, and blood pressure is 145/87mmHg. Physical examination shows
non-pitting edema and some ulceration on her hands. Her abdomen is not
distended. Bowel sounds are hypoactive. She does not have stridor but there is
some diffuse bilateral wheezing on auscultation with good air entry bilaterally.
Cardiac examination is normal. Which of the following organs is most likely to be
affected by the patients disease? Esophagus
● A 40-year-old woman comes to the clinic because of bilateral, symmetrical
swelling of her hands and fingers, as well as dysphagia with both solids and
liquids. Physical examination shows telangiectasis on her forehead (see picture),
and her phalanges are positive for Raynaud's phenomenon. Which of the
following antibodies are most likely to be found on laboratory testing?
Anti-centromere
● A 65-year-old woman comes to the clinic due to difficulty swallowing and
retrosternal pain. For the past 2 days, she has had pain with swallowing which
has progressively worsened to the point of avoiding all food and drinks. She has
not had similar symptoms in the past. Medical history is significant for coronary
artery disease, hyperlipidemia, seasonal allergies, and osteoarthritis. The patient
has smoked 1 pack of cigarettes per day for 30 years. Medications include
aspirin, metoprolol, loratadine, lovastatin, and piroxicam as needed. Temperature
is 37.0°C (98.6°F), pulse is 80/min, and blood pressure is 125/85 mmHg. The
oral mucosa is clear without erythema or exudate. Cardiac auscultation reveals
no murmurs, and ECG shows normal sinus rhythm without ischemic changes.
The rest of the physical examination is normal. Endoscopy is obtained and shows
multiple round ulcers in the proximal esophagus with relatively normal
surrounding tissue. Which of the following is the most likely diagnosis?
Pill-induced esophagitis
● A 55-year-old woman comes to the office due to difficulty swallowing for the past
2 months. The patient describes intermittent difficulty swallowing solids which
improves with water or by chewing her food more. She also reports feeling more
tired during this time period. Medical history is noncontributory. Temperature is
36.5°C (97.7°F), pulse is 110/min, and blood pressure is 135/85 mmHg. Physical
examination reveals pale conjunctiva. Which of the following may be seen on
further evaluation of this patient? Koilonychia
● A 58-year-old woman comes to the urgent care clinic because of progressive
dysphagia for solids and liquids over the last 6 months. She has putrid breath
and esophageal manometry shows elevated lower esophageal sphincter
pressure. A barium swallow study is obtained and the results of which are shown
below. Which of the following treatment options is the most appropriate
management choice for this patient? Botulinum injection
● A 58-year-old woman comes to the urgent care clinic because of a several month
history of progressive pain with swallowing. She has difficulties tolerating solids
or liquids and also has foul-smelling breath. She has no known allergies and has
recently traveled to South America. A barium swallow study is obtained and the
results of which are shown below. Which of the following is the most appropriate
next step in management for this patient? Injection of botox into the lower
esophageal sphincter
● A 2800-g (6.17-lb) female infant is born vaginally and at term to a 21-year-old
woman who failed to have close prenatal follow-ups throughout her pregnancy.
Shortly after birth, the infant begins to cough and subsequently undergoes
choking episodes. She is cyanotic. Resuscitation efforts are initiated and her
condition stabilizes. The following day time-lapsed images of barium swallow are
taken and are shown below. Which of the following is the most likely diagnosis?
Congenital tracheoesophageal fistula
● A 72-year-old man comes into the outpatient clinic because of difficulty
swallowing. For several years he had difficulty swallowing solids, but recently
developed problems swallowing liquids. He denies fever, chills, nausea, vomiting,
and unintentional weight loss. The patient does not smoke cigarettes and does
not drink alcohol. His temperature is 37.0°C (98.6°F), pulse is 80/min,
respirations are 15/min, and blood pressure is 125/85 mm Hg. Barium swallow
study with fluoroscopy shows a diverticulum arising from the midline of the
posterior wall of the distal pharynx. Which of the following is the most appropriate
next step in management? Cricopharyngeal myotomy and diverticulectomy
● A female neonate is delivered vaginally at full term to a 23-year-old woman who
has not had close prenatal follow-up throughout her pregnancy. Several hours
after birth, the girl begins to cough and subsequently experiences choking
episodes. Examination shows that the infant is cyanotic. Resuscitation efforts are
initiated and her condition stabilizes. Which of the following is the most likely
diagnosis? Type C: esophageal atresia with distal tracheoesophageal fistula
● A 55-year-old woman is brought to the emergency department with acute onset
chest pain for the past 30 minutes. The pain started suddenly during dinner. The
patient describes the pain as crushing substernal pain and rates it as 8/10. She
recalls a few occasions of similar pain episodes but did not seek treatment
because she thought it would pass. Medical history is significant for smoking 1
pack per day for 30 years. The patient has not noticed any relation to specific
foods and does not have any reflux symptoms. The patient took her partner's
sublingual nitroglycerin pill before the ambulance arrival, which provided partial
relief. Temperature is 37.0°C (98.6°F), pulse is 90/min, and blood pressure is
135/85 mmHg. ECG is performed and shows normal sinus rhythm with no
significant ST-changes. A barium swallow is performed and is shown below:
Peppermint oil
● A 3-year-old patient presents with ear pain which has persisted over the last
week. Her mother reveals she had a throat infection the week prior. The patient is
diagnosed with a middle ear infection. What is the most likely route of infection
spread from the child's pharynx to her tympanic cavity? Pharingothympatic tube
● The __musculooo____ is highlighted in the image below Stylopharyngeus
● The ______nervioooo_____ is highlighted in the image below Pharyngeal branch
of vagus
● The ________________ is highlighted in the image below Piriform fossa
● The ____________________________ is highlighted in the image below
Superior pharyngeal constrictor
● The _parece chicle pegado__ is highlighted in the image below Pharyngeal
tonsils
● The _cerca del arco aortico___ is highlighted in the image below L. Recurrent
laryngeal nerve
● A patient presents with hoarseness and neck pain, which began the day prior
when he was eating a salmon filet for lunch. Imaging reveals a fish bone stuck in
his piriform fossa. What nerve is most likely at risk of injury? Internal laryngeal
● A patient presents with injury to the glossopharyngeal nerve. What muscle would
most likely be affected by this injury? Stylopharingeous

● PLAYLIST 2
○ A 2-month-old infant is brought to the office because of vomiting and cough for a
month. The vomiting is worse following meals, and the infant is exclusively formula
fed. The mother states these symptoms have been worsening. She says there has
been no projectile or bilious vomiting, fever, sputum production, or lethargy. She
says she keeps the child upright while feeding and has tried to switching formulas
without improvement. Physical examination shows the infant is in the 5th percentile
for height and weight. The remainder of the examination shows no abnormalities.
Which of the following is the most appropriate diagnostic investigation? Esophageal
pH monitoring
○ A 60-year-old man comes to the emergency department because of sub-sternal
chest pain for an hour. He has experienced this pain before, typically after heavy
meals, during times of stress, and when he goes running in the park. He also feels
the pain at night when he is lying in bed and has previously been woken from sleep
by discomfort. He has not noticed any dyspnea, diaphoresis, or palpitations but is
currently experiencing some nausea and a sour taste in his mouth. Medical history
is non-contributory. ECG shows normal sinus rhythm. Which of the following is the
most likely diagnosis? Gastroesophageal reflux
○ A 42-year-old woman comes to the office because of hematemesis and melena for
the past week. She has been experiencing abdominal pain, which she currently
rates a 4 on a 10-point scale. She says that the pain worsens when she eats and
that she often develops nausea after eating. The pain is worse, particularly when
eating fried foods. Physical examination shows tenderness to palpation in the
epigastric area, but no other abnormalities. Which of the following is the most likely
cause of her symptoms? Hernia hiatal
○ A 6-month-old boy comes to the office because of spitting up his meals for the past
five months. He is also often unsettled, with colicky behavior after feeding. Other
than this, his mother has no concerns. He is developing normally, and his growth
has been stabilized at the 90th percentile for his height and weight. Examination
shows a well-appearing child. Abdominal examination shows no abnormalities.
Which of the following is the most likely diagnosis? Gastroesophageal reflux
disease
○ A 49-year-old man comes to the office because of difficulty swallowing for three
months. He says food has been getting "stuck" when he tries to swallow it. This
sensation only happens with food, especially bread, and not with liquids. He has
tried many over-the-counter remedies, including a medication which decreases
stomach acid secretion. These treatments have had little effect. His medical history
includes persistent "heartburn" for the past five years. A barium swallow is obtained.
Which of the following is the most likely diagnosis? Esophageal stricture
○ A 63-year-old man comes to the office because of heartburn for the past four
months. He experiences a burning feeling in his chest whenever he eats. He has
also had difficulty swallowing his food. The patient has lost 6.8-kg (15-lb) over the
past four months. A chest radiograph is obtained and shown below. Which of the
following is the most likely diagnosis? Sliding hiatal hernia
○ A 60-year-old man comes to the emergency department because of sub-
○ sternal chest pain for an hour. He has experienced this pain before, typically after
heavy meals, during times of stress, and when he goes running in the park. He also
feels the pain at night when he is lying in bed and has previously been woken from
sleep by discomfort. He has not noticed any dyspnea, diaphoresis, or palpitations
but is currently experiencing some nausea and a sour taste in his mouth. Medical
history is non-contributory. ECG shows normal sinus rhythm. Which of the following
is the most likely diagnosis? Gastroesophageal reflux
○ A 25-year-old man comes to the emergency department because of acute onset
hoarseness of voice within the past 3 days. He also reports coughing, frequent
throat clearing, and flu-like symptoms. He reports having an upper respiratory
infection from babysitting his younger cousin 5 days ago. A laryngoscopy is done
and shows erythematous and edematous laryngeal tissue. Which of the following is
the next best step in management for this patient? Drinking fluids and resting the
voice
○ A 55-year-old man comes to the office because of severe diarrhea for the past three
days. He says his diarrhea developed suddenly, and there have been no associated
symptoms. Medical history includes an over-the-counter medication for heartburn,
and regurgitation for the past five days. He says these have been present for the
past six months, and occur daily but are not getting worse. Examination shows an
afebrile man with a large body habitus. Abdominal examination is non-contributory.
A barium study is obtained. Which of the following medications is the patient most
likely taking for his heartburn? Magnesium hydroxide antacid
○ A 61-year-old man comes to the clinic because of dysphagia and “heartburn,” which
he states have become more troublesome over the past 5 months. Examination
shows mild tenderness to palpation over his epigastrium. Upper gastrointestinal
barium swallow fluoroscopy demonstrates a subdiaphragmatic gastroesophageal
junction, but apparent herniation of the gastric fundus superiorly and into the left
hemithorax. Which of the following is the most appropriate management of this
patient’s underlying condition? Surgical gastropexy
○ A 58-year-old woman comes to the office because of heartburn for two weeks. The
pain most often appears as she lies down to sleep. She also frequently wakes up
with burning in her throat and epigastric area. She sometimes has an unpleasant
bile taste in her mouth. She states that she has recently been having difficulty
swallowing in addition to her usual symptoms. She has hypertension and
hypothyroidism. Her current medications include Lopressor 50 mg daily, and
Synthroid daily. Her temperature is 36.8°C (98°F), pulse is 87/min, respirations are
15/min, and blood pressure is 134/86 mm Hg. Which of the following is the most
appropriate next step? Endoscopy
○ A 5-month-old infant is brought to the office because of a cough and vomiting for a
month. The vomiting is worse following meals, and the infant is exclusively formula
fed. The mother says these symptoms have been progressively worsening. She
says there has been no projectile or bilious vomiting, fever, sputum production, or
lethargy. She states she keeps the child upright while feeding and has tried to
switching formulas without improvement. Physical examination shows the infant is in
the 12th percentile for height and weight. The remainder of the examination shows
no abnormalities. Which of the following is the most appropriate initial management?
Formula thickening
○ A 55-year-old Caucasian man comes to the office because of progressively
worsening gastroesophageal reflux for the past nine months. His medical history
includes severe gastroesophageal reflux disease that was unresponsive to proton
pump inhibitors. Nissan fundoplication was performed a year ago and, initially
controlled his symptoms adequately. His current symptoms are worse than they
were before the operation. Upper gastrointestinal endoscopy shows high grade
dysplasia of the distal esophagus. Which of the following is the most appropriate
next step in management? Esophagectomy
○ A 65-year-old man comes to the office because of dull chest pain, dyspnea, and
heart palpitations. He occasionally feels his food "balling up" in his throat, causing
significant discomfort in his chest. He denies exacerbation of his chest discomfort
during exercise. He has tried taking calcium carbonate tablets to ease his symptoms
with some success. His chest X-ray is shown below. Which of the following is the
most appropriate surgical management of the patient's underlying condition? Nissen
fundoplication
○ A 47-year-old Japanese man comes to the clinic because of a 6-month history of
early satiety and mild epigastric pain, as well as an unintentional weight loss of
6.8-kg (15-lb) over the past year. He is examined by endoscopy, and biopsy shows
gastric carcinoma. A CT with contrast is obtained and shows an area that is
concerning for metastasis to the liver. Which of the following is the most appropriate
treatment? Surgical resection of tumors and adjacent
lymph nodes
○ A 55-year-old man comes to the office because of severe Diarrhea for
the past three days. He says his diarrhea developed suddenly, and
there have been no associated symptoms. Medical history includes an
over-the-counter medication for heartburn, and regurgitation for the
past five days. He says these have been present for the past six
months, and occur daily but are not getting worse. Examination shows
an afebrile man with a large body habitus. Abdominal examination is
non-contributory. A barium study is obtained. Which of the following
medications is the patient most likely taking for his heartburn?
Magnesium hydroxide antacid

● PLAYLIST 3
○ A 62-year-old man comes to the office because of painless rectal bleeding for the
past 3 months. He describes the intermittent occurrence of streaks of “bright red
blood” on the toilet paper after wiping, and blood on but not mixed within the stool.
Occasionally he has noted a small volume of blood within the toilet bowl, and
associates this with straining. For the past two weeks he has noticed an
“uncomfortable lump” in his anus when defecating, which goes away by itself
immediately afterwards. He says he has no abdominal pain, weight loss, or fevers.
He is a well-appearing man with obesity. Digital rectal examination shows bright red
blood on the examination glove following the procedure. Anoscopy shows enlarged
blood vessels above the pectinate line. Which of the following is the most
appropriate next-step in management? Rubber band ligation
○ A 65-year-old man comes to the emergency department for worsening abdominal
pain and intermittent black, tarry stools for the past three months. His medical
history is significant for chronic hepatitis B infection. Physical examination shows a
frail gentleman with decreased muscle mass, a protuberant abdomen, and icteric
sclera. Abdominal ultrasound shows enlarged lymph nodes in the hepatoduodenal
ligament. Which of the following structures is most likely to be compressed by these
lymph nodes? Portal traid
○ A 42-year-old woman who was hit by a car while riding her bike presents to the
emergency department with a pulse of 100 and a BP of 90/75. She is complaining of
pain in the left shoulder and wrist, with the latter showing an obvious deformity. A
FAST scan is positive for free fluid in the abdomen. Which of the following organs is
most likely injured? Spleen
○ A 52-year-old man is brought to the emergency department after complaining of
excruciating chest pain that radiates to the back. He also reports shortness of breath
and pain with swallowing. The symptoms began 30 minutes ago, after the patient
consumed a large quantity of alcohol and had several episodes of forceful vomiting.
Past medical history is notable for hypertension, alcohol use disorder, and liver
cirrhosis. His temperature is 37.2°C (99.0°F), blood pressure is 142/78 mmHg, and
pulse is 75/min. Physical exam shows crepitus on palpation of the upper chest wall.
Abdominal exam is notable for hepatosplenomegaly. Which of the following is the
next best step in the management of this patient? Perform contrast esophagram
○ A 2-year-old girl is brought to the clinic by her mother because of Skin spots. She
says the spots are patches of violaceous, compressible nodules on the girl's Lips
arms and chest as shown in the image. The mother says that her husband has
similar lesions and has been hospitalized several times. In addition to the skin,
which of the following is most commonly affected? Small bowel
○ A 45-year-old man is brought to the emergency room after an episode of
hematemesis. The patient was watching television when he felt nauseated and
produced bright-red vomitus. Past medical history is notable for an ankle sprain, for
which he is currently taking aspirin, and Helicobacter pylori infection, which was
successfully treated 7 months ago. The patient consumes 4-6 beers per week and
does not use tobacco products. His vitals are within normal limits. Endoscopy is
performed and examination of the stomach reveals a protruding 2.5-mm arteriole
that has eroded the overlying mucosa. There is active spurting of blood from the
vessel. Which of the following is the next best step in the management of this
patient? THERMAL COAGULATION OF BLEEDING SITE
○ A 55-year old woman comes to the emergency department because of light
headedness, fatigue, and a racing heart for 6 hours. She tells you that she has also
had very dark stools in the last day. Her medical history includes hyperlipidemia, and
daily cigarette smoking. Her temperature is 36.8°C (98°F), pulse is 110/min,
respirations are 23/min, and blood pressure is 90/65 mm Hg. Abdominal
examination is noncontributory. A biopsy from upper gastrointestinal endoscopy
shows partial replacement of the gastric mucosal epithelium by intestinal metaplasia
in the antrum. Which of the following most likely explains this patient’s problem?
Curvilinear Gram-negative rod
○ A 70-year-old woman comes to the emergency department because of blood in her
stool. She says that her stool has been black and tarry, but she can occasionally see
red streaking. She reports no recent weight loss or abdominal pain. Her past
medical history is significant for Hepatitis C infection. Stool guaiac cards are
positive. Endoscopy shows long, red streaky areas on the stomach. Which of the
following is the most likely diagnosis? Gastric antral vascular ectasia

● PLAYLIST 4
○ A 55-year-old homeless woman comes to the emergency department because of
acute confusion. She is jaundiced, and her breath smells of alcohol. Shortly after
arriving she falls into a stupor and becomes unresponsive. She is stabilized and the
following laboratory values are obtained:

Alanine aminotransferase (ALT): 45 U/L


Aspartate aminotransferase (AST): 83 U/L
Total bilirubin: 7 mg/dL
Direct bilirubin: 2 mg/dL
Prothrombin time (PT): 16 second
Which of the following is the most likely cause of the patient's increased PT?
Increased fibrinolysis

○ A 55-year-old man comes to the emergency department because of increasing right


upper quadrant pain and jaundice. Physical examination shows mild scleral icterus,
a 12-cm (4.72-in) liver span and vague right upper quadrant pain on palpation.
Laboratory studies show:

AST: 454 U/L, ALT: 206 U/L, Alkaline phosphatase: 126 U/L, γ-glutamyl
transpeptidase: 174 IU/L, Total bilirubin: 2.5 mg/dL, Direct bilirubin:1.8 mg/dL
Which of the following is the most likely diagnosis? Alcoholic liver disease

○ A 65-year-old man with a history of alcoholic cirrhosis and ascites comes to the
emergency department because of seven days of worsening confusion and impaired
memory. For the last two days, he has been increasingly agitated at night and
spends a large portion of the daytime asleep. He denies diarrhea, melena,
hematemesis, impaired balance, and double vision. He takes no medications. His
temperature is 38.2°C (100.8°F), pulse is 90/min, respirations are 16/min, and blood
pressure is 130/74 mm Hg. Abdominal examination shows a tender, distended
abdomen with shifting dullness. The rest of the examination and a computed
tomography scan of the head show no other abnormalities. Urinalysis and fecal
occult blood test show no abnormalities. Serum laboratory results are shown below:

Ethanol: 0.0 mg/dL, Leukocytes: 12,500/mm3, Total bilirubin: 0.9 mg/dL, AST: 75
U/L, ALT: 62 U/L, Phosphatase (alkaline): 87 U/L
Which of the following mechanisms is the most likely cause for this patient's
suspected acute illness? Translocation of bacteria through intestinal wall

○ A 52-year-old woman comes to the emergency department because of an episode


of melena. Physical examination shows a distended abdomen with shifting dullness
and a fluid wave. An esophagogastroduodenoscopy shows bleeding from
esophageal varices. The patient is diagnosed with decompensated cirrhosis and
placed on the liver transplant waiting list. Which of the following laboratory value
combinations are most appropriate in determining her priority on the waiting list?
Serum bilirubin, serum creatinine, and international normalized ratio
○ A 51-year-old man, with a 20 year history of alcoholism, comes to the emergency
department because of a loss of appetite, abdominal pain, and fever for the past 24
hours. He says that he had consumed twelve beers and a bottle of vodka two days
ago. Laboratory studies are obtained, and results show the following:

Sodium: 137 mEq/L, Potassium: 3.4 mEg/L, Alanine aminotransferase: 230 U/L
, Aspartate aminotransferase: 450 U/L
Which of the following is most likely associated with the current presentation?
Cytoplasmatic inclusión bodies with keratin

○ A 50-year-old alcoholic man comes to the emergency department because of an


episode of hematemesis. The patient looks disheveled and is disoriented to time
and place. Past medical history includes hepatitis C infection. Abdominal
examination shows abdominal distension with a fluid wave and caput medusae.
Examination of the extremities shows a bilateral “flapping” tremor, red palms, and
bilateral 2+ lower extremity edema. Which of the following is the most likely cause of
the patient's palmar erythema? Hyperestrinism
○ A 45-year-old woman comes to the emergency department because of dysfunctional
uterine bleeding. A hysterectomy is performed without complication. The morning
after her hysterectomy, she complains of nausea, headache, and tremulousness.
During examination, she suddenly loses consciousness and experiences a brief
tonic-clonic seizure that stops without intervention. The patient has never
experienced symptoms like these before. She has no history of seizures. Which of
the following examination findings is most likely to be observed in this patient?
Cirrhosis
○ A 65-year-old man with a history of cirrhosis and hepatocellular carcinoma (HCC)
comes to the emergency department because of 3 weeks of mild but increasing
abdominal pain. He has a history of ascites refractory to salt restriction and diuretic
therapy. He endorses intermittent chills, but no nausea or diarrhea. The patient
denies constipation and black stools, but endorses a notable decrease in how much
urine he has produced in the past week. Physical exam shows the patient’s
temperature is 37.8°C (100°F), with otherwise normal vital signs. Examination of his
abdomen shows shifting dullness by percussion. An abdominal ultrasound shows
ascites and no signs of hydronephrosis. Laboratory studies are shown below

Serum, WBC 7,000/μL, Creatinine 3.56 mg/dL, today 1.50 mg/dL,


at an outpatient provider visit 1 month ago
Urinalysis Negative for nitrites and leukocyte esterase 0-2 RBCs per high
powered field 0-1 WBC per high powered fieldNo evidence of casts or proteinuria
Which of the following is the most likely cause for this patient’s increased
creatinine? Hepatorenal syndrome

○ A 35-year-old woman comes to the emergency department because of nausea,


malaise, and a low-grade fever. She has a history of significant alcohol abuse and
still continues to drink. She came to the emergency department with an episode of
melena a month ago and was found to have esophageal varices and anemia.
Examination shows right upper quadrant tenderness and hepatomegaly. Which of
the following laboratory results are most consistent with the patient's symptoms?
Aspartate aminotransferase of 120 U/L and alanine aminotransferase of 60 U/L

● PLAYLIST 5
○ A 65-year-old man who immigrated from China to the United States 15 years ago
comes to the office because of increased abdominal girth, weight loss, and fever.
Physical examination shows for right upper quadrant tenderness on palpation and
organomegaly. A tissue biopsy is obtained and tests positive for a primary
malignancy. Which of the following tumor markers is most likely to be elevated in this
patient?α-fetoprotein (AFP)
○ A 60-year-old rock musician comes to the office because he has been feeling
increasingly tired for 6 months. He has a history of intravenous drug use and alcohol
abuse. He states that he feels quite tired, but otherwise has no complaints.
Examination is noncontributory. His laboratory investigations are normal aside from
elevated liver enzymes. Which of the following additional tests should you order
first? Hepatitis C virus RNA
○ A 13-year-old girl comes to the clinic with complaints of nausea, vomiting, diarrhea
and malaise. She has diffuse abdominal discomfort and hasn't eaten much in the
past few days. Her mother reports that she was eating normally only a few days
ago. Physical examination shows a temperature of 38.8oC (101.8oF). Laboratory
tests show a normal complete blood count, low bicarbonate, hypokalemia
hypernatremia and elevated aminotransferases. She appears pale and fatigued but
is not jaundiced What treatment should be started? IV fluids and supportive
therapy only
○ A 35-year-old woman comes to the emergency department because of fever,
abdominal pain, diarrhea, and weight loss for the past 3 weeks. She is a recent
immigrant from Mexico. She denies nausea, vomiting, or chills. Physical
examination shows severe right upper quadrant tenderness to palpation and bloody
diarrhea. Laboratory analysis reveals leukocytosis, elevated liver enzymes, and
elevated alkaline phosphatase. A CT is obtained and shows a hypodense avascular
liver mass with well-defined margins. Which of the following is the most likely
diagnosis in this patient? Entamoeba histolytica
○ A 19-year-old woman, gravida 2, para 1, presents to your clinic to follow up on her
first trimester laboratory results. She is now at 20 weeks of gestation and has had
an unremarkable pregnancy thus far. Her laboratory workup was negative for HIV,
chlamydia, gonorrhea, and syphilis. Her hepatitis panel demonstrated the following
results: HBsAg: positive, Anti-HBsAb:negative, Anti-HBcAb:negative. A hepatic
function panel drawn in clinic demonstrates normal AST and ALT values. What is the
next best step in the management of this patient? Obtain HBV DNA- anti-HBeAb-,
and HBeAg levels
○ A 25-year-old man comes to the emergency department because of fever, jaundice,
and pain localized to the right upper quadrant. Endoscopic retrograde
cholangiopancreatography shows alternating strictures and dilation of the bile ducts
with a characteristic "beading" appearance with the involvement of both intrahepatic
and extrahepatic bile ducts. Which of the following is the most likely diagnosis?
Primary sclerosing cholangitis
○ A 60-year-old man who grew up in rural Australia before moving to the east coast of
the United States last year comes to the office with gradual onset of jaundice. He
reports abdominal pain, particularly in the right upper quadrant area. Physical
examination shows hepatomegaly. Abdominal X-ray shows a single, large calcified
cyst in the liver that is partially obstructing the common bile duct. A PET scan is
negative. Based on the clinical presentation, which of the following treatments is
most likely indicated? Aspiration with ultrasound guidance
■ WBC: 10,000 cells/mm3
■ Hgb: 12.5g/dL
■ Hct: 36%
■ Platelets: 150,000/mm3
■ AST: 1000 U/L
■ ALT: 1400 U/L
■ Alkaline phosphatase: 150 U/L
■ Total bilirubin: 1.5 mg/dL
■ HBsAg: Positive
■ HBeAg: Positive
■ anti-HBsAb: Negative
■ Which of the following is the most likely diagnosis? Circulating
cryoglobulins
○ A 46-year-old woman comes to the clinic because of abdominal pain, unintentional
weight loss, and low-grade fever. The abdominal pain is dull and located in the right
upper quadrant, and she has lost 4.35-kg (10-lb) in the last three months. The
patient states she has intermittent yellowing of skin, dark urine, and clay-colored
stool. Examination shows mild hepatomegaly and jaundice. Right upper quadrant
ultrasound shows ductal dilatation and a mass along the lower margin of the liver.
Which of the following is the most appropriate next step in management? Conduct
CT abdomen and pelvis with contrast
○ A 26-year-old woman comes to emergency department with right upper quadrant
pain and fever. For the past two weeks, after returning from a trip to India, she has
been exhausted and "itchy all over". On physical exam, she has marked jaundice
with tenderness to palpation in the epigastric region and right upper quadrant.
Laboratory results are remarkable for elevated aminotransferases and bilirubin. If a
liver biopsy were performed, what would the acidophilic bodies seen on biopsy
represent? Hepatocyte apoptosis
○ A 62-year-old man is admitted to the hospital because he's been having fatigue, joint
pain, abdominal swelling, and bloating for 2 weeks. He has a history of hypertension
and was recently diagnosed with type 2 diabetes mellitus. He hasn't any recent
illnesses, fever, or chills. On exam, his skin is tan and cracked, and he has palmar
erythema. He also has scleral icterus and prominent abdominal distention with a
fluid wave. During his hospital stay, a liver biopsy is performed and Prussian blue
stain is applied to the slide (shown below). Given these results, which of the
following is the most appropriate treatment option? Begin serial phlebotomy
○ A 15-year-old boy comes to office with complaints of nausea, vomiting, diarrhea and
malaise. He has diffuse abdominal discomfort and has not eaten much in the past
few days. Physical examination reveals a temperature of 39o C (101.8o F).
Laboratory findings show a normal complete blood count, low bicarbonate levels,
hypokalemia, hypernatremia and elevated aminotransferases. He appears pale and
fatigued but is not jaundiced. Which of the following is most likely to reveal the
source of his symptoms? Hepatitis A IgG and IgM serum titers
○ A 15-year-old girl comes to the office for her annual visit. Physical examination
shows mild jaundice and scleral icterus. She says that she recently began taking
oral contraceptives. Laboratory studies show conjugated hyperbilirubinemia. Her
AST is 15 U/L and her ALT is 18 U/L. Upon oral cholecystography, the biliary system
cannot be visualized, even with double contrast. Which of the following is the most
likely diagnosis leading to these findings? Dubin johnson syndrome
○ A 33 year-old female comes to the office because of increasing fatigue and joint
pain for the past 3 months. She has also noticed a decreased appetite and a 4-kg
(8.8-lb) weight loss. She does not complain of abdominal pain, neurologic
symptoms, and she has not noticed any new rashes. She has always had irregular
periods, but her last “regular” period was 8 weeks ago. Her medical history is
noncontributory and she takes no medications. She has been sexually active with
her boyfriend for the past 4 months. Her temperature is 37.8°C (100°F), pulse is
77/min, and blood pressure is 132/88 mm Hg. Examination shows scleral icterus,
spider telangiectasias on the trunk, and mild hepatosplenomegaly. Urine pregnancy
test is negative. Laboratory studies show:*


■ ***Note: IgG value should read 46g/L
■ Which of the following is the most appropriate test to confirm the
diagnosis? Liver biopsy
○ A 33-year-old woman, gravida 2, para 1, comes to the office to follow up on her first
trimester laboratory results. She is now at 20 weeks of gestation and reports no
issues with her pregnancy. She reports that her morning sickness has largely
resolved and that her mood has been stable. Her laboratory workup was largely
unremarkable, except for her hepatitis panel, which returned the following results:
HBsAg: negative, HBeAg: negative, Anti-HBsAb:negative, Anti-HBcAb:negative.
When asked about her sexual history, the patient reports that she only has sexual
intercourse with her husband and has not been using any form of contraception
since becoming pregnant. Which of the following is the next recommended step in
the management of this patient? No intervention is necessary at this point
○ A 30-year-old woman comes to the clinic because of fever, joint pain, and a rash on
her lower extremities. She admits to intravenous drug use. Examination of her lower
extremities reveals palpable petechiae and purpura. Laboratory testing reveals a
negative antinuclear antibody, positive rheumatoid factor, and positive serum
cryoglobulins. Which of the following underlying conditions in this patient is
responsible for these findings? Hepatitis C infection
○ A 50-year-old man, who is currently homeless, is brought to the emergency
department because he was found unconscious. He appears intoxicated and unable
to provide a detailed history. Physical examination shows that he is febrile with a
temperature of 38.3º (101ºF) and appears malnourished. He has erythematous
macules and purpuric papules on his buttocks and lower extremities. He complains
of pain in numerous joints and joint tenderness upon palpation. His laboratory
workup is as follows:
WBC: 10,000 cells/mm3
Hgb: 12.5g/dL
Hct: 36%
Platelets: 150,000/mm3
AST: 1000 U/L
ALT: 1400 U/L
Alkaline phosphatase: 150 U/L
Total bilirubin: 1.5 mg/dL
HBsAg: Positive
HBeAg: Positive
anti-HBsAb: Negative
Which of the following is the most likely diagnosis? Circulating cryoglobulins

● PLAYLIST 6
○ A 45-year-old woman comes to the emergency department because of the sudden
onset of abdominal pain during the past hour. She reports occasional episodes of
abdominal pain that radiates to the right shoulder, especially after eating greasy
meals; however, her usual episodes are milder and shorter. She does not drink
alcohol or use illicit drugs. Her temperature is 39.4°C (103°F). Physical examination
shows scleral icterus. Which of the following is the most likely diagnosis?
Cholangitis
○ A 72-year-old woman comes to the emergency department because of intermittent
abdominal pain associated with persistent nausea and vomiting for the last five
days. She had an episode of vomiting with hematemesis six hours ago. Her past
medical history is significant for diabetes mellitus and hypertension. Her current
medications are metformin and lisinopril. Her temperature is 38.2°C (100.8°F), pulse
is 118/min, respirations are 22/min, and blood pressure is 110/60 mm Hg. Physical
examination shows abdominal distention and tenderness as well as increased bowel
sounds. Laboratory studies show:
Leukocyte count: 23,250/mm3
Hemoglobin: 12 g/dL
Hematocrit: 46%
Platelets: 400,000/µL
Total bilirubin, 0.8 mg/dL
Lipase: 140 U/L
AST, SGOT: 47 U/L
ALT, SGPT: 45 U/L
In addition to signs of partial or complete intestinal obstruction, which of the
following is a radiologic finding that might be present in this patient? Air in the
biliary tree on plain abdominal radiograph.
○ A 43-year-old woman comes to her primary care physician because of right upper
quadrant pain. The pain occurs only after eating a fatty meal. She denies any sick
contacts, fever, and chills. Her temperature is 37.6°C (99.5°F), pulse is 92/min,
respirations are 18/min, and blood pressure is 125/80 mm Hg. AST and ALT
concentrations are within normal limits. The right upper quadrant ultrasound of her
abdomen is inconclusive because of her body habitus and a follow-up CT scan of
the abdomen is ordered, the results of which are shown below. Which of the
following is the most likely diagnosis? Cholelithiasis
○ A 6-week-old infant boy, born in the United States, comes to the clinic because of
slow growth, weight gain, foul-smelling stools, and dark urine. His symptoms began
four weeks ago, and have continued to worsen. Physical examination shows an
enlarged spleen and jaundiced skin. Serum AST, ALT, and total bilirubin
concentrations are elevated. Abdominal ultrasound shows a triangular cord sign. A
HIDA scan shows obstruction of flow from the liver into the gallbladder and small
intestine. Which of the following is the most appropriate next step in management?
Hepatoportoenterostomy
○ A 64-year-old man comes to the outpatient clinic because of abdominal pain. He
reports that for the last few months, he has had postprandial pain that is made
worse by spicy foods. He states that the pain is oftentimes located in the right upper
portion of his abdomen and feels like it's traveling to his shoulder blade. These
episodes are sporadic and unpredictable. He denies any fevers. Physical
examination shows no abnormalities. Abdominal ultrasound is shown below. Which
of the following is the best treatment for this condition? Cholecystectomy
○ A 53-year-old man comes into the emergency department because of a sudden
onset of excruciating epigastric pain radiating to his upper back. The pain was
post-prandial and associated with multiple episodes of emesis without relief. The
vomit consists of digested food and a green mucous-like substance. Fever and
acute abdominal pain began as soon as he stopped vomiting. The patient has
familiar hypertriglyceridemia and chronic cholelithiasis. His temperature is 38.8°C
(101.8°F), pulse is 115/min, respirations are 16/min, and blood pressure is 145/85
mm Hg. Examination shows abdominal guarding and severe upper epigastric
tenderness radiating to his upper back. Cullen sign and Grey Turner sign are
absent. Which of the following diagnostic tests is the most appropriate next step in
management? Abdominal ultrasonography
○ A 43-year-old woman comes to the emergency department because of a 1-day
history of severe nausea and projectile vomiting and right upper quadrant pain. She
has had intermittent pain in the same area for the past two months, but her
symptoms have never been this severe before. Physical examination demonstrates
a positive Murphy’s sign. Ultrasonography of the abdominal region reveals multiple
gallstones. A laparoscopic cholecystectomy is recommended.Which of the following
structures is most likely to be found in the area bounded by the common hepatic
duct medially, cystic duct inferiorly, and lower edge of the liver superiorly? Cystic
artery
○ A 38-year-old man comes to the outpatient clinic because of severe right upper
quadrant pain and fever. His temperature is 38.8°C (101.8°F), pulse is 80/min,
respirations are 16/min, and blood pressure is 129/79 mm Hg. His examination
reveals a positive Murphy sign. Blood tests show elevated concentration of
leukocytes. Right upper quadrant ultrasound of the abdomen shows gallbladder wall
thickening of 6 mm. While the ultrasound is pressing against the right upper
quadrant, he complains of pain and guards his abdomen. Which of the following is
the most appropriate next step in management? Early cholecystectomy
○ A 42-year-old woman comes to the emergency department because of abdominal
pain, nausea, and vomiting. The pain is cramp-like, located in the right upper
quadrant, and radiates to her right shoulder. It is postprandial particularly after a fatty
meal. She has also experienced several episodes of non-bloody, non-bilious
vomiting. She endorses three previous episodes of similar pain in the past three
months, but states that her prior episodes lasted less than four hours. Her
temperature is 37.0°C (98.6°F), pulse is 85/min, respirations are 18/min, and blood
pressure is 128/80 mm Hg. Abdominal examination shows right upper quadrant
tenderness with palpation, and no guarding or rebound. An abdominal ultrasound is
performed, but it is difficult to visualize the gallbladder due to her body habitus.
Which of the following is the most appropriate next step step in management?
Cholescintigraphy (HIDA scan)
○ A 6-week-old infant born in the United States comes to the clinic because of slow
growth, weight gain, foul-smelling stools, and dark urine. His symptoms began four
weeks ago and have continued to worsen. He has an enlarged spleen and is
jaundiced. Serum AST, ALT, and total bilirubin concentrations are elevated.
Abdominal ultrasound shows a triangular cord sign. A HIDA scan shows obstruction
of flow from the liver into the gallbladder and small intestine. Which of the following
is the most likely diagnosis? Biliary atresia
○ A 42-year-old woman comes to the emergency department because of abdominal
pain. The pain began five hours ago, is postprandial, steadily increasing in intensity,
and associated with nausea and vomiting. Her temperature is 37.0°C (98.6°F), pulse
is 80/min, respirations are 16/min, and blood pressure is 125/85 mm Hg.
Examination shows an overweight middle-aged woman in mild distress. She has no
signs of jaundice. Her abdomen is exquisitely tender in the right upper quadrant.
Ultrasound shows several gallstones, an edematous gallbladder wall, and a positive
sonographic Murphy sign. For which of the following malignancies is this patient
most likely at an increased risk? Carcinoma of the gallbladder
○ A 35-year-old woman comes to the emergency department because of
progressively worsening right upper abdominal pain for two days. The patients
states that for the past 3 months she's had a similar pain 1-2 hours after meals, but
now the pain is more severe and constant. She admits to fever, dark-colored urine,
and clay-colored stools. She denies pain radiation, nausea, or vomiting. Physical
examination shows a middle aged female in moderate distress with mild jaundice.
Abdominal examination shows tenderness to palpation in the upper right quadrant.
A pregnancy test is negative. A complete blood count shows leukocytosis and
elevated alkaline phosphatase. Blood culture is positive for Escherichia coli. Based
on the patient's presentation, which of the following is the most likely diagnosis?
Ascending cholangitis
○ A 35-year-old woman comes to the emergency department because of episodic
abdominal pain. The pain is located in the right upper quadrant, lasts for about
30-minutes per episode, is usually postprandial, especially after fatty meals, and is
sometimes associated with nausea and vomiting. The patient does not have a family
history of hypertriglyceridemia and does not drink alcohol. Which of the following
findings is expected on abdominal examination and initial laboratory work? Physical
examination and laboratory findings without abnormalities
○ A 49-year-old man comes to the outpatient clinic because of right upper quadrant
pain, nausea, and occasional vomiting. His temperature is 37.0°C (98.6°F), pulse is
76/min, respirations are 16/min, and blood pressure is 125/75 mm Hg. Examination
shows right upper quadrant tenderness and jaundice. Laboratory tests show
elevated bilirubin and alkaline phosphatase, but WBC is normal. Pancreatic enzyme
levels are normal. Trans-abdominal ultrasound shows a stone blocking and dilating
the common bile duct. Which of the following is the most appropriate next step in
management? Perform an endoscopic retrograde cholangiopancreatography
(ERCP)
○ A 42-year-old woman comes to the emergency department because of postprandial
right upper quadrant pain (RUQ) radiating to the back, nausea, jaundice, and
vomiting over the past 12 hours. Her temperature is 38.8°C (101.8°F), pulse is
85/min, respirations are 15/min, and blood pressure is 125/75 mm Hg. Abdominal
examination shows non-distended abdomen with right upper quadrant tenderness
upon palpation. The liver, rectal, and pelvic examinations are normal. Ultrasound of
the right upper quadrant shows gallbladder wall thickening and biliary dilation.
Laboratory studies show:

Leukocyte count: 15,000/mm3


Direct bilirubin: 4.0 mg/dL
Alkaline phosphatase: 385 U/L
AST: 220 U/L
ALT: 240 U/L
Which of the following is the most appropriate next step in management? Admit
patient to the hospital, provide Intravenous hydration, begin antibiotic therapy,
and recommend ERCP

○ A 40-year-old woman comes into the emergency department because of increasing


fatigue over the past two weeks. She reports feeling ill a few weeks ago with fever,
chills, and abdominal pain. Her temperature is 37.9°C (100.2°F), pulse is 75/min,
respirations are 15/min, and blood pressure is 120/85 mm Hg. Physical exam shows
no abnormalities. Serum aminotransferase concentration is 290 U/L, and serum
alkaline phosphatase is 100 U/L. Cholangiography shows multifocal stricturing and
dilation of both the intrahepatic and extrahepatic bile ducts. Which of the following
may also be associated? Night blindness
○ A 75-year-old woman comes to the emergency department because of abdominal
pain and distention. The abdominal pain is colicky, and associated with nausea and
vomiting. For three days she has not been able to pass flatus or stool. Her
temperature is 37.0oC (98.6oF), pulse is 99/min, respirations are 15/min, and blood
pressure is 125/75 mm Hg. Abdominal examination shows high-pitched bowel
sounds. Abdomen CT is done. Which of the following is the most likely diagnosis?
Gallstone ileus
○ A 56-year-old man comes to the emergency department because of severe
abdominal pain for 3 hours. He states that he has never had anything like this before
and that he has vomited 4 times. He thinks his symptoms may be due to consuming
some eggs that were past their expiry day. His temperature is 36.8°C (98°F), pulse
is 87/min, respirations are 18/min, and blood pressure is 117/78 mm Hg.
Examination shows that his abdomen is soft and non-tender. A CT scan is obtained,
and a radiologist reports an incidental finding, but no acute intra-abdominal
pathologies. Based on the incidental finding shown, which of the following
malignancies is this patient most likely to develop? Gallbladder adenocarcinoma
○ A 40-year-old woman comes to the emergency department because of a 12-hour
history of severe abdominal pain in the right upper quadrant. In the past she has
experienced intermittent pain in the same location that lasted up to one hour, but it
has always been transient. She also notes vomiting her meal last night and has had
anorexia for one day. Her temperature is 37.8°C (100.0°F), pulse is 95/min,
respirations are 18/min, blood pressure is 120/80 mm Hg, and her oxygen saturation
is 97% on room air. She has tenderness and guarding in the right upper quadrant.
Which of the following is the most appropriate next diagnostic step? Sonography
○ An 85-year-old woman comes to the emergency department because of colicky
abdominal pain and constipation. She says she has not been able to pass gas or
stool for the past two days, and that her abdomen has expanded in the last few
days. Her temperature is 37.0°C (98.6°F), pulse is 85/min, respirations are 16/min,
and blood pressure is 130/75 mm Hg. Abdominal examination shows high-pitched
bowel sounds, distention, but no guarding or rebound. Her BMI is 45 kg/m2. CT
abdomen scan results are obtained and shown below. Which of the following
findings is being highlighted in this image? Rigler triad
○ A 42-year-old woman comes to the emergency department because of recurrent
abdominal pain that has persisted over the past 3 weeks. The pain is dull,
postprandial, and located in right upper quadrant. She denies nausea and vomiting.
Her temperature is 37.5°F (99.5°F), pulse is 75/min, respirations are 16/min, and
blood pressure is 125/75 mm Hg. Examination shows that her abdomen is
non-distended with minimal tenderness. The liver, rectal, and pelvic examinations
are normal. Laboratory studies show:

Leukocyte count: 13,000/mm3


Total bilirubin: 0.8 mg/dL
Direct bilirubin: 0.6 mg/dL
Alkaline phosphatase: 52 U/L
AST: 18 U/L
ALT: 15 U/L
Serum lipase and amylase concentrations are normal. Which of the following
anatomical landmarks, if obstructed, is most likely involved in this patient's disease
process? Cystic duct

○ fObstruction of the cystic duct

● PLAYLIST 7
○ A 40-year-old man comes to the emergency department because of increasing
upper abdominal pain that began two hours ago. He states the pain has been
progressive, with associated nausea and several episodes of vomiting. Physical
examination shows his temperature is 37.8ºC (100.2˚F), pulse is 105/min,
respirations are 18/min, and blood pressure is 115/75 mmHg. He appears very pale
and his abdomen is extremely tender to palpation. Laboratory studies show:
Serum amylase: 725 U/L
Serum lipase: 890 U/L
Alanine aminotransferase: 175 U/L
Aspartate aminotransferase: 120 U/L
Which of the following is the most likely underlying cause of this patient's condition?
Gallstone disease
○ A 45-year-old man comes to the emergency department because of severe upper
abdominal pain that began suddenly and has intensified over the past six hours. The
patient says that the pain is constant and scores it as a 9 on a 10-point scale. He
complains of nausea but has not vomited. The patient says that the pain radiates to
his back and is somewhat relieved by sitting up and bending forward. He has a
history of gallstones. Physical examination shows epigastric tenderness and
reduced bowel sounds. Which of the following is the most appropriate next step for
this patient? Aggressive fluid resuscitation, nil orally, and analgesia
○ A 65-year-old woman comes to the emergency department because of
mid-epigastric pain that radiates to the back. Despite attempts to alleviate the pain, it
has persisted for the past 8 hours. She has not had any melena. She has a history
of hyperparathyroidism. Examination shows temperature is 38°C (100.4°F), pulse is
102/min, and blood pressure is 100/70 mm Hg. Her abdomen is mildly distended,
tender to palpation, with no guarding. Complete blood cell count shows no
abnormalities. Serum lipase concentration is elevated. Which of the following is the
greatest risk factor for the likely diagnosis? Hypercalcemia
○ A 22-year-old woman comes to the clinic because of several days of worsening
abdominal pain. The patient has had chronic pain for about a year, but it has
worsened acutely in the past three days after contracting the flu. The pain is
localized in the epigastric region and radiates to the back. Her stools are oily and
float in the toilet. She has also been thirstier than usual and has been urinating more
frequently. Her fasting blood glucose concentration is significantly elevated, but
aminotransferase and alkaline phosphatase concentrations are within normal limits.
She drinks alcohol only on social occasions. Family history includes a mother with
diabetes and a brother with chronic abdominal pain. The patient is sent for an
abdominal CT scan, the results of which are obtained and shown below. Which of
the following is the most likely cause of this patient's symptoms? Mutation in
PRSS-1 gene
○ A 53-year-old-man comes to the office for follow-up after being hospitalized for his
first episode of acute alcohol-induced pancreatitis last month. He feels well and
denies any abdominal pain, nausea, anorexia, or diarrhea. His medical history is
significant for heavy chronic alcohol use, and he does not take any medications. His
immediate family members are all healthy. He smokes one pack of cigarettes per
day. His vitals are within normal limits. Laboratory results show serum electrolytes,
cholesterol and triglyceride levels are within normal limits. In addition to limiting
alcohol intake, which of the following is most likely to reduce this patient’s risk of
developing chronic pancreatitis? Smoking cessation
○ A 40-year-old man comes to the emergency department because of increasing
upper abdominal pain which began four days ago. He says the pain is constant, with
associated nausea and several episodes of vomiting. His temperature is 37.9ºC
(100.2˚F), pulse is 105/min, respirations are 18/min, and blood pressure is 115/75
mmHg. The patient appears very pale and his abdomen is extremely tender to
palpation. Laboratory studies show:
Serum amylase: 726 IU/L
Serum lipase: 890 U/L
Alanine aminotransferase: 178 U/L
Aspartate aminotransferase: 55 U/L
Which of the following is the most likely diagnosis? Acute pancreatitis
○ A 22-year-old previously healthy man comes to the emergency department because
of severe epigastric pain that began after returning to his hotel room after a dance
party. The pain is constant, radiates to his back, and he has vomited twice since the
onset of his symptoms. He consumes alcohol on a daily basis and attributes this to
his job as a disc jockey. His temperature is 38.6°C (101.5°F), pulse is 115/min,
respirations are 22/min, and blood pressure is 120/80 mm Hg. His abdomen is
tender to palpation over the epigastric area. Laboratory studies show:
Leukocyte count: 19,000/mm3
Glucose concentration: 160 mg/dL
Serum amylase: 6900 IU/L
Lipase: 350 IU/L
Which of the following findings most likely increases the risk of mortality for this
patient? The patient's leukocyte count
○ A 56-year-old man comes to the emergency department because of abdominal pain
for the past two days. He has a history of alcohol abuse and depression and the
pain began a day after a night of heavy drinking. He states that his pain moves from
his belly to his back and rates it as a 7 on a 10-point scale. He also complains of
nausea and vomiting. Current medications include 20 mg of fluoxetine daily. His
temperature is 38.0°C (100.5°F) and physical examination shows epigastric and
right upper quadrant tenderness with voluntary guarding but no rebound tenderness.
Which of the following is the most likely diagnosis? Acute pancreatitis
○ A 40-year-old woman with four children comes to the emergency department
because of severe epigastric pain that radiates to her back. She says that she has
been having pain which resolves within an hour in the right upper quadrant after
meals over the past few days. A day ago, she developed this same right upper
quadrant pain after eating which did not resolve and then moved to the epigastric
area. The pain is still severe and has not improved over the past day. She also has
nausea and vomiting. Physical examination shows she is afebrile. Her pulse is
110/min, blood pressure 118/62 mm Hg, and BMI is 32 kg/m2. She has
mid-epigastric tenderness with guarding. Laboratory studies show elevated serum
amylase and serum lipase concentrations. Which of the following is most important
part of her initial management? Intravenous fluid
○ A 55-year-old man comes to the emergency department because of a 4-week
history of greasy, foul-smelling diarrhea. He has had an alcohol use disorder for
several decades and has been hospitalized several times in the past for a "stomach
problem." He denies fever, hematochezia, or mucus in his stools. Which of the
following medication is most appropriate in the treatment of this patient? Lipase
○ A 44-year-old man who was treated four weeks ago for acute pancreatitis comes to
the office with a recurrent episode of abdominal pain, nausea, a "bloated feeling,"
and slight abdominal distention. His history is significant for long-term alcohol abuse.
Transabdominal ultrasound shows walled-off fluid collections in the pancreas which
are confirmed by a CT scan as pancreatic pseudocysts, the largest of which is
4.5cm. Which of the following is the most appropriate treatment plan for this patient?
Endoscopic drainage
○ A 20-year-old woman comes to the emergency department because of bilateral
lower abdominal pain for the past two days. She states that she had an intrauterine
device placed about 2 weeks before, and she has been experiencing irregular
vaginal bleeding and lower back pain ever since. Pelvic examination shows adnexal
tenderness and a mucopurulent cervical discharge. Gram stain shows gram-positive
filamentous rods, and a pregnancy test was negative. Which of the following is the
most likely microorganism causing the patient's condition? Actinomyces israelii
○ A 23-year-old nulliparous woman comes to the office because of chronic episodic
pelvic pain. The pain begins 2-3 days before menses and continues throughout the
menses after which it subsides. Menarche was at age 14. She has a regular
menstrual cycle of 25 days with 5-6 days of moderate bleeding. Medical history is
significant for recurrent migraine headaches with aura. She is not sexually active.
Physical examination shows a fixed anteverted uterus and nodularity in the posterior
cul-de-sac. The physician offers surgical therapy, which the patient declines at this
time. Which of the following is the most appropriate option to treat this condition
medically? Progestin-only pills
○ A 74-year-old woman arrives via ambulance to the emergency department
complaining of increasing abdominal pain, lethargy, and anorexia beginning three
days ago. She reports some nausea, but denies vomiting. Her last bowel movement
was two days ago. Upon arrival in the emergency department, her vital signs are as
follows: T 100.0 F P 75 bpm R 14 bpm BP 100/70 SaO2: 99% (room air). Physical
exam shows an ill-appearing elderly woman with a distended, diffusely tender
abdomen that is tympanic to palpation. An abdominal X-ray is ordered, which shows
the following: Reduction via flexible sigmoidoscopy
○ A 51-year-old woman comes to the clinic because of abdominal pain for the past 12
months. The pain has been vague, located in the epigastric region and left upper
quadrant of the abdomen. She says she has weekly nausea and vomiting, and
intermittent pain in her left shoulder. An arterial angiogram is obtained and is shown
below. Which of the following is the most likely diagnosis? Splenic artery
aneurysm
○ A 10-year-old girl comes to the outpatient clinic because of bloating and excessive
flatulence. Her symptoms worsen when she eats cheese or yogurt. She has been
growing at average pace and does not show signs of malnutrition. She is afebrile.
Physical examination shows abdominal distention, but no signs of tenderness or
guarding. Which of the following is the most appropriate next step in diagnosis?
Hydrogen breath test
○ A 60-year-old man with a history of coronary artery disease and tobacco use comes
to the emergency department because of acute abdominal pain. He has had diffuse
mild abdominal pain after meals for the past three months. However, recently he has
experienced more frequent episodes of abdominal pain and has seen blood in his
stool intermittently for the past two weeks. He rates the pain a 7 on a 10-point
scale. Abdominal examination shows no abnormalities. A computed tomography
scan of the abdomen scan shows bowel wall edema around the splenic flexure
extending to the descending colon. Decreased blood flow in which of the following
arteries would most likely account for the patient's clinical features? Inferior
mesenteric artery
○ A 38-year-old woman, with a history of gastric bypass two months ago, comes to the
emergency department because of colicky abdominal pain, severe nausea, and
diarrhea. She has a blood pressure of 86/50 mmHg, her heart rate is 120 bpm, and
her skin is sweaty. Her pupils are somewhat dilated but are equal, round, and
reactive to light. She reports a rapid onset of symptoms following a meal out with
friends at a local fast food restaurant, where she had a milkshake and several
cookies. Which of the following complication of bypass surgery is she experiencing?
Dumping syndrome
○ A 26-year-old woman comes to the emergency department because of lower
abdominal pain and vaginal bleeding for the past 24 hours. Her last menstrual
period was 7 weeks ago. She is sexually active with her boyfriend and they use
condoms intermittently. Physical examination reveals mild tenderness in the left
lower quadrant and left adnexa. The cervix is closed with dried blood visible at the
os; there is no active bleeding. A quantitative serum B-HCG is 1,500 IU/L.
Transvaginal sonography of the patient's left adnexa reveals an extra-uterine
gestational sac with a diameter of 2 cm. Which of the following is the most
appropriate management option for this patient? Intramuscular methotrexate
○ A 77-year-old woman comes to the emergency department because of acute
abdominal pain. She rates the pain a 10 on a 10-point scale and localizes it to her
umbilicus. She also says she experiences nausea and vomiting. She denies
diarrhea or constipation. She has a past medical history of congestive heart failure.
She has never had any surgeries. She has a 10-pack-year history of smoking
cigarettes. Her temperature is 37.0ºC (98.6ºF), pulse is 80/min, respirations are
16/min, and blood pressure is 120/90 mm Hg. Physical examination shows a soft,
nontender, nondistended abdomen with no rebound tenderness or guarding. The
lower extremities have 1+ pitting edema. Which of the following is the most likely
pathological process underlying this patient’s condition? Occlusion of vasculature
supplying the bowel
○ A 26-year-old woman, gravida 2, para 2, comes to the office because of episodic
right abdominal pain for the past week. She says the pain is dull and intermittent,
located in the right upper quadrant. Her medical history is significant for well
controlled hypertension, an appendectomy at the age of 14, and two prior cesarean
deliveries. Her medications include cilazapril and the combined oral contraceptive
pill. Her vital signs show no abnormalities. Examination shows moderate tenderness
in the right upper quadrant without guarding or rebound tenderness. A abdominal
computed tomography scan shows a 5cm heterogenous, well-circumscribed lesion
in the right lobe of her liver (shown below). Which of the following is the most likely
diagnosis? Hepatic adenoma
○ A 35-year-old woman comes to the emergency department because of right lower
quadrant abdominal pain that started suddenly and has been present for the past 3
hours. She also reports a small amount of vaginal bleeding. Her last period was 6
weeks ago. She denies fever, nausea, or vomiting. She reports that she is sexually
active and had a tubal ligation 2 years ago. Her temperature is 37°C (98.6°F), pulse
is 77/min, respirations are 12/min, and blood pressure is 110/70 mm Hg. A urine
pregnancy test is obtained and is positive. Serum hCG is 1950 mIU/mL. Which of
the following is the most appropriate next step for diagnosis? Transvaginal
ultrasound
○ A 35-year-old woman, grava 1, para 0, at 35 weeks' gestation comes to the
emergency department because of painful vaginal bleeding for the past 2 hours.
She says the pain began suddenly and is located in her lower abdomen and lower
back. She is also experiencing contractions every 10 minutes, which have not
increased in frequency or intensity. Her past medical history is significant for
hypertension that has been well managed throughout her pregnancy with labetalol.
She has smoked 10 cigarettes a day for the past 12 years, and has continued to
smoke throughout her pregnancy. She does not drink alcohol, and has not used illicit
drugs. Physical examination shows blood in the vaginal vault and a closed cervix.
Which of the following is the most likely diagnosis? Placenta abruption
○ A 12-year-old previously healthy boy is brought to the emergency department by his
mother because of nausea, vomiting, fever, non-bloody diarrhea and cramping
abdominal pain for the past 24 hours. His mother states that he woke up the day
before with a fever of 38.6°C (101.6°F) and has been having 5-6 loose stools per
day ever since. She also mentions that he was helping her cook chicken cutlets two
days ago. Physical examination shows a dehydrated male in moderate distress, and
abdominal examination shows diffuse tenderness to palpation without rebound.
Stool culture grows gram-negative, lactose-negative rod that form translucent
colonies on MacConkey agar. Which of the following is the best course of
treatment? Replace fluids and electrolytes
○ A 12-year-old boy comes to the emergency department with severe right lower
quadrant abdominal pain. Parents note the patient has felt hot for 3 days. His
temperature is 38.9°C (102.0°F). Laboratory studies show leukocytosis. The patient
is brought to the operating room for an appendectomy, and the surgeon finds visible
inflammation around the appendix, terminal ileum, and mesenteric lymph nodes, but
the appendix itself is normal. A week later the patient begins complaining of joint
pain, dysuria, headache, and malaise. He has had bloody diarrhea for the past
week. What is the most likely source of the pathogen causing his illness?
Contaminated pork and milk
○ A 29-year-old woman, gravida 1, para 0, at 19 weeks gestation, comes to the
emergency department because of nausea, vomiting, fever, and abdominal pain for
the past two days. She says she has lost her appetite. Her temperature is 38.8°C
(101.8°F), pulse is 92/min, respirations are 17/min, and blood pressure is 135/80
mm Hg. Physical examination shows moderate tenderness to palpation in the right
lower quadrant and right flank tenderness. Which of the following is the most
appropriate next step in management? Graded compression ultrasound
○ A 50-year-old woman comes to the office because of nausea, vomiting, and
decreased appetite for the past four days. Her medical history shows systemic lupus
erythematosus (SLE) and fibromyalgia, for which she requires chronic opiate
therapy. Current medications include prednisone, hydroxychloroquine, and a daily
multivitamin. She recently was treated for community-acquired pneumonia with
azithromycin. She works as a nurse at a local hospital and denies use of drugs,
alcohol, or tobacco products. Abdominal examination shows a soft, distended
abdomen that is diffusely tender to palpation. Which of the following is the most
likely underlying cause of her complaints? Hypomobility of the gastrointestinal
tract
○ A 35 year-old investment banker with a history of liver injury is undergoing surgery
for an appendectomy. You are his anesthesiologist; which of the agents below is
best suited for the patient’s anesthesia? Sevoflurane
○ A 60-year-old man comes to the emergency department because of fever and
severe right-sided upper and lower abdominal pain for the past 12 hours. He has
had no bowel action for the past 5 days. He was recently diagnosed with lymphoma
of the liver and was discharged several days ago after starting his first cycle of
Hyper-CVAD chemotherapy. His abdomen is severely distended and there rebound
and percussion tenderness exists. A nasogastric tube is inserted, he is started on IV
fluids and antibiotics, and a CT scan is ordered. Which of the following is the most
likely diagnosis? Neutropenic enterocolitis
○ A 22-year-old woman comes to the emergency department with lower abdominal
pain for the past five days. She says the pain is present on both sides of her lower
abdomen, and is sharp in nature. She also reports irregular vaginal bleeding and
lower back pain for the past seven days, before which she had an intrauterine
contraceptive device placed. Examination shows adnexal tenderness, uterine
tenderness and a mucopurulent cervical discharge. Urine dipstick shows no
abnormalities. Gram stain from a cervical swab shows Gram-positive filamentous
rods. Which of the following is the most appropriate pharmacological treatment for
this patient? Penicillin G
○ A 35-year-old woman comes to the clinic for a fertility consultation. The patient has
been married to her husband for the past 10 years and they have been trying to
have a child for the past 2 years without success. She has been receiving hCG
injections for the past 4 months and she says that she has been experiencing
nausea, abdominal bloating, excessive weight gain, darker urine, dyspnea, and
lower abdominal pain for the past 2 weeks. The patient says that symptoms have
been worsening. Which of the following is the most likely diagnosis? Ovarian
hyperstimulation syndrome
○ A 49-year-old man comes to the emergency department because of intense
abdominal pain. The pain is sharp in character and associated with nausea and
vomiting. He has type 1 diabetes, which he has managed with insulin since
childhood. He has also been experiencing abdominal pain all day but thought it was
secondary to the flu. His temperature is 38.8°C (101.8°F), pulse is 70/min,
respirations are 18/min, and blood pressure is 125/70 mm Hg. Abdominal
examination shows right lower quadrant tenderness with palpation and a positive
psoas sign. Which of the following is the most appropriate next step in
management? Laparoscopic removal of the infected organ
○ A 29-year-old pregnant woman comes to the emergency department because of
acute, severe right upper quadrant abdominal pain. She has been receiving regular
prenatal care and has had no complications in her pregnancy thus far. She denies
any tobacco use and has not consumed any alcohol for the past 11 months. Her
hepatojugular reflex is negative. Abdominal examination shows hepatomegaly with
an abdominal fluid wave. Urinalysis shows no abnormalities. Serum laboratory
studies show a creatinine concentration of 1.6 mg/dL and an albumin concentration
of 2.5 g/dL. Liver enzyme studies show alkaline phosphatase concentration of 40
U/L, AST concentration of 159 U/L, and ALT concentration of 184 U/L. Complete
blood count shows a leukocyte count of 8,500/mm3. Which of the following is the
most likely diagnosis? Hepatic vein thrombosis
○ A 62-year-old man with a history of atrial fibrillation and previous myocardial
infarction comes to the emergency department because of abdominal pain, nausea,
vomiting, and loss of appetite for the past five hours. The patient rates the pain an 8
on a 10-point scale. He says that the pain is constant and “all over” his abdomen,
unable to point to a specific location. He took 500 mg of acetaminophen but did not
notice any improvement in his symptoms. He also says he had an episode of
hematochezia prior to coming to the emergency department. Which of the following
is the most appropriate diagnostic step in the management of this patient? Contrast
CT
○ A 56-year-old man comes to the emergency department because of significant
abdominal pain, which started eight hours ago. On general examination, he is in
severe distress, diaphoretic, and nauseated. Physical examination reveals absent
bowel sounds and severe rebound tenderness. An abdominal computed
tomography is performed. Which of the following is the most likely diagnosis? Small
bowel infarction with subsequent lumenal air entering into portal circulation
○ A 48-year-old woman is reviewed on the surgical ward because of fever and wound
discharge for 30 minutes. She had an open appendectomy for gangrenous
appendicitis 5 hours ago and her post-operative recovery has been uneventful so
far. Her medications include metformin and intermediate-acting insulin. Her
temperature is 38.0°C (100.4°F), pulse is 87/min, respirations are 18/min, and blood
pressure is 117/78 mm Hg. Physical examination shows that the area around the
bandage has become extremely edematous and has taken on a dusky
grayish-purple color. There is also some thin, watery brown discharge that is
draining from the wound. Which of the following is the most appropriate next step in
management? Surgical debridement
○ A 66-year-old man is brought to the emergency department because of worsening
left lower quadrant pain and diarrhea over the past 2 days. His temperature is
38.7°C (101.6°F), pulse is 115/min, respirations are 20/min, and blood pressure is
145/85 mm Hg. Physical examination shows left lower quadrant tenderness.
Laboratory studies show leukocytosis. Which of the following is the most appropriate
next step in diagnosis? Computed tomography scan
○ A 64-year-old man comes to the emergency department because of a 2-day history
of uncontrollable nausea, vomiting, and generalized abdominal pain. His
temperature is 36.8°C (98.2°F), pulse is 112/min, respirations are 20/min, and blood
pressure is 104/64 mm Hg. Physical examination shows a distended abdomen with
decreased bowel sounds that is tympanitic to percussion. An abdominal radiograph
is obtained and is shown below. Which of the following is the most common
underlying cause of the patient's symptoms? Adhesions
○ A 24-year-old man presents to the emergency department with worsening
abdominal pain that began all of a sudden after eating pizza two nights ago. The
patient reports the pain has progressively worsened and is now 8 out of 10 in
severity. He reports no change in bowel movements during this time, but he has had
a decreased appetite and moderate nausea. Medical history includes lactose
intolerance. His temperature is 37.9°C (100.2°F), pulse is 72/min, respirations are
14/min, and blood pressure is 108/66 mmHg. Abdominal examination shows right
lower quadrant pain with passive right hip extension. Which of the following will most
likely provide definitive treatment for this patient’s condition? Surgical intervention
○ A 42-year-old man comes to the emergency department because of 3-day
abdominal pain. The pain occurs intermittently throughout the day, but is not
associated with meals or sleep. He has a history of rheumatoid arthritis, for which he
takes multiple non-opioid pain relief medications per day. He otherwise appears well
in spite of his acute pain. His vitals show no abnormalities and he has epigastric
tenderness without distention. Which of the following diagnostic tests is the most
appropriate next step in the management of this patient? Complete blood count
○ A 13-year-old boy is brought to the emergency department for evaluation of
testicular pain and swelling. The patient was running when he felt sudden onset
groin pain. Since then, he experienced severe scrotal pain with associated nausea
and vomiting. He has had similar episodes of pain over the past two months that
have all self-resolved without intervention. He has no significant past medical history
and has received all age-appropriate vaccinations. Temperature is 37.0°C (98.6°F),
pulse is 94/min, respirations are 21/min, and blood pressure is 141/80 mmHg.
Physical examination demonstrates an uncomfortable adolescent male in significant
pain. Examination demonstrates an edematous and erythematous right scrotum.
Stroking of the right inner thigh does not result in elevation of the right testis. The
pain is worsened with elevation of the scrotum, and the scrotum does not
transilluminate. Which of the following is the best next step in management?
Surgical fixation of the bilateral testicles
○ A 3-year-old boy is brought to the emergency department by his mother for
abdominal pain and irritability. She states that for the past two weeks he has been
increasingly lethargic and has had frequent spells of colicky abdominal pain with
nausea and vomiting. She also relates that he has recently been fixated on eating
ice and soil, and has been behaving strangely. The patient has not had any recent
dietary or lifestyle changes, and recently went with his parents on fishing trip to
Mexico. The patient's vital signs are within normal limits. Physical examination of the
patient shows a well developed male with marked pallor and lethargy. Laboratory
studies show a white blood cell count of 4,000/mm3, hemoglobin of 8.2 g/dL, and a
platelet count of 400,000/mm3. Which of the following is most likely to be associated
with the patient's clinical presentation? Lead poisoning
○ A 1-week-old infant is referred to a neonatal consultant because of abdominal
distension and rectal bleeding. The infant was observed in the neonatal intensive
care unit after premature birth at 35 weeks' gestation. Abdominal radiography series
shows pneumatosis intestinalis. Which of the following is the most likely diagnosis?
Necrotizing enterocolitis
○ A 25-year-old woman comes to the emergency department because of abdominal
pain. She describes it as intermittent dull, throbbing pain in the left lower quadrant.
She complains of nausea and vomiting in the morning that usually goes away in the
middle of the day. She denies the possibility of being pregnant as she uses oral
contraceptive pills. Her past medical history includes treatment of a sexually
transmitted infection with doxycycline one year ago, and a history of depression,
currently being treated with paroxetine. Serum beta-hCG levels are 2225 mIU/mL. A
transvaginal ultrasound scan (TVUS) is obtained and is negative for an intrauterine
gestation. Which of the following features on history put this patient at an increased
risk for the most likely diagnosis? Prior infection with Chlamydia trachomatis
○ A 66-year-old man comes to the emergency department because of severe
abdominal pain in his right lower quadrant. The abdominal pain is sharp in character
and has been present for three hours. His temperature is 37°C (98.6°F), pulse is
95/min, respirations are 18/min, and blood pressure is 130/80 mm Hg. Abdominal
examination shows tenderness with deep palpation at McBurney point. There are no
peritoneal signs. Laboratory studies show a leukocyte count of 16,000/mm3. Which
of the following is the definitive treatment? Appendectomy
○ A 58-year-old man comes to the emergency department because of severe
abdominal pain and vomiting. He has felt this pain for months, but it has become
intolerable over the last 8 hours. He has a history of gastroesophageal reflux
disease (GERD), hypertension, and asthma. He has not had a bowel movement or
passed flatus in 3 days. Since the pain worsened, he is now having trouble moving.
His temperature is 39.1°C (102.5°F), pulse is 115/min, and blood pressure is 135/75
mm Hg. Physical examination shows a distended abdomen with high pitched bowel
sounds. He appears to be in severe distress, secondary to pain, and refuses to
straighten his legs because it hurts too much. Which of the following is observed on
the barium follow-through due to this patient disease? Dilated small intestines
○ A 23-year-old previously healthy woman comes to the emergency department
because of right sided lower abdominal pain and vaginal bleeding for the past 24
hours. She denies nausea, vomiting, fever, anorexia, and diarrhea. Her last
menstrual period (LMP) was 7 weeks ago. She is sexually active and has multiple
sexual partners, using condoms inconsistently. She is afebrile, pulse is 80/min,
respirations are 14/min, and blood pressure is 115/80 mm Hg. Physical examination
shows mild tenderness in the right lower quadrant and right adnexa, with no
rebound tenderness nor guarding. The cervix is closed with scant blood visible at
the os; there is no active bleeding. A quantitative serum ßhCG is 2000 IU/L. Which
of the following is the most likely diagnosis? Ectopic pregnancy
○ A 62-year-old woman comes to her outpatient provider seeking evaluation for
postprandial abdominal pain. The symptoms started 6 months ago and have
progressively worsened. She reports that the pain begins 30-40 minutes after meals.
She has been eating less due to the pain, and she has experienced a 5 kg (11 lb)
weight loss since symptom onset. The patient denies nausea, vomiting, and
diarrhea. Past medical history is notable for hypertension and hyperlipidemia. The
patient has a 40-pack-year smoking history. Her younger sister was diagnosed with
colorectal cancer at age 54. Laboratory testing reveals the following findings:

Laboratory value Result

Hemoglobin 15.2 g/dL

Leukocyte count 6,500/mm3

Total bilirubin 0.6 mg/dL

Direct bilirubin 0.3 mg/dL

Alanine aminotransferase 14 U/L

Aspartate aminotransferase 17 U/L

Which of the following is the next best step in the management of this patient?
Perform CT angiography

○ A 3-year-old boy is brought to the clinic because of irritability, vomiting, fever, and
abdominal pain for the past month. His temperature is 39.2°C (102.56°F), pulse is
110/min, respirations are 25/min, and blood pressure is 110/70 mm Hg. Physical
examination shows an upper right quadrant abdominal mass on palpation. A CT
scan image is obtained and is shown below. Which of the following is the most likely
diagnosis? Wilms tumor
○ A 4-year-old boy is brought to the emergency department by his parents because of
severe abdominal pain and constipation for 2 days with no improvement. Prior to
this episode his mother states that he has become increasingly irritable and
lethargic over the past 2 weeks to the point where he has stopped speaking. Review
of the patient's medical history shows that he failed to meet developmental
milestones over the past year. When asked about this, the parents mention that he
has been having a hard time adjusting since they moved into an older home with
many of the original fixtures 15 months before. Physical examination of the patient
shows marked lethargy and is otherwise normal. Based on the patient's symptoms,
which of the following is the most appropriate treatment? Dimercaprol
○ A 25-year-old man comes to the emergency department because of excruciating
abdominal pain. The abdominal pain is localized to the right lower quadrant. A day
ago, the abdominal pain was located in the periumbilical region and was less
severe. His temperature is 37°C (98.6°F), pulse is 95/min, respirations are 18/min,
and blood pressure is 130/80 mm Hg. Abdominal examination shows tenderness
with deep palpation. Laboratory studies show a leukocyte count of 15,000/mm3.
Which of the following is the most sensitive finding for the patient's likely diagnosis?
Pain 3-5 cm from the anterior superior iliac spine on a straight line from the
anterior superior iliac spine to the umbilicus
○ A 27-year-old woman, gravida 2, para 1, abortus 1, comes to the emergency room
because of right lower quadrant pain and nausea. She has a history of ectopic
pregnancy and is concerned that this could be the problem. Her last menstrual
period (LMP) was 6 weeks ago. A quantitative b-hCG concentration is 1,851 IU.
Which of the following imaging modalities would you use for further investigation?
Transvaginal ultrasound
○ A 20-year-old woman comes to the emergency department with lower abdominal
pain for the past three days. She says the pain is present on both sides of her lower
abdomen, and is sharp in nature. She also reports irregular vaginal bleeding and
lower back pain for the past seven days, before which she had an intrauterine
contraceptive device placed. Examination shows adnexal tenderness, uterine
tenderness and a mucopurulent cervical discharge. Urine dipstick shows no
abnormalities. Gram stain from a cervical swab shows Gram-positive filamentous
rods. The organism responsible for causing this infection can be characterized by
the presence of which of the following? Sinus tracts containing sulfur granules
○ A 24-year-old woman presents to the emergency department with nausea, vomiting,
and worsening abdominal pain for the last 12 hours. She states the pain started all
of sudden with no inciting factors. Her symptoms were not associated with any
particular meal, and she denies any recent travel. The patient has a history of mild
asthma, for which she takes an albuterol inhaler as needed. Her temperature is
39°C (102.2°F), pulse is 84/min, respirations are 18/min, and blood pressure is
128/74 mmHg. Physical examination shows right lower abdominal pain that is
elicited with deep palpation. There is no hepatosplenomegaly. Cardiac and
pulmonary exams are noncontributory. A urine β-hCG test is ordered and returns
negative. Leukocyte count is 12,000/mm3. Which of the following is the next best
step in management? Computed tomography of the abdomen
○ A 56-year-old man comes to the emergency department because of onset of severe
epigastric pain. He has intractable retching without vomiting. Physical examination
shows a distended abdomen that is dull to percussion. An X-ray with barium
contrast is ordered and the results are shown below. Which of the following is the
most likely diagnosis? Organo-axial volvulus
○ A 26-year-old man is brought to the emergency department an hour after being
stabbed in the abdomen during a street fight. The patient is hemodynamically stable.
Abdominal examination shows a knife wound in the right lower quadrant, guarding,
and rebound tenderness. A full body computed tomography scan is done and shows
the following image Pneumoperitoneum
● PLAYLIST 8
○ A 30-year-old man comes to the emergency department because of an abrupt onset
of abdominal cramps, diarrhea, nausea, and vomiting that began last evening. He
reports returning from a trip to Mexico yesterday and feeling fine throughout the
travels. The patient feels an urgent need to defecate. Vitals signs are significant for
a temperature of 98.0°F (36.7°C). Which of the following is the most likely diagnosis
in this patient? Enterotoxigenic Escherichia coli
○ A 4-year-old girl is brought to her pediatrician because of abdominal cramps and
diarrhea for the past two days. The patient's mother noticed that her stools are
bloody, contain mucous and are small in volume. She also mentions that 2 other
children at her daughter's daycare have similar symptoms. Physical examination
shows a young girl in moderate distress with a fever of 39.6°C (103.3°F). Analysis of
the patient's stool sample shows polymorphonuclear leukocytes on a methylene
blue stain. A complete blood count was obtained and shows 14/mm3 white blood
cells. Which of the following is the most appropriate treatment for this patient's
condition? Azithromycin
○ A 35-year-old man comes to the emergency department because of sudden
explosive diarrhea and intermittent abdominal pain. His medical history is relevant
for recent traveling to Honduras. Upon further questioning, the patient mentions that
he feels an uncontrollable urgency to defecate that does not cease after defecation.
When the patient is asked about his stools, he describes them as "bloody and
foul-smelling." Physical examination shows decreased alertness, sunken eyes, dry
mouth, split skin on mouth corners, and abdominal tenderness on palpation. His
temperature is 38.6°C (101.48°F), pulse is 122/min, respirations are 24/min, blood
pressure is 90/70 mmHg. Laboratory studies show leukocytosis and elevated
alkaline phosphatase. Microscopy of a stool sample is obtained and shown below.
Which of the following is the most appropriate initial treatment for this patient? Oral
rehydration therapy
○ A 25-year-old woman comes to the emergency department because of a 2-day
history of nausea, vomiting, subjective fever, profuse diarrhea, and abdominal
cramping. She states that she had attended a large family picnic several days earlier
where there were several cold salads. Following admission, which of the following is
the most appropriate first step in management for this patient? Replacement of
fluids and electrolytes
○ A 15-year-old boy comes to the emergency department because of copious
amounts of watery diarrhea for the last 5 days. The patient states the diarrhea
began on the last day of his mission trip to rural Guatemala, where he was helping
build a new school with a group of people from his church. He describes the
diarrhea as watery and oily-appearing at times. He has also been feeling gassy
throughout the last few days. Stool microscopy shows the organisms seen in the
image below. Which of the following is the most appropriate treatment for this
patient's condition? Metronidazole
○ A 42-year-old man and his 12-year-old son come to the office because of
unremitting diarrhea for the past two weeks. They have both been having watery,
nonbloody, foul smelling, greasy diarrhea with abdominal pain and cramps. Two
weeks ago, they went on a father-son camping trip to the mountains where they
"roughed it." They ate only by scavenging for edible plants and spearing fish in the
river (which they thoroughly cooked over a campfire), and drank the mountain
stream water. A stool sample from each of them shows the organism below. Which
of the following pathogens is the most likely cause of this diarrhea? Giardia lamblia
○ A 3-year-old girl comes to the clinic because of an 8 day history of fever, vomiting
and watery diarrhea. There has been mild improvement in the last few weeks as the
frequency of the patient’s stool has decreased in the last few days, and the stool is
more formed. Her mother denies the patient has had any seizures and reports the
child has had a few sick contacts. The patient goes to a day care about 4 days a
week, and three other children have had similar symptoms. On physical examination
the patient has normal vital signs with a moderate amount of stool in her diaper. She
has an otherwise normal exam with no concerns regarding hydration. Fluids are
prescribed to the patient. Which of the following is the single best treatment option
available to help prevent further breakout of this patient's most likely condition?
Return back to daycare with stool well contained in diaper
○ A 2-year old girl comes to the clinic because of vomiting and diarrhea for 2 days.
Her mother describes the diarrhea as non-bloody and without mucus. The patient
appears to be in pain and is lethargic. Which of the following organisms is the most
likely culprit for the symptoms in this patient? Norovirus
○ A 14-year-old boy who recently immigrated from Cambodia presents to the clinic
with waxing and waning gastrointestinal symptoms of diarrhea, nausea, and
vomiting. On physical exam, the patient has a mild dry cough with some wheezing,
though the heart has regular rate and rhythm. There are serpiginous, raised
erythamatous tracks in the perianal area of the patient. A stool ova and parasite test
is done, the results of which are shown below. Which of the following is the best
treatment for this patient? Ivermectin
○ A 16-year-old African American boy comes to the outpatient clinic because of
watery, non-bloody diarrhea for a month. His diarrhea is associated with abdominal
cramps, bloating, and excessive flatulence, but is not associated with fever or
vomiting. He denies any changes in weight, recent travel history, sick contacts, or
recent viral infections. He says that he has been trying to "bulk up" for football
tryouts and has been drinking a lot of protein powder milkshakes in the past month.
Which of the following is the most appropriate next step in management? Lactose
breath hydrogen test
○ A 83-year-old woman comes to the emergency department because of fever,
nausea, vomiting, and profuse watery diarrhea for 2 days. Medical history includes
treatment with antibiotics for a hospital acquired pneumonia 8 days ago. Her
temperature is 37.8°C (100.0°F), pulse is 102/min, respirations are 20/min, and
blood pressure is 133/85 mm Hg. Physical examination shows a distressed
appearing woman. Cardiac auscultation is noncontributory, and pulmonary
auscultation shows reveals vesicular sounds. Abdominal examination shows mild,
diffuse abdominal tenderness. Laboratory investigations show the following:
A CT-scan of the abdomen is obtained and shows thickening of the colonic wall.
Which of the following classes of antibiotics was she most likely treated with for her
pneumonia? Fluoroquinolones
○ A 14-year-old girl comes to the office because of fatigue, anxiety, and balding for a
month. Her parents are missionaries and note that her appetite has been odd
roughly since they returned to the United States from a trip abroad. Among her odd
behaviors, she has a new found craving for all things made of ice and the other day
they say they caught her eating dirt. When questioned about her behavior the girl
expresses shame but states that she does these things "almost compulsively."
Examination shows a non-tender abdomen with areas of diffuse fullness, and
palpebral pallor. Peripheral smear shows anisocytic, microcytic, hypochromic red
blood cells. Eosinophilia and lymphocytosis are also shown. Iron studies show
decreased transferrin, and decreased ferritin levels. A trichobezoar is retrieved on
upper endoscopy. Which of the following is the most appropriate next step in
treatment? Ivermectin
○ A 30-year-old man comes to the clinic because he plans on traveling to Mexico for
three weeks. He inquires about prophylaxis for traveler diarrhea. Past medical
history reveals inflammatory bowel disease that is well controlled with diet and
medication. Which is the most appropriate prophylactic pharmacological
management in this patient? Ciprofloxacin
○ A 12-month-old infant boy comes to the pediatrician's office for a routine well-child
examination. The boy did not have any complications during pregnancy or delivery.
He passed meconium right after delivery. His weight and height are growing at a
normal pace; however, his mother is concerned that he has been irritable and
frequently passes gas after meals. His stool is found to be highly acidic, and he is
started on a special diet. Which of the following is the most likely diagnosis?
Lactase deficiency
○ A 4-year-old girl comes to the urgent care clinic because of vomiting and diarrhea
for the past 2 days. Her mother says that many of the other children at her
daughter's day care facility have also been experiencing the same symptoms. She is
concerned because some of the other children have experienced a week or more of
diarrhea. On examination, this patient is febrile to 38C with a heart rate of 150 bpm.
Blood pressure is 90/60. Her mucus membranes are somewhat dry, and abdominal
exam reveals hyperactive bowel sounds. In the interest of ensuring her
hemodynamic stability, she is admitted to the hospital and a sample of her stool is
sent for analysis. Bacterial cultures are negative, stool ova and parasites are both
negative, and viral polymerase chain reaction is positive for adenovirus. Which of
the following is the next best step in the treatment of this patient? Continued
support of hydration status
○ A 6-year-old boy comes to the emergency department because of 2 days of
diarrhea. His mother says that her son has also been vomiting occasionally
(non-bloody, non-bilious) for about the same amount of time. He has a heart rate of
120 bpm, a temp of 37.7*C, and a respiratory rate of 16 breaths/minute. The child
appears uncomfortable. His capillary refill is 4 seconds and his mucous membranes
are tacky. His abdomen is neither tender nor distended. The remainder of the exam
is unremarkable, and a complete blood count and electrolyte panel are normal.
Which of the following is the most appropriate first therapeutic step in the
management of this child? Oral rehydration therapy
○ A 20-year-old woman comes to the emergency department because of abdominal
pain, fever, and bloody diarrhea for 3 days. She recently immigrated to the United
states from Mexico. She denies nausea, vomiting, or chills. Physical examination
shows hepatomegaly and right upper quadrant tenderness. An indirect
hemagglutination titer confirms the presence of antibodies and an ultrasound shows
that she has an abscess in the right lobe of her liver. Which of the following
organisms is most likely responsible for this abscess? Entamoeba histolytica
○ A 4-year-old girl is brought to the office because of diarrhea for 12 hours. Her
mother states that she came home from daycare and developed fever, abdominal
pain, and diarrhea containing flecks of bright-red blood and pus. The girl did not
ingest any food at daycare, and her diet has been unchanged in the last week. A
stool specimen is collected and sent to the lab. Which of the following is the most
likely causative organism based on clinical findings? Shigella dysenteriae
○ A 20-year-old woman comes to the emergency department with a fever, headache,
and watery diarrhea. The patient's stomach has been hurting since last night and
she has noticed a small amount of blood in her most recent bowel movement. She
went on a picnic with her boyfriend two days ago where they ate peanut butter
sandwiches, chips, and had a couple of soft drinks. A stool sample is obtained and
is positive for a lactose-negative, hydrogen sulfide-producing bacteria. Which of the
following is the most likely infectious agent causing these symptoms? Salmonella
enteritidis
○ A 55-year-old man, who recently moved to the United States from Latin America,
comes to clinic because of a 1-week history of bloody diarrhea. Blood tests reveal
leukocytosis without eosinophilia, and liver function tests show elevated alkaline
phosphatase and transaminases. Physical examination of the abdomen shows
fever, right upper quadrant pain, and a palpable mass in the abdomen over the
ascending colon. On ultrasound of the liver, there is a round rim-enhancing lesion
which looks like a well-defined abscess with low density. Chest X-ray shows an
elevated right hemidiaphragm. Which of the following treatments would be most
appropriate? Immediately begin treatment with metronidazole
○ A 22-year-old female comes to the emergency department because of fever,
headache and watery diarrhea for 12 hours. She says her stomach has been hurting
since last night, and that she noticed a small amount of blood in her most recent
bowel movement. She states she went on a picnic with her boyfriend 2 days ago.
They ate sandwiches, chips, and consumed soft drinks. A stool sample is obtained
and cultured using MacConkey agar. The colonies show non-lactose fermenting,
non-hydrogen sulfide-producing bacteria. Which of the following organisms is the
most likely to cause such clinical symptoms? Shigella dysenteriae
○ A 16-year-old man comes to the clinic because of 7 days of flatulence and greasy,
foul-smelling diarrhea. He states he recently went camping and reports that his
friend collected water from a stream, but did not boil or chemically treat the water
before they consumed it. The patient also reports symptoms of nausea, weight loss,
and abdominal cramps followed by sudden diarrhea. Which of the following is the
most appropriate treatment option? Metronidazole
○ A 45-year-old man presents to the emergency department due to a several month
history of intermittent diarrhea that has been worsening over the past week. He has
about 10 watery stools per day. He also has mild abdominal pain, myalgia,
intermittent flushing, and diaphoresis. Physical examination shows a mildly
extended but non-tender abdomen, pulse is 110/min, dry mucous membranes and a
capillary refill of 6 seconds. Laboratory analysis indicate volume depletion with
hypokalemia and a non-anion gap acidosis with a gastric pH of 4. A tumor producing
which of the following endocrine hormones is most likely responsible for these signs
and symptoms? Vasoactive Intestinal Peptide (VIP)
○ A 45-year-old man comes to the emergency department because of severe
abdominal pain and copious non-bloody diarrhea for the last 24 hours. Two weeks
ago he was hospitalized for 7 days to treat acute pancreatitis, and was given
antibiotics during his stay. Several days after being discharged he started feeling
more tired and subsequently developed a fever. He has not traveled recently and
has not had any sick contacts since leaving the hospital. Physical examination
shows a male in moderate distress who appears significantly dehydrated.
Abdominal examination shows lower abdominal tenderness to palpation. Which of
the following is a finding associated with the most likely cause of this patient's
diarrhea? Pseudomembranous colitis
○ A 25-year-old man comes to the emergency department with a fever, headache, and
watery diarrhea. His abdomen has been hurting since last night. He noticed a small
amount of blood in his most recent bowel movement, 2 hours ago. A stool sample is
obtained and is positive for a lactose-negative, hydrogen sulfide-producing bacteria.
Which of the following mechanisms is most likely responsible for causing the
symptoms associated with the causative bacteria? Direct activation of the
immune response resulting in increased intracellular cAMP,
○ A 75-year-old woman comes to the emergency department with worsening
shortness of the breath for the past week. She is admitted with a diagnosis of
pneumonia. Her condition is managed with observation and intravenous levofloxacin
750 mg daily for 2 weeks. On the 15th day of hospitalization, she has three episodes
of diarrhea. The following day, her white blood cell count is 21,000/mm3, and she
begins complaining of abdominal pain. Her pulse is 110/min, respirations are 18/min,
and blood pressure is 92/50 mm Hg. Physical examination shows she is confused
and has diffuse abdominal tenderness without guarding, rebound, or rigidity. Which
of the following is the most appropriate response? Exploratory laparotomy

● PLAYLIST 9
○ A 54-year-old man comes to the emergency department because he is
currently confused, sluggish, unable to answer questions, and has rapid and
shallow y from colonoscopy shows noncaseating granulomas and skip lesions of
transmural ulcers. A barium swallow is done and the results are shown below.
Which of the following is the most appropriate medication to be administered?
Mesalamine (5-ASA)
○ A 37-year-old woman comes to the office because of multiple episodes of diarrhea
and bloating for several years. She also reports some itchy vesicles on her skin
around her arms and legs. Physical examination shows abdominal distention, but
the rest of the exam is normal except for mucosal and conjunctival pallor. Which of
the following is the most appropriate first-line treatment for her condition? Dietary
changes
○ A 22-year-old Jewish woman comes to the emergency department because of
severe right lower quadrant abdominal pain for the past two hours. She has had
diarrhea and weight loss for several weeks. A colonoscopy is performed, and an
image from the procedure is shown below. Which of the following is the most likely
complication of this patient's diagnosis? Perianal fistulas
○ A 43-year-old man with a recurrent fistula-in-ano comes to the office for monitoring
of his Crohn disease. An MRI is done to determine whether he has developed a
complex, multi-tract fistula. The MRI shows that his fistula only follows one tract. It
courses perpendicularly to the skin from its external perianal opening, then takes a
sharp 90° turn and burrows through both the external anal sphincter and the internal
sphincter to open into the anal canal. According to the Parks classification of
fistulas, which of the following most accurately describes this patient's fistula-in-ano?
Trans sphinteric fistula
○ A 25-year-old woman comes to the office because of painful new lesions on both of
her legs. The lesions are purple, firm nodules, and her legs are mildly swollen, as
seen in the image below. She has had loose stools, intermittent fevers, and weight
loss for the past three weeks. Which of the following is the most likely diagnosis?
Crohn disease
○ A 37-year-old woman comes to the office because of multiple episodes of diarrhea
and bloating for several years. She thinks she has lost weight because of this
indigestion. She also has some itchy vesicles on her arms and legs. Physical
examination shows abdominal distention as well as mucosal and conjunctival pallor.
Antibodies against which of the following are more likely associated with her
condition? Endomysium
○ A 15-year-old girl comes to the office because of abdominal pain, diarrhea,
flatulence, and borborygmi. The only known relieving factor is not eating, and as a
consequence, she has lost 6.8-kg (15-lb) in 6 weeks. She is earning excellent
grades in school and is involved in numerous extracurricular activities. She denies
any new stress or unusual behavior. She has noticed numerous fluid-filled blisters
along her arms and legs, as shown below. Her temperature is 37.6°C (99.7°F), pulse
is 88/min, respirations 13/min, and blood pressure 118/80 mm Hg. Serologic
evaluation is positive for anti-tissue transglutaminase IgA and anti-endomysial IgA.
Which of the following is most likely seen on abdominal radiograph with barium
contrast? Decreased jejunal folds, increased ileal folds
○ A 30-year-old man comes to the clinic because of a rash that appeared a week ago.
He says that it is so itchy that it has kept him up at night. He first noticed it on his
knees and since then, it has spread to both of his elbows, neck, and back. He
denies recent illness or exposure to other new products or chemicals. He has no
significant past medical history except for a cholecystectomy three years ago.
Physical examination shows many erythematous vesicles and plaques located on
his elbows, the extensor surface of his forearms, knees, neck, and back, as show
below. His mucous membranes are not involved and nail changes are not visible. A
biopsy of the affected skin and direct immunofluorescence reveals IgA deposits in
the dermal papillae. Which of the following additional symptoms is this patient most
likely to experience? Bloating and diarrhea
○ A 38-year-old man comes to the office because of recent onset of abdominal pain
around his umbilicus. He has a history of Crohn disease that was diagnosed a few
years ago. He says that his pants have been feeling a little loose, and he is worried
he might have lost some weight unintentionally. His temperature is 38°C (100.4°F).
Physical examination shows a palpable abdominal mass in the right lower quadrant.
Which of the following is the most appropriate next step in the management of this
patient? Abdominal CT
○ A 3-year-old girl comes to the office because her parents say that she frequently has
foul-smelling diarrhea, particularly after eating pasta or bread. Physical examination
shows excoriated papules and vesicles on her elbows, knees, back, and buttocks.
She is in the 10th percentile on the growth curve for height and weight. An
endoscopic small bowel biopsy is obtained and is shown below. Which of the
following gastrointestinal neoplasm is she at the greatest risk of developing if her
disease becomes refractory to treatment? T-cell lymphoma
○ A 40-year-old man comes to the office for a follow-up evaluation of his inflammatory
bowel disease. He asks what the risk factors are, and would like to know what
modifications he can make to his lifestyle that will decrease the risk of exacerbation.
Which of the following is the most appropriate physician response to his question?
Smoking may aggravate Crohn disease and relieve ulcerative colitis
○ A 20-year-old woman comes to the office because of a new rash that appeared two
days ago on her legs as shown below. The lesions are non-pruritic and tender to
touch. She was recently diagnosed with an inflammatory bowel disease that is still
being worked-up. Which of the following is the most likely diagnosis for her
dermatological condition? Erythema nodosum
○ A 29-year-old man comes to the emergency department because of right lower
abdominal pain, bloody diarrhea, and fever for the past week. He says that he has
been having these episodes of right lower abdominal pain and loose stools over the
course of the past year. However, this is the first time he has noticed blood in his
diarrhea. Physical examination shows tenderness along the sacroiliac joint. There
are raised erythematous lesions along his back that are painful to touch. A barium
swallow is obtained and the results of which are shown below:
A colonoscopy is performed and shows diffuse granulomatous inflammation of
ileocecal junction with transmural ulcerations. Biopsies show non-caseating
granulomas and skip lesions, with ulcers that are transmural. Which of the following
is most likely to result as a complication of this patient's disease? Fat and vitamin
malabsorption
○ A 30-year-old woman comes to the office because of increasing fatigue over the
past several months. She was diagnosed with Crohn disease as a teenager, for
which she had an ileocolic resection 3 years ago. She has not had any recent
diarrhea or abdominal pain and says that she has not seen any blood in her stool.
She eats a well-balanced diet and takes a multivitamin regularly. Her fecal occult
blood test is negative. Which of the following best explains this patient's fatigue?
Vitamin B 12 deficiency
○ A 40-year-old woman comes to the emergency department because of sudden
onset hemoptysis. She has started a new treatment for her Crohn disease and says
that she has had a low-grade fever and night sweats for the past five days. Which of
the following agents added to her treatment regimen is most likely responsible for
her new symptoms? Infliximab
○ A 29-year-old man comes to the office because of pain during urination. He was
treated for a urinary tract infection last month. He also says he has progressively
worsening diarrhea over the past two years. These episodes of diarrhea typically
last 1-2 weeks, then subside for several weeks to months. Physical examination
shows joint pain in many locations, especially in his sacroiliac joint. A urine sample
is collected and shows pneumaturia. Which of the following best explains the
patient's recurrent urinary tract symptoms? Enterovesical fistulization
○ A 29-year-old man comes to the office with complaints of frequent loose stools and
abdominal pain. He started having these symptoms a month ago, and they did not
respond to over-the-counter antidiarrheals. He became concerned when he noticed
an increase in urgency and a small amount of blood admixed with mucoid stools
yesterday. Past medical history is noncontributory. He has not recently travelled
outside the country. Vitals are within normal limits. Physical examination shows mild
abdominal tenderness in the left lower quadrant. Colonoscopy shows erythematous
rectal mucosa that is friable to touch, along with scattered ulcerations extending up
to the distal part of sigmoid colon. The remainder of the colon and ileum are normal.
Which of the following is the most appropriate pharmacotherapy for the patient at
this stage? 5-aminosalicylic acid (5-ASA)
○ A 3-year old girl is brought to the office because of a protuberant abdomen and
wasted extremities. Her mother says that her daughter has had recurrent watery
diarrhea, poor appetite, and multiple episodes of emesis. She also reports
new-onset bedwetting and increased drinking behaviors. At her 1-year evaluation, it
was noted that she was failing to thrive. The patient was diagnosed with celiac
disease following a duodenal biopsy. There are no other significant physical findings
on the exam. Which of the following tests should be conducted? Glucose
concentrations
○ A 29-year-old man comes to the emergency department because of right lower
abdominal pain and distension, diarrhea, and fever. He says that he has been
experiencing these episodes for two years due to his inflammatory bowel disease.
The episodes typically last 1-2 weeks and then subside. His condition is currently
being managed by a gastroenterologist. Despite medical management with 2
medications, his symptoms have worsened. A biopsy from colonoscopy at the time
of diagnosis showed the presence of noncaseating granulomas and skip lesions
with transmural ulcers. A barium swallow is obtained and shown below. Which of the
following treatments is the most appropriate next step for management of this
patient's condition? Surgical resection
○ A 28-year-old man is brought to the emergency department with lethargy and severe
abdominal pain since yesterday. He has had 4-5 episodes of small-volume bloody
diarrhea per day accompanied by lower abdominal cramps for the past 2 months.
Review of systems reveals a 4-kg (8.8-lbs) weight loss and loss of appetite. The
remainder of his history is noncontributory. Temperature is 38.8°C (102°F), pulse is
125/min, respirations are 18/min, and blood pressure is 90/55 mmHg. Physical
examination shows lethargy and dry mucous membranes. There is abdominal
distension, diffuse abdominal tenderness, and decreased bowel sounds without
rebound or guarding. Rectal examination reveals marked tenderness, with
guaiac-positive, maroon-colored, liquid stools in the vault. Laboratory results are
shown below
Laboratory value Result
Complete blood
count
Hemoglobin 9.9 mg/dL
Platelets 455,000/mm3
Leukocytes 19,200/mm3
Erythrocyte 67 mm/hr
sedimentation
rate

Fluid resuscitation with 0.9% normal saline improves the blood pressure to
103/59 mmHg. Which of the following is the best next step in management of this
patient? Plain abdominal x-ray
○ A 15-year-old boy comes to the office because of an intensely itchy rash on his
elbows and knees. He states that the rash began as small vesicles a week ago.
Family history is significant for Graves disease in his mother. Physical examination
shows excoriated papules and erosions that are crusted over on his elbows, knees,
and buttocks, as shown below. Which of the following is most likely to confirm the
diagnosis? Skin biopsy
○ A 28-year-old man of Jewish descent comes to the emergency department because
of abdominal pain, diarrhea, and fever. Physical examination shows a tender
abdomen and a perianal fistula. A biopsy is done and shows chronic inflammation in
the small intestine involving all layers of the intestinal wall. A monoclonal antibody
against which of the following cytokines would be most beneficial in the treatment of
this patient? TNFa
○ A 29-year-old man comes to the emergency department because of right lower
abdominal pain, diarrhea, and fever. He says that he has had episodes of this
condition for the past two years. The episodes typically last 1-2 weeks, then subside
for many weeks to months. During the episodes, he sometimes has bloody diarrhea.
Physical examination shows joint pain in many locations, especially in his sacroiliac
joint. There are multiple erythema nodosum lesions on his anterior shins. A barium
swallow is performed, and the result is shown below. Which of the following best
describes the clinical finding shown below? String sign of Kantor

● PLAYLIST 10
○ A 15-year-old girl comes to the office because of abdominal pain and Diarrhea for
the past three months. Her symptoms occur in the mornings before school. Physical
examination shows positive bowel sounds, no abdominal tenderness, and no mass
on palpation. Stool samples, blood work, and colonoscopy and upper endoscopy all
show no abnormalities. Which is the most likely diagnosis? Irritable bowel syndrome
○ A 24-year-old woman comes to the office because of intermittent diarrhea and
crampy abdominal pain over the past year. During these episodes, she notices an
increased frequency of bowel movements and softer stool consistency. Her
symptoms get worse with stress and improve when she takes loperamide. The
abdominal pain is decreased after defecation. She denies bleeding, weight loss,
early satiety, family history of colon cancer, and smoking.
Esophagogastroduodenoscopy and colonoscopy with biopsy are unremarkable.
Which of the following is the most likely diagnosis? Irritable bowel syndrome
○ A 15-year-old girl is brought to the office because she's been having intermittent
diarrhea with associated abdominal pain for the past year. Occasionally the
abdominal pain improves after she has a bowel movement. She experiences
frequent stomach aches that occur without warning, and has tried to avoid various
foods including milk, meats, grains, and caffeine, with only moderate and
inconsistent improvements in symptoms. She was diagnosed with obsessive
compulsive disorder three years ago which is now managed with 100 mg of
sertraline daily. She denies vomiting, fever, hematochezia, and melenaThere is no
family history of gastrointestinal disorders. She is currently in the 60th percentile for
weight given her height and her age. Physical examination shows no abnormalities.
A barium contrast enema shows no abnormalities. Which of the following is the most
likely diagnosis? Irritable bowel syndrome

● PLAYLIST 11
○ A 54-year-old man comes to the office five days after his first screening
colonoscopy. He has no acute complaints. He has a history of hypertension and
diabetic nephropathy, both treated with quinapril. His temperature is 37ºC (98.6ºF),
pulse is 89/min, respirations are 14/min, and blood pressure is 130/85 mm Hg. The
colonoscopy shows two 0.5 cm polypoid lesions in the descending colon, both
removed by polypectomy. Biopsy findings of one of the polyps is shown below. 3
years
○ A 75-year-old African American male comes to the gastroenterology department for
his scheduled screening colonoscopy. His medical history includes well-controlled
hypertension, type 2 diabetes mellitus, and long-term cigarette smoking. Family
history includes endometrial cancer in his mother. His endoscopic examination
shows a 2 cm (0.79 in), sessile polyp approximately 6 cm (2.36 in) above the
dentate line not amenable to endoscopic polypectomy. Which of the following is the
most appropriate next step? Multiple endoscopic biopsies of the polyp at the time of
initial colonoscopy
○ A 40-year-old man comes to the office because of multiple lesions on his forehead.
He states that he otherwise has no current medical issues. His medical history is
also noncontributory. Family history includes a father with small bowel cancer at age
48, and a sister aged 41 with endometrial cancer. Examination of his forehead is
shown below. His lesions are biopsied and are diagnosed as sebaceous adenomas.
Which of the following is an appropriate next step in the management of this
patient? Order a colonoscopy
○ A 55-year-old man comes to the office because of persistent fevers, fatigue, loss of
appetite, and transient chest pain for the past 2 months. He has also unintentionally
lost 7 kg (15 lb) over the same time period. His temperature is 38.8°C (102°F), pulse
is 87/min, respirations are 18/min, and blood pressure is 110/78 mm Hg. Physical
examination is non-contributory. Serial blood cultures grow Streptococcus
gallolyticus. Echocardiogram reveals vegetations on the mitral valve. Once the
patient is stabilized and treated for his bacteremia, which of the following additional
diagnostic tests should be performed? Colonoscopy
○ A 50-year-old man comes to the office because of an interest in treatment options
for his recent cancer diagnosis. He has a history of well-controlled hypertension,
psoriasis, and osteoarthritis. He has rectal carcinoma, which is approximately 3.0
cm from his anal verge with no evidence of metastatic disease. Which of the
following is the most appropriate treatment of choice for this mass? Abdominal
perineal resection with permanent colostomy
○ A 60-year-old woman is scheduled to undergo a routine screening colonoscopy for
colorectal cancer. She has no changes in bowel habits, unintentional weight loss, or
noticeable blood in the stool. Her previous colonoscopy 10 years ago showed no
abnormalities. Past medical history is significant for type II diabetes mellitus. There
is no family history of colorectal cancers. Colonoscopy is performed, which reveals a
single, sessile 10-mm polyp covered in mucous in the ascending colon. Histology of
this polyp subtype from a different patient is shown below: complete excision of the
polyp
○ A 61-year-old man comes to his physician for evaluation of excessive fatigue and
shortness of breath. He has also unintentionally lost 12-lb (5.4 kg) in the past 2
months. He has not had abdominal pain, melena, or changes in bowel habits. He
has not visited a physician in many years, but he was urged by his partner to make
an appointment. The patient recently retired from a battery-manufacturing factory,
which he worked at for 35 years. The patient has smoked a pack of cigarettes daily
for the past 40 years. His temperature is 37.0°C (98.6°F), pulse is 98/min,
respirations are 14/min, and blood pressure is 135/85 mmHg. Physical examination
is notable for skin and conjunctival pallor. Laboratory results are as follows:
Laboratory Value Result
Hemoglobin 9.8 g/dL
Hematocrit 40%
Mean corpuscular volume (MCV) 74 μm3
Which of the following diagnostic studies would be most helpful in confirming this patient’s
underlying diagnosis? Colonoscopy
○ A 55-year-old man comes to the office because of persistent fevers, fatigue, loss
of appetite, and transient chest pain for 2 months. He also mentions that he has
unintentionally lost 7 kg (15 lb) over the same time period. His temperature is
38.8°C (102°F), pulse is 87/min, respirations are 18/min, and blood pressure is
110/78 mm Hg. Examination is noncontributory. Serial blood cultures grow
Streptococcus bovis. Once the patient is stabilized and treated for his
bacteremia, which of the is the most appropriate next step? Colonoscopy

You might also like